You are on page 1of 392

RADIOLOGY

CASE REVIEW SERIES | Sp ine Imaging


Notice
Medicine is an ever-changing science. As new research and clinical experience broaden our knowledge, changes
in treatment and drug therapy are required. The authors and the publisher of this work have checked with sources
believed to be reliable in their e orts to provide information that is complete and generally in accord with the
standards accepted at the time of publication. However, in view of the possibility of human error or changes in
medical sciences, neither the authors nor the publisher nor any other party who has been involved in the preparation
or publication of this work warrants that the information contained herein is in every respect accurate or complete,
and they disclaim all responsibility for any errors or omissions or for the results obtained from use of the information
contained in this work. Readers are encouraged to con rm the information contained herein with other sources. For
example and in particular, readers are advised to check the product information sheet included in the package of each
drug they plan to administer to be certain that the information contained in this work is accurate and that changes
have not been made in the recommended dose or in the contraindications for administration. This recommendation is
of particular importance in connection with new or infrequently used drugs.
RADIOLOGY
CASE REVIEW SERIES | Spine Imaging

Allison Grayev, MD
Department of Radiology
University of Wisconsin
Madison, Wisconsin

Sayed Ali, MD
Associate Professor of Clinical Radiology
Temple University School of Medicine
Philadelphia, Pennsylvania

Reu b en Grech , MD
Neuroradiologist
Medical Imaging Department
Mater Dei Hospital
Malta

S ERIES ED ITO R

Rola n d Ta la n ow, MD, Ph D


President
Department of Radiology Education
Radiopolis, a subdivision of InnoMed, LLC
Stateline, Nevada

New York Chicago San Francisco Athens London


Madrid Mexico City Milan New Delhi Singapore
Sydney Toronto
Copyright © 2015 by McGraw-Hill Education. All rights reserved. Except as permitted under the United States Copyright Act of 1976, no part of this publication may be
reproduced or distributed in any form or by any means, or stored in a database or retrieval system, without the prior written permission of the publisher, with the exception
that the program listings may be entered, stored, and executed in a computer system, but they may not be reproduced for publication.

ISBN: 978-0-07-179824-2

MHID: 0-07-179824-2

The material in this eBook also appears in the print version of this title: ISBN: 978-0-07-179808-2,
MHID: 0-07-179808-0.

eBook conversion by codeMantra


Version 1.0

All trademarks are trademarks of their respective owners. Rather than put a trademark symbol after every occurrence of a trademarked name, we use names in an editorial
fashion only, and to the bene t of the trademark owner, with no intention of infringement of the trademark. Where such designations appear in this book, they have been
printed with initial caps.

McGraw-Hill Education eBooks are available at special quantity discounts to use as premiums and sales promotions or for use in corporate training programs. To contact
a representative, please visit the Contact Us page at www.mhprofessional.com.

TERMS OF USE

This is a copyrighted work and McGraw-Hill Education and its licensors reserve all rights in and to the work. Use of this work is subject to these terms. Except as
permitted under the Copyright Act of 1976 and the right to store and retrieve one copy of the work, you may not decompile, disassemble, reverse engineer, reproduce,
modify, create derivative works based upon, transmit, distribute, disseminate, sell, publish or sublicense the work or any part of it without McGraw-Hill Education’s
prior consent. You may use the work for your own noncommercial and personal use; any other use of the work is strictly prohibited. Your right to use the work may be
terminated if you fail to comply with these terms.

THE WORK IS PROVIDED “AS IS.” McGRAW-HILL EDUCATION AND ITS LICENSORS MAKE NO GUARANTEES OR WARRANTIES AS TO THE
ACCURACY, ADEQUACY OR COMPLETENESS OF OR RESULTS TO BE OBTAINED FROM USING THE WORK, INCLUDING ANY INFORMATION
THAT CAN BE ACCESSED THROUGH THE WORK VIA HYPERLINK OR OTHERWISE, AND EXPRESSLY DISCLAIM ANY WARRANTY, EXPRESS OR
IMPLIED, INCLUDING BUT NOT LIMITED TO IMPLIED WARRANTIES OF MERCHANTABILITY OR FITNESS FOR A PARTICULAR PURPOSE. McGraw-Hill
Education and its licensors do not warrant or guarantee that the functions contained in the work will meet your requirements or that its operation will be uninterrupted or
error free. Neither McGraw-Hill Education nor its licensors shall be liable to you or anyone else for any inaccuracy, error or omission, regardless of cause, in the work or
for any damages resulting therefrom. McGraw-Hill Education has no responsibility for the content of any information accessed through the work. Under no circumstances
shall McGraw-Hill Education and/or its licensors be liable for any indirect, incidental, special, punitive, consequential or similar damages that result from the use of or
inability to use the work, even if any of them has been advised of the possibility of such damages. This limitation of liability shall apply to any claim or cause whatsoever
whether such claim or cause arises in contract, tort or otherwise.
For my wonderful family and colleagues without
whom this would not have been possible.
— Allison Grayev, MD

To my parents, siblings, Chandra and my incredible sons


Rayhan and Kian, for your support in all that I do.
— Sayed Ali, MD

To my loved ones. Your continuous support was invaluable.


— Reuben Grech, MD
This page intentionally left blank
Contents

Series Preface ix
Preface xi

Easy Cases 1

Moderately Di cult Cases 155

Most Di cult Cases 299

Subject Index 367


Chapter Index 369
Subchapter Index 371
Di culty Level Index 373
Author Index 375
Acknowledgment Index 377

vii
This page intentionally left blank
Series Preface

M
aybe I have an obsession or cases, but when I was with images and questions and page 2 with the answers and
a radiology resident I loved to learn especially rom explanations. This approach avoids unintentional peeking at
cases, not only because they are short, exciting, and the answers be ore deciding on the correct answers yoursel .
un—similar to a detective story in which the aim is to get to We keep it strict: one case per page! This way it remains up
“the bottom” o the case—but also because, in the end, that’s to your own knowledge to f gure out the right answer.
what radiologists are aced with during their daily work. Another example that residents (including me) did miss
Since medical school, I have been ascinated with learning, in traditional case review books is that these books did not
not only or my own benef t but also or the sake o teaching highlight the pertinent f ndings on the images: sometimes,
others, and I have enjoyed combining my IT skills with my even looking at the images as a group o residents, we could
growing knowledge to develop programs that help others in not f nd the abnormality. This is not only rustrating but also
their learning process. Later, during my radiology residency, time-consuming. When you prepare or the boards, you want
my passion or case-based learning grew to a level where the to use your time as e iciently as possible. Why not show
idea was born to create a case-based journal: integrating new annotated images? We tackled that challenge by provid-
concepts and technologies that aid in the traditional learn- ing, on the second page o each case, the same images with
ing process. Only a ew years later, the Journal of Radiology annotations or additional images that highlight the f ndings.
Case Reports became an internationally popular and PubMed When you are preparing or the boards and managing
indexed radiology journal—popular not only because o your clinical duties, time is a luxury that becomes even more
the interactive eatures but also because o the case-based precious. Does the resident preparing or the boards truly
approach. This led me to the next step: why not tackle some- need lengthy discussions as in a typical textbook? Or does the
thing that I especially admired during my residency but that resident rather want a “rapid ire” mode in which he or she
could be improved—creating a new interactive case-based can “ ly” through as many cases as possible in the shortest
review series. I imagined a book series that would take into possible time? This is the reality when you start your work
account new developments in teaching and technology and a ter the boards! Part o our concept with the new series is
changes in the examination process. providing short “pearls” instead o lengthy discussions. The
As did most other radiology residents, I loved the tradi- reader can easily read and memorize these “pearls.”
tional case review books, especially or preparation or the Another challenge in traditional books is that questions
boards. These books are quick and un to read and ocus in a are asked on the f rst page and no direct answer is provided,
condensed way on material that will be examined in the f nal only a lengthy block o discussion. Again, this might become
boards. However, nothing is per ect and these traditional case time-consuming to ind the right spot where the answer
review books had their own intrinsic aws. The authors and is located i you have doubts about one o several answer
I have tried to learn rom our experience by putting the good choices. Remember: time is money—and li e! There ore,
things into this new book series but omitting the bad parts we decided to provide explanations to each individual ques-
and exchanging them with innovative eatures. tion, so that the reader knows exactly where to f nd the right
What are the eatures that distinguish this series rom answer to the right question. Questions are phrased in an
traditional series o review books? intuitive way so that they it not only the print version but
To save space, traditional review books provide two also the multiple-choice questions or that particular case in
cases on one page. This requires the reader to turn the page our online version. This system enables you to move back
to read the answer or the irst case but could lead to unin- and orth between the print version and the online version.
tentional “cheating” by seeing also the answer o the second In addition, we have provided up to 3 re erences or
case. Doesn’t this de eat the purpose o a review book? From each case. This case review is not intended to replace tra-
my own authoring experience on the USMLE Help book ditional textbooks. Instead, it is intended to reiterate and
series, it was well appreciated that we avoided such acciden- strengthen your already existing knowledge ( rom your train-
tal cheating by separating one case rom the other. Taking the ing) and to f ll potential gaps in your knowledge.
positive experience rom that book series, we decided that However, in a collaborative e ort with the Journal of
each case in this series should consist o two pages: page 1 Radiology Case Reports and the international radiology

ix
x Series Preface

community Radiolopolis, we have developed an online this series is structured on di f culty levels so that the series
repository with more comprehensive in ormation or each also becomes use ul to an audience with limited experience
case, such as demographics, discussions, more image exam- in radiology (nonradiologist physicians or medical students)
ples, interactive image stacks with scroll, a window/level up to subspecialty-trained radiologists who are preparing or
eature, and other interactive eatures that almost resemble their CAQs or who just want to re resh their knowledge and
a workstation. In addition, we are planning ahead toward use this series as a re erence.
the new Radiology Boards ormat and are providing rapid I am delighted to have such an excellent team o US and
ire online sessions and mock examinations that use the international educators as authors on this innovative book
cases in the print version. Each case in the print version is series. These authors have been thoroughly evaluated and
crosslinked to the online version using a case ID. The case selected based on their excellent contributions to the Journal
ID number appears to the right o the diagnosis heading at of Radiology Case Reports, the Radiolopolis community, and
the top o the second page o each case. Each case can be other academic and scientif c accomplishments.
accessed using the case ID number at the ollowing web It brings especially personal satis action to me that this
site: www.radiologycasereviews.com/case/ID, in which project has enabled each author to be involved in the over-
“ID” represents the case ID number. I you have any ques- all decision-making process and improvements regarding
tions regarding this web site, please e-mail the series editor the print and online content. This makes each participant not
directly at roland@talanow.in o. only an author but also part o a great radiology product that
I am particularly proud o such a symbiotic endeavor o will appeal to many readers.
print and interactive online education and I am grate ul to Finally, I hope you will experience this case review book
McGraw-Hill or giving me and the authors the opportunity as it is intended to be: a quick, pertinent, “get to the point”
to provide such a unique and innovative method o radiology radiology case review that provides essential in ormation or
education, which, in my opinion, may be a trendsetter. the radiology boards in the shortest time available, which, in
The primary audience o this book series is the radiol- the end, is crucial or preparation or the boards.
ogy resident, particularly the resident in the inal year who
is preparing or the radiology boards. However, each book in Roland Talanow, MD, PhD
Preface

S
pine imaging has always appealed to me as it crosses tions, but also prepare you or your career and aid you in
several di erent subspecialties and imaging modali- appreciating the value that a well-trained radiologist can add
ties, combining biomechanical and neurological rea- in spinal imaging.
soning to aid in patient imaging. This compilation o spine
cases is designed to both review common entities and chal- Allison Grayev, MD
lenge you to stretch your diagnostic abilities. It is my hope Sayed Ali, MD
that this will not only enable you to prepare or examina- Reuben Grech, MD

xi
This page intentionally left blank
RADIOLOGY
CASE REVIEW SERIES | Sp ine Imaging
This page intentionally left blank
Unrestrained passenger in motor vehicle accident

1. What should be included in the di erential


diagnosis?

2. What structures are involved?

3. What type o odontoid racture is associated


with the highest degree o nonunion?

4. What are the mechanisms o injury?

5. What are treatment options?

1
Fracture C2—type 2 2597
Case ranking/dif culty: Category: Vertebral body

Sagittal reconstructed CT image at bone windows demonstrates Sagittal reconstructed CT image at soft tissue windows
lucency through the base of the odontoid with associated demonstrates epidural hematoma associated with odontoid
displacement and angulation. fracture.

Answers
• The racture is usually the result o a hyper exion or
1. C2 ractures (all subtypes) and os odontoideum need
hyperextension injury, and may occur in the setting
to be considered in the setting o lucency within the C2
o high-speed trauma (motor vehicle accident) or
vertebral body.
lower-velocity trauma ( all rom a standing height,
2. Type 2 ractures traverse the base o the dens. particularly in the elderly).
• Type 2 odontoid ractures are most at risk or
3. Type 2 ractures have the highest incidence o nonunion.
nonunion.
The prevalence o nonunion in type 2 ractures
• Features associated with nonunion include increased
approaches 50%.
displacement (>5 mm anterior or >2 mm posterior),
4. Flexion loading is the most common mechanism, advanced patient age, comminution, and delay in
ollowed by extension loading. diagnosis.
• Classif cation o dens ractures:
5. Analgesia and halo f xation are appropriate f rst-line
• Type 1—through the odontoid tip
treatment methods; however, i the patient is older or
• Type 2—through the odontoid base
there is a greater degree o displacement (5 mm anterior
• Type 3—involvement o the vertebral body
or 2 mm posterior), these patients o ten require surgical
f xation.

Suggested Readings
Pearls Greene KA, Dickman CA, Marciano FF, Drabier JB, Hadley
• Type 2 odontoid ractures are the most common MN, Sonntag VKH. Acute axis ractures: analysis o
subtype o odontoid ractures and odontoid ractures management and outcome in 340 consecutive cases.
are the most common cervical spine racture, Spine. 1997;22:1843-1852.
accounting or up to 15% o all cervical spine Rao SK, Wasyliw C, Nunez DB. Spectrum o imaging
ractures. f ndings in hyperextension injuries o the neck.
Radiographics. 2005;25:1239-1254.
2
Neck pain

1. What should be included in the di erential


diagnosis?

2. What are common presenting symptoms?

3. What measurement is used to assess this


abnormality?

4. What is the etiology?

5. What are treatment options?

3
Platybasia 2598
Case ranking/dif culty: Category: Vertebral body

4. Many etiologies can cause platybasia, including


rickets, osteomalacia, trauma, osteogenesis imper ecta,
cleidocranial dysostosis, and Paget disease.
5. Conservative management is most appropriate or
isolated platybasia; however, i basilar invagination
is present, surgery may be indicated to reduce
complications.

Pearls
• Platybasia is def ned as a Welcher basal angle o
greater than 143°. The Welcher basal angle is the
angle ormed when tangents rom the clivus and
sphenoid bone intersect. The normal angle is between
125° and 143°.
• While platybasia may be an isolated f nding, there is a
strong association with basilar invagination.
• Platybasia can be seen with cleidocranial dysostosis,
osteogenesis imper ecta, Paget disease, trauma, or
rickets/osteomalacia.
• Platybasia unto itsel is asymptomatic; however, there
is an association with basilar invagination.
Sagittal CT image demonstrates a short hypoplastic clivus with • Basilar invagination re ers to acquired cephalad
associated platybasia. Incidentally noted is congenital fusion of displacement o the dens above the oramen magnum.
C2 and C3.
• Basilar impression re ers to congenital cephalad
displacement o the dens above the oramen magnum.
Answers
1. Abnormalities o the craniocervical junction should
Suggested Readings
be considered, including basilar impression, basilar
invagination, clival hypoplasia, occipital condyle Cronin CG, Lohan DG, Mhuircheartigh JN, Meehan CP,
hypoplasia, and atlantooccipital assimilation. Murphy J, Roche C. “CT evaluation o Chamberlain’s,
McGregor’s and McRae’s skull-base lines.” Clin Radiol.
2. Platybasia unto itsel is generally asymptomatic; 2009;64:64-69.
however, pain, syrinx, and myelopathy can be seen in Smoker WRK. “Craniovertebral junction: normal anatomy,
association with basilar invagination. craniometry, and congenital anomalies. Radiographics.
3. Platybasia is assessed using the Welcher basal angle, 1994;14:255-277.
which is ormed at the intersection o tangents drawn Smoker WRK, Khanna G. Imaging the craniocervical
rom the clivus and sphenoid bone. junction. Childs Nerv Syst. 2008;24:1123-1145.

4
Back pain

1. What should be included in the di erential


diagnosis?

2. What are common presenting symptoms?

3. What are components o the classif cation


system?

4. Which portions o the spine are a ected?

5. What are treatment options?

5
Osteoid osteoma 2482
Case ranking/dif culty: Category: Posterior elements

5. Conservative management, surgical resection, and/or


percutaneous radio requency ablation are all treatment
options. Depending on the instability associated with
surgical resection, f xation may be needed. Percutaneous
ethanol ablation is contraindicated given the close
proximity to spinal cord and nerve roots. Also, given
the natural tendency o osteoid osteomas to heal
spontaneously, conservative management on long term
NSAIDS has been success ul.

Pearls
• Osteoid osteoma is a benign primary bone tumor that
is most commonly seen in boys and young men.
• The lesion is characterized by a cortical location, a
mineralized nidus, and surrounding reactive sclerosis.
• These lesions most commonly occur in long bones,
with only approximately 10% within the spine. When
they do occur in the spine, the posterior elements are
most commonly involved.
• I lesions are greater than 1.5 cm, they are categorized
as osteoblastomas. Some argue that osteoblastomas
are di erent lesions as they are associated with a small
risk o malignant degeneration.
Sagittal reconstructed CT image at bone windows demonstrates • Patients o ten present with scoliosis related to pain and
well-circumscribed lesion in the inferior left pedicle of T10 with muscle spasm.
extension to the superior articulating facet, and with associated • Treatment traditionally consists o surgical resection;
sclerotic changes. however, there is a growing body o literature
looking at the use o percutaneous cryotherapy and
Answers radioablation.

1. Numerous entities may present as sclerotic oci within


the posterior elements, including osteoblastoma,
Suggested Readings
osteomyelitis, sclerotic metastasis, lymphoma, and
osteoid osteoma. Chai JW, Hong SH, Choi JY, et al. Radiologic diagnosis o
osteoid osteoma: rom simple to challenging f ndings.
2. The classic presentation is scoliosis and nighttime pain, Radiographics. 2010;30:737-749.
relieved by nonsteroidal anti-in ammatory medications. Gasbarrini A, Cappuccio M, Bandiera S, Amendola L,
3. The most commonly used classif cation scheme van Urk P, Boriani S. Osteoid osteoma o the mobile
(Kransdor et al) separates osteoid osteomas into spine. Spine. 2011;36:2089-2093.
medullary, cortical, and subperiosteal in location. Goto T1, Shinoda Y, Okuma T, et al. Administration
o nonsteroidal anti-in ammatory drugs accelerates
4. Osteoid osteoma can a ect any portion o the spine, spontaneous healing o osteoid osteoma. Arch Orthop
but is most o ten seen in the mobile spine, split evenly Trauma Surg. 2011;131(5):619-625.
between cervical, thoracic, and lumbar regions. Kransdor MJ, Stull MA, Gilkey FW, Moser RP. Osteoid
osteoma. Radiographics. 1991;11:671-696.

6
Back pain

1. What should be included in the di erential


diagnosis?

2. What is the classic imaging appearance?

3. What are common presenting symptoms?

4. What is the etiology?

5. What are treatment options?

7
Sacral chordoma 2512
Case ranking/dif culty: Category: Vertebral body

Axial T2 image demonstrates multilobulated T2 hyperintense mass. Axial T1 image following gadolinium administration demonstrates
irregular enhancement of the expansile sacral mass.

4. Chordomas arise secondary to notochordal remnant


degeneration.
5. Treatment includes surgical resection—gross total
resection increases overall survival—with adjuvant
radiotherapy. There is no role or chemotherapy or
plasmapheresis. Biopsy can increase disease recurrence
by seeding the tract.

Pearls
• Chordoma is a rare tumor arising rom notochordal
remnants.
• The sacrum is the most common site (30-50%),
ollowed by the skull base, and f nally the remainder
o the spine.
• They generally present as a T2 hyperintense, lobulated
mass with extensive osseous destruction.
• Symptoms at presentation are based on location;
Sagittal T1 image following gadolinium administration demonstrates however, sacral lesions o ten grow rather large be ore
irregular enhancement of the expansile sacrococcygeal mass. becoming symptomatic.
• Prognosis is based on size at diagnosis and pathologic
Answers f ndings, including necrosis.
• Treatment is surgical resection with adjuvant
1. Many lesions can present as an expansile T2 radiotherapy, and total resection increases survival.
hyperintense destructive osseous lesion, including
metastasis, plasmacytoma, chordoma, lymphoma,
chondrosarcoma and giant cell tumor.
Suggested Readings
2. Chordomas o ten present as a destructive osseous lesion
with an associated so t tissue mass, which demonstrates deBruine FT, Kroon HM. Spinal chordoma: radiologic
T2 hyperintensity and irregular septal enhancement. eatures in 14 cases. AJR. 1988;150:861-863.
Rich TA, Schiller A, Suit HD, Mankin HJ. Clinical and
3. Sacral chordomas can grow to a large size be ore pathologic review o 48 cases o chordoma. Cancer.
diagnosis and o ten present with slowly progressive 1985;56:182-187.
symptoms over months to years, including back pain,
constipation, radicular symptoms, and incontinence.
8
Snowmobile accident

1. What should be included in the di erential


diagnosis?

2. What is the etiology?

3. What are the presenting symptoms?

4. What is the next step in evaluation?

5. What are treatment options?

9
Atlantooccipital assimilation 2483
Case ranking/dif culty: Category: More than one category

4. Flexion/extension radiographs allow or identif cation


o unstable craniocervical junction anomalies, which
should be re erred or surgical f xation.
5. While observation and activity restriction are appropriate
or asymptomatic lesions, symptomatic lesions require
more aggressive management, including traction,
surgical decompression, and/or surgical usion.

Pearls
• Anomalies o the craniocervical junction are not
uncommon in clinical practice. It is important to be
able to di erentiate those that require surgical f xation
rom those that do not.
• Atlantooccipital usion is one o the more common
mal ormations, and while it is o ten asymptomatic,
there are reports o associated myelopathic symptoms
as basilar invagination develops.
• There is a risk o instability, and evaluation with
exion/extension radiographs may be indicated.
Coronal reconstructed image from a cervical spine CT demonstrates • I one usion anomaly is noted during evaluation o the
complete fusion of the occipital condyles and C1 arch. spine, have a higher index o suspicion to look or an
additional abnormality.

Answers
1. Atlantooccipital assimilation is an abnormality o Suggested Readings
vertebral usion along the Klippel Feil spectrum. Basilar Rande AV, Rai R, Prabhu LV, Kumaran M, Pai MM. Atlas
invagination is a complication o atlantooccipital assimilation: a case report. Neuroanatomy. 2007;6:32-33.
assimilation. Smoker WRK. Craniovertebral junction: normal anatomy,
2. Atlantooccipital assimilation arises secondary to a craniometry, and congenital anomalies. Radiographics.
segmentation ailure between the ourth occipital 1994;14:255-277.
sclerotome and the f rst spinal sclerotome.
3. Headaches, particularly occipital or those exacerbated
by cervical spine movement, myelopathic symptoms,
and cranial nerve or brainstem def cits, can be seen in
atlantooccipital assimilation. However, this is usually an
asymptomatic abnormality.

10
Positive VDRLtest in an HIV+ patient, sudden onset of lower extremity
weakness during lumbar puncture

1. What patient in ormation should be considered


prior to planning a lumbar puncture?

2. What are the pre erred levels or per orming


lumbar puncture?

3. What complications may lead to the


development o lower extremity paresthesias
during a lumbar puncture?

4. What are the indications or uoroscopic


guidance or lumbar puncture?

5. What are the treatment options?

11
Complication, lumbar puncture—inadvertent 2241
epidural anesthetic injection
Case ranking/dif culty: Category: Spinal canal

Axial CT image demonstrates focal gas in the anterior epidural Sagittal CT image demonstrates focal gas in the anterior epidural
space. space.

Answers
1. Laboratory evaluation or coagulopathy, evaluation Pearls
or increased intracranial pressure, review o patient • It is imperative to maintain excellent technique when
history or recent lumbar surgery, prior lumbar imaging per orming lumbar punctures to avoid complication.
studies, and the patient ability to give consent should be • Anesthetizing the so t tissues should be per ormed
considered in planning a lumbar puncture. only a ter placing negative pressure on the syringe to
avoid intra-arterial injection.
2. L2-L3 and L3-L4 are the pre erred levels; they are
• Epidural injection is more di f cult to exclude as by
generally below the level o the cord but the spinal canal
def nition there should not be return with negative
remains largest at these levels.
pressure.
3. Inadvertent administration o anesthetic may result in • In the case o probable epidural injection o anesthetic,
lower extremity paresthesia; however, consideration o imaging may need to be per ormed, particularly in the
epidural hematoma is important, particularly in a patient coagulopathic patient to exclude epidural hematoma.
with borderline coagulation values.
4. Fluoroscopic guidance may be o benef t in the obese
patient or postoperative patient, where landmarks may Suggested Readings
not be readily apparent. Additionally, i attempts without ACR-ASNR Practice guideline or the per ormance o
guidance are unsuccess ul, uoroscopy may increase the myelography and cisternography. Revised (2008).
success rate. Ru RL, Dougherty JH. Complications o lumbar puncture
5. Even i there is a small component o epidural ollowed by anticoagulation. Stroke. 1981;12:879-881.
hematoma, conservative management with spontaneous Yu SD, Chen MY, Johnson AJ. Factors associated with
resolution o symptoms is the rule. In the particularly traumatic uoroscopy-guided lumbar punctures: a
coagulopathic patient, alerting neurosurgery may be retrospective review. AJNR. 2009;30:512-515.
indicated in case o rapidly enlarging hematoma.

12
Radicular pain

1. What should be included in the di erential


diagnosis?

2. What are common presenting symptoms?

3. What are the components o the intervertebral


disc?

4. What is the natural history?

5. What are the treatment options?

13
Disc herniation 2187
Case ranking/dif culty: Category: Nerve roots/Nerve plexus/Peripheral nerves

4. Most patients will have spontaneous resorption with


conservative management; however, there is potential
or worsening disease and development o diskogenic
endplate changes or sequestered ragments.
5. Most patients will have resolution o symptoms
with conservative management/physical therapy.
Chemonucleolysis and diskectomy can be per ormed in
re ractory cases.

Pearls
• Disc herniations are one o the most common
etiologies o lumbar radiculopathy seen in an average
radiology practice.
• During the evaluation o disc herniations, it is
important to evaluate compression o the thecal sac,
the lateral recesses, and the neural oramina.
• A protrusion is a herniation o less than 50% o the
circum erence o the disc—this can be divided into
ocal (<25%) or broad based (25%-50%).
• A herniation o >50% is termed a bulge.
• Protrusions can be def ned as extrusions i the base o
the herniation is narrower than the remainder o the
herniated disc.
• I there is a piece o herniated disc that is not
continuous with the underlying disc, this can be
termed a sequestered disc ragment. This is important
to report as the surgeons may need to explore to f nd a
sequestered ragment.
• There are eight cervical nerve roots but only seven
cervical vertebrae; thus, at the C7-T1 level, the C8
AP radiograph following intrathecal administration of contrast
nerve roots are exiting.
demonstrates a left-sided lling defect.
• At the level o this disc protrusion (L5-S1), the disc
could impact the exiting L5 nerve root in the oramen
Answers or the traversing S1 nerve root in the lateral recess.
1. Extradural space-occupying lesions include epidural
abscess, epidural hematoma, disc herniation, epidural
f brosis, and nerve sheath tumor. Suggested Readings
2. Radiculopathy and back pain are the most common Weber H. Lumbar disk herniation: a controlled, prospective
presenting symptoms. Patients will mani est normal study with 10 years o observation. Spine. 1983;8:131-140.
to decreased re exes and may have lower extremity Weinstein JN, Tosteson TS, Lurie JD, et al. Surgical vs
weakness. nonoperative treatment or lumbar disk herniation. JAMA.
2006;296:2441-2450.
3. The central nucleus pulposus is surrounded by the
transitional zone and the peripheral annulus f brosis.

14
Progressive right ptosis

1. What is the most likely etiology o this lesion?

2. What is the common imaging appearance?

3. Where do these lesions commonly occur?

4. What are common presenting symptoms?

5. What are the treatment options?

15
Clival chordoma 2219
Case ranking/dif culty: Category: Spinal canal

Sagittal T2 image demonstrates expansile T1 hypointense clival


lesion.

Axial T2 image demonstrates expansile T2 hyperintense mass


involving the clivus and sphenoid sinuses with extension into the
Pearls
posterior orbits.
• Chordoma is a rare tumor arising rom notochordal
remnants.
Answers • The skull base is the second most common location,
1. Expansile T2 hyperintense destructive osseous lesions a ter the sacrum.
include chondrosarcoma, metastasis, chordoma, • Chordomas are classically T2 hyperintense lobulated
plasmacytoma, and lymphoma. Destructive lesions masses with extensive osseous destruction.
o the clivus also include nasopharyngeal carcinoma, • Presentation o clival chordomas o ten include
pituitary adenoma, pituitary carcinoma, and intraosseous headache and cranial nerve dys unction, particularly
meningioma. palsy o the sixth cranial nerve leading to diplopia.
• Skull base tumors are notoriously di f cult to resect,
2. While chordomas are typically T2 hyperintense and T1 and adjuvant radiotherapy plays an important role in
hypointense with avid enhancement, there can be oci management.
o T1 hyperintensity, representing oci o hemorrhage or • These tumors tend to spread primarily through direct
mineralization. spread, with distant hematogenous spread less likely.
3. While these lesions can occur at any location within
the spine, the sacrum and coccyx are most common,
ollowed by the clivus.
Suggested Readings
4. Clival chordomas generally present with headaches and Erdem E, Angtuaco EC, Van Hemert R, Park JS, Al-Me ty O.
occasionally associated cranial neuropathies, including Comprehensive review o intracranial chordoma.
diplopia and acial pain. There is potential to develop Radiographics. 2003;23:995-1009.
vertebrobasilar insu f ciency with posterior extension o Rich TA, Schiller A, Suit HD, Mankin HJ. Clinical and
tumor. pathologic review o 48 cases o chordoma. Cancer.
5. Treatment includes surgical resection—gross total 1985;56:182-187.
resection increases overall survival—with adjuvant
chemotherapy. There is no role or chemotherapy.
Biopsy can increase disease recurrence by seeding the
tract.

16
Sickle cell anemia, back pain

1. What should be included in the di erential


diagnosis?

2. What should be included in the di erential


diagnosis o vertebral body abnormalities in
sickle cell anemia?

3. What is the pathophysiology leading to this


abnormality?

4. What are additional osseous mani estations o


this disease?

5. What are the treatment options?

17
“Fishmouth” vertebra in sickle cell disease 2216
Case ranking/dif culty: Category: Vertebral body

2. Sickle cell anemia can produce osseous in arction,


pathologic racture, f sh mouth de ormity, and secondary
osteomyelitis.
3. Reactivation o red marrow leads to marrow expansion,
which thins the cortex and leaves it vulnerable to
compression rom the adjacent disc.
4. Additional osseous mani estations o sickle cell disease
include bone-in-bone vertebra, avascular necrosis, and
hair-on-end calvarium.
5. For asymptomatic concave vertebral bodies, no
treatment is required. I a patient presents in sickle cell
crisis, the key components o treatment are hydration,
supplemental oxygenation, and pain control.

Pearls
• The f sh mouth (Lincoln log or H shaped) vertebra
is ormed secondary to weakening o the endplates
rom marrow activation and associated medullary
expansion.
• As the medullary cavity expands, the endplates
become weakened and collapse.
• It is important to di erentiate this rom pathologic
racture, which is a recognized complication.
• Associated f ndings may include T1 and T2 marrow
hypointensity, secondary to reactivation, and
superimposed bone in arcts.
Lateral radiograph demonstrates biconcave deformity of the
vertebral bodies.
Suggested Readings
Answers Ejindu VC, Hine AL, Mashayekhi M, Shorvon PJ, Misra
1. Etiologies o hypointense marrow and vertebral RR. Musculoskeletal mani estations o sickle cell disease.
body mal ormations include osteopetrosis, leukemia, Radiographics. 2007;27:1005-1021.
pyknodysostosis, osteopathia striata, and sickle cell Ganguly A, Boswell W, Aniq H. Musculoskeletal
anemia. mani estations o sickle cell anaemia: a pictorial review.
Anemia. 2011:9.

18
New onset seizures

1. Which are associated abnormalities?

2. What are the presenting symptoms?

3. What should be included in the imaging


evaluation?

4. What are associated syndromes?

5. What are the treatment options?

19
Chiari I malformation 2206
Case ranking/dif culty: Category: Spinal cord

Axial T1 image demonstrates cerebellar tonsils crowding the


posterior foramen magnum.

Pearls
• Chiari I mal ormation is a heterogeneous group o
disorders.
• Chiari I is characterized by caudal extension o the
cerebellar tonsils more than 5 mm below the oramen
Sagittal T1 image demonstrates cerebellar tonsils extending magnum. Some argue that up to 6 mm can be normal,
through foramen magnum; no syringomyelia.
particularly in children.
• The tonsils develop a peg conf guration, instead o the
Answers normal rounded conf guration.
• The causative etiologies are variable, including both
1. Chiari I mal ormation is associated with abnormalities o
autosomal dominant and autosomal recessive genetics,
the skull base, skeleton, and ourth occipital sclerotome,
syndrome-associated mal ormations, and Chiari
including scoliosis, retro exed dens, kyphosis,
mal ormations associated with mal ormation o the
platybasia, and short clivus.
skull base.
2. Headache, gait disturbance, and myelopathy are the most • I the tonsils extend greater than 12 mm below the
common presenting symptoms. Additionally, Chiari oramen magnum, the patients are more likely to be
patients can develop cranial nerve palsy and ocular symptomatic.
disturbances.
3. MRI o the brain can be use ul in the evaluation o
hydrocephalus while spine imaging can be used to Suggested Readings
evaluate or the presence o syrinx. Cerebrospinal uid Elster AD, Chen MY. Chiari I mal ormations: clinical and
ow studies at the oramen magnum can be help ul radiologic reapparaisal. Radiology. 1992;183:347-353.
in the quantif cation o ow both preoperatively and Milhorat T, Chou MW, Trinidad EM, et al. Chiari I
postoperatively. mal ormation redef ned: clinical and radiographic
4. There are multiple syndromes associated with Chiari f ndings or 364 symptomatic patients. Neurosurgery.
I mal ormation, including Williams syndrome, 1999;44:1005-1017.
achondroplasia, and Klippel Feil syndrome.
5. Asymptomatic patients can o ten be ollowed with
imaging studies, while symptomatic patients o ten
require suboccipital craniectomy. Resection o the
posterior elements o C1 can be required depending on
the caudal extension o the cerebellar tonsils. Tonsillar
resection can be per ormed i needed.
20
Back pain, no radicular symptoms

1. What should be included in the di erential


diagnosis?

2. Why is it important to identi y degenerative


endplate changes?

3. What is the staging system?

4. What is the most common phase?

5. What are the treatment options?

21
Diskogenic endplate changes 2205
Case ranking/dif culty: Category: Vertebral body

Sagittal T1 image demonstrates hypointensity along the inferior Sagittal T2 image demonstrates hyperintensity along the inferior
endplate of L5 and superior endplate of S1 with adjacent disc endplate of L5 and hypointensity along the superior endplate of
degeneration. S1 with adjacent disc degeneration.

Answers
1. Di erential considerations include diskogenic endplate Pearls
changes and diskitis/osteomyelitis. • Degenerative endplate changes most commonly occur
in the lumbar spine adjacent to degenerative changes
2. It is important to f nd di erentiate diskogenic endplate
within the intervertebral disc.
changes rom metastasis or in ection. While these
• There are three recognized phases:
changes could serve as a potential source o back pain,
• Phase I (edema/vascular marrow)—T1 hypointense,
there are no f ndings that are su f cient to predict positive
T2 hyperintense
diskography results.
• Phase II ( atty inf ltration)—T1 hyperintense, T2
3. Type 1 changes are T1 hypointense and T2 hyperintense iso-to hyperintense
secondary to edema. Type 2 changes are T1 and T2 • Phase III (f brosis)—T1 hypointense, T2 hypointense
hyperintense secondary to atty inf ltration. Type 3 • Phase I changes may also demonstrate contrast
changes are T1 and T2 hypointense secondary to enhancement.
sclerosis. • It is important to recognize the characteristic imaging
appearance o these degenerative changes in order to
4. In patients undergoing MRI or degenerative disc
exclude either osseous lesion or in ection.
disease, approximately 15% will have Type 2 changes
(5% Type 1 and 1% Type 3).
5. Conservative management is usually indicated; spinal
Suggested Readings
usion may be considered or re ractory pain.
Albert HB, Manniche C. Modic changes ollowing lumbar
disc herniation. Eur Spine J. 2007;16:977-982.
Thompson KR, Dagher AP, Eckel TS, Clark M, Reinig JW.
Modic changes on MR images as studied with provocative
diskography. Radiology. 2009;250:849-855.

22
Prostate cancer and leg weakness

1. What should be included in the di erential


diagnosis?

2. What are common presenting symptoms?

3. What additional imaging studies should be


considered?

4. Which primary malignancies tend to have


sclerotic metastatic lesions?

5. What are the treatment options?

23
Di use marrow in ltration 2201
Case ranking/dif culty: Category: Vertebral body

Sagittal T1 image demonstrates di use marrow hypointensity. Sagittal T2 image demonstrates di use marrow hypointensity.

Answers
• However, it is an important entity to recognize as it
1. Etiologies o hypointense marrow include myelof brosis,
can be associated with malignancy-associated anemia
reactivation o red marrow, leukemia, osteopetrosis, and
secondary to marrow replacement.
metastatic marrow inf ltration.
• Look or an intervertebral disc that is brighter than the
2. Potential presenting symptoms o axial skeletal vertebral body marrow on T1 images.
metastasis include pain, pathologic racture, cord • Treatment is aimed at the primary malignancy.
compression, and radicular symptoms. • Advanced imaging techniques, including di usion-
weighted imaging, have been explored to evaluate the
3. CT or FDG-PET scan may be indicated or assessing
presence o marrow inf ltration.
extraosseous disease. Bone scan can be help ul or
identi ying appendicular sites o involvement.
4. Renal cell and thyroid cancers o ten present with lytic
Suggested Readings
lesions.
Padhani AR, van Ree K, Collins DJ, D’Sa S, Makris A.
5. Analgesia and treatment o the primary malignancy, Assessing the relation between bone marrow signal
including chemotherapy and radiation, are the mainstays intensity and apparent di usion coe f cient in di usion-
o treatment o di use metastatic disease. Vertebroplasty weighted MRI. AJR. 2013; 200:163-170.
may be indicated in the setting o superimposed Schmidt GP, Reiser MF, Baur-Melnyk A. Whole-body MRI
compression racture. or the staging and ollow-up o patients with metastasis.
Eur J Radiol. 2009;70:393-400.

Pearls
• Di use marrow inf ltration is a less common pattern
o osseous metastatic disease than ocal osseous
metastatic lesions.

24
Pain

1. What should be included in the di erential


diagnosis?

2. What are common presenting symptoms?

3. What is the classic clinical presentation?

4. What is the prognosis i untreated?

5. What are the treatment options?

25
Syringomyelia 2202
Case ranking/dif culty: Category: Spinal cord

Axial T2 image demonstrates anterior T2 hyperintensity within


the cord.

Sagittal T2 image demonstrates anterior T2 hyperintensity within


the cord.
hydromyelia rom syringomyelia on imaging;
pathologically, the ormer is lined by ependymal cells.
Answers • I there is extension o syringomyelia to the brain
stem, it can be termed syringobulbia.
1. Intramedullary T2 hyperintense lesions include cystic
• Syringomyelia can be seen as a complication o trauma;
intramedullary tumor, hydromyelia, myelomalacia, and
however, it can be a primary diagnosis that generally
syringomyelia.
a ects adolescents/young adults. In those cases, it is
2. Distal weakness and gait ataxia are the most common important to rule out obstruction to cerebrospinal uid
presenting symptoms; other symptoms include back ow, such as Chiari I mal ormation, or congenital
pain, radiculopathy, and sphincter dys unction. abnormality, such as Chiari II or tethered cord.
• Presenting symptoms are generally myelopathic with
3. The classic presentation is “cloak” distribution o pain
ataxia and distal weakness.
and loss o temperature sensation.
• The classic presentation consists o “cloak”
4. Primary syringomyelia generally has a slowly progressive distribution loss o pain and temperature sensation.
course without resolution. Treatment may halt progression, • Treatment is aimed at resolution o the underlying
but improves symptoms in less than hal o patients. disruption to ow; however, in patients with idiopathic
syringomyelia, placement o a syringosubarachnoid
5. For patients with Chiari I mal ormation, suboccipital
shunt may be necessary.
craniectomy is the treatment o choice to improve
cerebrospinal uid ow dynamics. Syringosubarachnoid
shunt placement may be considered or patients with
idiopathic syringomyelia. Suggested Readings
Klekamp J. Treatment o syringomyelia related to
nontraumatic arachnoid pathologies o the spinal canal.
Pearls Neurosurgery. 2013;72:376-389.
Williams B. On the pathogenesis o syringomyelia: a review.
• Syringomyelia re ers to cystic dilation within the
J Royal Soc Med. 1980;73:798-806.
spinal cord, which is not contiguous with the central
canal.
• Dilation o the central canal is re erred to as
hydromyelia. It can be di f cult to di erentiate

26
Chiari I malformation

1. What should be included in the di erential


diagnosis?

2. Why are T2 gradient echo sequences less


susceptible to cerebrospinal uid pulsation
arti act?

3. What are common arti acts encountered in


spine imaging?

4. How can cardiac gating be used to reduce


cerebrospinal uid pulsation arti act?

5. What are potential solutions when aced with


cerebrospinal uid pulsation arti act?

27
Cerebrospinal uid pulsation artifact 2198
Case ranking/dif culty: Category: Thecal sac

are important to recognize and be able to correct. For


example, motion arti act rom adjacent organs can o ten
be eliminated by applying strategic saturation bands and
swapping phase and requency encoding directions.
4. Cardiac gating reduces temporal phase-shi t e ects, both
cardiac dependent and unmasked, as well as decreasing
time-o - ight e ects.
5. Cerebrospinal uid pulsation arti act can usually be
diagnosed based on reviewing all sequences; however,
additional T2 gradient echo sequences or peripheral
cardiac gating may be necessary in the challenging
patient.

Pearls
• Cerebrospinal uid pulsation arti act is common in
children who tend to have hyperdynamic cerebrospinal
uid ow.
• It commonly occurs on T2 ast spin echo (FSE)
images, but resolves on other sequences.
• Using T2 gradient echo sequences can help minimize
the arti act.
• T2 gradient echo sequences are o ten pre erred in
Sagittal T2 image demonstrates T2 hypointense signal dorsal to imaging o the cervicothoracic spine where this
the cord. Note Chiari I malformation without syrinx. arti act is most likely to occur.

Answers
1. T2 hypointense signal could represent a ow void, Suggested Readings
calcif ed mass, or various stages o hemorrhage. Kwon JW, Yoon YC, Choi S-H. Three-dimensional isotropic
Pulsatility arti act can be conf rmed by the lack o T2-weighted cervical MRI at 3T: comparison with
f ndings on additional sequences. two-dimensional T2-weighted sequences. Clin Radiol.
2012;67:106-113.
2. Shorter echo time, gradient re ocusing pulse applied on
Low RN, Austin MJ, Ma J. Fast spin-echo triple echo dixon:
the read out gradient, and shorter repetition time make
initial clinical experience with a novel pulse sequence
the sequence less susceptible to pulsation arti act while
or simultaneous at-suppressed and non at-suppressed
maintaining T2 weighting.
T2-weighted spine magnetic resonance imaging. JMRI.
3. Common arti acts encountered in spine imaging include 2011;33:390-400.
Gibb truncation arti act, cerebrospinal uid pulsation Rubin JB, Enzmann DR, Wright A. CSF-gated MR imaging
arti act, respiratory motion arti act, cardiac motion o the spine: theory and clinical implementation.
arti act, and bowel peristalsis motion arti act. Arti acts Radiology. 1987;163:784-792.

28
Fall

1. What should be included in the di erential


diagnosis?

2. What structures are involved?

3. What are associated abnormalities?

4. What are the mechanisms o injury?

5. What are the treatment options?

29
Fracture C2—type 1 2195
Case ranking/dif culty: Category: Vertebral body

5. This racture is o ten managed by cervical


immobilization, either through cervical collar or halo
f xation.

Pearls
• Type 1 odontoid ractures are the most rare type o
dens racture, consisting o a racture isolated to the
cephalad portion o the dens without involvement o
the base or body.
• This type o dens racture is thought to result rom an
avulsion injury o the alar ligament.
Axial CT image demonstrates lucency through the anterior • These ractures may be treated conservatively using a
odontoid tip. cervical collar.
• Dynamic exion and extension views o the spine
should be obtained to exclude associated ligamentous
Answers
or osseous instability.
1. Etiologies o an odontoid lucency include racture and os
odontoideum. Os odontoideum results rom a congenital
non-union o the cephalad odontoid apophysis. These
lesions can be di erentiated by their sclerotic border and Suggested Readings
are usually f xed to the anterior arch o C1. Greene KA, Dickman CA, Marciano FF, Drabier JB, Hadley
MN, Sonntag VKH. Acute axis ractures: analysis o
2. Type 1 racture traverses the odontoid tip.
management and outcome in 340 consecutive cases.
3. Type 1 ractures can be seen in conjunction with Spine. 1997;22:1843-1852.
occipital condyle ractures and atlantooccipital Rao SK, Wasyliw C, Nunez DB. Spectrum o imaging
dislocation. f ndings in hyperextension injuries o the neck.
Radiographics. 2005;25:1239-1254.
4. This racture originates rom axial loading and
hyperextension.

30
Lumbar cutaneous hemangioma

1. What should be included in the di erential


diagnosis?

2. What are the open spinal dysraphisms?

3. What are common presenting symptoms?

4. What need to be evaluated or the classif cation


o congenital spinal dysraphism?

5. What are the treatment options?

31
Lipomyelocele 2194
Case ranking/dif culty: Category: More than one category

Axial T1 image demonstrates fat containing mass in the dorsal


spinal canal, in direct contact with the distal spinal cord. Marker
was placed over cutaneous hemangioma (external to patient).

Sagittal image demonstrates dorsal dysraphic defect extending


caudally from L3 to L4 with herniation of neural elements, Pearls
extending caudally to the L5 level, where it terminates in a fatty • Lipomyelocele is a subtype o closed spinal
mass.
dysraphism, suspected by the presence o an overlying
cutaneous lesion (hemangioma, hairy patch, skin
Answers dimple/pit).
• It is important to document the location o the
1. Closed spinal dysraphisms include lipomyelocele, placode–lipoma inter ace.
lipomyelomeningocele, f lum lipoma, diastematomyelia, • When the placode–lipoma inter ace is within the
and spina bif da occulta. spinal canal, the anomaly is a lipomyelocele; i it
2. Open spinal dysraphisms include meningocele, lies outside o the spinal canal, the anomaly is a
myelomeningocele, myeloschisis, hemimyelocele, and lipomyelomeningocele.
myelocele. • Patients can present with lower extremity weakness,
spasticity, neurogenic bladder, or bowel incontinence/
3. Common presenting symptoms include cutaneous constipation.
hemangioma, bowel incontinence/constipation, and • Many patients are diagnosed in the neonatal period
lower extremity spasticity. during evaluation o the cutaneous lesion.
4. Factors that need to be evaluated include location o • Treatment consists o resection o the lipomatous
the neural placode, presence o associated dysraphism, portion o the mass with release o the f lum and conus
presence o a cutaneous lesion, and documentation o the as needed.
conus position. A number o vertebral bodies are variable
and not included in the evaluation o congenital spinal
dysraphisms. Suggested Readings
5. Lipoma resection and cord untethering are per ormed Ru ener SL, Ibrahim M, Raybaud CA, Parmar HA.
with potential duraplasty to reduce the incidence o cord Congenital spine and spinal cord mal ormations—
retethering. Resection o the conus is contraindicated; pictorial review. AJR. 2010;194:S26-S37.
the goal is preservation o as much neural tissue as Sarris CE, Tomei KL, Carmel PW, Gandhi CD.
possible. Unless there is superimposed instability, there Lipomyelomeningocele: pathology, treatment and
is no indication or surgical usion. outcomes. Neurosurg Focus. 2012;33:E3.

32
Low back pain

1. What should be included in the di erential


diagnosis?

2. What are common presenting symptoms?

3. What associated abnormalities are seen?

4. What are the most common levels or this


abnormality?

5. What are the treatment options?

33
Disc fragment migration 2193
Case ranking/dif culty: Category: Disc

Axial T1 image demonstrates isointense soft tissue within the Sagittal T2 image demonstrates L3-L4 disc extrusion with superior
right lateral recess, consistent with L3-L4 disc extrusion. migration and sequestration.

Answers
1. Degenerative disc lesions include annular tear, disc Pearls
bulge, disc herniation, disc protrusion, and disc • Up to one in three patients has an asymptomatic disc
sequestration. herniation. Terminology is very important in the
discussion o degenerative disc disease.
2. Pain increased with sitting/bending/coughing,
• Disc bulge: >50% disc circum erence.
radiculopathy, and sphincter dys unction are all common
• Disc herniation: <50% disc circum erence.
presenting symptoms.
• Disc extrusion: Herniation where the base o the
3. Associated abnormalities include diskogenic protrusion is narrower than the herniated disc material.
endplate changes, enhancement o nerve roots, • Disc protrusion: Herniation where the base o the
epidural hematoma, and T2 hypointensity within the protrusion is wider than the herniated disc material.
intervertebral disc. • Disc sequestration: Free ragment o disc material.
• Treatment o disc disease is generally conservative—
4. L4-L5 and L5-S1 account or nearly 90% o all
only 10% o patients require surgical intervention
herniations.
secondary to persistent symptoms.
5. Conservative management is success ul in most patients
with surgery reserved or patients with recalcitrant
symptoms.
Suggested Readings
Chen CY, Chuang YL, Yao MS, Chiu WT, Chen CL, Chan WP.
Posterior epidural migration o a sequestered lumbar disk
ragment: MR imaging f ndings. AJNR. 2006;1592-1594.
Schellinger D, Manz HJ, Vidic B, et al. Disk ragment
migration. Radiology. 1990;175:831-836.

34
Neck pain

1. What should be included in the di erential


diagnosis?

2. What are common presenting symptoms?

3. What are the components o the classic triad?

4. What are associated abnormalities?

5. What are the treatment options?

35
Klippel-Feil syndrome 2190
Case ranking/dif culty: Category: Vertebral body

Sagittal CT image demonstrates fusion of the C2 and C3 vertebral Coronal CT image demonstrates fusion of the C2 and C3 vertebral
bodies. bodies.

Answers
• In addition, there are accelerated degenerative changes
1. Klippel-Feil syndrome, postoperative usion, ankylosing at the levels adjacent to the used vertebrae.
spondylitis, and postin ectious usion could all be • The classic triad has been described as limited cervical
considered. motion, short neck, and low hairline; however, this
2. Presenting symptoms include pain, alterations o cervical is present in less than hal o patients with vertebral
movement, myelopathy, radiculopathy, and neurological usions.
def cits in the setting o minor trauma. • Klippel Feil can be associated with Sprengel
de ormity, a rare congenital disorder mani ested by a
3. The components are low hairline, short neck, and high riding scapula, secondary to abnormal migration.
abnormal cervical motion. • I the patient has Sprengel de ormity, evaluation
4. Associated abnormalities include Sprengel de ormity, should be per ormed or the presence o an
cardiovascular abnormalities, hearing loss, and renal omovertebral bone (os omovertebrale), which is
abnormalities. present in up to 3/4 o patients, and extends rom
the superomedial scapula to the transverse process,
5. Generally activity restriction is the f rst step in spinous process, or lamina o C4 through C7.
management ollowed by bracing and traction as needed.
Surgical usion with decompression may be per ormed i
needed or neurological symptoms.
Suggested Readings
Pizzutillo PD, Woods M, Nicholson L, MacEwen GD. Risk
Pearls actors in Klippel-Feil syndrome. Spine. 1994;19:2110-2116.
Ulmer JL, Elster AD, Ginsberg LE, Williams DW. Klippel-
• Klippel Feil re ers to congenital usion o vertebral Feil syndrome: CT and MR o acquired and congenital
bodies. abnormalities o the cervical spine and cord. J Comp Asst
• Unto itsel it is harmless; however, there are alterations Tomog. 1993;17:215-224.
o biomechanics that place the patient at higher risk o
injury in the setting o trauma.

36
Low back and leg pain

1. What should be included in the di erential


diagnosis?

2. What are the presenting symptoms?

3. What imaging patterns have been described in


this abnormality?

4. What are the etiologies o this abnormality?

5. What are the treatment options?

37
Arachnoiditis 1501
Case ranking/dif culty: Category: Filum

Axial postmyelography CT image demonstrates “empty” Coronal postmyelography CT image demonstrates lateral
appearance of thecal sac with dorsal displacement of nerve roots. displacement and clumping of nerve roots. Note postoperative
Note postoperative changes of laminectomy and bilateral pedicle changes of laminectomy and bilateral pedicle screw
screw placement. placements.

Answers
1. Arachnoiditis can have nodular and linear enhancement Pearls
o nerve roots, simulating carcinomatous meningitis, or • Arachnoiditis is now most commonly seen in the
intradural extramedullary tumors (including nerve sheath postoperative patient population.
tumors and metastatic disease). Spinal stenosis can lead • It can be seen in conjunction with prior meningitis,
to ocal clumping o nerve roots at the level o stenosis. intrathecal chemotherapy, lumbar puncture, or
subarachnoid hemorrhage.
2. Pain and radicular symptoms are most common; others
• Arachnoiditis can demonstrate post-gadolinium
include bowel and bladder dys unction, paresis, and
enhancement in di erent patterns, which can make the
hypoesthesis.
correct diagnosis challenging.
3. Arachnoiditis can present with an empty thecal sac, • Patient history is critical in di erentiating causes o
secondary to peripheral adhesions o nerve roots, arachnoiditis.
distortion o the thecal sac, clumped nerve roots, and an • Arachnoiditis symptoms tend to be stable over time
intrathecal so t tissue mass. The intrathecal so t tissue without signif cant progression.
mass arises rom clumping and matting o nerve roots.
4. Potential causes or sterile arachnoiditis include surgery,
subarachnoid hemorrhage, intrathecal chemotherapy, Suggested Readings
myelography, and lumbar puncture; however, Van Goethem JWM, Parizel PM, Jinkins JR. Review article:
postmyelography arachnoiditis rates have dropped MRI o the postoperative lumbar spine. Neuroradiology.
ollowing the introduction o water-based contrast. 2002;44:723-729.
5. Treatment options include intrathecal steroid injection, Young PM, Berquist TH, Bancro t LW, Peterson JJ.
laminectomy and lysis o adhesions, and placement o Complications o spinal instrumentation. 2007;27:775-789.
a spinal cord stimulator. Prognosis is dismal despite
treatment. Over 50% o patients experience continued
pain.

38
Four-story fall onto wet grass

1. When does the anomaly leading to this occur?

2. Which radiograph is most help ul in


di erentiating this abnormality rom wedge
racture?

3. Where does this most commonly occur?

4. What is the etiology?

5. What are the treatment options?

39
Butter y vertebra 2189
Case ranking/dif culty: Category: Vertebral body

3. The thoracolumbar junction is the most common


location o butter y vertebra.
4. Butter y vertebrae arise secondary to persistence o the
notochord.
5. While some postulate that butter y vertebra can alter
spinal biomechanics and lead to advanced degenerative
changes, they are not unstable and do not generally
require any treatment.

Pearls
• Butter y vertebrae are unusual usion anomalies o the
vertebral bodies thought to be secondary to persistence
o the notochord.
• There are o ten additional osseous abnormalities
involving the ribs or spine.
• There is an association with genetic syndromes,
including Crouzon and Alagille syndromes.
• No treatment is necessary as this is not an unstable
f nding, but care should be taken not to con use it with
Coronal CT image demonstrates vertically oriented clefting of the an acute racture.
midportion of the T6 vertebral body.

Answers Suggested Readings


1. The two lateral ossif cation centers o the vertebral Garcia F, Florez MT, Conejero JA. A butter y vertebra or a
bodies usually use between the third and sixth weeks o wedge racture? Int Orthop. 1993;17:7-10.
gestation. Sonel B, Yalcin P, Ozturk E, Bokesoy I. Butter y vertebra:
a case report. Clin Imaging. 2001;25:206-208.
2. AP radiographs are most help ul or evaluation. A
butter y vertebra should demonstrate a lack o midline
usion; however, the two halves o the vertebral body
may be di erent sizes i there is an associated unilateral
arterial mal ormation.

40
Back pain and cold sweats

1. What are presenting symptoms?

2. What patient populations are most likely to


present with this abnormality?

3. What are the imaging f ndings within the


vertebral bodies?

4. What are the our types o this abnormality that


a ect humans?

5. What are the associated abnormalities?

41
Brucellosis 2181
Case ranking/dif culty: Category: More than one category

Axial T1 image following gadolinium administration demonstrates Sagittal T1 image following gadolinium administration demonstrates
right psoas and anterior epidural enhancing uid collections, abnormal enhancement within the L4 and L5 vertebral bodies
consistent with abscesses. and intervening disc, as well as the anterior epidural space and
prevertebral space.

Answers
1. The presentation is nonspecif c and includes ever,
malaise, night sweats, weight loss, and polyarthralgia.
Pearls
• Brucellosis is a relatively rare cause o
2. Brucellosis is a widespread zoonosis seen most spondylodiskitis; however, it can have an indolent
commonly in veterinarians and armers. course and mimic tubercular disease.
3. Classic f ndings o spondylitis include low T1 signal, • Brucellosis is most commonly seen in zoo keepers and
high T2 signal, and enhancement. armers.
• Diagnosis can be made by documenting positive
4. There are six species o Brucella but B abortus, Brucella serology, in either serum or cerebrospinal
B melitensis, B suis, and B canis are the our species uid.
that in ect humans. • Treatment generally consists o long-term
5. Associated abnormalities that can be seen in spinal combination antimicrobial therapy, including ri ampin,
brucellosis include paraspinal abscess, diskitis, epidural doxycycline, and trimoxazole.
abscess, nerve root enhancement, and vertebral body
destruction.
Suggested Readings
Namiduru M, Karaoglan I, Gursoy S, Bayazit N, Sirikei
A. Brucellosis o the spine: evaluation o the clinical,
laboratory, and radiological f ndings o 4 patients.
Rheumatol Int. 2004;24:125-129.
Tekkok IH, Berker M, Ozcan O, Ozgen T, Akalin E.
Brucellosis o the spine. Neurosurgery. 1993;33:838-844.

42
Systemic disease, evaluate for spine involvement

1. What should be included in the di erential


diagnosis?

2. What are common presenting symptoms?

3. What are associated abnormalities?

4. What portion o the spine is most commonly


a ected?

5. What are the treatment options?

43
Vertebra plana 2178
Case ranking/dif culty: Category: Vertebral body

Sagittal T1 image demonstrates loss of vertebral body height and Sagittal T2 image demonstrates loss of vertebral body height and
hypointensity of the C4 vertebral body. hypointensity of the C4 vertebral body.

Answers
• EG is limited to osseous lesions.
1. Vertebral body collapse can be seen in compression
• Hand-Schüller-Christian disease is an intermediate
racture, metastasis, osteomyelitis, and primary bone
orm o LCH with osseous and visceral lesions.
tumor.
• Letterer-Siwe disease is the most severe orm with
2. Potential presenting symptoms include ever, pain, extensive visceral involvement.
pathologic racture, and radiculopathy. Additionally, • EG generally presents be ore adolescence with
myelopathy can be seen i the spinal canal is multiple lytic lesions, which can be complicated by
compromised. collapse or pathologic racture leading to pain.
• The spine is one o the less common sites o osseous
3. Potential associated abnormalities in Langerhans
involvement, but when involved it o ten mani ests as
cell histiocytosis include cutaneous lesions,
vertebra plana.
lymphadenopathy, gastrointestinal bleeding, proptosis,
• EG is generally treated conservatively, and there are
and diabetes insipidus. Diabetes insipidus arises rom
even reports o spontaneous resolution o vertebra
involvement o the pituitary in undibulum.
plana.
4. Hal o cases occur in the thoracic spine with a third • More severe variants o LCH are treated with
within the lumbar spine. multiagent chemotherapy with radiation and stem cell
transplant reserved or recalcitrant lesions.
5. Potential treatment modalities depend on the severity
o disease and secondary complications and include
conservative management, chemotherapy, radiation,
surgical resection, and stem cell transplant. Suggested Readings
Denaro L, Longo UG, Papalia R, DiMartino A, Ma ulli
N, Denaro V. Eosinophilic granuloma o the pediatric
Pearls cervical spine. Spine. 2008;33:E936-E941.
Ropper AE, Cahill KS, Hanna JW, McCarthy EF, Gokaslan
• EG represents almost three-quarters o Langerhans
ZL, Chi JH. Primary vertberal tumors: A review o
cell histiocytosis (LCH) cases and is the most benign
epidemiologic, histological, and imaging f ndings, part I:
subtype.
benign tumors. Neurosurgery. 2011;69:1171-1180.

44
Back pain in patient with chronic steroid use

1. What should be included in the di erential


diagnosis?

2. What are common presenting symptoms?

3. What imaging f ndings can be help ul in


di erentiating epidural processes?

4. What part o the spine is most commonly


involved?

5. What are the treatment options?

45
Thoracic epidural lipomatosis 2176
Case ranking/dif culty: Category: Spinal canal

Sagittal T1 image demonstrates dorsal T1 hyperintensity with Sagittal T2 image with fat suppression demonstrates dorsal
anterior displacement of the dura. hypointensity with anterior displacement of the dura.

Answers
1. Processes o the epidural space, which may show Pearls
varying amounts o T1 hyperintensity, include • Epidural lipomatosis is becoming more common
lymphoma, metastasis, hemorrhage, abscess, and as the population becomes more obese; the most
lipomatosis. common cause remains exogenous steroid use.
• Epidural lipomatosis should be di erentiated rom
2. Usually this abnormality is asymptomatic; however,
other causes o T1 hyperintensity within the epidural
presenting symptoms include back pain, sensory def cits,
space.
and myelopathic symptoms.
• Fat suppressed images can be help ul in evaluation.
3. Fat will maintain hyperintensity on T1 and T2 • This is o ten an asymptomatic process; however, some
sequences, but should demonstrate decreased signal with patients develop radicular or myelopathic symptoms.
the application o at suppression. • In these cases, surgical decompression should be
considered i tapering o steroids or weight loss is not
4. The thoracic spine is most commonly involved.
possible.
5. As most patients are asymptomatic or have symptoms
only vaguely re erable to the epidural lipomatosis,
conservative management is an appropriate choice.
Suggested Readings
Noninvasive options such as weight loss and tapering
o steroids are also good choices with decompression Fassett DR, Schmidt MH. Spinal epidural lipomatosis: a
reserved or re ractory cases. review o its causes and recommendations or treatment.
Neurosurg Focus. 2004;16:1-3.
Quint DJ, Boulos RS, Sanders WP, Mehta BA, Patel SC, Tiel
RL. Epidural lipomatosis. Radiology. 1988;169:485-490.

46
Left leg numbness

1. What should be included in the di erential


diagnosis?

2. What are important items to review prior to


per orming myelography?

3. What are the keys to recognizing extrathecal


contrast injection?

4. What are the optimal lumbar levels to gain


access or myelography?

5. What are the treatment options?

47
Mixed injection on myelogram 1525
Case ranking/dif culty: Category: Spinal canal

Sagittal CT image following myelogram demonstrates contrast Axial CT image following myelogram demonstrates contrast
within the subdural, epidural, and intrathecal spaces. within the epidural (ventral right arrow), subdural (middle left
arrow), and intrathecal spaces (dorsal arrow).

Answers
1. Extradural hyperdensity seen on CT myelography
could be caused by subdural or epidural hematoma,
Pearls
epidural metastasis, or posterior longitudinal ligament • Mixed injections on myelography should be
ossif cation. recognized at the time o the procedure.
• I the contrast ails to distribute around the nerve roots
2. Important items to review include coagulation laboratory o the f lum, this is indicative o subdural injection;
values, current medications, allergies, and prior imaging however, it can sometimes be di f cult to discern.
studies. • A mixed injection is less troublesome than a
3. The best way to delineate early extrathecal contrast completely extrathecal injection.
injection is to evaluate or ocal pooling o contrast at the • CT ollowing myelography is routinely done, which
site o injection. can delineate the compartments that contain contrast.
4. L2-L3 and L3-L4 are the most optimal levels as they
should be below the level o the conus, yet in an area
where the thecal sac is still airly large. Suggested Readings
5. Conservative management is indicated. Epidural blood Harreld JH, McMenamy JM, Toomay SM, Chason DP.
patches are per ormed or cerebrospinal uid leaks. Myelography: a primer. Curr Probl Diagn Radiol.
2011;4:149-157.
Ozdoba C, Gralla J, Rieke A, Binggeli R, Schroth G.
Myelography in the age o MRI: why we do it, and how
we do it. Radiol Res Pract. 2011;329017:6.

48
Fall

1. What should be included in the di erential


diagnosis?

2. What is the classif cation o ractures in this


disorder?

3. What underlying disorders are associated with


this abnormality?

4. What disorders can lead to axial skeletal


osseous overproduction?

5. What are the treatment options?

49
Di use idiopathic skeletal hyperostosis with fracture 1511
Case ranking/dif culty: Category: More than one category

4. Seronegative spondyloarthritis, acromegaly, ochronosis,


uorosis, and trauma are additional causes o osseous
overproduction in the spine.
5. Patients with ractures superimposed on DISH generally
benef t rom multilevel usion given the inherently
altered biomechanics.

Pearls
• Di use idiopathic skeletal hyperostosis (DISH) is a
degenerative disorder characterized by ligamentous
and tendinous ossif cation.
• It is diagnosed when owing osteophytes bridge our
or more adjacent vertebral bodies.
• It also a ects the sacroiliac joints and has extraspinal
mani estations.
• It a ects men twice as o ten as women and usually
older patients (>40 years old).
Sagittal image demonstrates owing anterior osteophytes with
lucency extending into the C6 vertebral body. Note associated • The etiology is uncertain but there is a amilial pattern.
ossi cation of the posterior longitudinal ligament. • The abnormal ossif cation leads to altered
biomechanics and increased risk o racture at levels
o adjacent osseous usion.
Answers • Care ul evaluation o spinal imaging is imperative
1. The di erential includes ankylosing spondylitis, in patients with DISH as they may su er an atypical
psoriatic arthritis, and di use idiopathic skeletal racture, even in the setting o only minor trauma.
hyperostosis. • In addition, patients with DISH o ten have increased
morbidity and mortality in the setting o superimposed
2. Fractures are classif ed into our types: ractures in comparison to patients without an
Type 1—disc injury ankylosing spinal disorder.
Type 2—body injury
Type 3—anterior body or posterior disc injury
Type 4—anterior disc or posterior body injury Suggested Readings
Type 1 ractures are most common in DISH. Cammisa M, DeSerio A, Guglielmi G. Di use idiopathic
skeletal hyperostosis. Eur J Radiol. 1998;27:S7-S11.
3. Type 2 diabetes, cholelithiasis, hypertension, and Caron T, Brans ord R, Nguyen Q, Agel J, Chapman J,
hypercholesterolemia are all associated with DISH. Bellabarba C. Spine ractures in patients with ankylosing
spinal disorders. Spine. 2010;35:E458-E464.

50
Back pain

1. What should be included in the di erential


diagnosis?

2. Which spinal compartment is most commonly


a ected?

3. What portion o the spine is most commonly


a ected?

4. What signal characteristics would avor nerve


sheath tumor over neurof broma?

5. What are the treatment options?

51
Cystic schwannoma 1510
Case ranking/dif culty: Category: Meninges/Nerve sheath

Sagittal T2 image demonstrates a uid Sagittal T1 image following gadolinium Axial T2 image demonstrates a uid
intensity mass in the ventral spinal canal administration demonstrates a peripherally intensity mass in the ventral spinal canal
with associated cord displacement and enhancing mass in the ventral spinal canal with associated cord displacement and
compression. with associated cord displacement and compression.
compression.

Answers
• 70% o schwannomas occur in the intradural
1. Epidermoid and schwannoma are the most common
extramedullary compartment. The remainder are
intradural extramedullary lesions to demonstrate cystic
divided equally between the extradural and combined
signal characteristics. Meningioma and neurof broma
intradural–extradural compartment.
also occur within the intradural extramedullary
• Imaging characteristics that avor schwannoma include
compartment and should be considered within the
cystic degeneration, hemorrhage, and ocal lobulation.
di erential.
• Neurof bromas tend to be more usi orm in
2. 70% are intradural extramedullary in location with appearance.
the remainder equally divided between extradural and • Meningiomas do not usually demonstrate cystic
combined intradural and extradural locations. change and are more likely to have calcif cations and
an enhancing dural tail.
3. Thoracic spine is most common with equal prevalence in
• Schwannomas can be multiple, particularly
the cervical and lumbar spine.
in association with neurof bromatosis type 2,
4. Cystic degeneration, hemorrhage, and lobulated schwannomatosis, and Carney complex.
appearance are common imaging f ndings in • Patients generally present with ocal pain; however,
schwannoma. Neurof broma o ten demonstrates more myelopathic symptoms may occur in the setting o
usi orm enlargement. Calcif cation is more commonly cord compression.
seen in meningioma. • Treatment requires complete surgical resection, o ten
associated with sacrif ce o the associated nerve.
5. Surgical resection is the treatment o choice or
symptomatic lesions or pathologic diagnosis. This may
necessitate sacrif ce o the associated nerve.
Suggested Readings
Friedman DP, Tartaglino LM, Flanders AE. Intradural
Pearls schwannomas o the spine: MR f ndings with emphasis
on contrast-enhancement characteristics. AJR. 1992;158:
• Schwannomas are the most common intradural 1347-1350.
extramedullary mass in the spine. Parmar H, Patkar D, Gadani S, Shah J. Cystic lumbar nerve
sheath tumours: MRI eatures in f ve patients. Australas
Radiol. 2001;45:123-127.
52
Fall

1. In which parts o the spine does this disorder


occur?

2. What are synonyms or this abnormality?

3. What are inciting actors in this abnormality?

4. What are classic imaging eatures?

5. What are the treatment options?

53
Calcium pyrophosphate deposition 1508
Case ranking/dif culty: Category: Vertebral body

enhancement on MRI are imaging eatures o calcium


pyrophosphate deposition; however, the appearance is
variable depending on where the crystals are deposited.
5. I patients are symptomatic, treatment is geared at the
inciting cause. Surgical resection and usion can be
considered depending on severity o symptomatology
and site o involvement.

Pearls
• The incidence o calcium pyrophosphate deposition
(CPPD) increases with age and may occur
in association with other conditions (trauma,
surgery, hypophosphatasia, hyperparathyroidism,
hypomagnesemia).
• CPPD can also be seen in association with gout and
hydroxyapatite deposition.
• Involvement o the dens may become symptomatic,
either through compression at the oramen magnum
Sagittal CT image demonstrates calcium deposition posterior to secondary to deposition or instability secondary to
the dens with early erosive changes. dens erosion.
• On microscopic inspection, calcium pyrophosphate
crystals are rhomboidal in shape and demonstrate
Answers weak positive bire ringence under polarized light.
1. Deposition occurs in the intervertebral disc, ligamentum
avum, transverse ligament, and apophyseal joints with
di erent symptomatology depending on amount. Suggested Readings
2. CPPD is also called pseudogout or articular Bouvet JP, Le Parc JM, Michalski B, Benlahrache C, Auquier
chondrocalcinosis. L. Acute neck pain due to calcif cations surrounding the
odontoid process: the crowded dens syndrome. Arthritis.
3. Mechanical trauma and in ammation are elt to be
1985;28:1417-1420.
inciting actors in deposition o calcium pyrophosphate.
Salcman M, Khan A, Symonds D. Calcium pyrophosphate
4. So t tissue and intervertebral disc calcif cation, osseous arthropathy o the spine: case report and review o the
erosion, heterogeneous T2 signal, and peripheral literature. Neurosurgery. 1994;34:915-918.

54
Back pain with right L4 radiculopathy. MRI was contraindicated

1. What is this procedure called?

2. When is this procedure indicated?

3. Name the two common approaches used.

4. Name some recognized complications o this


procedure.

5. Describe the typical postprocedure headache.

55
Myelography 3216
Case ranking/dif culty: Category: Thecal sac

AP lm of the lumbar spine following the injection of 12 mLof Lateral projection con rms narrowing of L4-5 intervertebral disc
Iohexol (300 mgL/mL) in the lumbar subarachnoid space via a 22 space and a right posterolateral disc protrusion which indents the
G spinal needle. Myelography demonstrates disc space narrowing thecal sac.
(arrowhead) at L4-5 level associated with amputation of the right L4
nerve root sheath (arrow).
More serious complications include in ection,
hematoma, seizures, contrast reactions, and CSF
Answers leaks. The latter can be treated by a blood patch. Less
1. Myelography is a radiographic spinal examination that commonly the spinal cord may be damaged directly
involves injection o a contrast agent into the thecal sac particularly i the conus is low lying or i a cervical
prior to the acquisition o radiographs using standard approach is being used.
projections. Alternatively, a CT scan may be per ormed. 5. The typical postprocedure headache develops within the
2. The technique has been largely superseded by MRI; f rst ew hours and characteristically increases in severity
however, it is still a use ul technique whenever MRI in the upright position and improves in the recumbent
is contraindicated or when MR images are limited by position.
arti acts rom previous spinal f xation.
3. Two approaches, namely the mid-sagittal and the Pearls
parasagittal approaches, are commonly used. The
advantage o the ormer is that it is less pain ul as the • Myelography has been largely superseded by MRI but
ligament is poorly innervated compared to paraspinal is still use ul whenever MRI is contraindicated.
muscles. In the parasagittal approach, the needle is • Complications include in ection, hematoma, seizures,
advanced into the interlaminar space, and there ore contrast reactions, and CSF leaks.
it will not need to be orced through the interspinous • Two approaches, namely the mid-sagittal and the
ligament (which may be severely calcif ed) or negotiated parasagittal approaches, are commonly used.
through narrowed interspinous spaces. • Only nonionic contrast can be injected into the thecal sac.

4. Myelography is generally a sa e technique. Some


patients develop a headache, vertigo, and vomiting
Suggested Readings
postprocedure secondary to CSF loss. This may be
reduced with the use o a smaller needle and by orienting Bischo RJ, Rodriguez RP, Gupta K, Righi A, Dalton
the bevel o the spinal needle parallel to the longitudinal JE, Whitecloud TS. A comparison o computed
f bers o the thecal sac. The standard spinal needles are tomography-myelography, magnetic resonance imaging,
22- and 25-gauge. The latter is enough or intrathecal and myelography in the diagnosis o herniated nucleus
contrast injection but is insu f cient to allow CSF pulposus and spinal stenosis. J Spinal Disord. 1993
collection. Aug;6(4):289-295.
http://www.ir.ustl.du/neurorad/internal.sp?NavID=72.
Accessed 22nd June 2014.

56
Myelopathic symptoms in a patient with a known syndrome

1. What should be included in the di erential


diagnosis?

2. What are common presenting symptoms?

3. What portion o the cord is most commonly


a ected?

4. What are the subtypes o this abnormality?

5. What are the treatment options?

57
Cervical ependymoma 1504
Case ranking/dif culty: Category: Spinal cord

Sagittal T1 image following gadolinium administration demonstrates Sagittal T2 image demonstrates multiple T2 hyperintense expansile
multiple enhancing foci in the cervical cord; the inferior lesion foci within the cervical cord; the inferior lesion demonstrates a uid
demonstrates a nonenhancing cystic portion. intensity portion.

Answers
1. Any intramedullary lesion should be considered, Pearls
including ependymoma, astrocytoma, • Ependymomas are the most common intramedullary
hemangioblastoma, metastasis, and oligodendroglioma; spinal cord tumors in adults, accounting or
ependymoma is the most common intramedullary tumor approximately 60% o all intramedullary spinal cord
in the adult population. tumors.
• While there is an association with neurof bromatosis
2. Pain is the most common presenting symptoms; others
type 2 (NF-2), the tumors seen in conjunction with
include weakness, radicular symptoms, and sensory
NF-2 are not histologically di erent rom those that
def cits.
occur spontaneously.
3. The cervical cord is the most common location o • Diagnosis is o ten delayed secondary to nonspecif c
cellular ependymomas, ollowed by thoracic cord and presenting symptoms o pain and/or myelopathy.
conus. • These tumors are usually WHO grade II.
• Surgical resection is the treatment o choice with
4. The subtypes o ependymoma are cellular,
radiation therapy reserved or subtotal resection or
myxopapillary, clear cell, and tanycytic; the cellular
recurrent disease.
subtype is the most common intramedullary subtype.
• There is no role or adjuvant chemotherapy.
5. Most patients are cured with surgical resection; radiation
therapy can be used in patients with recurrent disease or
subtotal resection.
Suggested Readings
Bostrom A, von Lehe M, Hartmann W, et al. Surgery or
spinal cord ependymomas: outcome and prognostic
actors. Neurosurgery. 2011;68:302-309.
Whitaker SJ, Bessell EM, Ashley SE, Bloom HJG, Bell BA,
Brada M. Postoperative radiotherapy in the management o
spinal cord ependymoma. J Neurosurg. 1991;74:720-728.

58
Spinal cord injury s/p Harrington rod placement with 3 days of discomfort

1. What are the most common causes o this


abnormality?

2. What are potential etiologies?

3. What is the best modality to evaluate this


abnormality?

4. What are the expected MRI f ndings?

5. What are the treatment options?

59
Calci c arachnoiditis (arachnoiditis ossi cans) 1502
Case ranking/dif culty: Category: Filum

Axial CT image demonstrates calci cation of the thecal sac. Sagittal CT image demonstrates postoperative changes with
calci cation of the thecal sac.

Answers
1. Traditionally, spinal meningitis was the most common
Pearls
cause; however, in the modern era, spinal surgery
is the most common cause. Other causes include • Historically arachnoiditis arose secondary to spinal
lumbar puncture, subarachnoid hemorrhage, and prior in ection; however, it is now more commonly seen in
myelogram. the postoperative patient.
• It mani ests with clumping and thickening o the nerve
2. The predominant theory is that chronic in ammation roots.
leads to osseous metaplasia o the exuberant f brotic • It has been classically described as an “empty thecal
changes. Other theories include organization o sac.”
intradural blood products with ossif cation and seeding • Calcif c arachnoiditis has a worse prognosis than
o bone ragments. the noncalcif ed variant with a tendency toward
3. Noncontrast CT is the study o choice. progressive neurological decline.

4. Findings are similar to noncalcif ed arachnoiditis,


including clumping o the nerve roots, with
superimposed linear or nodular T1 hyperintensity and T2 Suggested Readings
hypointensity. However, signal abnormality is variable Chan CC, Lau PY, Sun LK, Lo SS. Arachnoiditis ossif cans.
depending on the hydroxylation status o the calcium. Hong Kong Med J. 2009;15:146-148.
5. Treatment options include analgesia and spinal Frizzell B, Kaplan P, Dussault R, Sevick R. Arachnoiditis
stimulation, as is per ormed or noncalcif ed ossif cans: MR imaging eatures in f ve patients. AJR.
arachnoiditis. Additionally, some patients benef t 2001;177:461-464.
rom decompressive laminectomy; however, this must
encompass the entirety o the abnormality, which is
important to document in a radiology report.

60
Back pain

1. What should be included in the di erential


diagnosis?

2. What are common presenting symptoms?

3. What are potential causative actors?

4. What part o the spine is most commonly


involved?

5. What are the treatment options?

61
Lumbosacral epidural lipomatosis 1500
Case ranking/dif culty: Category: Spinal canal

Axial T1 image demonstrates classic Y-shaped con guration of Sagittal T1 image demonstrates focal anterior epidural T1
the thecal sac, resulting from limitations of the epidural space by hyperintensity, consistent with fat, with posterior displacement
the meningovertebral ligaments. of the posterior longitudinal ligament. Note also the posterior
epidural fat most prevalent below the L4 level.

Answers
1. Processes that involve the epidural space and can
have T1 hyperintensity include abscess, hematoma,
Pearls
metastasis, and lipomatosis. • Epidural lipomatosis is becoming more common as
the population becomes more obese.
2. Back pain is the most commonly reported symptom; • The most common reason to develop epidural
others include myelopathy, radiculopathy, sensory lipomatosis is exogenous steroid use.
def cit, and incontinence. • This is o ten an asymptomatic process; however, some
3. The most common cause is exogenous steroid use; patients develop radicular or myelopathic symptoms.
however, endogenous overproduction is a cause. As the In these cases, surgical decompression should be
population becomes heavier, obesity is increasing in considered and tapering o steroids i possible.
requency as a causative actor. • Epidural lipomatosis is an important entity to
recognize and di erentiate rom other more serious
4. The thoracic spine is most commonly involved, ollowed epidural processes, such as abscess, hematoma, or
by the lumbar spine. metastatic disease.
5. Weight loss and tapering o exogenous steroids, i • Fat suppressed images can be help ul in the diagnosis.
possible, are noninvasive treatment options. I symptoms
are re ractory, surgical decompression should be
considered.
Suggested Readings
Fassett DR, Schmidt MH. Spinal epidural lipomatosis: a
review o its causes and recommendations or treatment.
Neurosurg Focus. 2004;16:1-3.
Quint DJ, Boulos RS, Sanders WP, Mehta BA, Patel SC, Tiel
RL. Epidural lipomatosis. Radiology. 1988;169:485-490.

62
Leg pain

1. What classif cation scheme is used to grade


this abnormality?

2. What are risk actors?

3. What are common presenting symptoms?

4. This abnormality most commonly occurs at


what spinal level?

5. What are the treatment options?

63
Spondyloptosis 1484
Case ranking/dif culty: Category: Posterior elements

Axial CT image demonstrates Sagittal CT image Sagittal T2 image demonstrates Sagittal PD image demonstrates
bilateral L5 spondylolysis. demonstrates Grade 4-5 Grade 4-5 anterolisthesis of L5 Grade 4-5 anterolisthesis of L5
anterolisthesis of L5 on S1. on S1 with uncovering of the on S1 with uncovering of the
disc and severe spinal canal disc and severe spinal canal
narrowing. narrowing.

Axial T2 image demonstrates bilateral L5 spondylolyses.

Answers Axial PD image demonstrates bilateral L5 spondylolyses.

1. Grade I: <25% o vertebral body displacement


Grade II: 25%-50% o vertebral body displacement Pearls
Grade III: 50%-75% o vertebral body displacement
• Spondyloptosis results when there is complete
Grade IV: 75%-100% o vertebral body displacement slippage o one vertebral body relative to the adjacent
Grade V: 100% vertebral body displacement vertebral body (Grade V spondylolisthesis).
Grade V spondylolisthesis is also re erred to as • This is generally due to high-velocity trauma, but can
spondyloptosis. occur rom progressive spondylolisthesis secondary to
spondylolysis.
2. Multiple actors can contribute to spondylolysis, • Symptoms usually result rom severe spinal canal,
including repetitive microtrauma, spina bif da occulta, lateral recess, and neural oraminal compromise.
abnormal sacral anatomy, and positive amily history. • Treatment consists o in situ f xation or reduction and
3. Back pain is the most common presenting symptom; f xation.
others include ocal kyphosis, myelopathy, and
radiculopathy.
Suggested Readings
4. Spondyloptosis is most common in the lumbar spine.
Curylo LJ, Edwards C, DeWald RW. Radiographic markers
5. Depending on patient symptomatology and risk or in spondyloptosis: implications or spondylolisthesis
progression, treatment options can include conservative progression. Spine. 2002;27:2021-2025.
management, in situ surgical f xation, and surgical Rodriguez DP, Poussaint TY. Imaging o back pain in
reduction and f xation. children. AJNR. 2010;31:787-802.
64
Painful kyphosis

1. What should be included in the di erential


diagnosis?

2. What are common presenting symptoms?

3. Which location within the spine is most


commonly a ected?

4. What are the imaging f ndings?

5. What are the treatment options?

65
Scheuermann disease 1482
Case ranking/dif culty: Category: Vertebral body

3. Thoracic spine is most commonly a ected with the


kyphosis apex centered between T7 and T9.
4. Scheuermann disease is related to abnormalities o
the epiphyseal growth plates and mani ests as anterior
wedge compression de ormity, apophyseal ring racture,
endplate irregularity, and Schmorl nodes.
5. Treatment options include analgesia, bracing, physical
therapy, and surgical usion. Surgical treatment is
indicated in cases o severe kyphosis.

Pearls
• Scheuermann disease is def ned as pain ul kyphosis
in association with three or more wedge compression
de ormities with endplate irregularity.
• It is most common in adolescents, in conjunction with
times o rapid growth.
• It is most commonly seen in the thoracic spine.
Lateral radiograph demonstrates multiple rounded defects in the • Treatment is generally nonoperative bracing; however,
anterior vertebral bodies at the thoracolumbar junction. surgical usion may be indicated or patients with
severe kyphosis.

Answers
1. A combination o kyphosis and endplate irregularity can Suggested Readings
be seen in compression racture, in ection, osteogenesis
Jagtap SA, Manuel D, Kesavdas C, et al. Scheuermann
imper ecta tarda, and spondyloepiphyseal dysplasia
disease presenting as compressive myelopathy. Neurology.
tarda.
2012;78:1279.
2. Pain and kyphosis are the most common presentations; Rodriguez DP, Poussaint TY. Imaging o back pain in
others include scoliosis and myelopathy. children. AJNR. 2010;31:787-802.

66
Breast cancer

1. What should be included in the di erential


diagnosis o a vertebral body lesion in an
adult?

2. What are common presenting symptoms?

3. What are the most common cancers to


metastasize to bone?

4. What portions o the spinal column are most


commonly a ected?

5. What are the treatment options?

67
C1 osseous metastasis 1477
Case ranking/dif culty: Category: Vertebral body

T1 hypointense lesion right lateral T2 hyperintense mass right lateral mass C1. Enhancing mass right lateral mass C1.
mass C1.

Answers
1. Vertebral body lesions include hemangioma, multiple Pearls
myeloma, metastasis, and in ection. The diagnosis • 20% o patients with a primary malignancy will
may be urther complicated by the presence o a develop symptomatic osseous metastatic lesions.
superimposed compression racture. • Up to hal o patients will have skeletal metastases
diagnosed at autopsy.
2. Although nonspecif c, one o the most common
• Treatment options or osseous metastatic lesions
presentations o osseous metastatic disease is night pain,
(in addition to treating the primary cancer) include
o ten severe enough to awaken the patient rom sleep.
radiation, stereotactic radiosurgery, or corpectomy/
3. The top f ve cancers metastatic to bone are prostate, resection with usion.
breast, lung, kidney, and thyroid. • I osseous metastatic lesions are identif ed, it is critical
to evaluate or extraosseous extension, particularly
4. Osseous metastatic lesions o ten involve the anterior and
into the epidural space.
middle columns, while sparing the posterior elements.
5. A multimodality treatment approach is usually
employed, including analgesia, chemotherapy, radiation/
Suggested Reading
radiosurgery, and resection.
Hage WD, Aboulaf a AJ, Aboulaf a DM. Incidence, location
and diagnostic evaluation o metastatic bone disease.
Orthop Clin North Am. 2000;31:515-528.

68
Back pain with right L4 radiculopathy

1. What is the most likely diagnosis?

2. What is meant by the term protrusion?

3. What is meant by a “ ocal” herniation?

4. What does the term herniation imply?

5. Which is the most common location o disc


herniation?

69
Disc herniation—protrusion 3051
Case ranking/dif culty: Category: Disc

Sagittal T2-weighted sequence of the Sagittal T1-weighted sequence of the Axial T2-weighted image at the level
lumbar spine. There is a right foraminal lumbar spine showing a disc protrusion of L4-L5 intervertebral disc showing a
disc protrusion (arrow) at L4-L5. The (arrow) at L4-L5. The vertebral body right foraminal disc protrusion (arrow).
right L4 exiting nerve root is seen heights and marrow signal are within Note that the dome is smaller than the
just caudal to the protrusion and the normal limits. base of the herniation.
surrounding fat is e aced.

Answers 3) Foraminal (Subarticular): Foraminal and


1. Disc “protrusion.” In 1995 a multidisciplinary task orce extra oraminal herniations account or less than 10%
rom the North American Spine Society recommended o disc herniations. Herniation o disc material into
the use o a standardized nomenclature. the intervertebral oramen is o ten symptomatic due
to compression o dorsal root ganglia.
2. I the distance between the edges o the base o the disc 4) Extra oraminal (Lateral): Disc herniations in this
herniation is larger than that between the edges o the region are uncommon.
herniated material, then it is termed a protrusion (ie, the
base is wider than its dome). The herniated disc material
beyond the disc space remains continuous with disc Pearls
within the normal disc space.
• Herniation re ers to the localized displacement o disc
3. Protrusions are urther classif ed as “ ocal” or “broad material beyond the limits o the intervertebral disc space.
based” depending on whether the base is less than 25% or • I the distance between the edges o the base o the disc
between 25% and 50% o the circum erence o the disc, herniation is larger than that between the edges o the
respectively. Circum erential extension (50%-100%) o herniated material, then it is termed a protrusion (ie, the
disc material beyond the vertebral ring apophysis is not base is wider than its dome).
considered a orm o herniation and is termed disc bulging. • Protrusions are urther classif ed as “ ocal” or “broad
4. Herniation re ers to the localized displacement o based.”
disc material (which includes the nucleus pulposus,
annular tissue, ragmented apophyseal bone, cartilage,
or any combination thereo ) beyond the limits o the Suggested Readings
intervertebral disc space. Fardon DF, Milette PC; Combined Task Forces o the
North American Spine Society, American Society o
5. The axial plane is used to localize herniated discs and
Spine Radiology, American Society o Neuroradiology.
the ollowing terminology is used:
Nomenclature and classif cation o lumbar disc pathology.
1) Central (Medial): Disc herniations are o ten Recommendations o the Combined task Forces o the
slightly eccentric to the le t or right as the posterior North American Spine Society, American Society o Spine
longitudinal ligament (PLL) is thickest centrally. Radiology, and American Society o Neuroradiology.
2) Paramedian (Lateral recess): This is the most Spine (Phila Pa 1976). 2001 Mar 1;26(5):E93-E113.
common location or disc herniations as the PLL is RF Costello, DP Beall. Nomenclature and standard reporting
thinner in this region. terminology o intervertebral disc herniation. Magn Reson
Imaging Clin N Am. 2007;15:167-174.
70
Valsalva-induced headaches

1. What are common presenting symptoms?

2. What are associated f ndings?

3. Where does syrinx ormation most o ten


occur?

4. What are the theoretical causes o this


abnormality?

5. What are the treatment options?

71
Chiari I malformation with syrinx 1475
Case ranking/dif culty: Category: Spinal cord

Sagittal T1 image demonstrating inferior Sagittal T2 image demonstrates Sagittal T2 image demonstrates
displacement and elongation of cerebellar tissue multiloculated T2 hyperintense uid caudal extension of syrinx into
with crowding at the foramen magnum and T1 collection within the cervical cord. the thoracic cord.
hypointense uid collection within the cervical cord.

Answers
1. Symptoms include cough-induced headache, ocal Pearls
sensory loss, lower extremity spasticity, and back pain. • While only 50% o patients with Chiari I
mal ormation have an associated syrinx, up to 90% o
2. Findings in Chiari I mal ormation include short clivus,
symptomatic patients present with syrinx.
hydrocephalus, scoliosis, and platybasia.
• Treatment o Chiari I mal ormations is primarily
3. While syrinx may occur at any level, it is most common directed at relieving obstruction and improving ow
at C4-C6. dynamics at the oramen magnum.
• This is generally accomplished using a suboccipital
4. The hydrodynamic/water hammer theory postulates that
craniectomy, and may be augmented by resection o
the low-lying cerebellar tonsils result in pressure orcing
the posterior arch o C1.
cerebrospinal uid into the central canal. Another
• O tentimes, this will lead to decrease in size o the
leading theory is the cranial-spinal pressure dissociation
associated syrinx; however, in cases in which the
theory, which postulates that sudden changes in venous
syrinx is persistent with associated myelopathic
pressure orce cerebrospinal uid into the central canal.
symptoms, syringosubarachnoid shunts can be placed.
5. Treatment is primarily directed at relieving the
obstruction to ow at the oramen magnum with
suboccipital craniectomy and posterior C1 resection;
Suggested Readings
however, shunt placement may be needed in re ractory
cases, which may be placed within the ventricles or the Oldf eld EH, Muraszko KM, Shawker TH, Patronas NJ.
syrinx itsel . Pathophysiology o syringomyelia associated with Chiari
I mal ormation o the cerebellar tonsils. J Neurosurg.
1994;80:3-15.
Strahle J, Muraszko KM, Kapurch J, Bapuraj JR, Garton
HJL, Maher CO. Chiari I mal ormation Type 1 and syrinx
in children undergoing magnetic resonance imaging.
J Neurosurg Pediatrics. 2011;8:205-213.

72
Chronic neck pain secondary to disc degeneration

1. What is this procedure called?

2. Name some indications or this procedure.

3. What are the disadvantages o spinal usion?

4. Name some contraindications to this


procedure.

5. Describe ADR device characteristics and


surgical implications.

73
Arti cial disc replacement (ADR) 3101
Case ranking/dif culty: Category: Disc

AP radiograph of the cervical spine. There Lateral radiograph of the cervical Oblique radiograph of the cervical
is a prosthetic intervertebral disc between spine demonstrates an arti cial spine shows a prosthetic disc in
C6 and C7. The arti cial disc comprises two disc replacement (arrow) at C6-C7. situ. The adjacent intervertebral
metallic endplates (arrowheads) that are foramina are widely patent.
separated by a more pliable core (arrow).

Answers preserved during surgery, which helps maintain the


1. Artif cial disc replacement is a surgical procedure in stability o the spinal column. Single discs or several
which artif cial mechanical devices are inserted to disc levels may be replaced during the same surgery.
replace degenerated intervertebral discs in the cervical Artif cial disc replacement only gained FDA approval or
and lumbar spine. use in the United States in 2004.

2. It aims to improve or eliminate chronic, severe neck and


back pain secondary to degenerative disc disease. It is
Pearls
an alternative to spinal usion, which allows continued
motion between the a ected vertebral bodies and • Artif cial disc replacement aims to improve or
there ore prevents premature breakdown in adjacent eliminate chronic, severe neck and back pain
spinal levels. secondary to degenerative disc disease.
• The artif cial disc comprises two metallic endplates
3. Disadvantages o spinal usion include loss o motion that are separated by a more pliable core.
and exibility, which permanently alter the motion • Contraindications include central or lateral
characteristics and biomechanics o the entire spinal recess stenosis, herniated nucleus pulposus with
column. It has been shown that spinal usion causes radiculopathy, osteoporosis, scoliosis, acet arthrosis,
accelerated degeneration o the intervertebral discs spondylolysis or spondylolisthesis, and def ciency o
above and below the used level. the posterior vertebral elements.
4. Relative and absolute contraindications to the • Single discs or several disc levels may be replaced
procedure include central or lateral recess stenosis, during the same surgery.
herniated nucleus pulposus with radiculopathy,
osteoporosis, scoliosis, acet arthrosis, spondylolysis
or spondylolisthesis, and def ciency o the posterior Suggested Readings
vertebral elements.
Huang RC, Lim MR, Girardi FP, Cammisa FP Jr. The
5. Depending on the prosthesis used the a ected disc prevalence o contraindications to total disc replacement
is either partially or ully excised. The artif cial disc in a cohort o lumbar surgical patients. Spine
comprises two metallic endplates that are separated by (Phila Pa 1976). 2004 Nov 15;29(22):2538-2541.
a more pliable core. The latter is usually a plastic spacer McDonald CP, Chang V, McDonald M, Ramo N, Bey MJ,
made o a polyethylene core and is designed to simulate Bartol S. Three-dimensional motion analysis o the cervical
the biomechanical properties o the nucleus pulposus. spine or comparison o anterior cervical decompression
The vertebral endplates and paraspinal ligaments are and usion versus artif cial disc replacement in 17 patients.
J Neurosurg Spine. 2013 Dec 20.
74
Increasing back pain

1. What are common imaging f ndings o this


lesion?

2. A posterior position o an intradural


extramedullary mass would support the
diagnosis o which entity?

3. What is the classic imaging appearance?

4. What portion o the spine is most commonly


a ected?

5. What are the treatment options?

75
Meningioma, myelography 1359
Case ranking/dif culty: Category: Spinal canal

Frontal radiograph demonstrates focal meniscus at the cephalad Lateral radiograph demonstrates lling defect in the anterior
edge of intrathecal contrast. spinal canal with severe narrowing of the thecal sac.

Answers
1. Meningiomas are more likely to calci y than Pearls
schwannomas, which are more likely to undergo cystic • Meningiomas are the second most common tumor o
degeneration. Meningiomas are most common in the the extramedullary, intradural space.
thoracic spine, but are overall rarer than intracranial • They are usually benign and do not present until
meningiomas. symptoms o cord or nerve root compression develop.
• The classic imaging appearance is an extramedullary
2. Nerve roots exit the spinal cord in the ventral spinal
intradural mass with calcif cations and an enhancing
canal; there ore, a mass in the posterior intradural
dural tail.
extramedullary space is unlikely to represent a nerve-
• Treatment is surgical resection; however, i the
based lesion and a meningioma would be avored.
mass has been present or a long time, there may be
3. Meningiomas are usually isointense to the spinal cord secondary myelomalacia.
and demonstrate homogeneous enhancement and an
enhancing dural tail. Calcif cation can occur; cystic
degeneration is uncommon and avors a diagnosis o
Suggested Readings
schwannoma.
Beall DP, Googe, DJ, Emery RL, et al. Extramedullary
4. Approximately 80% o meningiomas occur in the intradural spinal tumors: a pictorial review. Curr Probl
thoracic spine, ollowed by the lumbar and cervical spine. Diagn Radiol. 2007;36:185-226.
5. Meningiomas are generally amenable to surgical Lee JW, Lee IS, Choi K-U, et al. CT and MRI f ndings
resection with excellent outcomes. However, recurrence o calcif ed spinal meningiomas: correlation with
can occur i there is subtotal resection, especially pathological f ndings. Skeletal Radiol. 2010;39:345-352.
in the case o an en plaque or inf ltrative lesion and
radiotherapy and/or radiosurgery can be considered.

76
Incontinence, history of imperforate anus

1. What should be included in the di erential


diagnosis?

2. What are common presenting symptoms?

3. What are potential causative actors?

4. What are di erences between Group 1 and


Group 2 patients?¡

5. What are the treatment options?

77
Caudal regression 1491
Case ranking/dif culty: Category: More than one category

Sagittal T2 image demonstrates agenesis of the coccyx and lower Coronal T2 image demonstrates a blunted conus, terminating at
sacrum with a blunted conus, terminating at the L1 level. the L1 level.

Answers
1. The di erential diagnosis includes tethered cord, Pearls
myelomeningocele, and terminal lipoma. • Caudal regression syndrome results rom an insult
prior to the ourth week o gestation.
2. Presenting symptoms include urinary incontinence, club
• There are several etiologies postulated to cause these
eet, and lower extremity weakness. In addition, there
abnormalities, including hyperglycemia or ischemic
are associated anomalies o the gastrointestinal and
insult.
genitourinary tracts, as well as the heart.
• There is an association with maternal diabetes
3. The exact etiology is uncertain; however, there is a and VACTERL (Vertebral anomalies, Anorectal
signif cant increase in incidence in o spring o diabetic mal ormations, TracheoEsophageal f stulas, Renal,
mothers, particularly those who are insulin dependent. and Limb anomalies).
Other causative actors include vascular insult, toxic • While most cases are sporadic, there is an inherited
exposure, and maternal in ection. orm as well.
• Treatment is geared at untethering the cord (i present)
4. Group 1 patients are at the more severe end o the
with physical therapy or the lower extremities and
spectrum with more severe sacral agenesis. They
treatment or associated incontinence.
uni ormly have a high location o the conus, which is
usually blunted or wedge shaped. Group 2 patients have
less severe sacral anomalies and usually have normal
to low-lying conus position with occasional tethering Suggested Readings
lesion. Gehlot P, Mandliya J. Caudal regression syndrome. IJBAS.
5. I present, cord tethering should be released. Treatment is 2011;1:126-130.
then directed at symptomatic lower extremity problems. Smith AS, Grable I, Levine D. Caudal regression
syndrome in the etus o a diabetic mother. Radiology.
2004;230:229-233.

78
Back pain

1. What should be included in the di erential


diagnosis?

2. What are common presenting symptoms?

3. Which location within the spine is most


commonly a ected?

4. What actors should be considered in selecting


treatment options?

5. What are the treatment options?

79
Kummell disease 1514
Case ranking/dif culty: Category: Vertebral body

Sagittal CT image demonstrates vacuum disc at L3-L4 with extension Coronal CT image demonstrates gas within the L3 vertebral body
into the inferior endplate of L3. adjacent the inferior endplate.

Answers
• In this case, there is a compression de ormity o the
1. The di erential diagnosis includes in ection, Kummell
L4 vertebral body; however, the L3 vertebral body
disease, and Schmorl node.
height is maintained.
2. The most common presentation is back pain and • Given the intrabody air, this vertebra is at high risk or
kyphosis. Presence o bowel and bladder dys unction, collapse, which can lead to a pain ul kyphosis.
myelopathy, and radiculopathy implies compromise o • Patients tend to be elderly osteopenic women who
the spinal canal. present with pain ul ocal kyphosis.
• The presence o gas within the vertebral body makes
3. The thoracic and lumbar spine are most commonly
pathologic racture much less likely.
a ected.
• Treatment can consist o conservative management
4. Patient age, degree o kyphosis, level o disability, and analgesia, to vertebroplasty/kyphoplasty or usion.
pain level, and neurological compromise should all be
considered in deciding on treatment options.
5. Potential treatment options include conservative Suggested Readings
management, analgesia, bracing, and vertebroplasty.
Kim SW, Kim H-S. A case o posterior element racture in
Kummell’s disease. Osteoporos Int. 2012;23:1641-1644.
Ma R, Chow R, Shen FH. Kummell’s disease: delayed post-
Pearls traumatic osteonecrosis o the vertebral body. Eur Spine J.
2010;19:1065-1070.
• Kummell disease is avascular necrosis o the vertebral
body, which o ten starts in a delayed ashion ollowing
trauma.
• The imaging hallmark is the presence o a gas-f lled
cle t within the vertebral body.

80
Chronic neck pain with upper extremity weakness

1. What is the most likely diagnosis?

2. How does the condition usually mani est?

3. What should be included in the di erential


diagnosis?

4. Which imaging modality best images the


condition?

5. Which treatments may be benef cial in this


condition?

81
Cervical spondylotic myelopathy (CSM) 3313
Case ranking/dif culty: Category: Spinal cord

Sagittal T2-weighted sequence of Sagittal T1-weighted sequence Axial T2-weighted sequence at the level of C4-C5
the cervical spine demonstrates of the cervical cord. Note a intervertebral disc. There is a broad-based disc/osteophyte
degenerative changes in the mid prominent disc/osteophyte protrusion (arrow) that impinges the cervical cord and
and lower cervical spine. There is bar at C4-C5 (arrow) that results in moderate canal stenosis. Increased intramedullary
a disc/osteophyte protrusion at projects posteriorly into the signal intensity is also noted (arrowhead).
C4-C5 (arrow) that impinges the spinal canal. The associated
cervical cord. Intramedullary high cord signal abnormality is not altered intramedullary signal intensity particularly on
signal intensity (arrowhead) is appreciated on this sequence. T2-weighted images.
noted at C5 level in keeping with 5. Conservative measures aim at providing symptomatic
cervical spondylotic myelopathy.
relie and include physical therapy, cervical
Answers immobilization, systemic and epidural steroids, and pain
relie . Surgery, on the other hand, aims at preventing
1. Cervical spondylotic myelopathy re ers to spondylotic
urther neurological deterioration and may achieve some
changes within the cervical spine that result in
unctional recovery.
neurologic def cits due to cervical cord injury.
2. A ected individuals o ten present with cervical neck
pain, sti ness, and crepitus, which may be associated
Pearls
with or predate brachialgia, upper extremity weakness, • Cervical spondylotic myelopathy re ers to spondylotic
and impaired dexterity, and is later ollowed by changes within the cervical spine, which result in
neurological dys unction in the lower limbs. Sensory neurologic def cits due to cervical cord injury.
loss occurs below the a ected spinal level and typically • A clinically symptomatic dys unction o the
results in pain and temperature disturbances with relative cervical cord and radiological demonstration o cord
sparing o proprioception, vibration, and touch. Gait impingement or compression are required to make the
abnormalities (broad-based, hesitant, shu ing gait) and diagnosis.
bowel and bladder instability result rom involvement o • Several conditions may mimic CSM, and there ore the
the long tracts o the cord. clinical diagnosis is usually not straight orward.
• Newer techniques like di usion tensor imaging
3. Myelopathy could be secondary to causes other than promise to increase the sensitivity or detecting CSM.
spondylotic including extradural compression o the
cord by hemorrhage, abscess ormation, and neoplasia.
Other causes include in ammatory conditions, vascular Suggested Readings
disease, drug intoxication, autoimmune conditions, König SA, Spetzger U.Surgical management o cervical
previous trauma, and metabolic abnormalities. spondylotic myelopathy—indications or anterior, posterior
4. Unlike radiography and CT that can only demonstrate or combined procedures or decompression and stabilisation.
cervical spondylotic changes, MRI can evaluate directly Acta Neurochir (Wien). 2014 Feb;156(2):253-258.
the cervical neural structures. It allows or direct Miyazaki T, Sudo H, Hiratsuka S, Iwasaki N. Cervical
visualization o cord impingement and can demonstrate spondylotic myelopathy with subacute combined
degeneration. Spine J. 2014 Feb 1;14(2):381-382.
82
Involved in a motor vehicle accident

1. What are thoracic paraspinal lines?

2. What is the importance o this so t tissue sign?

3. Describe the radiological appearance o both


the right and le t paraspinal lines.

4. How o ten are thoracic paraspinal lines seen?

5. Name some causes o displacement or bulging


o thoracic paraspinal lines.

83
Thoracic paraspinal lines (TPL) 3246
Case ranking/dif culty: Category: Vertebral body

Posteroanterior (PA) chest radiograph in a Dedicated thoracic spine radiographs (AP Axial CT scan at the level of T6 shows a
patient involved in a motor vehicle accident. and lateral) were taken, which con rmed wedge fracture of T6 vertebral body (arrow)
There is an abnormal focal bulge of the the abnormal right paraspinal bulge and with an associated paraspinal hematoma
right paraspinal line (arrow) at the level of a possible upper endplate fracture of T6. (arrowhead).
T6 vertebral body. Otherwise the lungs are
clear and there are no pleural e usions.

Answers and metastatic deposits), abscess ormation, and


1. Thoracic paraspinal lines (TPL) are a eature o rontal posttraumatic hematoma. The latter three entities are
chest radiographs and are ormed by the inter ace o ten associated with changes in the adjacent vertebrae,
between the lungs and posterior mediastinal so t tissues. which should there ore be assessed thoroughly.
Extensive mediastinal at in Cushing disease and obese
2. The ability to recognize so t tissue signs on the chest patients may also displace the paraspinal lines.
radiograph can provide valuable clues to spinal and
paraspinal pathology. The lines should there ore not
be overlooked as they may reveal potentially serious
Pearls
traumatic injuries or posterior mediastinal abnormalities.
A localized bulge or di use displacement o the • Thoracic paraspinal lines (TPL) are a eature o rontal
thoracic paraspinal lines without an obvious incidental chest radiographs and are ormed by the inter ace
explanation should there ore alert the radiologist o these between the lungs and posterior mediastinal so t
possibilities and urther imaging may be warranted. tissues.
• Displacement or bulging o paraspinal lines may be due
3. The le t paraspinal line extends rom the arch o the to several etiologies including degenerative changes in
aorta to the diaphragm (sometimes extends below it) and the thoracic spine, posterior mediastinal lesions, abscess
lies medial to the lateral aortic wall. Occasionally, it may ormation, and posttraumatic hematoma.
be seen lateral to the descending thoracic aorta. • Abnormal thoracic paraspinal lines should not be
The right paraspinal line is seen as a straight vertical overlooked as they may reveal potentially serious
line just lateral to T8 through T12 vertebral levels, and traumatic injuries or posterior mediastinal abnormalities.
projects below the diaphragm. The lines can be better
appreciated on computed tomography.
4. The le t paraspinal line is seen in about 35% o chest Suggested Readings
radiographs and is seen more commonly than its right Casullo J, Fallone BG. On the perception o the le t thoracic
counterpart, which is only seen in 25% o examinations. paraspinal line. Acad Radiol. 1995 Mar;2(3):215-221.
Gupta SK, Mohan V. The thoracic paraspinal line: urther
5. Displacement or bulging o paraspinal lines may be
signf cance. Clin Radiol. 1979 May;30(3):329-335.
due to several etiologies including degenerative change
in the thoracic spine, posterior mediastinal lesions (eg,
lymphadenopathy, neurogenic tumors, extramedullary
hematopoiesis, esophageal pathology, descending
thoracic aortic aneurysm, granuloma ormation,

84
Chronic low back pain. Known case of prostatic carcinoma

1. What is this radiological f nding called?

2. Name some common causes in adults.

3. Name some common causes in children.

4. Describe the typical imaging f ndings.

5. How can the di erential diagnoses be


narrowed down?

85
Ivory vertebra sign 3222
Case ranking/dif culty: Category: Vertebral body

AP radiograph of the lumbar spine. Lateral radiograph of the Sagittal T2-weighted Sagittal T1-weighted
There is a homogenously dense T12 lumbar spine. Again seen sequence of the lumbar sequence of the lumbar
vertebral body (arrow) that retains is di use sclerosis a ecting spine. T12 vertebral body spine. The normal high
its normal size and contours without T12 vertebral body (arrow) (arrow) appears very signal of the fatty bone
change in the adjacent intervertebral with no vertebral expansion hypointense in keeping marrow is replaced by
discs. The pedicles are not identi ed in or cortical destruction. with extensive sclerosis. extensive sclerosis in T12
the a ected vertebra (arrowhead) due The adjacent discs are not Also note an incidental vertebral body (arrow) and
to overlapping sclerosis. Appearances a ected. oval-shaped lesion in the appears very hypointense.
are compatible with an “ivory vertebra.” spinal canal (arrowhead), Again seen in an incidental
which was con rmed to be meningioma (arrowhead).
a meningioma.
Answers
1. An “ivory vertebra” re ers to a solitary homogenously
dense vertebral body that otherwise retains its normal size intensity on STIR. The a ected vertebra may show
and contours without change in the adjacent intervertebral increased radiotracer uptake on scintigraphy.
discs. It was f rst described by Souques in 1925. 5. A thorough clinical history, physical examination, and
2. In adults, the most common causes are osteoblastic serological tests may help narrow down the di erential
metastases (particularly rom breast and prostate diagnoses as some o these are systemic conditions.
carcinoma), lymphoma, in ection, post-radiation
necrosis, and Paget disease o bone. Less commonly
chordoma, SAPHO syndrome, sarcoma, and di use Pearls
condensing osteosis may be responsible. • An “ivory vertebra” re ers to a solitary homogenously
3. In children, lymphoma, osteosarcoma, osteoblastoma, dense vertebral body that otherwise retains its normal
and metastasis ( rom neuroblastoma, medulloblastoma size and contours without change in the adjacent
and Ewing sarcoma) are o ten responsible. intervertebral discs.
4. The a ected vertebra appears as a sclerotic dense
vertebra with possible involvement o the posterior Suggested Readings
elements. The vertebral body is not expanded and its
contours are preserved. Similar changes are noted on Cli ord PD, Jose J. Ivory vertebra sign. Am J Orthop
computed tomography where di use sclerotic change is (Belle Mead NJ). 2010 Aug;39(8):400-402.
seen to involve most o the vertebral body. MRI reveals Graham TS. The ivory vertebra sign. Radiology. 2005
T1 low signal intensity within the a ected vertebral May;235(2):614-615.
body, which replaces the normal hyperintense atty
bone marrow. The vertebra may demonstrate high signal
86
Chronic low back pain

1. What is the most likely diagnosis?

2. What is meant by disc degenerative changes?

3. Describe clinical and genetic implications o


this condition.

4. What MRI changes may be seen in this


condition?

5. What is the etiology o loss o disc height?

87
Disc desiccation 3104
Case ranking/dif culty: Category: Disc

Sagittal T2-weighted sequence of the lumbar spine shows a Sagittal T1-weighted sequence of the lumbar spine. The vertebral
hypointense intervertebral disc (arrow) at L4-L5 with loss of body heights and marrow signal are within normal limits.
distinction between the inner and outer parts of the disc (Grade 4). Incidental note is made of a retroverted uterus (arrow).
A“sandwich-like”con guration of the discs with low signal intensity
bands (Grade 2) is also noted (arrowheads) at the more cranial levels.
Disc degeneration with a small protrusion is also noted at L5/S1. III—The inner parts o the disc are o inhomogeneous
intermediate signal intensity.
Answers IV—The central parts o the disc are o intermediate to
1. Disc desiccation is a very common degenerative change low signal intensity with loss o distinction between the
o intervertebral discs. It is usually the earliest sign o inner and outer parts o disc.
more severe disc degenerate changes. V—Disc collapse.
2. Intervertebral discs show degenerative changes earlier than 5. It has been shown that loss o intervertebral disc height
other connective tissues in the body. Disc degeneration is secondary to annular bulging and bowing o the
may mani est as disc space narrowing, altered signal vertebral endplates rather than a reduction in the nucleus
intensity on MRI, annular tears, Schmorl nodes, disc pulposus volume.
bulging, protrusion, extrusion, and sequestration.
3. The incidence o disc desiccation increases with age
and is o ten considered as a natural process o aging. Pearls
Pathologically, the hydrophilic glycosaminoglycans
• Pathologically, the hydrophilic glycosaminoglycans
within the nucleus pulposus are replaced with
within the nucleus pulposus are replaced with
f brocartilage, which reduces the exibility o the disc.
f brocartilage, which reduces the exibility o the disc.
Desiccation a ects particularly mobile sections o the
• Desiccation a ects particularly mobile sections o the
spine and is hence most commonly observed in the
spine and is hence most commonly observed in the
cervical and lumbar levels.
cervical and lumbar levels.
4. On MRI, the central T2 hyperintensity within the disc is • On MRI, the central T2 hyperintensity within the disc
lost and the horizontal midline hypointense cle t can no is lost and the horizontal midline hypointense cle t can
longer be appreciated. no longer be appreciated.
Several classif cations have been proposed to quanti y
the extent o disc desiccation. One such classif cation has
f ve grades as ollows: Suggested Readings
I—The nucleus pulposus is homogeneously hyperintense Alomari RS, Corso JJ, Chaudhary V, Dhillon G. Computer-
and clearly distinct rom the hypointense annular f bers. aided diagnosis o lumbar disc pathology rom clinical
II—The nucleus pulposus is inhomogeneous with lower spine MRI. Int J Comput Assist Radiol Surg. 2010
horizontal low signal bands in a sandwich-like May;5(3):287-293.
conf guration. Dähnert W. Radiology Review Manual. Philadelphia, PA:
Lippincott Williams Wilkins; 2007.

88
Traumatic burst fracture of C4

1. What is this procedure called?

2. Describe VBR device characteristics and their


surgical implications.

3. Name some disadvantages o carbon f ber


devices.

4. What are the objectives o surgical treatment


or spinal tumors?

5. Name some recognized complications o this


procedure?

89
Vertebral body replacement (VBR) 3102
Case ranking/dif culty: Category: Vertebral body

Lateral radiograph of the cervical spine. There is a vertebral body Anteroposterior radiograph of the cervical spine. A VBR device
replacement (VBR) device in situ at C4 (arrow). The device spans (arrow) is seen at C4 level. Anterior spinal instrumentation is also
the interbody distance between C3 and C5 and its edges abut noted (arrowhead).
the endplates of the respective vertebrae. Anterior xation of the
spine (arrowhead) is seen as this device was not designed for use
as a standalone device. 4. The objectives o surgical treatment or spine tumors
include pain amelioration, decompression o the neural
Answers elements, spinal stabilization, and wide resection o the
primary or metastatic tumors.
1. Vertebral body replacement re ers to the use o prosthetic
devices to replace collapsed, damaged, or unstable 5. Complications o vertebral body replacement may result
vertebral bodies due to tumor or trauma. rom either the surgical approach or the placement
o spinal instrumentation. These include hemorrhage
2. VBR devices come in a variety o lengths, widths, and secondary to vascular injury, in ection, injury to internal
heights to accommodate most patient anatomies. Others organs, the lumbosacral plexus, and the sympathetic
can be set to the required settings during surgery and chain.
may have an expansion range enough to replace one or
Most manu acturers recommend annual radiographic
two vertebral bodies.
assessment and pain assessment scores in order to detect
Although implant devices may have f xed degrees o loss o device height and implant ailure.
lordosis, some have an advanced endplate design, which
allows or in situ angulation o up to 10° during surgery
in order to precisely match the patient’s anatomic need.
Pearls
A variety o materials have been used in the construction
o vertebral body implants, which include titanium cages • Vertebral body replacement re ers to the use o
and meshes, ceramic, ceramic/glass, and carbon f ber. prosthetic devices to replace collapsed, damaged, or
unstable vertebral bodies due to tumor or trauma.
Some titanium mesh cages have spikes at their end tips
• Several implants with di erent specif cations and
and sharp edges that allow them to be anchored into the
characteristics have been devised and are available on
adjacent vertebral bodies. This provides extra torsional
the market.
stability. Most expandable cages are not designed or use
as a standalone device, and there ore anterior (with or
without additional posterior) instrumentation is required.
Suggested Readings
3. Carbon f ber has the advantage o being radiolucent and
Daentzer D, Bianchi N, Böker DK, Deinsberger W.
there ore allows better postoperative usion assessment.
Multilevel segmental interbody usion versus vertebral
However, carbon f ber implants have several disadvantages
body replacement: comparison o two operative methods.
including in ammatory responses with possible oreign
Orthopade. 2013 Dec 19.
body reactions, material brittleness that can result in
Rohlmann A, Dreischar M, Zander T, Graichen F, Bergmann
breakage, as well as composite material ailure.
G. Loads on a vertebral body replacement during
locomotion measured in vivo. Gait Posture. 2013 Oct 19.
90
Whiplash injury

1. What is the most likely diagnosis?

2. Where do most disc herniations occur?

3. Name subdivisions o this condition.

4. What is the etiology o this condition?

5. Integrity o which anatomical structure is used


to di erentiate “contained” rom “uncontained”
disc herniations?

91
Annular rupture 3052
Case ranking/dif culty: Category: Disc

Axial images at the level of T3-T4. There is a paracentral disc


herniation (arrow, panel A) lling the left lateral recess and
displacing the dural sac (arrowhead, panel B) medially.

Sagittal T2-weighted sequence of Sagittal T1-weighted sequence


the cervical and upper thoracic of the cervical and upper The term is not synonymous with ruptured disc, which is
spine. There is disc herniation thoracic spine. Note that the equivalent to disc herniation.
(arrow) at T3-T4 intervertebral herniated material (arrow)
disc space that is of the same follows the same signal 5. Intervertebral disc herniations are also characterized as
signal intensity as the nucleus intensity as the central part “contained” when the displaced disc material is covered
pulposus. The herniated material of the intervertebral disc and by the intact outer annulus and “uncontained” when such
is contained posteriorly by the represents herniated nucleus covering is absent. Although the disc material may still
posterior longitudinal ligament pulposus. There is mild loss of be covered by the posterior longitudinal ligament (PLL)
(PLL). Note is also made of mild the a ected intervertebral disc and the peridural membranes, the term uncontained is
degenerative changes in the height. still recommended i the outer annulus is breeched.
lower cervical spine.

Answers Pearls
1. The annulus f brosus is composed o 15-20 consecutive • Rupture o the annulus re ers to disruption o the
rings called laminae composed o type 1 collagen, which annulus f brosus and the term implies a posttraumatic
run obliquely to span the distance between adjacent etiology.
vertebrae. The direction o such f bers alternates between • Annulus f brosus ruptures may be urther subdivided
one lamina and another with their orientation with into circum erential or radial ruptures.
respect to the vertical being approximately 65°-70°.
The inner portion o the annulus f brosus is composed
o weaker f brocartilage that blends gradually with the Suggested Readings
central nucleus pulposus.
Fardon DF, Milette PC; Combined Task Forces o the
2. The annulus is thinner and the collagen f bers are more North American Spine Society, American Society o
disorganized posterolaterally, rendering this part o the Spine Radiology, American Society o Neuroradiology.
disc weaker. The latter accounts or the increased risk o Nomenclature and classif cation o lumbar disc pathology.
disc herniation posterolaterally. Recommendations o the Combined task Forces o the
North American Spine Society, American Society o Spine
3. Annulus f brosus ruptures may be urther subdivided into
Radiology, and American Society o Neuroradiology.
circum erential or radial ruptures.
Spine (Phila Pa 1976). 2001 Mar 1;26(5):E93-E113.
4. In 1995, a multidisciplinary task orce rom the Standring S (Ed.). Gray’s Anatomy. 39th ed. London:
North American Spine Society addressed the lack o Elsevier. Churchill Livingstone; 2011.
standardization in the terminology o intervertebral disc
disorders and recommended a nomenclature or use in
clinical practice.
Rupture o the annulus re ers to disruption o the annulus
f brosus and the term implies a posttraumatic etiology.

92
Incidental nding on a chest radiograph

1. What is the most likely diagnosis?

2. How does the condition usually mani est?

3. Describe the imaging f ndings in this condition.

4. Describe clinical and pathological


characteristics o this condition.

5. What is meant by a false cervical rib?

93
Cervical rib 3050
Case ranking/dif culty: Category: Miscellanous

4. They may be uni- (33%) or bilateral (66%) and occur in


0.5% o the general population. They are more commonly
ound in emales. Up to 15% o patients with Klippel-
Feil syndrome have a cervical rib. Although they arise
most commonly at the level o C7 (95%), they have been
described as high as C4 level. The condition is sometimes
called dorsalization o the cervical spine. Cervical ribs
vary signif cantly in their shape, size, and anatomical
course, and it is impossible to determine radiographically
whether cervical ribs will be symptomatic or not. This
is because short cervical ribs may be associated with
long f brous bands that cause compression o neural or
vascular structures.
5. Elongation o the transverse process o C7 that curves
and tapers distally but lacks a costotransverse and
costovertebral joints is called a false cervical rib or
apophysomegaly. The latter is rarely symptomatic but
may still have associated f brous bands that may impinge
into the scalene triangle.

Frontal chest radiograph. There is a right cervical rib (arrow) that


articulates via a costotransverse joint (arrowhead) with C7 vertebra.
Pearls
• Cervical ribs are supernumerary ribs that orm true
Answers articulations with the transverse process o the seventh
1. Cervical ribs are supernumerary ribs that orm true cervical vertebra.
articulations with the transverse process o the seventh • They are o ten asymptomatic but may result in
cervical vertebra. thoracic outlet syndrome due to compression o the
ipsilateral brachial plexus or subclavian vessels.
2. When symptomatic, cervical ribs o ten present
• They may be uni- (33%) or bilateral (66%) and occur
around the ourth or f th decade and the dominant
in 0.5% o the general population.
arm is more requently a ected. Neurological
• A costotransverse joint needs to be present to make
symptoms (97%) predominate (pain, wasting o hand
the diagnosis.
muscles, paresthesiae) with venous (2%) and arterial
• The length o cervical ribs varies rom a rudimentary
mani estations (1%) being much rarer. Venous occlusion
stump to a ully ormed rib that articulates anteriorly
by either mechanical compression or venous thrombosis
with the sternoclavicular junction.
results in ipsilateral upper limb swelling whereas arterial
• Coronal oblique MR sequences will also depict the
compromise o ten results in Raynauds-like presentation.
relation between the cervical rib and the thoracic
Dynamic tests including Wright test and Adson test may
outlet, neural and vascular structures.
conf rm the presence o thoracic outlet syndrome.
3. The diagnosis can be made on plain f lms o the cervical
spine. A costotransverse joint needs to be present to Suggested Readings
make the diagnosis. The length o cervical ribs varies
Millan G, Casal D, Sagaribay A, Marques V, Martins
rom a rudimentary stump to a ully ormed rib that
JE. Neurogenic thoracic outlet syndrome associated
articulates anteriorly with the sternoclavicular junction.
with cervical rib. Acta Reumatol Port. 2013 Apr-
Fusion with the f rst rib is common as is pseudoarthrosis
Jun;38(2):98-103.
o the cervical rib. Coronal oblique MR sequences will
Weber AE, Criado E. Relevance o bone anomalies in
also depict the relation between the cervical rib and the
patients with thoracic outlet syndrome. Ann Vasc Surg.
thoracic outlet, neural and vascular structures. Deviation
2013 Dec 5.
o brachial plexus roots, ocal vessel dilatations, vascular
deviations or kinks, and collateral circulation may be seen.

94
Chronic low back pain

1. What is the most likely diagnosis?

2. Describe clinical and pathological


characteristics o this condition.

3. How does the condition mani est clinically?

4. Describe the typical imaging f ndings.

5. Which treatments may be benef cial in this


condition?

95
Spondylolysis (pars defect) 3049
Case ranking/dif culty: Category: Posterior elements

AP radiograph of the lumbar spine. The vertebral endplates and Lateral radiograph of the lumbar spine demonstrates a defect in
pedicles are intact. Note that the pars interarticularis defect is not the pars interarticularis (arrow) of L4 vertebra that has resulted in
seen on this projection. grade 1 anterolisthesis of L4 vertebra (arrowhead).

Answers brace) is also e ective. Nonsteroidal anti-in ammatory


1. Spondylolysis re ers to a de ect in the neural arch, drugs (NSAIDs) are used to relieve the pain and reduce
specif cally in the pars interarticularis. in ammation. Surgical options include laminectomy
and posterior lumbar usion and may be o ered
2. Spondylolysis is a stress racture o the pars to symptomatic patients who do not respond to
interarticularis secondary to repeated microtrauma conservative measures.
particularly in individuals involved in strenuous sports
including ootball and gymnastics. Heredity is also
believed to play an important role. The condition has
Pearls
a male predominance and is more common in patients
with spina bif da occulta. The pars interarticularis is very • Spondylolysis re ers to a de ect in the neural arch,
vulnerable when the spine is subjected to an axial orce specif cally in the pars interarticularis.
in the extended position such as when landing on one’s • Most cases (95%) occur at L5 level but spondylolysis
eet. Such pressure can cause a racture in susceptible may a ect any lumbar vertebra and less commonly the
individuals. Bilateral spondylolysis can progress to thoracic vertebrae.
spondylolisthesis. • Pars de ects are commonly asymptomatic but some
patients may experience pain with extension or
3. Pars de ects are commonly asymptomatic but some rotation o the lumbar spine.
patients may experience pain with extension or rotation
o the lumbar spine.
4. The diagnosis can be made on radiography. Lateral Suggested Readings
projections are the most sensitive whereas oblique views
Standaert CJ, Herring SA. Spondylolysis: a critical review.
are the most specif c or detection o pars ractures.
Br J Sports Med. 2000 Dec;34(6):415-422.
Oblique radiographs characteristically demonstrate a
Syrmou E, Tsitsopoulos PP, Marinopoulos D, Tsonidis C,
collar around the neck o the “Scottie dog.” Radiography
Anagnostopoulos I, Tsitsopoulos PD. Spondylolysis: a
may, however, be limited by its inability to demonstrate
review and reappraisal. Hippokratia. 2010 Jan;14(1):17-21.
stress reactions in the a ected pars interarticularis
be ore progression to a complete racture. MRI, CT, and
scintigraphic studies are much more sensitive to detect
these early changes.
5. Treatment involves physiotherapy and rest rom
strenuous activities. A Boston overlap brace (antilordotic

96
Fall from height

1. What is the most likely diagnosis?

2. Why shouldn’t this f nding be overlooked?

3. Which are the our subtypes described by


Castellvi et al?

4. Describe the typical imaging f ndings.

5. Which landmarks are used to characterize


lumbosacral transitional vertebrae?

97
Lumbosacral transitional vertebrae (LSTV) 3048
Case ranking/dif culty: Category: Vertebral body

AP radiograph of the lumbar spine. There are six rib-free lumbar- Lateral radiograph of the lumbar spine. There are upper end plate
type vertebrae (labelled L1—LSTV) in keeping with a lumbosacral fractures of L2 and L3 vertebrae (arrows). The vertebral alignment
transitional vertebra. Posttraumatic upper end plate fractures of L2 is preserved. The most caudal ribs (arrowheads) articulate with
and L3 are seen, which are better appreciated on the lateral lm. Th12 vertebra. Note that there are six lumbar vertebrae in keeping
with an LSTV. Squaring of the lumbarized upper sacral segment is
Answers noted. The intervertebral disc above the LSTV shows some loss of
disc height.
1. Lumbosacral transitional vertebrae (LSTV) are common
congenital spinal anomalies seen in 4%-30% o the 5. A sagittal cervicothoracic MR localizer may be used to
general population. accurately characterize transitional vertebrae. Another
2. They have been implicated as a cause o chronic back technique to correctly number LSTV is to identi y the
pain. Lumbosacral transitional vertebrae should not be iliolumbar ligament, which usually arises rom the
overlooked as ailure to recognize the anomaly or a poor transverse process o L5 and is a relatively constant
description in the radiological report may lead to spinal landmark. The use o other anatomical markers including
interventions or surgery being per ormed at an incorrect the aortic bi urcation, right renal artery, and conus
level. medullaris has been described, but these are less reliable.
3. Castellvi et al proposed a classif cation o lumbosacral
transitional vertebrae:
Pearls
Type 1—a dysplastic transverse process with a height
o >90 mm • Lumbosacral transitional vertebrae (LSTV) are
common congenital spinal anomalies seen in 4%-30%
Type 2—incomplete lumbarization or sacralization
o the general population.
Type 3—complete lumbarization or sacralization with • Failure to recognize LSTV may lead to spinal
complete usion with the neighboring sacral basis interventions or surgery being per ormed at an
Type 4—mixed type incorrect level.
4. Lumbarization o S1 is less common than sacralization
and is encountered in about 2% o the population. Six
rib- ree lumbar-type vertebrae are seen on radiography. Suggested Readings
Anomalous (o ten rudimentary) acet joints and Castellvi AE, Goldstein LA, Chan DPK. Lumbosacral
intervertebral disc between S1 and S2 are seen. Other transitional vertebra and their relationship with lumbar
radiographic f ndings include squaring o the lumbarized extadural de ects. Spine. 1983;9:493-495.
upper sacral segment or wedging o the sacralized lowest Konin GP, Walz DM. Lumbosacral transitional vertebrae:
lumbar segment. The intervertebral disc above the LSTV classif cation, imaging f ndings, and clinical relevance.
o ten shows increased degenerative changes. AJNR Am J Neuroradiol. 2010 Nov;31(10):1778-1786.
98
Unremitting back pain

1. What is the most likely diagnosis?

2. What is meant by the term extrusion?

3. What might herniated disc material consist o ?

4. What does the term herniation imply?

5. Name some orms o disc “herniation.”

99
Disc herniation—extrusion 3047
Case ranking/dif culty: Category: Disc

Sagittal T2-weighted sequence of Sagittal T1-weighted sequence of Axial T2-weighted image at the level of T9 vertebral
the lumbar spine. There is a disc the lumbar spine showing a disc body. There is an extradural lesion (arrow) within the
extrusion (arrow) at T9-T10 with extrusion at T9-T10 (arrow). spinal canal, which represents extruded disc material.
cranial migration. The herniated
disc material is in continuity with Sequestration is a subtype o extrusion in which
the parent disc. A Schmorl node herniated disc material has lost continuity with the disc
(arrowhead) is noted in the superior o origin. Migration implies that the herniated disc
endplate of T12. material is displaced away rom the site o extrusion
both i continuation with the parent disc is maintained
Answers or whether there is sequestered ragment. Hence, the
term migration re ers only to the position and not to
1. Disc extrusion.
continuity o the extruded material.
2. Herniated discs may be urther subclassif ed as protrusion
or extrusion according to the shape o herniated material.
Extrusion occurs when the distance between edges o Pearls
herniated disc material is greater than the edges o the
• Disc herniation is def ned as displacement o disc
base o the herniation in any particular plane (ie, it has a
material beyond the normal limits o the intervertebral
broader dome than neck).
disc space.
3. The herniated material may consist o nucleus • Extrusion occurs when the distance between edges o
pulposus, annular tissue, f brocartilage, and ragmented herniated disc material is greater than the edges o the
apophyseal bone. base o the herniation in any particular plane (ie, it has
a broader dome than neck).
4. Disc herniation is def ned as displacement o disc material
beyond the normal limits o the intervertebral disc space.
The latter is def ned craniocaudally by the adjacent
Suggested Reading
vertebral endplates and peripherally by the vertebral ring
apophysis excluding any osteophytic ormations. Fardon DF, Milette PC; Combined Task Forces o the
North American Spine Society, American Society o
Displacement o disc material may only occur when
Spine Radiology, American Society o Neuroradiology.
there is a disruption in the annulus f brosus or a break
Nomenclature and classif cation o lumbar disc pathology.
in the vertebral endplate resulting in intravertebral
Recommendations o the Combined task Forces o the
herniation, which is also known as a Schmorl node.
North American Spine Society, American Society o Spine
5. When disc material extends circum erentially beyond the Radiology, and American Society o Neuroradiology.
vertebral ring apophysis, the term bulging is used, which Spine (Phila Pa 1976). 2001 Mar 1;26(5):E93-E113.
is not considered a orm o disc herniation.
100
Intermittent fever and severe neck pain

1. What is the most likely diagnosis?

2. How does this condition present clinically?

3. Name some predisposing conditions.

4. Which imaging modality best images this


condition?

5. Which is the most common pathogen


cultivated in this condition?

101
Epidural abscess 3000
Case ranking/dif culty: Category: Spinal canal

Sagittal T2-weighted sequence (with fat Sagittal T1-weighted sequence of the Axial T2-weighted sequence con rms the
suppression) of the cervical and upper cervical and upper thoracic spine. Note epidural location of the abscess (arrow).
thoracic spine. There is a T2 hyperintense that the hypointense epidural collection The collection extends along the extradural
dorsal epidural collection that lifts the (arrow) is di cult to distinguish from the space into the left exiting foramen
dura into the spinal canal (arrow). Note the adjacent CSF signal. (arrowhead). Note the high signal intensity
hyperintensity in the paraspinal muscles along the right side of the spinous process.
(arrowhead) posterior to the collection in
keeping with adjacent myositis.
may lead to direct inoculation o the epidural space. The
source o in ection remains unidentif ed in many cases.
Answers
4. MRI is the imaging modality o choice. It will not only
1. The anatomy o the spinal epidural space is crucial to conf rm the presence o an epidural abscess but also
understand the development and progression o epidural determine the extent o involvement and the presence o
abscesses. Anteriorly the dura is tightly adherent to canal or cord compromise. It may also help in planning
ligaments and vertebral bodies whereas posteriorly the surgical treatment.
dura is more lax allowing in ections to spread over several
levels. Abscess ormation is there ore more requent in the 5. Staphylococcus aureus is commonly cultivated and
posterior epidural space, particularly in the thoracic region. accounts or up to 65% o cases.
Spread o in ection to the posterior space is o ten
hematogenic rom a distant source o in ection.
Pearls
2. The initial presentation is o ten nonspecif c and includes
back pain and malaise. Additional symptoms may • Staphylococcus aureus is commonly cultivated and
include ever, radiculopathy, and sphincter dys unction. accounts or up to 65% o cases.
Headache and neck pain may occur with cervical epidural • Early diagnosis and treatment are associated with
abscesses. Some studies have ound that posterior improved outcomes.
spinal epidural abscesses are more likely to present with
severe neurologic def cits compared to ventral epidural
abscesses. It is estimated that up to 50% o cases are Suggested Readings
initially misdiagnosed or have a delayed diagnosis. Huang PY, Chen SF, Chang WN, et al. Spinal epidural
3. Hematogenic spread o in ection to the epidural spread may abscess in adults caused by Staphylococcus aureus:
occur rom bacterial endocarditis, urinary tract in ections, clinical characteristics and prognostic actors. Clin Neurol
peritoneal and retroperitoneal in ections, and in ected Neurosurg. 2012 Jul;114(6):572-576.
indwelling catheters. The anterior epidural space is, on Lang IM, Hughes DG, Jenkins JP, St Clair Forbes W,
the other hand, prone to direct extension rom vertebral McKenna F. MR imaging appearances o cervical epidural
osteomyelitis, particularly in adults. Iatrogenic causes abscess. Clin Radiol. 1995 Jul;50(7):466-471.
include epidural injections or catheter placement, which

102
Acute onset of severe back pain and paraplegia

1. What is the most likely diagnosis?

2. What should be included in the di erential


diagnosis?

3. How does the condition mani est clinically?

4. Which imaging modality best images this


condition?

5. Name some predisposing actors.

103
Spontaneous epidural hematoma 2999
Case ranking/dif culty: Category: Spinal canal

Sagittal T2-weighted Sagittal T1-weighted Axial T2-weighted sequence at the level of the lower thoracic
sequence of the thoracic sequence of the thoracic spine. Note that the thecal sac (arrowhead) is displaced ventrally
spine showing an spine again shows a by a low signal intensity collection (arrow), which causes severe
extensive extradural heterogenous epidural canal narrowing.
collection (arrow) dorsal collection (arrow)
to the thoracic cord posterior to the cord
either be secondary to an underlying lesion or contrast
(arrow). The collection is (arrowhead). Incidental
heterogenous and mostly note is made of multiple extravasation rom leaking vessels. A CT scan should be
of low signal intensity. scattered vertebral obtained i MRI is contraindicated or unavailable.
Incidental note is made hemangiomas. 5. A venous source is thought to be responsible in
of multiple scattered
spontaneous spinal EDH or that ollowing minimal
vertebral hemangiomas.
trauma. Predisposing actors that have been implicated
include coagulopathy, anticoagulant and thrombolytic
Answers drugs, spinal vascular anomalies, intervertebral disc
1. Spontaneous spinal epidural hematoma (EDH) is a rare herniation, Paget disease o bone, Valsalva maneuver,
condition requiring urgent management. It has been and hypertension. Rupture o valveless veins in Batson
reported to occur in all age groups but it is extremely plexus even by the slightest change o posture may
rare in the pediatric population. cause epidural bleeding. Less commonly the source
o bleeding is arterial rom spinal epidural arteries
2. Di erential diagnoses include acute intervertebral disc
and paraspinal muscular branches. The latter is more
herniation, spinal cord in arction, epidural tumor or
common in postoperative EDH.
abscess, spondylitis, transverse myelitis, dissecting
aortic aneurysm, and acute myocardial in arction.
3. Patients typically present with an acute onset o severe
Pearls
stabbing back pain with rapidly developing signs o
spinal cord or cauda equina compression (including • Spontaneous spinal epidural hematoma (EDH) is a
paraparesis and quadriparesis). rare condition requiring urgent management.
• MRI is the gold standard imaging modality and is
4. MRI is the gold standard imaging modality and is use ul in establishing the diagnosis.
use ul in establishing the diagnosis. It demonstrates
biconvex epidural hematomas in the epidural space
with well-def ned borders tapering superiorly and
Suggested Readings
in eriorly. Subacute hematomas show characteristic
T1 hyperintensity. T2 heterogeneous hyperintensity Baek BS, Hur JW, Kwon KY, Lee HK. Spontaneous
with ocal hypointensity should suggest the diagnosis spinal epidural hematoma. J Korean Neurosurg Soc.
o acute spinal EDH. Central or peripheral contrast 2008;44(1):40-42.
enhancement may be seen. Hyperemia o the adjacent Fukui MB, Swarnkar AS, Williams RL. Acute spontaneous
dura mater has been postulated as a cause o peripheral spinal epidural hematomas. AJNR Am J Neuroradiol.1999
enhancement o spinal EDH. Central enhancement may Aug;20(7):1365-1372.

104
Chronic low back pain

1. What is the most likely diagnosis?

2. Which radiological f ndings are typical o this


condition?

3. Which spinal levels are particularly a ected?

4. Which imaging modality best images this


condition?

5. Which treatments may be benef cial?

105
Bertolotti syndrome 2996
Case ranking/dif culty: Category: Posterior elements

4. Radiographs may be diagnostic and demonstrate


anomalous enlargement o the transverse process o a
transitional vertebra.
MRI is the gold standard examination. It will not
only characterize the articulation or usion but also
demonstrate pre erential disc degeneration at the L4/5
level. Disc herniation and exiting nerve impingement
may also be evaluated. Coronal images are particularly
help ul to demonstrate the abnormal articulation.
5. Posterolateral usion or resection o the anomalous
articulation may be o ered. Nonsurgical treatments
include guided steroid injections within the cavity o the
pseudarthrosis or radio requency sensory ablation.
Bertolotti syndrome should be included in the list o
di erential diagnoses in young people presenting with
lower back pain.

Frontal radiograph of the pelvis shows anomalous enlargement


of the left L5 transverse process. The patulous transverse Pearls
process (arrow) forms a pseudoarticulation with the left sacral
ala (arrowhead). • Bertolotti syndrome re ers to lower back pain
secondary to lumbosacral transitional vertebrae.
• The transitional L5 vertebra will have unilateral
Answers spatulas enlargement o its transverse process, which
1. Bertolotti syndrome re ers to lower back pain secondary may articulate or use with the sacrum or ilium.
to lumbosacral transitional vertebrae. • Presentation is with sciatica or radiculopathy, which
corresponds to the f th lumbar nerve root.
2. The transitional L5 vertebra will have unilateral patulous • MRI is the gold standard examination and both
enlargement o its transverse process, which may surgical and conservative approaches are used.
articulate or use with the sacrum. Less commonly it • Bertolotti syndrome should be included in the list o
may articulate with the ilium. di erential diagnoses in young people presenting with
3. The anomalous articulation results in limited motion at lower back pain.
the lumbosacral articulation. To compensate or this,
increased movement occurs at a more cranial lumbar
segment, resulting in increased strain on the L4-L5 disc. Suggested Readings
Presentation is o ten with a sciatic or radicular type o
pain, which corresponds to the f th lumbar nerve root. Aihara T, Takahashi K, Ogasawara A, Itadera E, Ono Y,
Moriya H. Intervertebral disc degeneration associated
Various studies have shown that the intervertebral discs
with lumbosacral transitional vertebrae: a clinical and
immediately above the transitional vertebra were much
anatomical study. J Bone Joint Surg Br. 2005;87:687-691.
more degenerate than the disc between the transitional
Castellvi AE, Goldstein LA, Chan DPK. Lumbosacral
vertebra and the sacrum. The articulation was also
transitional vertebra and their relationship with lumbar
associated with increased strain and degenerative
extadural de ects. Spine. 1983;9:493-495.
changes in the opposite acet joint.
Quinlan JF, Duke D, Eustace S. Bertolotti’s syndrome. A
Aihara et al ound that the iliolumbar ligaments cranial cause o back pain in young people. J Bone Joint Surg Br.
to the transitional vertebrae were much thinner and 2006 Sep;88(9):1183-1186.
weaker than in una ected individuals. The same authors
postulated that this f nding may explain the accelerated
degeneration o the lumbar discs due to hypermobility
and abnormal torque o the intervertebral spaces above
the transitional vertebra.

106
Chronic low back pain

1. What is this procedure called?

2. Name some indications or this procedure.

3. Name some contraindications to the procedure.

4. Are there any recognized complications?

5. What is the purpose o this procedure?

107
Percutaneous vertebroplasty 2982
Case ranking/dif culty: Category: Vertebral body

Sagittal sequences of the lumbar spine (Panel A = T2 Axial T2-weighted sequence at the Lateral radiograph of
weighted, Panel B = STIR, and Panel C = T1 weighted). level of L2. Again seen is a cystic the lumbar spine. There
There is a well-de ned cystic lesion (arrow) occupying lesion (arrow) occupying most of the is radiopaque material
most of L2 vertebral body, which was histologically cross-sectional area of the vertebral (arrow) lling the previously
con rmed to be a solitary bone cyst. body. demonstrated cystic lesion in
L2 vertebral body.

4. Complications o percutaneous vertebroplasty


include neurologic def cits, ractures, hemorrhage,
in ection, allergic reactions, pulmonary embolism, and
pneumothorax.
5. Percutaneous vertebroplasty not only provides pain relie
but may also improve unctional outcomes.

Pearls
• Percutaneous vertebroplasty is a minimally invasive
treatment used in pain ul osteoporotic and malignant
Frontal radiograph of the lumbar spine. Note that the
ractures re ractory to medical therapy.
polymethylmethacrylate (arrow) lls both sides of the vertebral body.
• Complications include neurologic def cits, ractures,
hemorrhage, in ection, allergic reactions, pulmonary
Answers embolus, and pneumothorax.
1. Percutaneous vertebroplasty involves image-
guided percutaneous injection o cement
(polymethylmethacrylate—PMMA) into the a ected Suggested Readings
vertebral bodies.
Buchbinder R, Osborne RH, Ebeling PR, et al. A randomized
2. Percutaneous vertebroplasty is used in pain ul trial o vertebroplasty or pain ul osteoporotic vertebral
osteoporotic and malignant ractures re ractory to ractures. N Engl J Med. 2009 Aug 6;361(6):557-568.
medical therapy. The procedure has also been used in Deramond H, Depriester C, Galibert P, Le Gars D.
vertebral angioma and as a prophylactic procedure in Percutaneous vertebroplasty with polymethylmethacrylate.
pathologically weakened vertebrae. Technique, indications, and results. Radiol Clin North Am.
1998 May;36(3):533-546.
3. The procedure is contraindicated in active osteomyelitis,
allergy to PMMA, and patients with uncorrectable
coagulopathy.
Major complications occur in <1% o patients treated
or osteoporotic ractures and in <5% o neoplastic
vertebral ractures.
108
Loss of bowel and bladder control

1. What is the most likely diagnosis?

2. Describe clinical and pathological


characteristics o this condition.

3. Which other spinal f ndings are associated with


this condition?

4. Which imaging modality best images


this condition?

5. Which imaging f ndings are typical o


this condition?

109
Tethered cord 2980
Case ranking/dif culty: Category: Spinal cord

stretching o the spinal cord, which leads to vascular


insu f ciency at the level o the conus medullaris. I
le t untreated, tethered cord syndrome will have a
progressive course. Surgical release may improve
neurological unction dramatically.
3. Cord tethering may be associated with f lar lipoma and
cyst, diastematomyelia, and imper orate anus. Scoliosis
is seen in up to 20% o a ected individuals.
4. MRI is diagnostic o the condition and is regarded as the
gold standard imaging modality.
5. MR will conf rm a low-lying conus medullaris (ie,
caudal to L2), def ne the thickness o the f lum terminale
(>2mm), and evaluate or associated spinal dysraphism.
An abnormal course o the nerve roots relative to the
spinal cord (>15°) may also be appreciated.
In the pediatric population, ultrasonography is also an
important screening tool. The lack o ossif ed posterior
elements in normal in ants provides an excellent acoustic
window. Reduced or absent pulsatile movement o the
cord and nerve roots is observed on M-mode scanning.
Sagittal T2-weighted sequence Sagittal T2-weighted sequence
of the lumbar spine showing of the sacrum. A slightly more
a low-lying spinal cord. Note caudal sequence was acquired
that the conus medullaris to exclude associated lesions Pearls
(arrow) is abnormally low and such as myelomeningocele
lies at the level of L5 vertebral and cord lipoma. Note • Tethered cord syndrome describes the constellation o
body. The lum terminale an incidental Tarlov cyst symptoms secondary to tethering o the spinal cord.
(arrowhead) is thickened, (arrowhead) within the sacral • The condition may be urther subdivided into primary
and there is no evidence of canal. The conus medullaris when it is seen as an isolated anomaly or secondary
associated spinal dysraphism. lies at the level of the superior when it occurs in the setting o other abnormalities.
endplate of L5. • The condition commonly presents between 5 and 15
years, particularly during periods o growth spurts.
• It may, however, go undiagnosed until adulthood and
Answers present with sensory, motor, or autonomic dys unction.
1. Tethered spinal cord re ers to incomplete involution o • Cord tethering may be associated with f lar lipoma and
the distal spinal cord with ailure o ascent o the conus cyst, diastematomyelia, and imper orate anus.
medullaris. Abnormal tissue attachments within the • MRI is diagnostic o the condition and is regarded as
spinal canal limit the developmental ascent o the spinal the gold standard imaging modality.
cord. Tethered cord syndrome describes the constellation • In the pediatric population, ultrasonography is also an
o symptoms secondary to tethering o the cord. The important screening tool.
condition may be urther subdivided into primary when • Surgical release may improve neurological
it is seen as an isolated anomaly or secondary when unction dramatically.
it occurs in the setting o other abnormalities such as
myelomeningocele and lipoma o the f lum terminale.
2. Presentation in childhood is with the cutaneous stigmata Suggested Readings
o associated spinal dysraphism. The condition more Ng WH, Seow WT.Tethered cord syndrome preceding syrinx
commonly presents between 5 and 15 years, particularly ormation—serial radiological documentation. Childs
during periods o growth spurts. The condition may, Nerv Syst. 2001;17(8):494-496.
however, go undiagnosed until adulthood and present Raghavan N, Barkovich AJ, Edwards M et al.MR imaging
with sensory, motor, or autonomic dys unction. in the tethered spinal cord syndrome. AJR Am J
Neurological dys unction is secondary to chronic Roentgenol.1989;152(4):843-852.

110
Unremitting back pain

1. What is the most likely diagnosis?

2. Which bones are o ten a ected in this


condition?

3. Which symptoms does this condition


present with?

4. What are the characteristic imaging f ndings?

5. Which treatments may be benef cial in


this condition?

111
Solitary plasmacytoma of bone (SPB) 2979
Case ranking/dif culty: Category: Vertebral body

T2-weighted sequence of the lumbar spine shows a hyperintense Axial T2-weighted sequence again demonstrates the hyperintense
lesion in T10 (arrow) that extends to involve the posterior lesion (arrow) centered within the vertebral body with direct
elements (arrowhead). extension into the right pedicle and transverse process.

Answers
1. Solitary plasmacytoma o bone (SPB) is a rare Pearls
hematologic malignancy. It represents the solitary • Solitary plasmacytoma o bone (SPB) is characterized
orm o the spectrum o plasma cell neoplasms that are by neoplastic proli eration o a monoclonal plasma
characterized by neoplastic proli eration o a monoclonal cell inf ltrate.
plasma cell inf ltrate. • It has a predilection or the axial skeleton, particularly
2. The lesion has a predilection or the axial skeleton, the thoracic spine.
particularly the thoracic spine. The ribs, sternum, • Targeted radiotherapy is the treatment o choice.
clavicle, and scapula are also commonly a ected.
3. Compression o the cord may also occur and the Suggested Readings
neurological def cits are sometimes the presenting
symptoms. More commonly presentation is with A onso PD, Almeida A. Solitary plasmacytoma o the spine:
unremitting back pain or pathological ractures. Up to an unusual presentation. AJNR Am J Neuroradiol. 2010
50% o solitary plasmacytomas have been shown to Jan;31(1):E5.
evolve into multiple myeloma within 10 years (average Dimopoulos MA, Moulopoulos LA, Maniatis A, Alexanian
3.5 years) rom the initial diagnosis. R. Solitary plasmacytoma o bone and asymptomatic
multiple myeloma. Blood. 2000 Sep 15;96(6):2037-2044.
4. The lesion appears as a purely lytic lesion with a narrow
zone o transition on plain f lms. MRI is the imaging
modality o choice and demonstrates a ocal area o
bone marrow replacement. Signal characteristics are
not specif c since the lesion appears hypointense on T1-
weighted and hyperintense on T2-weighted and STIR
sequences. An extraosseous so t-tissue component may
be demonstrated and possible impingement on the cord
or spinal nerves should be evaluated.
5. Targeted radiotherapy is the treatment o choice in
solitary plasmacytoma o bone and o ten relieves the
associated symptoms.

112
Worsening back pain

1. What is the most likely diagnosis?

2. How does the condition usually present?

3. What may give rise to this condition?

4. Which imaging modality best images


this condition?

5. How many subtypes o this condition


are described?

113
Annular ssure 2951
Case ranking/dif culty: Category: Disc

Sagittal T2-weighted sequence of the lumbar spine showing Axial T2-weighted image at the level of L5-S1 intervertebral
a hyperintense linear focus (arrow) within the otherwise disc. There is a linear hyperintensity (arrow) in the posterolateral
hypointense annulus brosus. Multilevel disc desiccation is noted aspect of the annulus brosus, which is compatible with an
between L3 through to S1. annular ssure.

Answers Type 2 (radial tears)—f ssures that extend rom the


1. Annular f ssure re ers to the def ciency in the f bers o the periphery o the annulus f brosus to the nucleus pulposus,
annulus f brosus. The term f ssure is pre erred over tear which disrupt the longitudinal f bers. The abnormality is
as the latter may erroneously imply acute disc injury. seen as a hyperintense ocus on T2-weighted sequence.
Type 3 tears (transverse tears)—disruption o Sharpey
2. Annular f ssures are very common and most are f bers adjacent to their vertebral insertion. They are also
asymptomatic. However, ingrowth o nerve endings and seen as ocal T2 high signal intensity zones.
ormation o granulation tissue within the de ect may
result in pain. This occurs particularly when f ssures
occur close to dorsal root ganglia.
Pearls
3. Disruptions may be due to separation between the
• Annular f ssure re ers to def ciency in the f bers o the
annular f bers, avulsion o the annular f bers rom their
annulus f brosus.
vertebral insertions, and breaks that involve one or more
• The breaks may extend in a radial, transverse, or
layers o the annular lamellae. The breaks may extend in
concentric ashion.
a radial, transverse, or concentric ashion.
• Ingrowth o nerve endings and ormation o
4. They are seen as T2 hyperintense regions in the granulation tissue within the de ect may result in pain.
otherwise low signal annulus on MRI. Contrast
enhancement is seen in up to 96% o cases. Discography
can urther characterize annular f ssures and may Suggested Readings
distinguish a partial- rom a ull-thickness tear. The
clinical relevance o the latter is, however, disputed. RF Costello, DP Beall. Nomenclature and standard reporting
terminology o intervertebral disc herniation. Magn Reson
5. Yu et al described three types o annular f ssures Imaging Clin N Am. 2007;15:167-174.
as ollows: Yu S, Sether LA, Ho PS, Wagner M, Haughton VM. Tears
Type 1 (concentric tears)—rupture o the transverse o the anulus f brosus: correlation between MR and
f bers connecting adjacent annular lamellae. The pathologic f ndings in cadavers. AJNR Am J Neuroradiol.
longitudinal f bers are intact and these tears are not 1988;9:367-370.
visualized on MR.

114
Lower limb weakness and incontinence

1. What is the most likely diagnosis?

2. What should be included in the


di erential diagnosis?

3. Which primary tumors are o ten implicated?

4. Which imaging modality best images


this condition?

5. Which treatments may be benef cial in


this condition?

115
Leptomeningeal metastasis (lung adenocarcinoma) 2944
Case ranking/dif culty: Category: Nerve roots/Nerve plexus/Peripheral nerves

Sagittal T2-weighted sequence of the Contrast-enhanced T1-weighted Contrast-enhanced axial T1-weighted image at the
lumbar spine. Note subtle nodularity sequence of the lumbar spine. level of L2. Note pathological nodular enhancement
(arrow) along the cauda equina roots. There are enhancing nodules (arrow) along the cauda equina roots in this fat
The cord terminates at T12 and the (arrow) and di use leptomeningeal saturated sequence.
lumbar vertebrae have preserved enhancement (arrowhead) along
body heights and marrow signal. the cauda equina roots.

Answers 5. Treatment o ten involves ocal radiotherapy to


1. Leptomeningeal metastasis rarely complicates cancer symptomatic sites in combination with systemic
and re ers to the invasion and proli eration o neoplastic chemotherapy. Intrathecal chemotherapy requires
cells within the subarachnoid space. normal CSF ow dynamics. It is rarely e f cacious
except or treatment o hematopoietic neoplasms.
2. Di erential diagnoses o leptomeningeal enhancement Some studies have also demonstrated the benef cial
include in ectious, granulomatous, neoplastic, and use o intrathecal therapy in leptomeningeal spread
connective tissue disorders. complicating breast cancer.
3. The most common cancers to metastasize to the
leptomeninges are breast cancer, lung cancer, and
malignant melanoma. Leptomeningeal metastasis occurs Pearls
both with solid tumors when it is called leptomeningeal • Leptomeningeal metastasis re ers to the invasion
carcinomatosis and with nonsolid tumors when the and proli eration o neoplastic cells within the
condition is re erred to as tumor or lymphomatous/ subarachnoid space.
leukemic meningitis. • The most common cancers to metastasize to the
4. Contrast-enhanced MRI is the imaging investigation o leptomeninges are breast cancer, lung cancer, and
choice and demonstrates pathological leptomeningeal malignant melanoma.
enhancement, which extends into the cerebral sulci and • Contrast-enhanced MRI is the imaging investigation o
cerebellar olia or presents as enhancing subarachnoid choice and demonstrates pathological leptomeningeal
tumor nodules. Multi ocal or di use inf ltration in a enhancement.
sheet-like ashion can be seen along the spinal cord and
cauda equina roots. Associated f ndings in the spine
include cord enlargement, intraparenchymal nodules, Suggested Readings
and epidural compression. Lumbar puncture is the Bruna J, Simó M, Velasco R. Leptomeningeal metastases.
standard diagnostic procedure. Multiple punctures may Curr Treat Options Neurol. 2012 Aug;14(4):402-415.
be required as positive cytology is only demonstrated in Chamberlain MC. Leptomeningeal metastasis. Curr Opin
50%-70% on initial sampling. The CSF opening pressure Neurol. 2009 Dec;22(6):665-674.
and CSF protein may also be increased.

116
Worsening back pain

1. What is this procedure called?

2. Which approaches are commonly used?

3. What are the potential benef ts o the


coaxial technique?

4. Name some recognized complications o this


procedure.

5. Name some contraindications to this procedure.

117
Vertebral biopsy 2943
Case ranking/dif culty: Category: Vertebral body

Axial CT image with the patient in the prone Percutaneous biopsy of L3 vertebral body via a transpedicular approach. A
position showing a destructive lesion centered in 14G biopsy needle was advanced until it reached the periosteum (arrow,
the body of L3 vertebra and extending into the panel A). It was then advanced along the pedicle (arrow, panel B) into the
right pedicle. central part of the vertebral body.

Answers 5. Vertebral biopsy is contraindicated in patients with


1. Vertebral biopsy is indicated when a def nite diagnosis uncorrected bleeding diathesis.
cannot be made on neuroimaging and when histological
evaluation o a vertebral lesion is required. Open
biopsy is associated with signif cant morbidity and Pearls
complications. On the other hand, percutaneous vertebral
biopsy has been shown to be a sa e and highly accurate • Percutaneous vertebral biopsy has been shown to be a
procedure. It can be per ormed under intravenous sa e and highly accurate procedure.
sedation and local anesthesia. General anesthetic may be • It can be per ormed under intravenous sedation and
used in patients with severe back pain. local anesthesia. CT-guided vertebral biopsy has a
reported accuracy o 67%-97% with a complication
2. Three approaches are commonly used—the rate o 0%-26%.
posterolateral, transpedicular, and transcostotransversal— • Fluoroscopic guidance may also be used where rapid
the latter being used in the thoracic region. The choice CT scanning is not available.
o the approach o ten depends on the location o the • Complications o percutaneous vertebral biopsy
lesion. The posterolateral approach is o ten used i include in ections, pulmonary and neurologic
the lesion is located in the lower part o the vertebra complications.
or i it is predominantly located in the disc space. The • Three approaches are commonly used—
transpedicular approach was shown to be more e ective the posterolateral, transpedicular, and
i the pedicle and the posterior hal o the vertebral body transcostotransversal—the latter being used in the
were involved. The accuracy o vertebral biopsies using thoracic region.
the transpedicular approach was shown to be higher • The coaxial technique is pre erred as it o ten allows
compared to that o the posterolateral approach. intact specimens o adequate size to be obtained.
3. The coaxial technique is pre erred as it o ten allows • Vertebral biopsy is contraindicated in patients with
intact specimens o adequate size to be obtained. It also uncorrected bleeding diathesis.
enables multiple biopsies to be obtained through a single
tract, increasing the diagnostic accuracy. Bleeding at the
biopsy site may also be controlled by the insertion o Suggested Readings
Gel oam through the cannula.
Pierot L, Boulin A. Percutaneous biopsy o the thoracic and
4. CT-guided vertebral biopsy has a reported accuracy lumbar spine: transpedicular approach under uoroscopic
o 67%-97% with a complication rate o 0%-26%. guidance. AJNR Am J Neuroradiol. 1999 Jan;20(1):23-25.
Fluoroscopic guidance may also be used where rapid CT Ya e D, Greenberg G, Leitner J, Gipstein R, Shapiro M,
scanning is not available. Complications o percutaneous Bachar GN. CT-guided percutaneous biopsy o thoracic
vertebral biopsy include in ections, pulmonary and and lumbar spine: a new coaxial technique. AJNR Am J
neurologic complications. Neuroradiol. 2003 Nov-Dec;24(10):2111-2113.
118
Incidental nding

1. What is the likely diagnosis?

2. The lesion most commonly occurs at


what level?

3. The lesion is typically located proximal to the


dorsal root ganglion. True or False?

4. What is the typical appearance on conventional


or CT myelography?

5. Nerve f bers are usually within the walls o


these lesions. True or False?

119
Tarlov cyst 2921
Case ranking/dif culty: Category: Thecal sac

Osseous remodeling. Similar ndings. Nerve bers are within the walls, but di cult
to appreciate.

Answers
1. The presence o a cerebrospinal uid (CSF) f lled lesion Pearls
in the sacral canal with remodeling o bone is diagnostic
• Tarlov cysts are very common, a ecting 5%-10% o
or a Tarlov cyst.
the general population.
2. The vast majority o Tarlov cysts occur at the S1-4 level, • Most are asymptomatic, but a large size (greater than
although they uncommonly occur elsewhere in the spine. 1.5 cm) may result in symptoms.
This helps di erentiate rom meningeal diverticulae, • Symptoms include loss o bladder, bowel and sexual
which usually occur in the thoracic spine. unction, chronic back pain, and sensory def cit.
• Lesions are most commonly at the S1-4 level, and are
3. False. Tarlov cysts occur at or distal to the junction o
requently multiple.
the posterior nerve root and dorsal root ganglia unlike
• CT and MRI will show a uid-f lled structure with bone
meningeal diverticulae, which occur proximally.
remodeling in some cases. The nerve roots are in the
4. Most Tarlov cysts have a potential or limited cyst wall and sometimes within the cyst cavity. These
communication with the subarachnoid space. There ore, cysts can extend into the oramina and cause expansion.
with oil-based contrast myelographic contrast material • Cysts occur distal to the dorsal root ganglia, unlike
that has been used in the past, there may be no f lling or arachnoid diverticulae
delayed f lling o the cyst. The delayed f lling is due to
exudation though the cyst wall, or via CSF hydrostatic
pressure. This f nding is less pronounced when using
water-soluble contrast media. Suggested Readings
Meningeal diverticulae, with their larger communication, Paulsen RD, Call GA, Murtagh FR. Prevalence and
usually f ll immediately. percutaneous drainage o cysts o the sacral nerve root
sheath (Tarlov cysts). AJNR Am J Neuroradiol. 1994
5. True. Nerve f bers are usually within the walls o Feb;15(2):293-297; discussion 298-299.
Tarlov cysts, and sometimes within the cyst cavity Sen RK, Goyal T, Tripathy SK, Chakraborty S. Tarlov cysts:
itsel , helping to di erentiate them rom meningeal a report o two cases. J Orthop Surg (Hong Kong). 2012
diverticulae, which have no neural elements. Apr;20(1):87-89.
Xu J, Sun Y, Huang X, Luan W. Management o symptomatic
sacral perineural cysts. PLoS One. 2012 Jul;7(6):e39958.

120
Generalized bone pain

1. What are the imaging eatures o this entity?

2. What are the complications o this disease?

3. This disease is caused by clonal proli eration


o which type o cell?

4. What are risk actors or this condition?

5. What are the possible treatments?

121
Multiple myeloma 2920
Case ranking/dif culty: Category: More than one category

Di use osteopenia, with multiple focal areas Heterogenous T1 signal. Signal is not signi cantly increased on
of osteolysis. Collapse of a thoracic vertebra, T2-weighted images.
with mild wedging of several others.

Answers Pearls
1. MRI is the most sensitive test or myeloma. Marrow • Multiple myeloma is the most common primary bone
inf ltration by myeloma results in low T1 marrow signal, tumor. It must always be considered in the di erential
although the STIR signal can be variable. diagnosis o multiple lytic lesions, especially in the
Pathological ractures result rom the weakening o bone appropriate age group (over 50 years).
due to lytic lesions. • A plasmacytoma is a ocal collection o myelomatous
cells and may represent early stage disease, with a high
Myeloma has typically low uptake on radionuclide
incidence o conversion to di use multiple myeloma.
scans; however, a lesion may show increased uptake in
It has a better prognosis than multiple myeloma.
the presence o a pathological racture.
• A skeletal survey is o ten per ormed to stage the
2. Myeloma does not cause thrombocytosis. As a consequence extent o the disease.
o marrow inf ltration by plasma cells, thrombocyte • MRI is, however, the gold standard where resources
production is reduced, leading to thrombocytopenia. permit, as the ull extent o marrow replacement can
Secondary amyloidosis can cause e ects in multiple be appreciated, as well as any associated so t tissue
organs including liver, kidneys, and lungs. components.
Cord compression can result rom spinal deposits with • Imaging eatures consist o di use lytic lesions on
epidural so t tissue masses. Pathological ractures are plain f lms and di use marrow inf ltration with low T1
relatively common. signal lesions on MRI.
• The MRI signal on T2-weighted images is variable and
3. Monoclonal proli eration o plasma cells results in
should not be relied upon exclusively or diagnosis.
multiple myeloma. The immunoglobulin light chains are
• Similarly lesions can be “warm” or “hot” on a bone
detected in the serum and urine (Bence-Jones protein),
scan in the presence o a racture.
which orms the basis o laboratory diagnosis.
• Complications o myeloma include pathological
4. There are no def nite risk actors or multiple myeloma, racture; hence, it is as important to highlight to the
although there is a very slightly higher incidence in clinician sites prone to racture, as it is to comment on
f rst-degree relatives, and the disease is more common in the presence o established ractures.
blacks. There is no known genetic predisposition, with
mutations acquired, rather than hereditary. Hence, it is
extremely rare below 40 years o age. Suggested Readings

5. Stem cell transplant o ers the best chance o disease Li SD, Wang YF, Qi JY, Qiu LG. Clinical eatures o bone
remission, although it can o ten not be tolerated in the complications and prognostic value o bone lesions
elderly population that tend to have the disease. There detected by X-ray skeletal survey in previously untreated
is no possibility o cure with surgery, as the disease patients with multiple myeloma. Indian J Hematol Blood
by def nition is a systemic disorder o plasma cell Transfus. 2010 Sep;26(3):83-88.
monoclonal proli eration. The complications o bone pain, Sedlic A, Chingkoe C, Lee KW, Duddalwar VA, Chang SD.
hypercalcemia, and pathological racture can be treated. Abdominal extraosseous lesions o multiple myeloma:
imaging f ndings. Can Assoc Radiol J. 2012 Mar;65(1):2-8.
122
Motor vehicle accident with neurologic symptoms

1. What are the major radiologic f ndings?

2. What is the likely mechanism o injury?

3. Which part o the spine is most commonly


a ected?

4. With regard to hyperextension and


hyper exion injuries, which are more stable?

5. When neurologic compromise occurs, what are


the usual symptoms?

123
Extension teardrop fracture 2919
Case ranking/dif culty: Category: Vertebral body

Extension teardrop fracture at C3, with Fracture can be di cult to appreciate on axial Prevertebral soft tissue swelling.
mild retrolisthesis. images, especially if the orientation of the axial
slices is suboptimal.

Answers
• Fractures are more stable than their hyper exion
1. There is an extension teardrop racture at the anterior
counterparts in the lower cervical spine.
in erior C3 body, with prevertebral so t tissue
• Cord compression, when it occurs, is due to
swelling and mild retrolisthesis o C3 resulting in cord
retrolisthesis or compression rom the hyperkyphotic
compression.
vertebral segment. The anterior longitudinal ligament
2. The most likely mechanism, in an upper vertebral body (ALL) avulses a triangular ragment o bone rom the
corner racture in an older patient, is hyperextension. anterior in erior vertebral body. The ragment height is
equal to or greater than its transverse dimension.
3. The upper cervical spine, especially C2, is most
• The acet and interspinous distances may be widened.
commonly a ected in hyperextension injuries.
• Prevertebral so t tissue swelling is usually present, but
Hyper exion injuries predominate in the lower cervical
may be minimal or absent in older patients.
spine.
• Treatment is usually with cervical orthosis, but
4. Hyperextension injuries are more stable. Hyper exion cervical usion is indicated in some cases.
injuries, which predominate in the lower cervical spine,
are more unstable and o ten result in cord compression.
5. The cord compression and edema classically results in Suggested Readings
an anterior cord syndrome, with quadriplegia and loss
Rao SK, Wasyliw C, Nunez DB. Spectrum o imaging
o pain, touch and temperature but with preservation o
f ndings in hyperextension injuries o the neck.
posterior column unction such as proprioception and
Radiographics. 2005;25(5):1239-1254.
vibration.
Torretti JA, Sengupta DK. Cervical spine trauma. Indian J
Orthop. 2007 Oct;41(4):255-267.

Pearls
• Extension teardrop ractures usually occurs in older
patients with osteoporotic bone.
• C2 and C3 are most o ten a ected. Hyper exion
injuries predominate in the lower cervical spine.

124
Back pain

1. What are the radiographic f ndings?

2. What is the di erential diagnosis, and what is


the most likely diagnosis?

3. What are the major orms o this condition,


using the American College o Rheumatology
(ACR) criteria?

4. The pauciarticular type o this condition


typically a ects which joints?

5. In this case, what is the typical range o


HLA-B27 positivity?

125
Juvenile idiopathic arthritis 2917
Case ranking/dif culty: Category: More than one category

Sacroiliac joint, vertebral and posterior element fusion. Posterior element fusion.

Answers
1. There is ankylosis o the cervical spine with tarsal ankylosis. • The systemic orm is characterized by mainly systemic
Although the predental space can be widened and the dens symptoms, with arthralgia, pleuritis, and pericarditis
eroded, these eatures are not present in this case. among a range o other symptoms.
2. Extensive bony ankylosis can be seen in juvenile • So t tissue swelling, e usions, and periosteal reaction
idiopathic arthritis (JIA) and ankylosing spondylitis. This are typical. Hyperemia may result in overgrowth o the
case demonstrates extensive spinal involvement that can epiphyses.
be seen in both entities, and along with SI joint usion, can • Bony ankylosis is characteristic and may a ect the small
make the diagnosis di f cult. The presence o extensive and large joints. Ankylosis is not typically a eature o
tarsal ankylosis, however, suggests JIA as the diagnosis. adult rheumatoid arthritis, except in the cervical spine.
• Cervical spine involvement in JIA is more common
3. The three major types o JIA recognized by the ACR are than lumbar and thoracic spine. In these patients,
the polyarticular, pauciarticular, and systemic types. HLA-B27 is positive in up to 80% o cases, and SI
4. The pauciarticular type typically a ects the large joints joint usion is also common. There is o ten a amily
such as the knees and ankles. The polyarticular type history o spondyloarthropathy.
a ects the small joints o the hands and eet. • Cervical spine eatures include acet erosions and
ankylosis, predental space widening, dens erosion, and
5. HLA-B27 is requently positive in patients with late atlantoaxial subluxation.
onset pauciarticular JIA, with the development o • Ultrasound and MRI are use ul in evaluating early
enthesitis and sacroiliitis. HLA-B27 is positive in peripheral changes, and monitoring the response to
approximately 80% o these cases. therapy.

Pearls
Suggested Readings
• Juvenile idiopathic arthritis has three main orms:
polyarticular, pauciarticular, and systemic. Johnson K. Imaging o juvenile idiopathic arthritis. Pediatr
• Polyarticular orm a ects the small joints o the hands Radiol. 2006 Aug;36(8):743-758.
and eet. Johnson K, Wittkop B, Haigh F, Ryder C, Gardner-Medwin
• Pauciarticular orm a ects larger joints such as knees JM. The early magnetic resonance imaging eatures o
and ankles. the knee in juvenile idiopathic arthritis. Clin Radiol. 2002
Jun;57(6):466-471.
126
Short stature

1. What are the f ndings on the radiographs?

2. What is the most likely diagnosis?

3. What orm o dwarf sm is demonstrated in this


condition?

4. What is the inheritance pattern?

5. What are the main causes o mortality in these


patients?

127
Achondroplasia 2916
Case ranking/dif culty: Category: Vertebral body

The pedicles are short and broad. Short broad metacarpals and phalanges, with a trident hand.

Answers
1. Narrowed interpedicular distance, “Champagne” glass Pearls
pelvis, horizontal sacrum, and acetabulae with short, • Achondroplasia is a proximal (rhizomelic) dwarf sm.
broad emoral heads and necks. • Endochondral ossif cation is a ected, with normal
2. The f ndings are consistent with achondroplasia. periosteal and membranous bone growth.
• A ected bones are short with normal thickness.
3. Achondroplasia is a disproportionate rhizomelic • There is metaphyseal aring with a ball in socket
(proximal) dwarf sm. epiphysis.
4. 85% o cases are a result o a spontaneous genetic • There are also trident hands, which are f ngers o equal
mutation, and 15% are due to an autosomal dominant length that are widely opposed
inheritance. • The vertebrae are bullet shaped with posterior
scalloping and spinal stenosis. Brainstem compression
5. In patients under 4 years, brainstem compression is the may occur.
leading cause o death. Between 5 and 24 years central • A “champagne” glass pelvis is seen.
nervous system and respiratory causes predominate, and • The oramen magnum is narrowed, which can lead to
between 25 and 54 years cardiovascular complications brainstem compression.
are the leading cause o death. • Mortality is increased, usually due to brainstem
compression in young patients, and respiratory and
cardiovascular complications in older patients.

Suggested Readings
Kao SC, Waziri MH, Smith WL, Sato Y, Yuh WT, Franken
EA. MR imaging o the craniovertebral junction, cranium,
and brain in children with achondroplasia. AJR Am J
Roentgenol. 1989 Sep;153(3):565-569.
Song HR, Choonia AT, Hong SJ, Lee SH, Suh SW, Cha
IH, Park JT. Rotational prof le o the lower extremity in
achondroplasia: computed tomographic examination o 25
patients. Skeletal Radiol. 2006 Dec;35(12):929-934.
128
Chronic back pain

1. What are the salient MRI f ndings?

2. What is the diagnosis?

3. What are contributory actors to this entity?

4. The condition is most commonly seen in the


lumbar spine. True or False?

5. Surgical excision o the spinous processes is


curative. True or False?

129
Baastrup phenomenon 2915
Case ranking/dif culty: Category: Posterior elements

Note interspinous bursitis Another patient with a compression fracture and loss of Note the enlarged spinous
(arrowheads). vertebral height. The extent to which associated vertebral processes.
and disc disease is contributory is uncertain.

Answers
1. The spinous processes are enlarged with interspinous Pearls
bursitis. Although degenerative disc disease with height • Baastrup phenomenon is a condition where the
loss is a requent association, it is not present in this case. spinous processes o the spine (usually the lumbar)
become enlarged, at, and sclerotic and are in contact
2. The f ndings are compatible with a diagnosis o Baastrup with each other.
phenomenon. • The contact between the spinous processes is
3. Baastrup phenomenon is associated with advancing accentuated in extension.
age, with a prevalence o more than 80% above age 80. • Adventitial bursae orm between the spinous
Degenerative disc disease or vertebral height loss are processes, and are readily detected on MRI.
requent associations; however, the condition may be • Associated degenerative disc disease with loss o disc
seen when the rest o the spine is entirely normal. The height is requent. The degree to which Baastrup is
contact between the spinous processes is accentuated in contributory to back pain is there ore uncertain.
extension. • The incidence increases with advancing age, with a
prevalence o more than 80% over age 80.
4. True. Baastrup phenomena is almost always seen in • Treatment is typically conservative.
the lumbar spine, although the thoracic spine may be
a ected very in requently.
5. False. Although there are undoubtedly nociceptors
Suggested Readings
ormed at the spinous processes o these patients, the
result o surgical excision o the spinous processes has Kwong Y, Rao N, Latie K. MDCT f ndings in Baastrup
been disappointing. There ore, ailure to address other disease: disease or normal eature o the aging spine? AJR
causes o back pain in these individuals may result in Am J Roentgenol. 2011 May;196(5):1156-1159.
ongoing pain. Maes R, Morrison WB, Parker L, Schweitzer ME, Carrino
JA. Lumbar interspinous bursitis (Baastrup disease)
in a symptomatic population: prevalence on magnetic
resonance imaging. Spine (Phila Pa 1976). 2008
Apr;33(7):E211-E215.

130
Chronic low back pain

1. What is this procedure called?

2. Name some indications or this procedure.

3. Name some contraindications to the procedure.

4. Which drugs are used or inf ltration?

5. Are there any recognized complications o this


procedure?

131
Facet joint injection 2937
Case ranking/dif culty: Category: Posterior elements

The procedure is per ormed under imaging guidance


using uoroscopy, computed tomography, ultrasound, or
MR guidance, which conf rms the needle placement and
increases the precision o the procedure.
4. The procedure is per ormed under local anesthetic using
strict aseptic precautions. The joint is inf ltrated with a
long-acting steroid depot preparation and a long-acting
local anesthetic agent via a 23G spinal needle inserted
percutaneously. The injected anesthetic agent acts on
the nociceptive f bers within the synovium, whereas the
corticosteroids reduce synovial in ammation.
The position o the needle tip may be conf rmed by
injecting contrast within the capsule prior to injection
o the mixture, although studies have shown that precise
Axial CT image with the A small amount of contrast intra-articular injection is not required.
patient in the prone position. injection con rmed the 5. Complications are very rare and include septic arthritis,
Bilateral percutaneous needle intracapsular position of
spondylodiscitis, and allergy to the injectates. The outcome
placement into L4-5 facet the needle tips (arrow and
o acet joint injection is variable and some patients may
joints. The tip of the rst arrowhead). The joints were
needle is already placed within then in ltrated using 20 mg not achieve the desired benef t. In patients who experience
the capsule (arrow) and the of Depo-Medrol and 0.5 mL of symptom relie , the therapeutic benef t may be short lived
second needle (arrowhead) is 0.5% Bupivacaine under strict and may necessitate repeated injections.
being advanced to reach the aseptic technique. The capacity o the normal acet joint is about 2 mL, and
left sided L4-5 facet joint. care must be taken when injecting the joint not to cause
capsular disruption and spillage o the injected mixture
Answers
into the epidural space and adjacent so t tissues.
1. Facet joint injection is a spinal interventional procedure
used or both treatment and diagnosis o radicular pain
syndromes. Pearls
The procedure has a high diagnostic accuracy and • Facet joint injection is used or both treatment and
reproducibility. The long-term clinical outcome is, diagnosis o radicular pain syndromes.
however, variable. • The joint is inf ltrated with a long-acting steroid depot
Facet joints are a common cause o chronic pain preparation and a long-acting local anesthetic agent
and were proven to be responsible or up to 45% o via a spinal needle inserted percutaneously.
lower back pains. Facet joints are synovial joints and • Complications are rare and include septic arthritis,
in ammation or trauma may lead to pain ul back spondylodiscitis, and allergy to the injectates.
movement. This may initiate a vicious cycle o physical
deconditioning with irritation o the acet nerves and
muscular spasm. The joints are richly innervated by the
dorsal ramus o spinal nerves. Suggested Readings
Boyajian SS. Using image-guided techniques or
2. Indications or acet joint inf ltrations are multiple and
chronic low back pain. J Am Osteopath Assoc. 2007
include clinical suspicion o acet syndrome, chronic low
Nov;107(10 suppl 6):ES3-ES9.
back pain with negative radiology, tenderness over the
Peh W. Image-guided acet joint injection. Biomed Imaging
acet joints, and persistent low back pain postsurgery.
Interv J. 2011 Jan-Mar;7(1).
3. Although there are no absolute contraindications, the
procedure is o ten avoided in patients with bleeding
diathesis, allergy to contrast or to the injectates, in ection
particularly i a ecting the puncture site, and pregnancy.

132
Chronic back pain

1. What are the major radiographic f ndings?

2. What determines the severity o the condition?

3. What are the types o vertebral osteoporotic


compression ractures?

4. What are some o the radiographic pit alls


when diagnosing these ractures?

5. When is vertebral augmentation therapy


indicated?

133
Osteoporotic compression fractures 2914
Case ranking/dif culty: Category: Vertebral body

Pearls
• Osteoporotic vertebral compression ractures are
extremely common, occurring in up to 50% o the US
population a ter 50 years o age.
• Radiologic signs o osteoporosis include loss o bone
density, cortical thinning, and accentuation o the
vertical trabeculae.
• Fracture patterns include wedge racture, biconcave
racture, and crush ractures. Severity is determined by
the degree o height loss:
• Mild: 20%-25%
• Moderate: 26%-40%
• Severe: 40%
Similar ndings in the thoracic spine.
• Acute ractures can demonstrate a cortical break and
trabecular impaction, but these are o ten di f cult to
see especially on radiographs.
Answers • Fractures are most common in the lower thoracic
1. The bones are di usely osteopenic, with cortical and upper lumbar spine (T12-L1), where a kyphotic
thinning and accentuated vertical trabeculae. Multiple de ormity may occur.
biconcave and wedge de ormities are seen. Minimal • Radiologic pit alls include simulation o endplate
retropulsion is seen at T12 and L2. depression by an oblique lateral projection, scoliosis
and the presence o multiple Schmorl nodes.
2. Although many actors are taken into consideration
when determining severity, osteoporotic vertebral
ractures are generally classif ed as mild, moderate, or
severe depending on the degree o height loss. Suggested Readings
Kondo KL. Osteoporotic vertebral compression ractures
3. Fractures are classif ed depending on which part o the
and vertebral augmentation. Semin Intervent Radiol. 2008
vertebral body is involved: wedge (anterior), biconcave
Dec;25(4):413-424.
(middle), or crush (posterior).
Lenchik L, Rogers LF, Delmas PD, Genant HK. Diagnosis o
4. Several entities may mimic osteoporotic compression osteoporotic vertebral ractures: importance o recognition
ractures. These include a lateral f lm that is not a and description by radiologists. AJR Am J Roentgenol.
true lateral, ie, an oblique f lm, scoliosis, cupid’s bow 2004 Oct;183(4):949-958.
(a developmental variant), limbus vertebra, multiple
Schmorl nodes (Scheuermann disease), and H-shaped
vertebrae in sickle cell disease.
5. Failure to respond to medical therapy and intractable
pain are the most important reasons or vertebral
augmentation. These procedures include vertebroplasty
where cement (polymethylmethacrylate or PMMA)
is injected into the vertebral body percutaneously, or
kyphoplasty where there is balloon in ation to restore
vertebral height prior to injection o the cement.
Kyphoplasties are per ormed in patients with vertebral
ractures and associated kyphosis.
Complications include those related to needle placement,
iatrogenic ractures, cement extravasation, in ection, and
bleeding.

134
Motor vehicle accident

1. What are the major radiographic f ndings?

2. What is the diagnosis?

3. MRI is usually indicated in the workup o


these individual. True or False?

4. What is the most commonly a ected level?

5. Which patients are prone to this injury?

135
Chance fracture 2913
Case ranking/dif culty: Category: More than one category

Horizontal component Posterior element Fracture well demonstrated.


well demonstrated. involvement.

Answers
1. There is anterior compression o the T11 vertebral body, Pearls
with a horizontal split racture involving the body and • Chance ractures are a result o a exion distraction
extending into the posterior elements. injury.
• The use o lap seatbelts alone is implicated. With
2. The f ndings are compatible with a Chance racture.
the advent o combined lap and shoulder belts, the
3. False. Neurological compromise is rare, and MRI is only incidence is alling.
indicated in the patient with unexplained neurological • The thoracolumbar junction, especially L2, is most
symptoms or in the polytrauma patient. Findings may commonly a ected.
include contusion o the conus medullaris, or cauda • Associated abdominal injuries are common, occurring
equina compression. in up to 50% o the pediatric population.
• Concomitant neurological compromise is rare.
4. The most commonly a ected level is L2. In children,
• Treatment is usually conservative with reduction in
where the center o gravity is lower, the racture is
extension, and placement o a cast. However, surgical
usually in the midlumbar spine.
usion may be indicated where body habitus precludes
5. Patients using lap seatbelts alone in a motor vehicle adequate reduction, or in the polytrauma patient.
accident are prone to this racture. Fall rom a height is
another common mechanism. In addition, those with a
rigid spine, eg, patients with ankylosing spondylitis are
more susceptible to a Chance racture. Suggested Readings
Bernstein MP, Mirvis SE, Shanmuganathan K. Chance-type
ractures o the thoracolumbar spine: imaging analysis in 53
patients. AJR Am J Roentgenol. 2006 Oct;187(4):859-868.
Da ner RH. Chance racture o the upper thoracic spine.
AJR Am J Roentgenol. 2005 Aug;185(2):555.
Davis JM, Beall DP, Lastine C, Sweet C, Wol J, Wu D.
Chance racture o the upper thoracic spine. AJR Am J
Roentgenol. 2004 Nov;183(5):1475-1478.

136
Neck pain

1. What is the major radiographic f nding?

2. What is the diagnosis?

3. What measurement typically constitutes


longitudinal instability?

4. What measurement typically constitutes


rotational instability?

5. At what level in the pediatric cervical spine is


anterolisthesis considered normal?

137
Ligamentous instability 2909
Case ranking/dif culty: Category: More than one category

Pearls
• Ligamentous instability o the spine has numerous
etiologies, most commonly traumatic, degenerative, or
related to ligamentous laxity.
• Instability can be longitudinal or rotational.
• In the cervical spine, anterior translation o more
than 3.5 mm between exion and extension implies
longitudinal instability.
• Rotational instability is implied i sagittal rotation is
greater than 11°.
• White and Panjabi proposed a checklist system or
each region (cervical, thoracic, and lumbar).
• White and Panjabi criteria:
• Anterior elements = 2
• Posterior elements = 2
Treated with an anterior cervical fusion. • Sagittal translation 3.5 mm or 20% o vertebra = 2
• Sagittal rotation greater than 11° = 2
• Positive stretch test = 2
Answers • Cord damage = 2
1. There is signif cant anterolisthesis o C4 on C5 on • Root damage = 1
exion, as well as an increase in angulation. • Abnormal disc narrowing = 1
• Dangerous loads anticipated = 1
2. The f ndings are consistent with ligamentous instability. • Unstable i total score greater than 5.
3. Longitudinal instability in the cervical spine is implied i • Denis proposed a three-column concept or the spine
the total translational motion exceeds 3.5 mm. The total (anterior, middle, and posterior), with injury to two or
translational motion includes adding any retrolisthesis on more columns implying instability.
extension to the anterolisthesis on exion. • Treatment is usually with surgical usion.

4. Sagittal rotational instability re ers to the angular change


at the level o the disc on exion and extension, and is
considered abnormal i greater than 11°. Suggested Readings
Denis F. Spinal instability as def ned by the three-column
5. There can be up to 3 mm o anterolisthesis o C2 on
spine concept in acute spinal trauma. Clin Orthop Relat
C3 in the pediatric cervical spine (patients less than
Res. 1984 Oct;65-76(189):65-76.
8 years old) as a normal f nding, due to ligamentous
White AA, Panjabi MM. Clinical Biomechanics of the Spine.
laxity and more horizontal acets. Care must be taken
2nd ed. Philadelphia, PA: Lippincott; 1990:30-342.
when evaluating the pediatric trauma patient, as
placement on a standard trauma board may cause the
neck to ex due to the large head size, accentuating this
pseudosubluxation. A trauma board that allows slight
extension should be used.
Pseudosubluxation may also occur to a lesser extent
at C3-4.

138
Pain in neck after gardening

1. What is the main radiographic f nding?

2. What is the double spinous process (or ghost)


sign?

3. What is the proposed mechanism o injury?

4. Which vertebral level is most commonly


a ected?

5. The injury is unstable. True or False?

139
Clay shoveler fracture 2908
Case ranking/dif culty: Category: Vertebral body

“Double spinous process” sign at C7. Displaced C7 spinous process fracture.

Answers
1. There is a displaced C7 spinous process racture, a “clay Pearls
shoveler” racture. • Clay shoveler racture was originally described in clay
shovelers in Australia in the 1930s.
2. The displaced racture ragment and the native intact
• It is an avulsion racture o the spinous process o C7,
base give the appearance o two spinous processes at the
and less commonly C6 or T1.
a ected level on the AP f lm. There are several described
• The supraspinous ligament is believed to be
“ghost” signs, eg, meniscal radial tear, osteomyelitis o
responsible or the avulsion injury.
the Charcot oot etc, but there is only one described in
• AP f lm may show a “ghost” or “double spinous
the spine!
process” sign, representing the racture ragment and
3. This racture is believed to be an avulsion racture o the the intact base.
spinous process by the power ul supraspinous ligament. • Lateral f lm shows that the racture ragment is
in eriorly displaced.
4. C7 is most commonly a ected, although C6 and T1 are
• Treatment is conservative with immobilization using
not uncommon. Other levels are much less common.
an orthotic device.
5. False. This avulsion racture a ects only the posterior
column o the spine, and is considered a stable racture.
Suggested Reading
Tehranzadeh J, Bonk RT, Ansari A, Mesgarzadeh M. E f cacy
o limited CT or nonvisualized lower cervical spine
in patients with blunt trauma. Skeletal Radiol. 1994
Jul;23(5):349-352.

140
Back pain and sti ness

1. What are classical eatures o this condition?

2. Which diseases are associated with HLA-B27


positivity?

3. What are the possible treatments o this


condition?

4. What are the local and systemic complications


o this condition?

5. Which groups o patients have the highest


levels o HLA- B27 concordance?

141
Ankylosing spondylitis 2905
Case ranking/dif culty: Category: Vertebral body

“Bamboo” spine. Subsequent fracture through the rigid immobile spine. Another patient with Romanus lesions.

Answers
1. Entheseal in ammation is the basic pathology o • Due to the high relative incidence o the condition
ankylosing spondylitis. Atlantoaxial subluxation is a eature relative to other causes o sacroiliitis, unilateral or
o rheumatoid arthritis. Sacroiliitis is usually bilateral and asymmetric sacroiliitis is still more likely to be due to
classically symmetric, although requently asymmetric: i ankylosing spondylitis than due to other disorders.
sacroiliitis is unilateral, in ection must always be excluded. • Classical eatures include syndesmophytes, Romanus
Peripheral joint ankylosis is not a eature. lesions, squaring o vertebral bodies, Andersson
lesions, and bamboo spine.
2. Seronegative spondyloarthropathies include ankylosing • Syndesmophyte ormation is due to ossif cation o the
spondylitis, psoriasis, in ammatory bowel disease, and outer f bers o the annulus f brosus. It is best assessed
reactive disease. on a rontal radiograph.
3. Treatment or ankylosing spondylitis is generally • Chance ractures can occur through ossif ed
directed at symptom control. Surgical correction o a intervertebral discs with relatively little trauma, as
kyphotic de ormity is not generally per ormed. Novel these are the sites o mechanical weakness.
approaches include anti-TNF therapy, which is intended • Early spinal eatures include in ammation at
to treat active in ammation mediated by TNF-alpha costovertebral and costotransverse joints, which is best
cytokines. assessed on a uid-sensitive MRI sequence.
• It is important to look or extraarticular eatures and
4. Ankylosing spondylitis can predispose to spinal racture associated eatures o HLA-B27 disorders, eg, apical
and cord compression. Plantar ascitis is another lung f brosis and enthesitis. Also in ammatory bowel
association. disease and psoriasis may be di f cult to di erentiate,
Only 1% o cases are associated with an apical lung so eatures o these conditions should be sought a ter.
f brosis.
5. Over 90% o Caucasians with ankylosing spondylitis are
Suggested Readings
HLA-B27 positive. The male: emale ratio is 3:1.
Braun J, Baraliakos X. Imaging o axial spondyloarthritis
including ankylosing spondylitis. Ann Rheum Dis. 2011
Mar;(70 suppl 1):i97-i103.
Pearls Ostergaard M, Poggenborg RP, Axelsen MB, Pedersen PJ.
• Bilateral symmetrical sacroiliitis is highly suggestive Magnetic resonance imaging in spondyloarthritis—how
o ankylosing spondylitis. to quanti y f ndings and measure response. Best Pract Res
Clin Rheumatol. 2010 Oct;24(5):637-657.
142
History of chronic skin condition with back pain

1. What are the radiographic f ndings?

2. What is the most likely diagnosis?

3. What are other spinal eatures o this entity?

4. What proportion o patients with cutaneous


disease have an in ammatory arthritis?

5. What are the possible treatments o this


condition?

143
Psoriatic spondylitis 2904
Case ranking/dif culty: Category: More than one category

Erosions in the anterior Right erosive sacroiliitis. No de nite erosive changes in Bulky asymmetric lateral
vertebral body in the left SI joint; however, MRI would be more sensitive syndesmophyte.
another patient. in detecting early changes.

Pearls
• Psoriatic arthritis is one o the most common
Answers spondyloarthropathies, a ter primary ankylosing
spondylitis.
1. There are bulky asymmetric syndesmophytes, with • It may have many orms, but the most common
erosions in the right sacroiliac joint consistent with mani estations are an asymmetric arthritis a ecting
sacroiliitis. a ew joints, and a seronegative symmetric arthritis,
2. Bulky and asymmetric syndesmophytes, ie, paraspinal clinically indistinguishable rom rheumatoid arthritis.
ossif cation that ows rom one vertebral body to the • The most severe orm o peripheral psoriatic arthritis
next (unlike the thin symmetric annular ossif cation seen is termed arthritis mutilans. Characteristic eatures o
in ankylosing spondylitis), along with an asymmetric arthritis mutilans are severe erosions in a “pencil-in-
sacroiliitis, is characteristic o the seronegative cup” pattern, “sausage digits,” and a distribution more
spondyloarthropathies. The presence o a chronic skin distal than a typical rheumatoid pattern.
condition suggests psoriasis is the diagnosis. • Enthesitis (eg, Achilles) and dactylitis are
characteristic o psoriatic arthritis.
3. Along with thick asymmetric syndesmophytes and • In the spine, it is characterized by bulky asymmetrical
an asymmetric sacroiliitis, psoriatic spondylitis may syndesmophytes unlike the smooth thin symmetric
mani est as vertebral erosions and periostitis with syndesmophytes o ankylosing spondylitis.
eventual ankylosis. The ankylosed spine is prone to • Erosions and periostitis in the vertebral bodies may be
ractures. seen.
4. Psoriatic arthritis is a relatively common generalized • There is o ten a bilateral asymmetric sacroiliitis.
mani estation o psoriasis, occurring in up to one- • Patients are requently HLA-B27 positive.
third o patients with the skin condition. It is possible,
however, to have arthritis as the f rst presentation o the Suggested Readings
disease, prior to the development o any skin lesions.
Approximately 20% o psoriatic arthritis patients will Amrami KK. Imaging o the seronegative
develop a spondylitis. spondyloarthopathies. Radiol Clin North Am. 2012
Jul;50(4):841-854.
5. Methotrexate and azathioprine are classic disease- Paparo F, Revelli M, Semprini A, et al. Seronegative
modi ying steroid-sparing antirheumatic drugs spondyloarthropathies: what radiologists should know.
(DMARDs). Intra-articular injections have the benef t o Radiol Med. 2013 Nov.
avoiding or minimizing severe systemic corticosteroid Sudoł-Szopinska I, Urbanik A. Diagnostic imaging
side e ects. Surgery is reserved or severe unctionally o sacroiliac joints and the spine in the course o
limiting cases. spondyloarthropathies. Pol J Radiol. 2013 Apr;78(2):43-49.

144
Chronic medical condition on treatment

1. What is the major vertebral f nding?

2. What is the di erential diagnosis?

3. What is the presumed etiology?

4. What appearance may be seen in a


radionuclide bone scan?

5. The changes are decreased a ter renal


transplantation. True or False?

145
Renal osteodystrophy: rugger jersey spine 2898
Case ranking/dif culty: Category: Vertebral body

4. A “superscan” appearance with increased bone uptake


and so t tissue uptake (in extraskeletal calcif cation), but
no renal excretion o radiopharmaceutical can be seen
in renal osteodystrophy. The so t tissue uptake will help
di erentiate this superscan appearance rom generalized
metastatic disease.
5. False. The osseous changes o renal osteodystrophy are
o ten paradoxically increased a ter renal transplantation.

Pearls
• A rugger jersey spine appearance is seen in patients
with renal osteodystrophy. It re ers to the appearance
o sclerotic endplates with central lucency.
• A similar appearance can be seen in Paget disease or
osteopetrosis (sandwich vertebrae).
• The etiology is poorly understood, but there is usually
Sclerosis of the endplates of numerous vertebral bodies. an increase in cancellous bone in these patients, as
well as an increase in number and thickness o the
trabeculae within the cancellous bone.
Answers • Bones with a high concentration o cancellous bone,
1. Alternating sclerotic endplates with more lucent central such as the vertebrae, are there ore o ten a ected.
vertebral bodies is consistent with a “rugger jersey • Deposition o amorphous calcium phosphate rather
spine” appearance, typical o renal osteodystrophy. than calcium hydroxyapatite in bone may also be
contributory.
2. A “sandwich vertebra” appearance is seen in • Changes o renal osteodystrophy are typically seen
osteopetrosis, and can have a similar appearance; when more than 50% o renal unction is lost.
however, the age group and medical history will help
di erentiate. Paget disease can result in a “picture–
rame” appearance; however, the disease is usually not
generalized, and the bones are typically expanded. Suggested Readings
Metastatic disease shows generalized osteosclerosis. Beladi Mousavi. Renal bone disease among patients with
Fluorosis and myelof brosis can have a similar ESRD. Nephrourol Mon. 2013 Jul;5(3):849-850.
appearance, but there is usually vertebral/paraspinal Liu Y. Super-superscan on a bone scintigraphy. Clin Nucl
calcif cation/ossif cation in uorosis, and the spleen is Med. 2011 Mar;36(3):227-228.
enlarged in myelof brosis. Wittenberg A. The rugger jersey spine sign. Radiology. 2004
Feb;230(2):491-492.
3. An increase in the amount o cancellous bone, and an
increase in the thickness and number o trabeculae are
typically seen. There is also increased deposition o
amorphous calcium phosphate.

146
Motor vehicle accident

1. What are the imaging f ndings?

2. What is the diagnosis?

3. What is the typical mechanism o injury?

4. This injury is more common in young children.


True or False?

5. Neurological compromise is very common.


True or False?

147
Hangman fracture 2895
Case ranking/dif culty: Category: Vertebral body

Fracture of the pars C2 pars fracture well Bilateral C2 fractures, extending


interarticularis of C2. demonstrated. into the foramen transversarium
bilaterally.

Answers
1. There are bilateral ractures o the pars inter-articularis • Anterolisthesis o C2 on C3 and pre-vertebral so t
o C2, with prevertebral so t tissue swelling and mild tissue swelling are associated eatures. CT is indicated
anterolisthesis o C2 on C3. to assess the integrity o the oramen transversarium.
• I the racture extends into the oramina, CT
2. Bilateral pars or pedicle ractures o C2 is coined a angiography is indicated to assess the condition o the
“Hangman” racture. vertebral arteries.
3. Extension and distraction are the typical mechanism o • MRI is mandatory in any patient with neurologic
injury in “Hangman” ractures. compromise. The so t tissues, discs, and spinal cord
are readily assessed.
4. False. Injury in children under 8 years is uncommon • Neurological def cit is not as common as one would
because ractures pre erentially occur through the expect because o the autodecompression o the spinal
un used synchondrosis at the base o the dens. This canal that occurs in bilateral pars interarticularis or
synchondrosis usually uses at 6-7 years. pedicle ractures.
5. False. The autodecompression o the spinal canal that • Treatment is usually with a Halo-vest, and in some
occurs with bilateral C2 ractures reduces the incidence instances, with spinal usion. When the mechanism
o spinal cord injury. involves extension and distraction, as in most cases,
cervical traction should be avoided.

Pearls
Suggested Readings
• Hangman racture is a racture, typically bilateral, o
Clark CR, Igram CM, el-Khoury GY, Ehara S. Radiographic
the pedicles or pars interarticularis o C2.
evaluation o cervical spine injuries. Spine (Phila Pa
• The injury is uncommon in children under 8 years,
1976). 1988 Jul;13(7):742-747.
as ractures pre erentially occur through the un used
Hua Q, Ma WH, Zhao LJ, Fang Y. [Clinical application o
synchondrosis at the base o the dens.
multi-spiral CT thinner scanning and reconstruction in
• Up to 33% o patients have additional cervical spine
the diagnosis o atlantoaxial racture and dislocation].
injury and 10% additional nonspinal injury.
Zhongguo Gu Shang. 2009 May;22(5):349-352.
• Cranio acial and vertebral artery injuries are quite
common associated injuries.

148
Fall from a height

1. What are the major imaging f ndings?

2. What is the diagnosis at L1?

3. What is the most common location or these


injuries?

4. What is the management or compression


ractures o the spine?

5. Retropulsion into the spinal canal always


requires surgical intervention. True or False?

149
Burst fracture 2894
Case ranking/dif culty: Category: Vertebral body

Marked comminution of the L1 vertebral body Similar ndings. Multiple vertebral fractures, with the
with retropulsion, similar changes at L4 and L5. retropulsion at L1 causing compression and
The L1 fracture involves all three columns. edema in the conus medullaris.

Answers
1. There are multiple thoracic and lumbar compression Pearls
and burst ractures, with retropulsion and spinal canal • Burst ractures occur a ter a all rom a height or
compromise seen at L1 and L4. motor vehicle accident.
• The mechanism o injury is axial loading, o ten with a
2. Involvement o all three columns is seen; there ore, this is
exion component.
a burst racture. Compression ractures involve only the
• A compression racture involves only the anterior
anterior column; burst ractures involve the anterior and
column; however, a burst racture involves the anterior
middle columns and sometimes the posterior column.
and middle columns and sometimes all three columns.
3. Most vertebral ractures occur at the thoracolumbar • I there is neurologic compromise, more than 40%
junction because this is the mechanical transition point spinal canal compromise or greater than 25% kyphotic
between the rigid thoracic spine and the more mobile de ormity, or greater than 50% vertebral height loss,
lumbar spine. T5-T8 is also a common location or burst the racture will require surgical intervention.
ractures. • I the degree o spinal canal compromise is less than
40%, remodeling and reabsorption o the bony ragment
4. Compression ractures only involve the anterior
occur and the patient can be treated with an orthosis.
column, unlike burst ractures. There ore, the posterior
• Follow-up radiographs should always be done in the
ligamentous complex is typically intact and the racture
standing position to assess or progression o kyphosis.
is deemed stable. However, i there is more than 50%
anterior vertebral height loss or greater than 25%
kyphosis, the posterior ligamentous complex is likely to
be compromised and surgery is indicated. Suggested Readings
Atlas SW, Regenbogen V, Rogers LF, Kim KS. The
5. False. I the retropulsed ragment involves less than
radiographic characterization o burst ractures o the
40% o the spinal canal and there is no neurologic
spine. AJR Am J Roentgenol. 1986 Sep;147(3):575-582.
compromise, then reabsorption and remodeling o up to
Da ner RH, Deeb ZL, Roth us WE. The posterior vertebral
50% o the bone ragment may occur, and the patient can
body line: importance in the detection o burst ractures.
be treated conservatively.
AJR Am J Roentgenol. 1987 Jan;148(1):93-96.
Shuman WP, Rogers JV, Sickler ME, et al. Thoracolumbar
burst ractures: CT dimensions o the spinal canal relative
to postsurgical improvement. AJR Am J Roentgenol. 1985
Aug;145(2):337-341.

150
Incidental nding

1. What is the name given to the plain


radiographic appearance?

2. What is the likely diagnosis?

3. The lesions are never purely sclerotic. True or


False?

4. What is the typical scintigraphic appearance?

5. What are the expected angiographic f ndings?

151
Hemangioma 2886
Case ranking/dif culty: Category: Vertebral body

Increased fat within Increased fat and coarse striations T2 hyperintensity. Coarse vertical “Polka dot” appearance on CT
the T10 vertebral body well appreciated. No expansion or trabeculae give a “polka dot” (di erent patient).
(arrowhead), with posterior element extension. appearance.
coarse striations but no
bony expansion. Similar
smaller lesion at T8.
Pearls
Answers • Vertebral hemangiomas are extremely common,
1. The rein orced vertical trabeculae secondary to occurring in 10% o the adult population in one
the resorbed horizontal trabeculae give the classic cadaver study. The most common location is the
“corduroy” appearance. thoracic spine, ollowed by the lumbar spine.
• Plain radiographs show coarsened vertical trabeculae,
2. Although lymphangioma, metastasis, and in ection are in giving a corduroy appearance.
the di erential, the appearance is classic or a vertebral • Axial CT scan shows the rein orced vertical trabeculae
hemangioma. en ace, leading to a polka dot appearance.
3. False. Vertebral hemangiomas may appear sclerotic with • T1 signal intensity is variable depending on the amount
coarse vertical striations, giving an “ivory vertebra” o adipose tissue. The T2 signal is usually increased.
appearance. The epithelioid subtype o hemangioma is Lesions typically enhance a ter intravenous contrast.
typically lucent, although there are rare reported cases o • Most lesions are asymptomatic. Symptomatic lesions
di use osteosclerosis resembling metastases, lymphoma, may be a result o bony expansion and posterior
or chronic in ection. element involvement, with extension into the epidural
space. Increased distention rom activity, pregnancy,
4. The scintigraphic appearance is variable, with classically or menstruation may also lead to symptoms.
a photopenic lesion on Tc-99m MDP bone scanning.
Lesions can, however, show normal or even increased
activity especially i there is an associated pathologic
Suggested Readings
racture.
Friedman DP. Symptomatic vertebral hemangiomas: MR
Tc-99m RBC scan will show increased uptake.
f ndings. AJR Am J Roentgenol. 1996 Aug;167(2):359-364.
5. Angiography will show increased vascularity, and is Park HJ, Jeon YH, Rho MH, et al. Incidental f ndings o
usually per ormed in conjunction with embolization o the lumbar spine at MRI during herniated intervertebral
symptomatic lesions, prior to surgical removal. disk disease evaluation. AJR Am J Roentgenol. 2011
May;196(5):1151-1155.
Rodallec MH, Feydy A, Larousserie F, et al. Diagnostic
imaging o solitary tumors o the spine: what to do and
say. Radiographics. 2008 Jul-Aug;28(4):1019-1041.
152
Back pain for several months

1. What is the most likely diagnosis in this


patient?

2. What is the most appropriate initial


management?

3. Which solid organ malignancies may present


with this appearance?

4. What is the appropriate treatment or this


condition?

5. What conditions can have an identical


appearance?

153
Osteoblastic metastases 2885
Case ranking/dif culty: Category: Vertebral body

Numerous sclerotic Similar ndings. Numerous sclerotic Multiple foci of increased


vertebral lesions, vertebral lesions with radiopharmaceutical
with “ivory” vertebra “ivory” vertebrae uptake.
(arrowhead). (arrowheads).

Answers
1. Osteoblastic metastases. The common sclerotic Pearls
metastases are rom prostate, breast, colorectal, and lung • Generalized osteosclerosis in an adult is o ten
carcinoma. There is no real di erential in an elderly secondary to metastatic disease. In a male, this is
patient with a rising PSA other than prostatic carcinoma. usually rom prostate carcinoma. In a emale, this is
usually rom breast carcinoma.
2. A bone scan with urological re erral is most appropriate
• Other causes are rarer and are suggested by other
or a patient with di use sclerotic lesions and a rising
imaging eatures, eg, uorosis—enthesopathy; sickle
PSA. A technetium bone scan is cheap and very sensitive
cell disease—H-shaped vertebrae; osteopetrosis—age
in the detection o osteoblastic metastases. Other
and Erlenmeyer ask de ormity.
modalities, eg, SPECT or PET-CT, would be needlessly
• The axial skeleton is involved f rst with prostatic
expensive when the diagnosis is in little doubt.
metastases.
3. Breast, prostate, and colorectal carcinoma are the most • Progressive sclerosis is not easy to monitor on imaging
common osteoblastic skeletal metastases. Lung and renal studies when comparison o treatment response is
cell metastases are typically lytic. sought. For this purpose, serum PSA is an appropriate
test, as changes rom the baseline can be used to better
4. Metastatic prostate cancer could be appropriately
monitor the response to therapy.
managed with palliation, radiotherapy or local
• Bone scans, plain radiographs, CT scans, and MRI
or skeletal complications, hormonal therapy, or
scans can all be used to demonstrate the extent o
chemotherapy. However, radical curative surgery is not
disease. Technetium-99 radionuclide scans are still f rst-
appropriate.
line investigations or mapping the extent o disease,
5. Breast and prostatic carcinoma metastases can have although they may lack specif city, being positive in
identical appearances. Other causes or di use sclerosis, other instances, eg, post-trauma and in ection.
eg, sickle cell disease, may look similar, although
the patient demographics would be very di erent.
Osteopetrosis has a more uni ormly sclerotic appearance Suggested Readings
and is very rare in comparison. Feydy A, Carlier R, Vallée C, et al. [Imaging o osteosclerotic
metastases]. J Radiol. 1995 Sep;76(9):561-572.
Hackländer T, Scharwächter C, Golz R, Mertens H. [Value
o di usion-weighted imaging or diagnosing vertebral
metastases due to prostate cancer in comparison to other
primary tumors]. Rofo. 2006 Apr;178(4):416-424.

154
Spina bi da, status post myelomeningocele repair

1. What is the most likely diagnosis?

2. What are associated maternal conditions?

3. What germ cell layers are involved?

4. What are associated cranial and brain


mal ormations?

5. What are treatment options?

155
Chiari II malformation 2600
Case ranking/dif culty: Category: More than one category

Sagittal T2 image demonstrates low-lying cerebellar tissue with Sagittal T2 image demonstrates low-lying conus with complex
small cervicothoracic syrinx. brotic mass, consistent with postoperative change following
myelomeningocele repair.

Answers
1. In this patient with a known history o Pearls
myelomeningocele repair, the only di erential • Chiari II mal ormation is universally present in
possibility is Chiari II mal ormation. patients with myelomeningoceles.
• Imaging o the brain and neuroaxis is necessary to
2. Folate def ciency or de ects in processing o olate can
document all associated abnormalities. Supratentorial
lead to Chiari II mal ormation. Folate supplementation
abnormalities include dysgenesis o the corpus
is recommended prior to conception, through at least
callosum, towering cerebellum, and interdigitation o
6 weeks o gestational age.
the gyri.
3. Chiari II mal ormations involve abnormalities o the
mesoderm (cranium) and neuroectoderm (both supra-
and in ratentorial).
Suggested Readings
4. Clivus shortening, beaking o the tectum, McLone DG, Dias MS. The Chiari II mal ormation: cause
cervicomedullary kinking, dysgenesis o the corpus and impact. Childs Nerv Syst. 2003;19:540-550.
callosum, and a small posterior ossa may be seen in Stevenson KL. Chiari type 2 mal ormation: past, present, and
conjunction with Chiari II mal ormation. uture. Neurosurg Focus. 2004;16:Article 5.
5. All patients undergo myelomeningocele repair;
others may require additional treatment, such as
suboccipital craniectomy and ventriculoperitoneal or
syringosubarachnoid shunt placement.

156
Left leg numbness

1. What should be included in the di erential


diagnosis?

2. What imaging f ndings can be seen in these


patients?

3. What is the most common level where this


occurs?

4. What are common presenting symptoms?

5. What are treatment options?

157
Conjoined nerve root 2599
Case ranking/dif culty: Category: Nerve roots/Nerve plexus/Peripheral nerves

4. Radicular symptoms and pain are the most common


presenting symptoms. Conjoined nerve roots are at a
higher risk or compression in the lateral recess. Patients
may also present with back pain.
5. Traditional diskectomy is o ten not su f cient in the
setting o conjoined nerve roots and increased exposure
via hemilaminectomy, oraminotomy, or pediculectomy
may be required. Increased traction is contraindicated as
conjoined nerve roots are o ten f xed in place and may
result in dural leak or nerve root avulsion.

Pearls
• Conjoined nerve roots are relatively common
abnormalities, which are requently under-recognized.
• Conjoined nerve roots most commonly occur in the
lumbar spine.
• Conjoined nerve roots can be clinically signif cant
in the setting o radiculopathy resulting rom
compression.
Coronal T1 image demonstrates a common origin of the left L5 • To identi y conjoined nerve roots, search or
and S1 nerve roots. the common nerve origin or enlargement o the
surrounding nerve root sheath.

Answers
1. Nerve sheath tumor, synovial cyst, conjoined nerve root, Suggested Readings
and disc herniation can be included in the di erential
diagnosis. Scuderi GJ, Vaccaro AR, Brusovanik GV, Kwon BK,
Berta SC. Conjoined lumbar nerve roots: a requently
2. Imaging f ndings can include enlargement o the neural underappreciated congenital abnormality. J Spinal Disord
oramen, so t tissue within the lateral recess, hypoplastic Tech. 2004;17:86-93.
pedicles, and enlargement o the nerve sleeve. Spinal Trimba R, Spivak JM, Bendo JA. Conjoined nerve roots o
stenosis can be associated with conjoined nerve roots, the lumbar spine. Spine J. 2012;12:515-524.
particularly in the setting o hypoplastic pedicles.
3. Conjoined nerve roots most commonly occur at the
L5-S1 level.

158
One year status post lumbar fusion with draining wound

1. What should be included in the di erential


diagnosis?

2. What are common presenting symptoms?

3. What are risk actors?

4. What are the common organisms that are


involved?

5. What are the treatment options?

159
Postoperative infection lumbar spine 2240
Case ranking/dif culty: Category: Hardware/Lines/Tubes

4. The most common are Staphylococcus (both aureus and


epidermidis) and Enterococcus.
5. Spinal in ection is treated with antibiotics and
consideration o percutaneous abscess drainage, open
debridement, and hardware removal.

Pearls
• Postoperative in ections are relatively uncommon in
the setting o spinal instrumentation; however, they
can occur over 2 years ollowing surgery.
• Recent studies have shown obesity to be an
independent risk actor or the development o
in ection.
• Clinical eatures that should raise concern or in ection
include cutaneous redness, pus drainage, or pain.
• Evaluation may include CT or CT myelography to
evaluate or intraspinal extension. MRI may be used
Axial CT image demonstrates postoperative changes of posterior but can be limited by adjacent metal arti act.
spinal fusion from L3 to S1 with complex surrounding uid
• Superf cial in ections can be treated medically;
collection and draining tract extending to posterior midline
incision.
however, extension below the ascia may necessitate
surgical drainage/debridement and, in some cases,
removal o in ected hardware. This can be problematic
Answers in patients who may now have spinal instability.
1. Focal so t tissue collection and gas can be seen in the
setting o recent operative procedure, diskitis, cellulitis,
and osteomyelitis. Suggested Readings
2. Common presenting symptoms include ever, pain, Pull ter Gunne A, Cohen DB. Incidence, prevalence, and
cutaneous erythema, and incisional pus. Radicular analysis o risk actors or surgical site in ection ollowing
symptoms may result rom intraspinal extension o adult spinal surgery. Spine. 2009;34:1422-1428.
in ection. Young PM, Berquist TH, Bancro t LW, Peterson JJ.
3. Risk actors or developing postoperative in ection Complications o spinal instrumentation. Radiographics.
include diabetes, previous in ection, and blood loss 2007;27:775-789.
greater than 1 L. Anterior spinal approach decreases the
risk o in ection; multilevel usion may increase the risk
as it increased operative time.

160
Neck pain

1. What should be included in the di erential


diagnosis?

2. What is the most common intramedullary


spinal cord tumor in an adult?

3. What are common presenting symptoms?

4. What is the most common location within the


cord?

5. What are the treatment options?

161
Glioblastoma multiforme, cervical cord 2239
Case ranking/dif culty: Category: Spinal cord

Sagittal T2 image demonstrates expansion and T2 hyperintensity Axial T1 image following contrast administration demonstrates
within the spinal cord at the cervicothoracic junction. focal enhancement in the left cord.

Answers
1. T2 hyperintense expansion o the cord with ocal Pearls
enhancement can be caused by astrocytoma, • Spinal cord astrocytomas are the second most common
hemangioblastoma, ependymoma, demyelinating intramedullary neoplasm, but the most common in
disease, or metastasis. An atypical cord contusion could children.
also present with a similar appearance. • They tend to be low grade (WHO I-II); however,
the rare high-grade tumors (WHO III-IV) have an
2. Ependymomas are the most common intramedullary
extremely poor prognosis.
tumor in adults; astrocytomas are the most common
• Patients generally present with nonspecif c symptoms
intramedullary tumor in children.
o pain or myelopathy. Treatment is directed a ter
3. Myelopathy is the most common presenting symptom; tissue diagnosis is made.
other symptoms include pain, radicular symptoms, • O tentimes, the lower-grade tumors are subject to
sensory dysesthesia, and bowel/bladder incontinence. serial ollow-up i the patient does not have symptoms.
• High-grade lesions are generally treated with
4. The cervical spine is the most common site o most
chemotherapy and radiation with resection reserved
primary intramedullary tumors.
or unctional decline. Despite treatment, the survival
5. Biopsy is the f rst step to determine tumor pathology. is generally around 5 years.
For low-grade tumors, conservative management may be
appropriate. In higher-grade tumors, chemotherapy and
radiation have been shown to prolong survival. Resection
Suggested Readings
is reserved or patients with progressive neurological
symptoms. Cohen AR, Wiso JH, Allen JC, Epstein F. Malignant
astrocytomas o the spinal cord. J Neurosurg. 1989:70:50-54.
Liu X, Germin BI, Ekholm S. A case o cervical spinal cord
gliobastoma diagnosed with MR di usion tensor and
per usion imaging. J Neuroimaging. 2011;21:292-296.

162
Back pain

1. What should be included in the di erential


diagnosis?

2. What are common presenting symptoms?

3. What are the most common organisms in


children?

4. What are the most common organisms in


adults?

5. What are the treatment options?

163
Bacterial spinal meningitis 2185
Case ranking/dif culty: Category: More than one category

Sagittal T1 image following gadolinium administration Axial T1 image following gadolinium administration
demonstrates di use dural enhancement with enhancement demonstrates enhancement along the nerve roots (red arrow) and
along the conus and nerve roots. Note epidural abscess and ventral epidural abscess (blue arrow).
enhancement of L4 vertebral body as well.

Answers
• Organisms are generally spread hematogenously;
1. Etiologies or abnormal spinal enhancement include
however, contiguous spread or direct inoculation may
bacterial meningitis, idiopathic hypertrophic
be seen.
pachymeningitis, leptomeningeal metastasis, Guillain
• Imaging demonstrates di use dural and
Barre, and viral meningitis.
pachymeningeal enhancement.
2. Common presenting symptoms include back pain, ever, • Lumbar puncture is necessary to isolate an organism
and headache. and di erentiate other causes o enhancement
(metastatic disease, viral in ection, etc).
3. Group B Streptococcus predominates in neonates with
• Antibiotic and supportive therapy must be initiated as
Haemophilus in older children. Other etiologies include
rapidly as possible, even be ore the per ormance o
Listeria, Neisseria meningitides, and Streptococcus
lumbar puncture.
pneumoniae.
4. Staphylococcus is also seen in adult patients.
5. Antibiotics and com ort measures are the mainstays Suggested Readings
o treatment. While lumbar puncture is required or Morris BJ, Fletcher N, Davis RA, Mencio GA. Bacterial
diagnosis, there is no benef t or therapeutic uid removal. meningitis a ter traumatic thoracic racture-dislocation:
two case reports and review o the literature. J Orthop
Trauma. 2010;24:e49-e53.
Pearls Pai S, Welsh CT, Patel S, Rumboldt Z. Idiopathic hypertrophic
spinal pachymeningitis: report o two cases with typical
• Spinal meningitis o ten presents with neck pain and MR imaging f ndings. AJNR. 2007;28:590-592.
sti ness, particularly in the setting o ever and headache.
• Patients may rapidly progress to mental status changes
and seizures.

164
Back pain following posterior fusion

1. What should be included in the di erential


diagnosis?

2. What are risk actors?

3. What associated abnormalities are seen?

4. What are common presenting symptoms?

5. What are the treatment options?

165
Sacral insu ciency fracture 2218
Case ranking/dif culty: Category: Vertebral body

4. Back, buttock, and groin pain, as well as palpable


tenderness and radiculopathy, are all potential presenting
symptoms; however, pain is most common.
5. Treatment options include analgesia, conservative
management, physical therapy, and sacroplasty. Surgical
f xation is generally not employed given the underlying
osteoporosis.

Pearls
• Sacral insu f ciency ractures are important to
recognize as they have a signif cant association with
increased morbidity and mortality.
• It is important to evaluate or associated injuries,
such as vertebral body compression ractures and hip
ractures.
• The classic distribution o insu f ciency ractures is the
H-shaped ractures with vertical components through
the sacral ala in conjunction with a transverse racture
across the sacral body. However, any variation o this
conf guration may be seen.
Coronal CT image demonstrates vertically oriented right sacral
• Treatment is generally conservative with immobility
sclerosis, surrounding a linear lucency.
and pain management, progressing to physical therapy
and treatment o cause or osteopenia.
Answers • Sacroplasty may be per ormed or pain management.
1. Mixed lucent and sclerotic sacral lesions include
insu f ciency racture, metastasis, osteomyelitis,
sacroiliitis, and traumatic racture. Suggested Readings
2. Osteoporosis, hyperparathyroidism, multiple myeloma, Lyders EM, Whitlow CT, Baker MD, Morris PP. Imaging
Paget disease, renal osteodystrophy, and rheumatoid and treatment o sacral insu f ciency ractures. AJNR.
arthritis all increase the risk o developing sacral 2010;31:201-210.
insu f ciency ractures. In addition, sacral insu f ciency Odate S, Shikata J, Kimura H, Soeda T. Sacral racture
ractures may also occur ollowing hip arthroplasty as a ter instrumented lumbosacral usion. Spine.
patients begin to bear weight. 2013;38:E223-E229.
3. Sacral insu f ciency ractures can be associated with
other osteoporotic ractures, including emur and
vertebral body compression ractures.

166
Neonate with increased abdominal girth

1. What should be included in the di erential


diagnosis?

2. What are associated clinical syndromes?

3. What are common presenting symptoms?

4. What are the components o the Evans


anatomic staging system?

5. What are the treatment options?

167
Neuroblastoma 2215
Case ranking/dif culty: Category: Paraspinal so t tissue

Coronal CT image demonstrates partially calci ed (arrowhead) Axial T2 weighted image post-primary tumor resection
right paraspinal mass. demonstrates extension through the right T9-T10 neural foramen.

Answers
1. Etiologies or a paraspinal mass in a child • There are several clinical syndromes that patients
include neuroblastic tumor, adrenal hemorrhage, may present with—or these lesions are occasionally
lymphadenopathy, lymphoma, and adrenocortical tumor. diagnosed antenatally with ultrasound.
2. Blueberry mu f n syndrome, Horner syndrome, Hutchinson • Treatment includes surgery, chemotherapy, and
syndrome, Kerner-Morrison syndrome, and Pepper radiation.
syndrome are all syndromes associated with neuroblastoma. • Prognosis is based on Evans anatomic staging system.
• Stages 1-3 are considered “locoregional” disease.
3. Diarrhea results rom increased VIP secretion (Kerner- (1—conf ned to organ; 2—extraorgan extension;
Morrison syndrome). Opsoclonus-myoclonus and ataxia 3—extraorgan extension, crossing midline).
result rom reaction o antineuroblastoma antibodies • Stage 4 is systemic or metastatic disease.
with cerebellar Purkinje cells. Myelopathy can occur • Stage 4S, which is seen in patients less than 12 months
rom intraspinal extension. at diagnosis, with metastatic disease limited to skin,
4. Stages 1-3 are considered “locoregional” disease. liver, and bone marrow. Stage 4S patients have an
(1—conf ned to organ; 2—extraorgan extension; excellent prognosis versus Stage 4 patients who have
3—extraorgan extension, crossing midline). Stage 4 is approximately 10% chance o survival.
systemic or metastatic disease. There is a subtype o • Neuroblastoma is along the spectrum o neuroblastic
Stage 4—Stage 4S—which is seen in patients less than tumors, which includes ganglioneuroma and
12 months at diagnosis with metastatic disease limited ganglioneuroblastoma (which are more benign).
to skin, liver, and bone marrow. Stage 4S patients have • Ganglioneuroma patients o ten have complete cure
an excellent prognosis versus Stage 4 patients who have ollowing resection.
approximately 10% chance o survival. • Ganglioneuroblastoma patients have a prognosis
5. Primary treatment is surgery, chemotherapy, and based on the percentage o ganglioneuroma and
radiation; steroids may be used to treat symptoms o neuroblastoma within their tumor.
adrenal insu f ciency. • There are cases o extension both ways along
the spectrum, with malignant degeneration and
spontaneous di erentiation.
Pearls
• Neuroblastoma is a common solid malignancy o
in ancy and childhood, with most cases presenting Suggested Readings
be ore the age o 5. Fisher JPH, Tweddle DA. Neonatal neuroblastoma. Semin
• The classic imaging appearance is a paraspinal mass, Fetal Neonatal Med. 2012;17:207-215.
o ten demonstrating extension into the spinal canal Wilne S, Walker D. Spine and spinal cord tumors in children:
through multiple neural oramina. a diagnostic and therapeutic challenge to healthcare
systems. Arch Dis Child Educ Pract Ed. 2010;95:47-54.
168
Prima gravida with abnormal screening ultrasound

1. What should be included in the di erential


diagnosis?

2. What are risk actors?

3. What associated abnormalities are seen?

4. What portion o the spine is most commonly


a ected?

5. What are the treatment options?

169
Spinal dysraphism, in utero 2214
Case ranking/dif culty: Category: More than one category

Sagittal T2 SSFSE image demonstrates an open neural tube defect Axial T2 SSFSE image demonstrates an open neural tube defect in
in the lumbosacral spine with herniation of meninges. the lumbosacral spine with herniation of meninges.

Answers
• There is an association with osseous spinal
1. Etiologies or a cystic lumbosacral lesion seen on
abnormalities as well.
prenatal ultrasound include anterior sacral meningocele,
• The incidence o open neural tube de ects has
closed spinal dysraphism, myelocele, myelocystocele,
decreased with olate supplementation.
and sacrococcygeal teratoma.
• Postnatal imaging is not routinely indicated with
2. Risk actors include Chiari II mal ormation, maternal surgical repair o the open neural tube de ect within
olate def ciency, trisomy 13, and trisomy 18. the f rst days o li e.
• Some centers per orm in utero repair, which may help
3. Associated abnormalities include club oot, dermal sinus,
decrease hydrocephalus in patients with Chiari II.
hydrocephalus, scoliosis, and syrinx.
• Per the MOMS (Management o Myelomeningocele
4. Nearly hal o myelomeningoceles are lumbosacral in Study) trial, the best chance or improved outcome
location. arises with repair prior to 25 weeks’ gestational age.
• Postoperative imaging ocuses on identi ying cases o
5. Primary myelomeningocele closure is the f rst treatment
retethering.
with untethering o spinal cord. Ventriculoperitoneal
shunt placement may be needed in the setting o
associated hydrocephalus.
Suggested Readings
Bulas D. Fetal evaluation o spine dysraphism. Pediatr
Pearls Radiol. 2010;40:1029-1037.
Peruzzi P, Corbitt RJ, Ra el C. Magnetic resonance imaging
• Meningomyelocele arises secondary to ailure o
versus ultrasonography or the in utero evaluation o
closure o the neural tube.
central nervous system anomalies. J Neurosurg Pediatrics.
• Meningomyelocele is o ten picked up on routine
2010;6:340-345.
screening ultrasound with additional imaging
per ormed using MRI.
• Evaluation should include images o the posterior
ossa and supratentorial compartment to evaluate
or associated Chiari II mal ormation and/or
hydrocephalus.

170
Neck pain and radicular symptoms

1. What should be included in the di erential


diagnosis?

2. What are common presenting symptoms?

3. What are common etiologies?

4. What are the common organisms that are


involved?

5. What are the treatment options?

171
Septic facet arthritis 2184
Case ranking/dif culty: Category: More than one category

Parasagittal T1 image following contrast administration Axial T1 image following contrast administration demonstrates
demonstrates enhancement surrounding the C3-C4 left facet joint. enhancement surrounding the C3-C4 left facet joint with epidural
extension.

Answers
• Classically, patients present with pain and symptoms
1. The di erential diagnosis should include metastatic
o in ection; however, there may be associated
disease, osteoarthritis, rheumatoid arthritis, and synovial
radicular or myelopathic symptoms depending on
cyst.
spread.
2. Fever and pain are the most common presenting • In addition to the f ndings o arthritis at the acet joint,
symptoms; variable radiculopathy and myelopathy can the presence o enhancement raises the question o
be seen depending on extent o in ection. in ection.
• It is important to evaluate or epidural and paraspinal
3. Hematogenous spread is the most common etiology.
extension.
Other etiologies include contiguous spread rom
• Treatment is usually medical; however, aggressive
meningitis, diskitis, or appendicitis, as well as direct
in ections may lead to osseous destruction and
inoculation.
instability requiring surgical intervention and
4. Staphylococcus aureus accounts or over 80% o cases. stabilization.
Streptococcus is the second most common organism.
5. These lesions are primarily treated medically with
analgesia, antibiotics, and/or percutaneous drainage o Suggested Readings
so t tissue abscess; surgical intervention is indicated in Jones JL, Ernst AA. Unusual cause o neck pain:
cases o spinal instability or spinal cord compromise. septic arthritis o a cervical acet. Am J Emer Med.
2012;30:2094.1-2094.4
Weingarten TN, Hooten WM, Huntoon MA. Septic acet
Pearls joint arthritis a ter a corticosteroid acet injection. Pain
Med. 2006;7:52-56.
• Facet joint septic arthritis is rare and may be the result
o hematogenous or contiguous spread. In addition, it
can rarely be seen a ter therapeutic injections.

172
Back pain

1. What should be included in the di erential


diagnosis?

2. What are common presenting symptoms?

3. What are the stages o this abnormality within


the cord?

4. What are treatment options or spinal


arachnoid cyst?

5. What are the treatment options or


myelomalacia?

173
Myelomalacia 2208
Case ranking/dif culty: Category: More than one category

4. Asymptomatic lesions do not require treatment.


Symptomatic lesions are pre erentially treated with
resection; however, i this is not possible, enestration
or marsupialization o the cyst can be per ormed.
Alternative options include placement o a cyst-
subarachnoid space shunt.
5. In chronic myelomalacia, conservative management and
physical therapy are the best treatment options. Surgery
is reserved or relieving compressive lesions.

Pearls
• Myelomalacia is a generic term that re ers to gliosis
within the spinal cord.
• It can be the result o external compression (as in this
case), secondary to trauma with cord contusion or the
result o a proximal lesion with subsequent Wallerian
degeneration.
• It is important to di erentiate high T2 signal rom
gliosis rom edema related to underlying lesion.
Myelomalacia will progress to volume loss in the
chronic phase.
• Patients will generally present with myelopathic
Sagittal T2 image demonstrates dorsal cystic lesion (cephalad symptoms related to the level o insult.
arrow) with compression of the spinal canal and compression and • Treatment is aimed at primary etiology, i possible, or
increased T2 signal within the spinal cord (caudal arrow). supportive.

Answers
Suggested Readings
1. Etiologies o intramedullary T2 hyperintensity include
astrocytoma, ependymoma, multiple sclerosis, acute Ergun T, Lakadamyali H. Multiple extradural spinal
disseminated encephalomyelitis, and Wallerian arachnoid cysts causing di use myelomalacia o the
degeneration rom a proximal lesion. spinal cord. Neurologist. 2009;15:347-350.
Kumar A, Sakia R, Singh K, Sharma V. Spinal arachnoid
2. Myelopathic symptoms predominate in presentation, cyst. J Clin Neurosci. 2011;18:1189-1192.
including distal weakness and numbness, and bowel and
bladder dys unction.
3. There are our stages o Wallerian degeneration:
Stage 1—axonal destruction
Stage 2—myelin protein breakdown
Stage 3—myelin lipid breakdown and gliosis
Stage 4—volume loss

174
Back pain

1. What should be included in the di erential


diagnosis?

2. What are common presenting symptoms?

3. What are associated complications?

4. What portion o the spine is most commonly


a ected?

5. What are the treatment options?

175
Paget disease (multifocal) 2203
Case ranking/dif culty: Category: Vertebral body

Lateral radiograph demonstrates sclerosis, expansion, cortical Whole-body bone scan demonstrates increased radiotracer
thickening and trabecular coarsening within the a ected uptake in the thoracolumbar spine, as well as femurs, pelvis, and
thoracolumbar vertebral bodies. humeral heads.

Answers
1. Etiologies or increased density within a vertebral
Pearls
body include metastasis, hemangioma, osteopetrosis, • Paget disease a ects up to 1 out o every 10 patients
lymphoma, and osteosarcoma. over 80 years o age with spinal involvement in up to
hal o all a ected patients.
2. Pain and pathologic racture are most common; however, • Patients most commonly present with asymmetric
osseous expansion can lead to scoliosis and radicular multi ocal involvement; however, occasionally there
symptoms. Myelopathy can occur secondary to cord may be a solitary site o involvement.
ischemia rom shunting. • In the spine, vertebral body involvement is the rule
3. Potential complications include basilar impression, with variable extension to the posterior elements.
compression racture, kyphosis, scoliosis, and • Radiographic presentations include the ivory vertebra
spondylosis. and the picture rame vertebra.
• Patients o ten present with pain or pathologic racture;
4. The lumbar spine is more commonly involved than the however, there may be serious sequelae secondary to
thoracic spine, which is more commonly involved than basilar invagination, cord ischemia, or compression.
the cervical spine. • Treatment is primarily medical, using bisphosphonates
5. Potential treatments include conservative management, and calcitonin; however, surgery may be used to
bisphosphonate therapy, calcitonin therapy, and relieve spinal cord compression or stabilize ractures.
arthroplasty. There is no role or radiation in the
treatment o Paget disease.
Suggested Readings
Cortis K, Micalle K, Mizzi A. Imaging Paget’s disease o
bone— rom head to toe. Clin Radiol. 2011;66:662-672.
Theodorou DJ, Theodorou SJ, Kakitsubata. Imaging o Paget
disease o bone and its musculoskeletal complications:
review. AJR. 2011;196:S64-S75.

176
Unrestrained passenger in single car accident, GCS 3 at the scene

1. What should be included in the di erential


diagnosis?

2. What is the def nition o Wachenheim line?

3. What is Powers ratio?

4. What underlying disorders are associated with


a higher incidence o this abnormality?

5. What are the treatment options?

177
Atlantooccipital dislocation 2200
Case ranking/dif culty: Category: Vertebral body

Lateral CT scout image demonstrates prevertebral soft tissue Sagittal CT reconstruction con rms atlantooccipital dislocation
swelling and inferior displacement of the dens. with inferior displacement of the dens. An additional fracture
fragment is noted cephalad to the C1 anterior arch.

Answers
1. Atlantooccipital dislocation is identif ed by disruption o
• Upper cervical spine injuries are particularly worrisome
Wachenheim line and increased atlantooccipital condyle
given the potential instability and associated cord injury.
distance. While commonly atal, this can be a survivable
• Injuries above the level o C3 are potentially lethal,
injury; however, death is usually secondary to brain stem
as the C3 through C5 spinal nerves give supply to the
injury.
phrenic nerve and innervate the diaphragm.
2. Wachenheim line is drawn along the dorsal sur ace o the • Imaging should be undertaken immediately upon arrival
clivus and should intersect the odontoid tip. It is used to while the patient is immobilized i clinical concern exists.
assess or anterior or posterior subluxation based on the • Prompt neurosurgical or orthopedic evaluation
position o the odontoid relative to the line. It is independent is recommended or consideration o surgical
o head positioning and does not vary based on patient age. intervention in unstable cases.
3. Powers ratio is used to determine anterior subluxation • Atlantooccipital dislocation can also be associated
and is def ned as the distance rom the basion to the with injury to the vertebral arteries at the skull base,
midvertical posterior lamina o the axis divided by the leading to vertebrobasilar in arction or insu f ciency.
distance rom the opisthion to the midvertical portion o Consideration o vascular imaging is recommended.
the anterior atlas ring (BC/OA). • Atlantooccipital dislocation is def ned using the
Powers ratio, which determine anterior subluxation
4. Down syndrome and rheumatoid arthritis have an
and is def ned as the distance rom the basion to the
increased incidence o atlantooccipital instability.
midvertical posterior lamina o the axis divided by the
5. Rigid cervical collar should be maintained until the distance rom the opisthion to the midvertical portion o
patient is taken to the operating room, or surgical the anterior atlas ring (BC/OA). Normal is less than 1.
f xation (generally rom occiput to C2) and halo • The basion is def ned as the midpoint anterior oramen
placement, which allows adjustment to obtain reduction. magnum; the opisthion is def ned as the midpoint
So t cervical collars do not provide enough stability and posterior oramen magnum.
traction is contraindicated as it may increase the risk o
vascular and spinal cord injury.
Suggested Readings
Bucholz RW, Burkhead WZ. The pathological anatomy o atal
Pearls atlanto-occipital dislocations. JBJS. 1979;61-A:248-250.
• Cervical spine injuries are commonly encountered in Watanabe M, Sakai D, Yamamoto Y, Sato M, Mochida
the setting o trauma. J. Upper cervical spine injuries: age-specif c clinical
eatures. J Orthop Sci. 2010;15:485-492.
178
Surveillance study performed for follow-up of anaplastic
medulloblastoma

1. What should be included in the di erential


diagnosis?

2. What are common presenting symptoms?

3. Early radiation changes can be seen how long


a ter radiation?

4. What is the threshold dose that leads to


permanent atty marrow replacement?

5. What are the treatment options?

179
Radiation changes 2197
Case ranking/dif culty: Category: More than one category

3. Early changes can be seen as early as 2 weeks ollowing


radiation.
4. Above doses o 50 Gy, reconversion to active marrow is
unlikely.
5. There is no indication to discontinue radiation as most
patients require no treatment. Vertebroplasty could be
considered in the setting o pathologic racture.

Pearls
• Spinal irradiation causes characteristic changes within
the vertebral bodies, with conversion o active marrow
to atty marrow.
• The key to diagnosis is recognizing the geographic
pattern o involvement.
• This is particularly apparent in pediatric patients,
but can be seen in adult patients, especially in those
Sagittal T2 image demonstrates geographic hyperintensity
patients who have reactivation o bone marrow.
throughout the posterior vertebral bodies and posterior
elements, consistent with marrow edema related to proton beam
• There may be long-term recovery o the marrow with
radiation. a reconversion to active marrow.

Answers
Suggested Readings
1. Focal lesions within the vertebral bodies include
hemangioma, in ection, metastasis, normal atty Hwang S, Le kowitz R, Landa J, et al. Local changes in bone
marrow, and primary bone tumor; however, the key to marrow at MRI a ter treatment o extremity so t tissue
diagnosis is the geographic distribution and T1 and T2 sarcoma. Skeletal Radiol. 2009;38:11-19.
hyperintensity. Stevens SK, Moore SG, Kaplan ID. Early and late bone-
marrow changes a ter irradiation: MR evaluation. AJR.
2. Fatty marrow replacement does not cause symptoms; 1990;154:745-750.
however, extensive conversion o active marrow could
produce anemia, leukopenia, and thrombocytopenia.
Pathologic ractures have been reported in vertebral
bodies that receive high doses.

180
Numbness and macrocytic anemia

1. What should be included in the di erential


diagnosis?

2. What are common presenting symptoms?

3. What etiologies can lead to the development o


this abnormality?

4. What is the prognosis?

5. What are the treatment options?

181
Subacute combined degeneration 2192
Case ranking/dif culty: Category: Spinal cord

4. Resolution o symptoms is related to severity and duration


o symptoms; the longer and worse the symptoms, the less
likely the complete resolution. Partial resolution ollowing
treatment is the general rule.
5. Parenteral B12 is the treatment, with treatment o
underlying cause i present.

Pearls
• Subacute combined degeneration occurs in the setting
o impaired B12 metabolism or def ciency.
• Nitrous oxide exposure can also result in subacute
combined degeneration; however, the pathway is
poorly understood.
• B12 is a critical coenzyme in pathways responsible
or myelin maintenance, and def ciency leads to
demyelinization, vacuolization, and eventually axonal
loss.
• There is signif cant crossover with copper def ciency
myelopathy.
• There is pre erential involvement o the dorsal
Sagittal T2 image demonstrates hyperintensity within the dorsal columns with the resultant presentation o loss o
and lateral columns. proprioception and vibration with paresthesias.
• Involvement o the lateral columns leads to spasticity
and hyperre exia.
Answers • T2 hyperintensity and mild expansion are seen without
1. Etiologies or intramedullary T2 hyperintensity signif cant enhancement.
include copper def ciency myelopathy, HIV vacuolar • Treatment is aimed at B12 supplementation with
myelopathy, in arction, multiple sclerosis, and transverse general resolution o symptoms.
myelitis; however, multiple sclerosis generally does not
present with long segment involvement.
2. Common presenting symptoms are ataxia and Suggested Readings
parasthesias. Additional symptoms include loss Naidich MJ, Ho SU. Case 87: Subacute combined
o position and vibration sense, spasticity, and degeneration. Radiology. 2005;237:101-105.
hyperre exia. Tsang BK-T, Crump N, Macdonel RA. Subacute combined
3. Pernicious anemia is the most common cause in the United degeneration o the spinal cord despite prophylactic vitamin
States. Other etiologies include ileal resection, nitrous B12 treatment. J Clin Neuroscience. 2012;19:908-910.
oxide exposure, partial gastrectomy, and vegan diet.

182
Low back pain

1. What should be included in the di erential


diagnosis?

2. This abnormality occurs rom an insult be ore


what age?

3. What portion o the spine is most commonly


a ected?

4. What are common presenting symptoms?

5. What are the treatment options?

183
Limbus vertebra 2191
Case ranking/dif culty: Category: Vertebral body

Sagittal T1 image demonstrates a small osseous fragment Sagittal T2 image demonstrates a small osseous fragment
adjacent to the superior anterior corner of the L2 vertebral body. adjacent to the superior anterior corner of the L2 vertebral body.
There is no adjacent edema to suggest fracture or instability.

Answers
1. Vertebral body racture, osteophyte racture, Schmorl
Pearls
node, diskitis/osteomyelitis, and limbus vertebra could
all be considered; however, the lack o T2 hyperintense • Limbus vertebrae are common incidental f ndings in
marrow would argue against most o these entities. spine imaging that result rom incomplete usion o
the rim apophysis and vertebral body.
2. The rim apophysis usually uses around 20 years o age; • The etiology is elt to be a traumatic insult during
a limbus vertebra results rom an injury prior to usion. childhood, be ore 20 years old (when usion is
3. The lumbar spine is most commonly involved, complete)—o ten the trauma is not signif cant enough
specif cally, the anterosuperior endplates. to be remembered.
• It is important to di erentiate this rom a racture,
4. Pain, decreased range o motion, and kyphosis are the which can be done by noting the lack o adjacent
most common presenting symptoms; however, this may edema.
also be an incidental f nding. • These lesions require no urther imaging or
5. Treatment is conservative with physical therapy management.
and analgesia. There is no indication or surgical
intervention.
Suggested Readings
Carr RB, Tozer Fink KR, Gross JA. Imaging o trauma: Part I,
pseudotrauma o the spine. AJR. 2012;199:1200-1206.
Runge M. Traps in spinal MR imaging. Diagn Interv
Imaging. 2012;93:993-999.

184
Back pain

1. What should be included in the di erential


diagnosis?

2. What are common presenting symptoms?

3. What location within the spine is most


commonly a ected?

4. Which portion o the spine is most commonly


a ected?

5. What are the treatment options?

185
Pott disease 2182
Case ranking/dif culty: Category: More than one category

Sagittal T2 image demonstrates compression deformity of Axial T1 image following gadolinium administration demonstrates
the T8 vertebral body with increased signal within the T8-T9 enhancement of the lateral aspect of the T8 vertebral body with a
intervertebral disc. peripherally enhancing right paraspinal collection.

Answers
• Spread is primarily hematogenous to the vertebral
1. Lesions that may involve the vertebral body,
body endplates. From there, the in ection can spread
intervertebral disc, and paraspinal so t tissues include
into the disc space and paraspinal so t tissues.
metastatic disease and in ectious spondylitis (pyogenic,
• Factors that can di erentiate tuberculous spondylitis
ungal, tubercular, Brucellar).
rom other etiologies include calcif cation within
2. Presenting symptoms include ever, night sweats, back paraspinal abscesses and lack o periosteal reaction.
pain, radiculopathy, and myelopathy. • Tuberculosis also characteristically spares the
posterior elements, helping di erentiate it rom
3. Most spinous involvement is centered at the
metastatic disease.
thoracolumbar junction.
• Treatment consists primarily o long-term multidrug
4. The vertebral body and intervertebral disc are commonly regimens.
involved. • Surgical resection and decompression may be
indicated in up to one- ourth o patients i neurological
5. Long-term antituberculosis agents are the mainstay o
compromise or spinal de ormity is present.
treatment. Abscess drainage and surgical debridement
may be considered in some cases.

Suggested Readings
Pearls Burrill J, Williams CJ, Bain G, Conder G, Hine AL, Misra
RR. Tuberculosis: a radiologic review. Radiographics.
• Tuberculous spondylitis is increasing in
2007;27:1255-1273.
requency in developed nations, primarily within
Harisinghani MG, McLoud TC, Shepard J-A O, Ko JP,
immunocompromised populations.
Shro MM, Mueller PR. Tuberculosis rom head to toe.
• The spine is involved in hal o patients with osseous
Radiographics. 2000;20:449-470.
tuberculosis involvement, most commonly around the
thoracolumbar junction.

186
Acute -onset paraplegia in patient with lupus

1. What should be included in the di erential


diagnosis?

2. What entities are associated with a higher


incidence o this abnormality?

3. What is the best prognostic actor o recovery?

4. What is the most common location within the


cord?

5. What are the treatment options?

187
Hemorrhagic cord infarction 2180
Case ranking/dif culty: Category: Spinal cord

Sagittal T2 image demonstrates intramedullary low signal, Axial T2 image demonstrates focus of signal dropout, consistent
consistent with hemorrhage, with surrounding T2 hyperintensity with hemorrhage.
(caudal arrow), consistent with vasogenic edema.

Answers
• The prognosis or spinal cord in arction is poor,
1. Etiologies o T2 hyperintensity within the cord include
generally with permanent loss o unction.
astrocytoma, ependymoma, vascular mal ormation,
• Di usion-weighted imaging may help in diagnosis,
in arction, and demyelinating disease; however, it would
particularly in the sagittal plane.
be unusual to see hemorrhage with demyelinating disease.
• Treatment is geared at restoring blood ow with
2. Thoracic aortic aneurysm and thoracic aortic aneurysm anticoagulation (although this should be used
repair, atherosclerosis, vasculitis, and trauma are all judiciously in the setting o cord hemorrhage) and
associated with cord in arction. steroids to reduce edema.
3. The greater the symptoms are at presentation, the worse
the prognosis.
4. The most common location is within the thoracic cord. Suggested Readings
Robertson CE, Brown RD, Wijdicks EFM, Rabinstein AA.
5. Treatment options include conservative management,
Recovery a ter spinal cord in arcts: long-term outcome in
physical therapy, and steroid and anticoagulant
115 patients. Neurology. 2012;78:114-121.
administration. There is no role or surgical resection,
Sang BK, Foster E, Kam A, Storey E. Di usion weighted
although hematoma evacuation could be considered in
imaging with trace di usion weighted imaging, the
select cases.
apparent di usion coe f cient and exponential images in
the diagnosis o spinal cord in arction. J Clin Neurosci.
2013;20:1630-1632.
Pearls Tator CH. Update on the pathophysiology and pathology o
acute spinal cord injury. Brain Pathol. 1995;5:407-413.
• Spinal cord in arction should be considered in
a patient with the sudden onset o paralysis or
myelopathic symptoms.
• Risk actors include the same risk actors or intracranial
in arction, including atherosclerosis and vasculitis.

188
Continued back pain following diskectomy

1. What should be included in the di erential


diagnosis?

2. What are common presenting symptoms?

3. What is the prognosis?

4. What portion o the spine is most commonly


a ected?

5. What are the treatment options?

189
Peridural brosis 2177
Case ranking/dif culty: Category: More than one category

4. Peridural f brosis is most commonly seen in the lumbar


spine.
5. Potential treatment options include physical therapy,
analgesia, corticosteroid injection, spinal cord
stimulation, and surgical resection; however, surgical
resection is unlikely to be benef cial in cases o pure
f brosis. It can be help ul or removal o recurrent disc
herniation and is more success ul i no peridural f brosis
is present.

Pearls
• Peridural f brosis can occur in up to 10% o patients
ollowing diskectomy.
• The classic presentation is progressive back pain/
radicular symptoms in a patient with prior diskectomy
with good results.
• This entity is one o the reasons that gadolinium
Axial T1 image following gadolinium administration demonstrates administration is recommended in postoperative
enhancement in the left lateral recess, surrounding the traversing ollow-up imaging o the lumbar spine.
S1 nerve root. Note the postoperative changes of prior left L5 • Imaging f ndings include so t tissue and enhancement
hemilaminectomy. surrounding a nerve root within the lateral recess.
• It is important to di erentiate peridural f brosis rom
recurrent or residual disc ragment.
Answers • Repeat surgery is more success ul i a recurrent or
1. Epidural so t tissue masses include epidural abscess, residual disc ragment is identif ed, particularly in the
epidural f brosis, epidural hematoma, epidural absence o associated f brosis.
metastasis, and recurrent disc herniation.
2. Potential presenting symptoms o epidural f brosis
include back pain, radiculopathy, loss o re exes, Suggested Readings
and dysesthesia. The key is the history o previous
Hue tle MG, Modic MT, Ross JS, et al. Lumbar spine:
success ul surgery and the onset o gradually progressive
postoperative MR imaging with Gd-DTPA. Radiology.
symptoms.
1988;167:817-824.
3. Generally, the symptoms are stable to slowly Lee YS, Choi ES, Song CJ. Symptomatic nerve root changes
progressive. Approximately one-third o patients have on contrast-enhanced MR imaging a ter surgery or
improvement o symptoms ollowing repeat surgery, lumbar disk herniation. AJNR. 2009;30:1062-1067.
while about hal o patients have improvement ollowing
spinal cord stimulator placement.

190
Fixed torticollis

1. What are etiologies o this abnormality?

2. What is the classif cation system according to


the Fielding and Hawkins system?

3. What syndromes are associated with increased


incidence?

4. What ligaments are disrupted?

5. What are the treatment options?

191
Rotatory subluxation 1522
Case ranking/dif culty: Category: Vertebral body

Axial CT image demonstrates asymmetry of the C1-C2 articulation. Coronal CT image demonstrates asymmetry of the C1-C2
articulation.

Answers
1. Trauma is the most common cause; however, prior Pearls
surgery may lead to laxity and nasopharyngeal in ection • Rotatory subluxation is the result o ligamentous
may spread to the craniocervical junction joints. disruption at the C1-C2 articulation.
• It is more common in children, secondary to the
2. Fielding and Hawkins system:
inherent ligamentous laxity, and relatively large head
Type 1—anterior atlas displacement <3mm size with respect to the remainder o the body.
Type 2—anterior atlas displacement 3-5 mm • On static imaging, this injury is suspected by the o set
Type 3—anterior atlas displacement >5 mm o C1 with respect to the dens and C2.
• I there is concern, consideration o dynamic three
Type 4—posterior atlas displacement
phase CT is recommended—including neutral and
There has been a ref nement with the Pang system based maximum head turning to each side. This should only
on three position CT. be undertaken under advisement o neurosurgery to
3. Down syndrome, Mar an syndrome, Morqui syndrome, avoid permanent patient injury.
and rheumatoid arthritis all have a higher incidence o
ligamentous laxity leading to increased incidence o
rotatory subluxation. Suggested Readings
4. The alar and transverse ligaments are disrupted; Beier AD, Vachhrajani S, Bayerl SH, et al. Rotatory
however, this may occur in the setting o other traumatic subluxation: experience rom the Hospital or Sick
injuries and evaluation o the remaining ligaments is Children. J Neurosurg Pediatrics. 2012;9:144-148.
critical. Haque S, Bin Bilal Shaf B, Kaleem M. Imaging o torticollis
in children. Radiographics. 2012;32:557-571.
5. Initial treatment is bracing; patients who ail bracing
Pang D, Li V. Atlantoaxial rotatory f xation: Part 3—a
may require traction and additional bracing, halo
prospective study o the clinical mani estation, diagnosis,
f xation, or posterior usion.
management, and outcome o children with alantoaxial
rotatory f xation. Neurosurgery. 2005;57(5):954-972.

192
Pedestrian versus train accident

1. What is the motor innervation supplied by the


brachial plexus trunks and cords?

2. What are the common presenting symptoms o


compressive brachial plexopathy?

3. What are some o the common causes o


compressive brachial plexopathy?

4. What are imaging landmarks used to evaluate


the segments o the brachial plexus?

5. What is the next most appropriate step in


management?

193
Brachial plexus hematoma and retracted nerve roots 1494
Case ranking/dif culty: Category: Nerve roots/Nerve plexus/Peripheral nerves

Coronal T1 image demonstrates Coronal T2 image demonstrates mixed Coronal T2 image demonstrates
mixed intensity mass, consistent with intensity mass, consistent with hematoma. pseudomeningoceles at C8 and T1.
hematoma.

Answers
1. The trunks and posterior cords innervate the shoulder, Pearls
the medial cord innervates the hand, and the lateral cord • Traumatic brachial plexus injuries usually present with
innervates the upper chest. motor symptoms.
• The radiologist’s role is to def ne the distance between
2. Compressive plexopathies are generally more di f cult to
avulsed nerve segments, i possible.
diagnose clinically as they present with mixed motor and
• In addition, it is important to describe the location o
sensory symptomatology.
avulsion: the more proximal the avulsion, the more
3. Extrinsic compression o the brachial plexus by tumor di f cult the surgical reanastomosis.
is more common than primary tumors o the brachial • Evaluation should include mention o associated
plexus. The most common etiologies o compression injuries, including to the cervical spine, spinal cord, or
include hematoma, lymphoma, leukemia, and metastatic shoulder joint.
lymphadenopathy.
4. Roots are located in the neural oramina; trunks are
located between the scalene muscles; divisions are Suggested Readings
located posterior to the clavicle; cords are located Castillo M. Imaging the anatomy o the brachial
in erior to the clavicle. Knowing the anatomic location is plexus: review and sel -assessment module. AJR.
important or identi ying abnormalities. 2005;185:S196-S204.
5. Evaluation or a vascular injury may be indicated Sureka J, Cherian RA, Alexander M, Thomas BP. MRI o
depending on the clinical history. Analgesia and brachial plexopathies. Clin Radiol. 2009;64:208-218.
physical therapy are additional appropriate management
steps in this patient, with the consideration o surgical
reanastomosis i appropriate.

194
Fever, abdominal pain, and emesis

1. What should be included in the di erential


diagnosis?

2. What are common presenting symptoms?

3. What viruses are most commonly associated


with this abnormality?

4. What part o the spinal cord is most commonly


involved?

5. What are the treatment options?

195
Enterovirus myelitis 2183
Case ranking/dif culty: Category: Spinal cord

Sagittal T2 image demonstrates ventral T2 hyperintensity extending Axial T2 image demonstrates T2 hyperintensity within the ventral
from the craniocervical junction through the cervical cord. horns.

Answers
• In this case, there is long segment involvement o the
1. Etiologies or T2 hyperintensity within the cord include
bilateral ventral horns, typical o viral myelitis.
polio, idiopathic transverse myelitis, multiple sclerosis,
• Demyelinating disease tends to have a more patchy
viral myelitis, and acute disseminated encephalomyelitis.
distribution and each individual lesion tends to be less
2. Given the ventral horn distribution o the signal than two vertebral body heights in length.
abnormality, it is not surprising that motor symptoms • Polio has a very similar appearance but is rarely
predominate. However, it is possible to have sensory encountered in the post-vaccine era.
symptoms as well. • Gadolinium administration is rarely help ul as any o
these etiologies may demonstrate enhancement.
3. The enteroviruses are the most common (coxsackie,
• Patients with viral myelitis tend to present with
echovirus, rubella, measles, mumps, hepatitis); other
weakness, which may progress to paralysis. Sensory
etiologies include herpes, West Nile, and human
disruption may also be seen.
immunodef ciency viruses.
• Lumbar puncture is key to diagnosis—the
4. The cervical and thoracic cords are most o ten involved cerebrospinal uid demonstrates a typical pattern or
with extension to the craniocervical junction. O tentimes viral in ection with elevated protein and mononuclear
the signal abnormality may even extend cephalad to the cells. Additionally, the uid can be sent or viral PCR.
brainstem.
5. Treatment options include supportive management,
antiviral agents, steroids, and intravenous Suggested Readings
immunoglobulin. Ooi MH, Wong SC, Lewthwaite P, Cardosa MJ, Solomon
T. Clinical eatures, diagnosis and management o
enterovirus 71. Lancet Neurol. 2010;9:1097-1105.
Pearls Shen WC, Tsai C-H, Chiu H-H, Chow K-C. MRI o
enterovirus 71 myelitis with monoplegia. Neuroradiology.
• T2 hyperintense signal within the cord is seen in a
2000;42:124-127.
variety o pathologies.
• The distribution can be help ul to di erentiate
diagnoses.

196
Neuro bromatosis, type 1

1. What should be included in the di erential


diagnosis?

2. What are common presenting symptoms?

3. What are risk actors or development?

4. What are common sites o metastases?

5. What are the treatment options?

197
Malignant peripheral nerve sheath tumor 1519
Case ranking/dif culty: Category: Nerve roots/Nerve plexus/Peripheral nerves

5. Surgical resection is the mainstay o treatment with


adjuvant chemotherapy and radiation; however,
reconstruction and gra ting are not routinely
recommended.

Pearls
• Malignant peripheral nerve sheath tumors are
uncommon in the general population with an incidence
o approximately 0.001%; however, patients with
neurof bromatosis, type 1, have a li etime incidence o
up to 10%.
• Hal o all malignant peripheral nerve sheath tumors are
diagnosed in patients with neurof bromatosis, type 1.
• Malignant degeneration o a neurof broma should be
Axial T1 image following gadolinium administration suspected in the setting o a rapid increase in size or
demonstrates a large left paraspinal soft tissue mass with development o pain.
heterogeneous enhancement; note extension through the left
• MRI is the imaging study o choice or evaluation o
C2-C3 neural foramen.
the involved nerves; however, PET imaging can be
help ul to di erentiate benign rom malignant tumors
Answers in select cases.
• The primary treatment is surgical resection; however,
1. The di erential diagnosis includes malignant peripheral
total resection can be problematic.
nerve sheath tumor, lymphadenopathy, pleomorphic
• Adjuvant radiation therapy is used or improved local
undi erentiated sarcoma, neurof brosarcoma, and
control, but does not extend li e.
plexi orm neurof broma.
• Chemotherapy can be added or distant metastatic
2. Common presenting symptoms include pain, rapid disease; however, it has limited benef t.
enlargement, radiculopathy, peripheral edema, and motor
def cit.
3. Risk actors or developing malignant peripheral Suggested Readings
nerve sheath tumors include neurof bromatosis, type
Anghileri M, Miceli R, Fiore M, et al. Malignant peripheral
1, neurof bromatosis, type 2, previous radiation o a
nerve sheath tumors: prognostic actors and survival in a
ganglioneuroma or pheochromocytoma, and presence o
series o patients treated at a single institution. Cancer.
a plexi orm neurof broma.
2006;107:1065-1074.
4. Common sites o metastases rom malignant peripheral Gupta G, Maniker A. Malignant peripheral nerve sheath
nerve sheath tumors include bone, brain, liver, lung, and tumors. Neurosurg Focus. 2007;22:E12.
lymph nodes.

198
Motor vehicle accident

1. What should be included in the di erential


diagnosis?

2. What are common mechanisms o injury?

3. What structures should be evaluated during


cervical spine imaging or trauma?

4. What are the indications or cervical spine


MRI in the setting o trauma?

5. What are the treatment options?

199
Ligamentous injury 1516
Case ranking/dif culty: Category: More than one category

4. Indications or MRI in the setting o trauma include


racture, persistent neck pain, weakness, altered
sensation, and instability on dynamic imaging.
5. Cervical collar/immobilization is the f rst-line therapy
or patients without malalignment or instability. Surgical
f xation is reserved or unstable injuries. Steroids can be
used to alleviate cord edema in selected patients.

Pearls
• Evaluation o the cervical spine in trauma is not
complete without considering the so t tissues.
• While the ligaments are di f cult to assess on CT, there
are clues that there is injury, including prevertebral
edema and epidural hematoma.
• Further evaluation in these patients should include
MRI, including sagittal T2-weighted sequences with
at suppression (either T2 with at saturation or STIR).
Sagittal STIR image demonstrates focal disruption of the anterior
• Management in ligamentous injury depends on degree
and posterior longitudinal ligaments and the ligamentum avum
with associated prevertebral edema. Also note hyperintensity o disruption, as well as stability during dynamic
within the T2 -T4 vertebral bodies, concerning for fractures. imaging.
• Some o these injuries can be treated with bracing/
cervical collar, while some require surgical
Answers stabilization.
1. Prevertebral so t tissue enlargement can be seen in
ligamentous injury, prevertebral hematoma, longus colli
tendonitis, and retropharyngeal abscess. Suggested Readings
2. Hyperextension, hyper exion, rotatory exion, rotatory Duane TM, Cross J, Scarcella N, et al. Flexion-extension
extension, and lateral exion can lead to ligamentous cervical spine plain f lms compared with MRI in the
disruption. diagnosis o ligamentous injury. Am Surg. 2010;76:595-598.
3. It is critical to evaluate osseous structures, ligaments, Schoen eld AJ, Bono CM, McGuire KJ, Warholic N,
paraspinal musculature, vasculature, and spinal cord in Harris MB. Computed tomography alone versus
the setting o cervical spine trauma. computed tomography and magnetic resonance imaging
in the identif cation o occult injuries to the cervical
spine: a meta-analysis. J Trauma Inj In ect Crit Care.
2010;68:109-114.

200
Right L3-L4 radicular symptoms

1. What should be included in the di erential


diagnosis?

2. What are common presenting symptoms?

3. What actors should be evaluated in order to


assess or the presence o this abnormality?

4. What are the etiologies?

5. What are the treatment options?

201
Dural ectasia, Marfan syndrome 1515
Case ranking/dif culty: Category: More than one category

Sagittal proton density image demonstrates di use dural ectasia Axial T1 image demonstrates dilation of the thecal sac and multiple
with dilation of the thecal sac and marked posterior vertebral nerve root sleeves, as well as an anterior sacral meningocele.
body scalloping.

Answers
1. The di erential o a cystic mass includes dural ectasia, Pearls
arachnoid cyst, epidermoid cyst, and schwannoma. • Dural ectasia is common in patients with Mar an
syndrome.
2. Dural ectasia can cause back pain and scoliosis rom
• It is diagnosed when the thecal sac diameter is greater
erosion o the vertebral bodies and pedicles, which can
at the S1 level than at the L4 level.
be complicated by pathologic racture.
• It is one o the major criteria or the diagnosis o
3. Scalloping o the posterior bodies as well as enlarged Mar an disease, which is an autosomal dominant
thecal sac and nerve root sleeve diameter can be used to connective tissue disorder.
diagnose dural ectasia. • Dural ectasia can also be seen in Ehlers-Danlos
syndrome, neurof bromatosis type 1, and ankylosing
4. Dural ectasia can be seen in ankylosing spondylitis,
spondylitis.
Ehlers-Danlos, neurof bromatosis type 1, and Mar an
• It is important to recognize as it can be associated with
syndrome.
posterior vertebral body scalloping, which can lead to
5. Conservative management is most appropriate unless racture, spondylolisthesis, or scoliosis.
complications such as racture, spondylolisthesis, or • I there are associated complications, spinal usion can
scoliosis have occurred, which can necessitate surgical be considered.
f xation.

Suggested Readings
Habermann CR, Weiss F, Schoder V, et al. MR evaluation
o dural ectasia in Mar an syndrome: reassessment o the
established criteria in children, adolescents and young
adults. Radiology. 2005;234:535-541.
Lundy R, Rand-Hendriksen S, Hald JK, et al. Dural ectasia
in Mar an syndrome: a case control study. AJNR.
2009;30:1534-1540.

202
Thoracic pain, nephrolithiasis

1. What should be included in the di erential


diagnosis?

2. What WHO grade are these lesions?

3. In patients with this lesion as part o a


syndrome, what are other components?

4. What is the most common location within the


cord?

5. What are the treatment options?

203
Hemangioblastoma 1506
Case ranking/dif culty: Category: Spinal cord

Sagittal T2 image demonstrates expansion and hyperintensity Sagittal T1 image after contrast administration demonstrates
within the lower thoracic cord. enhancing nodule at the T9-T10 level.

Answers
• Most hemangioblastomas are sporadic (2/3).
1. Hypervascular enhancing lesions in this location
• 1/3 occurring in conjunction with Von Hippel Lindau
include astrocytoma, ependymoma, metastasis, vascular
syndrome. These patients usually present at a younger
mal ormation, and hemangioblastoma.
age with multiple lesions.
2. Hemangioblastomas are WHO grade I and generally do • These are low-grade tumors and treatment may consist
not degenerate into higher grade tumors. o serial imaging ollow-up.
• I lesions are symptomatic, surgical resection or
3. Von Hippel Lindau syndrome includes
stereotactic radiosurgery may be per ormed with
hemangioblastoma, endolymphatic sac tumors, retinal
consideration o preoperative embolization to decrease
angiomas pheochromocytoma, hepatic/pancreatic and
complications.
epididymal cysts, and renal cell carcinoma.
4. The thoracic spine is the most common location.
5. I patients are symptomatic, surgical resection can Suggested Readings
be considered, usually a ter embolization to decrease Chamberlain MC, Tredway TL. Adult primary intradural
intraoperative complications. Stereotactic radiosurgery spinal cord tumors: a review. Curr Neurol Neurosci Rep.
can be used in patients with multiple lesions. 2011;11:320-328.
Rogers SR, Phalke VV, Anderson J, Riccelli LP, Gonda
S, Pollock JM. HEALSME: di erential diagnosis or
Pearls intramedullary spinal cord lesions. Neurographics.
2012;2:13-226.
• The classic imaging appearance o a
hemangioblastoma is a dorsal-enhancing nodule with
an intraspinal cyst.
• While ow voids may be seen, these are usually
not present until the tumor is greater than 2 cm in
dimension.

204
Two-month history of right upper extremity swelling and pain

1. What should be included in the di erential


diagnosis?

2. What are the most common primary


malignancies to metastasize to the brachial
plexus?

3. What are the presenting symptoms?

4. What is one o the earliest imaging signs o


brachial plexus involvement by a Pancoast
tumor?

5. What are the treatment options?

205
Brachial plexus metastatic in ltration 1498
Case ranking/dif culty: Category: Nerve roots/Nerve plexus/Peripheral nerves

Coronal T2 image demonstrates hyperintensity and soft tissue Coronal T1 postgadolinium image demonstrates enhancement of
along the right brachial plexus. the right brachial plexus with nodular soft tissue.

Answers
1. The di erential diagnosis includes radiation plexopathy, Pearls
primary nerve tumor, metastasis, and in ammatory • Metastatic inf ltration o the brachial plexus may occur
plexopathy. Metastasis to the brachial plexus o ten has via hematogenous, lymphatic, or direct spread.
a more nodular enhancement pattern than radiation or • In patients who have undergone treatment, it
in ammatory plexopathy. is important to di erentiate post-radiation and
in ammatory enhancement rom metastatic
2. Breast and lung carcinoma, as well as lymphoma,
inf ltration.
leukemia, and multiple myeloma, are among the most
• When di erentiating metastatic inf ltration rom post-
common to involve the brachial plexus. Head and neck
radiation or in ammatory plexopathy, the pattern o
cancers may also involve the brachial plexus.
enhancement is key.
3. Pain, Horner syndrome, and hand weakness are all • Generally, metastatic inf ltration demonstrates a more
symptoms o metastatic inf ltration o the brachial nodular pattern o contrast enhancement.
plexus. Lymphedema is more commonly seen in
association with post-radiation plexopathy.
4. E acement o the interscalene at pad is one o the Suggested Readings
earliest signs o brachial plexus involvement by Pancoast
Castillo M. Imaging the anatomy o the brachial plexus. AJR.
tumor.
2005;185:S196-S204.
5. Radiation and chemotherapy are the primary modalities Sureka J, Cherian RA, Alexander M, Thomas BP. MRI o
used to treat brachial plexus metastatic inf ltration, brachial plexopathies. Clin Radiol. 2009;64:208-218.
depending on the primary malignancy. Surgical resection
o adjacent masses is possible; however, resection
or sclerosis o the brachial plexus can be potentially
devastating with loss o unction o the upper extremity.

206
Lower extremity paraplegia following fall from standing height

1. What is the di erential diagnosis o ocal T2


hyperintense signal within the spinal cord?

2. What are indications or obtaining MRI in the


setting o cervical spine trauma?

3. What are the imaging f ndings o this


abnormality?

4. What are the grades o the Frankel scale?

5. What are the treatment options?

207
Cervical cord contusion 1505
Case ranking/dif culty: Category: Spinal cord

4. Grade A—complete loss o unction


Grade B—intact sensation, loss o motor unction
Grade C—intact sensation, intermediate motor unction
(2-3/5)
Grade D—intact sensation, near complete motor
unction (4/5)
Grade E—normal unction
This Frankel classif cation model can be used in
conjunction with imaging to help determine patient
prognosis.
5. Treatment is directed at decreasing the cord edema,
while treating associated injuries and/or instability.
There is no known role o cord hematoma evacuation.

Pearls
• Multi actorial spinal canal stenosis is most common in
Sagittal T2 image demonstrates cord expansion and T2 the elderly.
hyperintensity. • This leads to a higher incidence o spinal cord injuries
in the setting o relatively minor trauma.
• A high index o suspicion is required to make the
Answers diagnosis, and advanced imaging with MRI is usually
required.
1. T2 hyperintensity within the cord is nonspecif c and may
• Focal T2 hyperintensity within the cord is nonspecif c,
represent any o the etiologies listed, making correlation
but can be evaluated in the clinical context.
with patient’s history critical.
2. MRI should be considered in settings in which there is
concern or ligamentous and/or cord injury.
Suggested Readings
3. The signal intensity within the cord in acute / subacute Goldberg A, Kershah SM. Advances in imaging o vertebral
injury can vary depending on the amount o edema and spinal cord injury. J Spinal Cord Med. 2010;33:105-116.
(T1 hypointensity, T2 hyperintensity) and hemorrhage ter Haar M, Naidoo SM, Govender S, Parag P, Esterhuizen
(T1 hyperintensity, T2 hypointensity). In the acute TM. Acute traumatic cervical spinal cord injuries:
to subacute phase, the cord is usually normal size to correlating MRI f ndings with neurological outcome.
enlarged, but in the chronic phase, myelomalacia can SA Orthop J. 2011;10:35-41.
develop.

208
Neck mass

1. What is the etiology?

2. What are included in the classif cation scheme?

3. What are key di erences between cervical and


lumbosacral myelomeningoceles?

4. What should be included in the di erential


diagnosis o a cystic neck mass in a neonate?

5. What are the treatment options?

209
Cervical myelomeningocele 1492
Case ranking/dif culty: Category: Thecal sac

urological dys unction, this can be evaluated ollowing


closure o the spinal dysraphic de ect.
4. There are many etiologies or a cystic neck mass
in a neonate, including cystic hygroma, cystic
lymphadenopathy, myelomeningocele, branchial
cle t mal ormation, and lymphangioma; however,
myelomeningocele and hygroma are o ten more
posteriorly distributed.
5. Initial treatment should consist o surgical resection
with intradural exploration and cord untethering.
There is no indication to spare the neural tissue within
the de ect as it has been shown to be non unctional.
Syringosubarachnoid shunt placement may be indicated
in cases o re ractory syrinx; however, it is not a f rst-line
treatment option.

Pearls
• Cervical myelomeningocele is a rare subtype o
myelomeningoceles, representing less than 10% o all
Sagittal STIR image demonstrates posterior exophytic mixed
myelomeningoceles.
uid intensity mass with connection to underlying thecal sac and
• Like classic lumbosacral myelomeningoceles, these
distortion of the cord.
may occur in conjunction with Chiari II mal ormation.
• However, they may be seen as an isolated anomaly or
Answers in association with hydrocephalus or syringomyelia.
• The pathogenesis is secondary to abnormal
1. Cervical myelomeningoceles are secondary to abnormal
neurulation with incomplete usion o the neural old.
neurulation with ailure o the neural old to use.
• Patients generally have distal, but variable,
2. Cervical myelomeningoceles have been subdivided into neurological def cits.
meningoceles, meningocystoceles, and limited dorsal • Treatment consists o surgical resection o the
myeloschisis. Most recently, these have been urther meningeal sac; however, most patients achieve better
divided into three subgroups: cystic spinal dysraphism neurological outcome with the addition o untethering
with a stalk, cystic spinal dysraphism without o the cord.
meningocele or stalk, and myelocystocele. There are ew
accepted universal classif cations.
3. Cervical myelomeningoceles have a higher incidence Suggested Readings
o associated neurological def cits; however, there is a Habibi Z, Nejat F, Tajik P, Kazmi SS, Kajba zadeh
lower rate o association with Chiari II mal ormation AM. Cervical myelomeningocele. Neurosurgery.
or hydrocephalus. While preoperative imaging o 2006;58:1168-1175.
the spine is usually not indicated in lumbosacral Odebode TO, Udo a SU, Nzeh AD. Cervical
myelomeningoceles, it can be help ul in cervical lesions. myelomeningocele and hydrocephalus without
While both myelomeningoceles can be associated with neurological def cit: a case report. AEJSA. 2007;2:60-62.

210
Left hip and sacroiliac pain

1. What should be included in the di erential


diagnosis?

2. What are common presenting symptoms?

3. What is the most sensitive f nding on physical


exam?

4. Which entities tend to have unilateral or


asymmetric sacroiliac joint involvement?

5. What are the treatment options?

211
Pelvic osteomyelitis with associated sacroiliitis 1489
Case ranking/dif culty: Category: More than one category

Axial T2 image demonstrates hyperintense uid within the Axial T1 image following contrast administration demonstrates
sacroiliac joint, extending into the pelvis, with associated marrow peripherally enhancing uid collection within the sacroiliac joint,
edema within the ilium and sacrum. There is extension into the extending into the pelvis, with associated marrow edema within
gluteal musculature. the ilium and sacrum.

Answers
• The in erior one-third o the sacroiliac joint is lined by
1. Metastatic disease, primary bone tumor, ankylosing
synovium and may have uid; however, any uid seen
spondylitis, psoriatic arthritis, and in ection may all
in the upper two-thirds is always abnormal.
involve the sacroiliac joints, either unilaterally or
• The most common pathogen is Staphylococcus
bilaterally.
aureus (up to 75% o patients) with additional isolates
2. Fever and limp are the predominant symptoms. Other including Streptococcus, Haemophilus inf uenzae,
symptoms may include back pain, lower quadrant pain, Escherichia coli, and Salmonella.
and radiculopathy. • Imaging relies heavily on MRI or evaluation o so t
tissue and osseous involvement.
3. The FABERE test ( exion-abduction-external rotation-
• Initial treatment consists o appropriate antibiotic
extension) o the hip is usually positive or eliciting pain.
coverage with consideration o so t tissue abscess
4. Unilateral or asymmetric sacroiliac involvement can drainage.
be seen in Reiter syndrome, pyogenic sacroiliitis, and • Debridement o the sacroiliac joint may be per ormed,
psoriatic arthritis. Ankylosing spondylitis is usually i extensive bone involvement or intraspinal extension
bilateral and symmetric with prominent ankylosis is visualized, but o ten leads to instability requiring
( usion) across the sacroiliac joints. surgical usion.
5. Primary treatment is antibiotics with supplemental
drainage o associated so t tissue abscesses and
debridement. Surgical f xation is o ten necessary Suggested Readings
ollowing debridement to avoid pelvic instability.
Murphey MD, Wetzel LH, Bramble JM, Levine E, Simpson
KM, Lindsley HB. Sacroiliitis: MR imaging f ndings.
Radiology. 1991;180:239-244.
Pearls Quintana AM, Gutierrez BM, Lovillo MSC, Neth O,
Santaella IO. Pyogenic sacroiliitis in children—a
• Pyogenic sacroiliitis is uncommon and o ten presents
diagnostic challenge. Clin Rheumatol. 2011;30:107-113.
in patients with suppressed immune system.
• Up to 10% o patients report a prior history o trauma
to the sacroiliac joint.
• It can be di f cult to diagnose as the symptoms overlap
with a variety o abdominal and pelvic pathologies.

212
Chronic low back pain and radiculopathy

1. What is this procedure called?

2. Name some indications or this procedure?

3. Describe the major surgical steps o this


procedure.

4. Which are the disadvantages o this procedure?

5. Name potential complications o this


procedure.

213
Posterior lumbar interbody fusion (PLIF) 3312
Case ranking/dif culty: Category: Vertebral body

4. PLIF has certain disadvantages. The posterior approach


does not allow comprehensive evacuation o the
intervertebral disc space, which decreases the sur ace area
available or usion. An anterior approach also allows the
use o a larger spinal implant, which results in superior
stabilization. The posterior approach may render surgery
much more di f cult in patients with spinal de ormity and,
rarely, the cage may retropulse into the spinal canal.
5. The main risk o the procedure is nonunion, which may
require urther surgery to attempt spinal usion. Fusion
rates o PLIF are, however, reported to be as high as
95%. The risk is increased in patients who have had
previous spinal surgery, in multiple-level usion surgery,
in previous radiotherapy, and in obese patients. Other
AP radiograph of the lumbar Lateral radiograph of the complications o the procedure include in ection and
spine in a patient who was lumbar spine shows normal hemorrhage. The patient’s pain may not subside even
previously treated with PLIF. alignment of the lumbar when success ul spine usion has been achieved.
Note short rods (arrow) vertebrae. L4-L5 disc has been
between cannulated pedicular evacuated and replaced with
screws at L4-L5 used for spinal a synthetic cage (arrowhead) Pearls
stabilization. A synthetic cage which restores the disc space • Posterior lumbar interbody usion (PLIF) involves a
was placed in the evacuated height. Pedicular screws lumbar discectomy ollowed by placement o bone
intervertebral disc space (arrow) and rods were used to gra t into the evacuated intervertebral disc space in
(arrowhead). stabilize the spine.
order to achieve spinal usion.
• PLIF is indicated in patients with symptomatic disc
Answers prolapse or oraminal stenosis in which previous
1. Posterior lumbar interbody usion (PLIF) is a surgical spinal surgery has ailed, in patients with signif cant
procedure that involves a lumbar discectomy ollowed by discogenic or acet joint pain that has not responded
placement o bone gra t into the evacuated intervertebral to conservative measures, in spondylolisthesis, and in
disc space in order to achieve spinal usion. It is o ten spinal instability.
supplemented by simultaneous posterolateral spine • The procedure is per ormed via a posterior midline
usion surgery. incision.
2. PLIF is indicated in patients with symptomatic disc • A discectomy is then per ormed ollowed by insertion
prolapse or oraminal stenosis in which previous o a cage made o either allogra t bone or synthetic
spinal surgery has ailed, in patients with signif cant material into the intervertebral disc space.
discogenic or acet joint pain that has not responded • Pedicle screws may be inserted at the levels above and
to conservative measures, in spondylolisthesis, and in below in order to stabilize the spine.
spinal instability. • The main risk o the procedure is nonunion, which
may require urther surgery to attempt spinal usion.
3. The procedure is per ormed via a posterior midline • The patient’s pain may not subside even when
incision. The erector spinae muscles are stripped o success ul spine usion has been achieved.
the laminae on both sides be ore a laminectomy is
per ormed. The acet joints may also be undercut to
provide more room or exiting nerve roots. A discectomy Suggested Readings
is then per ormed ollowed by insertion o a cage made Ahsan MK, Hossain MA, Sakeb N, Khan SI, Zaman N.
o either allogra t bone or synthetic material (PEEK Instrumented posterior lumbar interbody usion (PLIF)
or titanium) into the intervertebral disc space. Pedicle with interbody usion device (cage) in degenerative disc
screws may be inserted in the levels above and below in disease (DDD): 3 years outcome. Mymensingh Med J.
order to stabilize the spine. The procedure is di erent 2013 Oct;22(4):798-806.
rom posterolateral usion in which the bone gra t is Pannell WC, Savin DD, Scott TP, Wang JC, Daubs MD.
placed between the transverse processes to stimulate Trends in the surgical treatment o lumbar spine disease in
usion between adjacent transverse processes. the United States. Spine J. 2013 Oct 31.
214
Back pain

1. What should be included in the di erential


diagnosis?

2. What are common presenting symptoms?

3. Which location within the spine is most


commonly a ected?

4. What are common patterns o extension?

5. What are the treatment options?

215
Osteosarcoma 1488
Case ranking/dif culty: Category: Vertebral body

Sagittal STIR image demonstrates compression of L3 vertebral Sagittal T1 image following gadolinium administration
body with heterogeneous signal, and an epidural soft tissue mass. demonstrates enhancement of the vertebral body with anterior
epidural extension.

Answers
• The risk actors include Paget disease and prior
1. Potential causes o vertebral body lesions and pathologic
radiation therapy; however, these risk actors are more
racture include chondrosarcoma, lymphoma, metastasis,
commonly seen in extremity osteosarcoma.
multiple myeloma, and plasmacytoma.
• Patients with primary spinal osteosarcoma tend to be
2. Presenting symptoms depend on vertebral body and slightly older than extremity osteosarcoma patients.
intraspinal extension and include pain, radiculopathy, • Patients o ten present with pain or pathologic racture.
myelopathy, and pathologic racture. • Evaluation or paravertebral so t tissue aids in making
the diagnosis o pathologic racture, rather than benign
3. 80% o spinal lesions arise in the posterior elements.
compression racture.
Sacral lesions generally arise in the body and ala.
• Primary treatment is surgical resection (with f xation
4. Potential patterns o extension include adjacent vertebral as needed) with adjuvant radiation and chemotherapy.
body, paravertebral so t tissue, epidural space, and neural
oramina.
5. Surgical resection is the primary treatment, with Suggested Readings
stabilization as needed. Adjuvant radiation and
chemotherapy may be used. Makhdoomi R, Nayil K, Ramzan A, Baba K, Bhat S, Sheikh
S. Primary osteosarcoma o the cervical spine: a case report
and literature review. Neurosurg Q. 2010;20:250-252.
Schwab J, Gasbarrini A, Bandiera S, et al. Osteosarcoma o
Pearls the mobile spine. Spine. 2012;37:E381-E386.
• Osteosarcoma rarely occurs in the spine; however, it is
the second most common primary osseous neoplasm
in the spine a ter multiple myeloma.

216
Neck pain after being kicked in neck 2 months prior

1. This abnormality can be associated with which


other bone lesions?

2. What parts o the vertebral body can be


involved?

3. What are common presenting symptoms?

4. What is the classic imaging appearance?

5. What are the treatment options?

217
Cervical aneurysmal bone cyst 1486
Case ranking/dif culty: Category: Posterior elements

Axial CT image in bone windows demonstrates Axial CT image in soft tissue windows Sagittal CT image demonstrates
expansile and lucent lesion involving the demonstrates expansile and lucent lesion expansile and lucent lesion
bilateral laminae and spinous process. involving the bilateral laminae and spinous involving the C2 spinous process.
process.

Answers
1. Aneurysmal bone cyst is classif ed as primary i Pearls
not associated with another lesion and secondary i • Aneurysmal bone cysts (ABC) are benign osseous
associated with another lesion, such as osteoblastoma, lesions o uncertain etiology.
telangiectatic osteosarcoma, giant cell tumor, or • They represent approximately 1% o primary bone
chondroblastoma. tumors, and up to 20% o ABC are ound in the spine.
• ABC are classif ed as primary i they are isolated
2. Aneurysmal bone cyst can a ect any part o the vertebral
lesions, or secondary i another lesion is ound in
body.
association with the ABC.
3. The most common presenting symptom is pain; others • ABC involve all spinal levels, as well as multiple parts
include palpable mass, scoliosis/kyphosis, myelopathy, o the vertebral column.
and radiculopathy. Depending on location and size, • They generally appear as expansile lucent lesions on
aneurysmal bone cyst can present with any o these radiographs or CT with complex signal on MRI and
symptoms. possible uid- uid levels, given the predilection or
superimposed hemorrhage.
4. Classically isolated aneurysmal bone cyst presents as
a lucent expansile lesion without signif cant internal
matrix. The lesion may be multiloculated with uid- uid
levels depending on previous hemorrhage. Suggested Readings
5. Surgical excision is the treatment o choice, but given Boriani S, De Iure F, Campanacci L, et al. Aneurysmal
the vascular nature o the lesions this is sometimes bone cyst o the mobile spine: report on 41 cases. Spine.
preceded by arterial embolization. In cases where 2001;26:27-35.
complete surgical excision is not possible, curettage and/ Papagelopoulus PJ, Currier BL, Shaughnessy WJ, et al.
or radiotherapy may be employed. Aneurysmal bone cyst o the spine: management and
outcome. Spine. 1998;23:621-628.

218
Back pain

1. What should be included in the di erential


diagnosis?

2. What other lesions can be seen in conjunction


with this abnormality?

3. What are common presenting symptoms?

4. What portion o the spine is most commonly


a ected?

5. What are the treatment options?

219
Sacral aneurysmal bone cyst 1485
Case ranking/dif culty: Category: Vertebral body

Axial CT image in bone Axial CT image in soft tissue Axial T1 image demonstrates multiple loculations with focal
windows demonstrates window demonstrates soft tissue areas of spontaneous T1 hyperintensity, consistent with
erosion of right S1 vertebral mass without internal matrix or blood products.
body and lamina with calci cation. Mass extends into
extension to the sacroiliac the spinal canal and displaces
joint. the thecal sac and traversing
nerve root in the lateral recess.

Axial T2 image demonstrates multiple areas of uid intensity,


consistent with cysts, interspersed between low T2 septae.
Axial T1 post-contrast image demonstrates enhancement of the
solid septae within the mass.
Pearls
Answers • Aneurysmal bone cysts only represent approximately
1. The di erential o lytic expansile osseous lesions include 1% o all primary bone tumors; however, one-f th o
aneurysmal bone cyst, lytic metastasis, telangiectatic the lesions are ound within the spine.
osteosarcoma, and unicameral bone cyst. In skeletally • Classif cation o lesions is dependent on the absence/
mature patients, giant cell tumor and chondrosarcoma presence o an associated lesion (primary/secondary).
should also be considered. Chordoma should be • The classic imaging appearance is an expansile lytic
included, but is classically more midline in location. lesion that demonstrates lobulation and internal
2. Secondary aneurysmal bone cyst can be seen in uid- uid levels.
conjunction with chondroblastoma, giant cell tumor, • Within the spine, any portion o the spinal column
osteoblastoma, and telangiectatic osteosarcoma. may be involved.
• Treatment is geared at resection with usion as needed
3. Pain is the most common presenting symptoms; others
or stability.
include palpable mass, myelopathy, radiculopathy, and
scoliosis.
4. Thoracic and lumbar spine account or approximately Suggested Readings
two-thirds o cases. The sacrum is the least likely
Boriani S, De Iure F, Campanacci L, et al. Aneurysmal
location (approximately 10%).
bone cyst o the mobile spine: report on 41 cases. Spine.
5. Embolization and resection are the primary treatment 2001;26:27-35.
options with f xation as needed to maintain stability. Papagelopoulus PJ, Currier BL, Shaughnessy WJ, et al.
Curettage and packing may also be employed. Aneurysmal bone cyst o the spine: management and
Conservative management may be considered as the outcome. Spine. 1998;23:621-628.
lesions generally stop growing a ter childhood and there
is no risk o malignant degeneration.
220
Back pain

1. In which location may these lesions occur?

2. What should be included in the di erential


diagnosis?

3. What are the etiologies?

4. What is included in the classif cation scheme?

5. What are the treatment options?

221
Arachnoid cyst 1483
Case ranking/dif culty: Category: Spinal canal

Sagittal T2 image demonstrates uid intensity mass in the dorsal Sagittal T1 image demonstrates uid intensity mass in the dorsal
spinal canal with mass e ect on and deformity of the cord. spinal canal with mass e ect on and deformity of the cord.

Answers
1. Intracranial arachnoid cysts are ar more common than Pearls
spinal arachnoid cysts. Within the spine, arachnoid • Arachnoid cysts are uncommon lesions that may occur
cysts may occur within the intradural extramedullary or within the extradural or intradural extramedullary
extradural compartments. Fluid collections within the spaces.
oramina are nerve sheath cysts; however, an intraspinal • They are usually located dorsal to the cord and o ten
lesion may extend through a oramen. extend over two to our vertebral segments.
2. Transdural spinal cord herniation occurs secondary to a • They are most common within the thoracic spine and
dural tear, usually at the level o a disc herniation. Facet generally ollow cerebrospinal uid signal on all MRI
synovial cyst tends to be more localized at the level o sequences. Heavily T2-weighted sequences can be
the acet joint. Dural ectasia also presents with vertebral help ul in evaluation.
body remodeling; however, this uid space reely • Filling o arachnoid cysts with contrast ollowing
communicates with the thecal sac. myelography is variable but usually delayed.
3. Arachnoid cyst is divided into primary and secondary.
Primary arachnoid cyst is elt to arise rom a congenital
Suggested Readings
diverticulum or dural de ect. Secondary arachnoid cysts
are secondary to in ammatory changes or dural tear. Hoy RJ, Faulder KC. Spinal arachnoid cysts. Aust Radiol.
1968;12:344-354.
4. The Nabors classif cation system: Kendall BE, Valentine AR, Keis B. Spinal arachnoid cysts:
Type 1A: Extradural arachnoid cyst clinical and radiological correlation with prognosis.
Type 1B: Sacral meningocele Neuroradiology. 1982;22:225-234.
Type 2: Tarlov cyst Kumar A, Sakia R, Singh K, Sharma V. Spinal arachnoid
cyst. J Clin Neuroscience. 2011;18:1189-1192.
Type 2: Spinal nerve root diverticulum Nakagawa A, Kusaka Y, Jokura H, Shirane R, Tominaga T.
Type 3: Intradural arachnoid cyst Use ulness o constructive inter erence in steady state
5. Complete resection is the pre erred treatment; (CISS) imaging or the diagnosis and treatment o a
however, i this is not easible, cyst enestration or large extradural spinal arachnoid cyst. Minim Invasive
marsupialization or shunt placement can be per ormed. Neurosurg. 2004;47:369-372.
222
Progressive bilateral lower extremity weakness

1. What should be included in the di erential


diagnosis?

2. What are common presenting symptoms?

3. What are the most help ul f ndings or


di erentiating this abnormality rom cord
in arction?

4. What is the classif cation system?

5. What are the treatment options?

223
Dural arteriovenous stula 1478
Case ranking/dif culty: Category: Spinal cord

Sagittal T2 image Parasagittal T2 Sagittal oblique contrast AP image from a selective right
demonstrates image demonstrates enhanced MRA image T6 intercostal artery injection
multiple ow voids multiple ow voids demonstrates tangle of demonstrates a tangle of vessels.
in the thecal sac. surrounding the abnormal vessels overlying the
thoracic cord and thecal sac.
hyperintensity and
enlargement of the
distal cord.
Pearls
Answers • Dural arteriovenous f stulas are the most common spinal
1. Linear areas o signal heterogeneity within the vascular mal ormation and are classif ed as Type 1.
cerebrospinal uid could be caused by dural arteriovenous • Patients are generally middle aged to elderly males
f stula, arteriovenous mal ormation, cerebrospinal uid who present with progressive lower extremity
pulsation arti act, nerve roots, and nerve sheath tumors. weakness, though to originate rom myelopathic
2. The most common presentation is slowly progressive changes rom venous hypertension.
lower extremity weakness; other symptoms include back • These lesions most o ten occur between the T5 and L3
pain, impotence, bowel and bladder incontinence, and levels.
subarachnoid hemorrhage. Hemorrhage is unusual. • Imaging workup usually consists o MRI, including
dynamic angiography i possible.
3. Both dural arteriovenous f stula and cord in arction can • Digital subtraction angiography can be used to ref ne
present with cord T2 hyperintensity and enhancement. delineation o eeding vessels, as well as per orm
Spinal cord in arction should have an acute presentation embolization o lesions.
and is less likely to have associated dilated pial veins. • Surgical resection or obliteration may also be required.
4. Type 1: Dural arteriovenous mal ormation
Type 2: Intramedullary glomus type arteriovenous
Suggested Readings
mal ormation
Type 3: Juvenile type arteriovenous mal ormation Da Costa L, Dehdashti AR, TerBrugge KG. Spinal cord
vascular shunts: spinal cord vascular mal ormations and
Type 4: Intradural extramedullary/perimedullary dural arteriovenous f stulas. Neurosurg Focus. 2009;26:E6.
arteriovenous f stula Muralidharan R, Saladino A, Lanzino G, Atkinson
5. Treatment options consist o onyx embolization and surgical JL, Rabinstein AA. The clinical and radiological
resection. Polyvinyl alcohol and Gel oam embolization are presentation o spinal dural arteriovenous f stula. Spine.
contraindicated given the high rate o recanalization. 2011;36:E1641-1647.
224
History of ovarian cancer with bilateral upper extremity weakness

1. What should be included in the di erential


diagnosis?

2. What are common presenting symptoms?

3. What location within the spinal cord is most


commonly a ected?

4. What are common imaging appearances?

5. What are the treatment options?

225
Intramedullary metastasis 1476
Case ranking/dif culty: Category: Spinal cord

Sagittal T2 image demonstrates heterogeneous, centrally necrotic Sagittal T1 image following gadolinium administration demonstrates
intramedullary mass extending from C2 to C5 with surrounding peripheral and nodular enhancement of the intramedullary lesion.
T2 hyperintensity, consistent with edema.

• The most common malignancies to metastasize to the


Answers spinal cord are lung and breast, but any primary may.
1. An enhancing intramedullary lesion with surrounding • Patients usually present with pain and weakness,
vasogenic edema can represent astrocytoma, which may be insidious or rapid in onset.
ependymoma, metastasis, demyelinating disease, and • There are no imaging eatures that clearly def ne a
hemangioblastoma. As in the brain, metastases o ten metastatic lesion, and biopsy is o ten required or
have edema out o proportion to lesion size. diagnosis.
2. Potential presenting symptoms include pain, myelopathy, • Metastases generally enhance and o ten have edema
bowel and bladder dys unction, paraparesis, and Brown- out o proportion to lesion size.
Sequard syndrome. • Spinal cord metastases a ect the cervical spine most
commonly and are usually solitary, although they can be
3. The cervical cord is most commonly a ected, ollowed seen in conjunction with other sites o metastatic disease.
by thoracic cord and then conus. • Treatment is geared at the primary malignancy with
4. Spinal cord metastases are generally solitary lesions that adjunct steroids to reduce cord edema.
have edema greater than would be expected or lesion • Surgical resection and radiotherapy may be considered
size. These lesions tend to be rapidly progressive and are in some patients to alleviate symptoms.
usually seen in patients with di use metastatic disease. • Overall survival ollowing diagnosis is poor, generally
5. Treatment options can include conservative less than 1 year.
management, analgesia, radiation, chemotherapy, and
surgical resection, depending on patient symptomatology
Suggested Readings
and overall prognosis.
Do-Dai DD, Brooks MK, Goldkamp A, Erbay S, Bhadelia
RA. Magnetic resonance imaging o intramedullary spinal
cord lesions: a pictorial review. Curr Probl Diagn Radiol.
Pearls 2010;39:160-185.
• While spinal cord metastases are rare, they should Rogers SR, Phalke VV, Anderson J, Riccelli LP, Gonda
be considered in the di erential o a cord lesion in a S, Pollock JM. HEALSME: Di erential diagnosis or
patient with a primary malignancy. intramedullary spinal cord lesions. Neurographics.
2012;2:13-26.
226
Motor cycle collision with left arm paralysis

1. What are common presenting symptoms?

2. What are etiologies o this abnormality?

3. What orms the brachial plexus?

4. What are associated abnormalities?

5. What are the treatment options?

227
Brachial plexus avulsion with pseudomeningocele 1496
Case ranking/dif culty: Category: Nerve roots/Nerve plexus/Peripheral nerves

Coronal T2 image demonstrates uid intensity collections arising Coronal T1 image demonstrates uid intensity collections arising
from the left T1 nerve root. from the left T1 nerve root.

Answers
1. The brachial plexus has both motor and sensory Pearls
components and both can be a ected by injury. • Brachial plexopathy symptoms tend to be vague and
Diaphragmatic paralysis can occur rom injuries nonspecif c.
involving the C3, C4, and C5 nerve roots. • Cases o neuropraxic (stretching) injuries and
avulsions tend to present with motor symptoms.
2. Potential mechanisms o brachial plexus abnormalities
• Pseudomeningoceles develop in approximately 80%
include neuropraxia, avulsion, inf ltration, and
o traumatic avulsions and result rom a tear in the
compression.
meningeal sheath with extravasation o cerebrospinal
3. During the f th to sixth week o gestation, the ventral uid.
rami o C5 to T1 orm the brachial plexus. • A radiologist should attempt to def ne the gap distance
between the avulsed nerve segments, which can be
4. Associated injuries can include spinal cord injury.
important in ormation or a surgeon planning potential
Diaphragmatic paralysis results rom injury to the
reanastomosis.
C3-C5 nerve roots. Scapular winging results rom injury
• In the neonatal population, consider brachial plexus
to C5 to C7. Horner syndrome results rom injury to
injuries related to delivery, which can result in Erb
in raclavicular plexus.
palsy (injury to C5 and C6) or Klumpke palsy (injury
5. For neuropraxic injuries, treatment consists o physical to C8 and T1).
therapy. I there is a brachial plexus avulsion, this can
be treated with either primary reanastomosis and/or
nerve gra ting depending on the gap distance. Prolonged
Suggested Readings
immobilization should be avoided because it can lead to
a rozen joint. Castillo M. Imaging the anatomy o the brachial
plexus: review and sel -assessment module. AJR.
2005;185:S196-S204.
Sureka J, Cherian RA, Alexander M, Thomas BP. MRI o
brachial plexopathies. Clin Radiol. 2009;64:208-218.

228
Scoliosis

1. What should be included in the di erential


diagnosis?

2. What are common presenting symptoms?

3. What are associated complications?

4. What are 5-year survival rates?

5. What are the treatment options?

229
Juvenile pilocytic astrocytoma 1493
Case ranking/dif culty: Category: Spinal cord

Sagittal T2 image demonstrates irregular uid collection within Sagittal T1 image following gadolinium administration demonstrates
the thoracic cord, which is expanded. intramedullary enhancing mass.

Answers
1. The di erential or multisegment T2 hyperintensity Pearls
within the cord includes astrocytoma, ependymoma, and • Astrocytoma is the most common spinal cord tumor
syringomyelia. Di erentiating syrinx rom underlying in children with the majority being low grade (WHO
tumor is an important role or contrast administration in grade I and II).
spinal imaging. • Holocord astrocytomas are less common and usually
demonstrate large cystic components.
2. The symptoms o holocord astrocytoma are nonspecif c
• Presenting symptoms are usually nonspecif c and
and include pain, progressive scoliosis, myelopathy,
include progressive scoliosis, pain, and myelopathy.
radiculopathy, and dysesthesia.
• Once the diagnosis is conf rmed, treatment can consist
3. Potential complications o holocord astrocytoma include o chemotherapy with consideration o surgical
permanent neurological def cit, disseminated metastatic resection and radiotherapy.
disease, osseous remodeling o the spinal canal, and • Large cysts associated with holocord astrocytoma can
scoliosis. mimic syrinx; this is the reason to give contrast to
identi y enhancing tumor.
4. For WHO grade I astrocytoma, the 5-year survival is
approximately 80%, in contrast to the higher-grade
tumors, which average 30%.
Suggested Readings
5. Microsurgical resection is o ten employed or debulking
ollowed by chemotherapy. However, radiation therapy Merchant TE, Kiehna EN, Thompson SJ, Heideman RL,
may be used in re ractory cases. Given their slow- San ord RA, Kun LE. Pediatric low-grade and ependymal
growing nature, conservative management might be spinal cord tumors. Pediatr Neurosurg. 2000;32:30-36.
appropriate. Schittenhelm J, Ebner FH, Tatagiba M, et al. Holocord
pilocytic astrocytoma—case report and review o the
literature. Clin Neurol and Neurosurg. 2009;111:203-207.

230
Back pain and a low-grade fever

1. What is this procedure called?

2. Which approaches are commonly used?

3. Why is this procedure per ormed?

4. What is the diagnostic yield o this procedure?

5. How can the diagnostic yield o this procedure


be improved?

231
Percutaneous disc aspiration and biopsy (PDAB) 3231
Case ranking/dif culty: Category: Disc

Axial CT image at the level of L3-L4 intervertebral disc. There CT-guided biopsy of L3-L4 intervertebral disc using a
are erosive endplate changes (arrow) and a small paravertebral posterolateral approach (arrowhead). The tip of the needle was
collection (arrow). Previous MRI of the lumbar spine showed advanced into the intervertebral disc (arrow) before three core
probable spondylodiscitis at L3-L4. samples were obtained.

Answers the vertebra be ore spreading to the disc. Sampling o


1. Spondylodiscitis commonly a ects elderly and paravertebral abscesses is o ten negative as the uid
immunocompromised patients and the clinical aspirated is usually sterile. Even when the diagnosis o
mani estations are nonspecif c o ten leading to a delay in spondylodiscitis is made on imaging, a microbiologic
the diagnosis. diagnosis is desirable as this helps target antibiotic
therapy according to drug sensitivities. Biopsy also
2. Two approaches, namely the transpedicular and provides a sample or histopathological assessment.
posterolateral approaches, are commonly used and one Histology may conf rm spondylodiscitis even when no
should aim to include both subchondral bone and disc specif c in ectious agent is isolated on microbiology and
material in the biopsy track. The procedure is per ormed a combination o the two is there ore the optimum.
under either uoroscopic or CT guidance. CT shows the
needle position better and is generally considered to be
a sa er option. In some institutions, percutaneous disc
Pearls
aspiration and biopsy or suspected spondylodiscitis has
become a routine procedure. • Spondylodiscitis commonly a ects elderly and
immunocompromised patients.
3. Although MR imaging is quite sensitive and specif c to • Biopsy should be per ormed be ore initializing
diagnose spondylodiscitis, it has certain limitations. Early antibiotic treatment.
stages o spondylodiscitis are notoriously di f cult to • Two approaches namely the transpedicular and
di erentiate rom Modic type 1 changes. Other di erential posterolateral approaches are commonly used and one
diagnoses, particularly in early stages, include dialysis- should aim to include both subchondral bone and disc
related disc changes, seronegative spondyloarthropathy, material in the biopsy track.
amyloidosis, and crystal deposition disease.
4. Microorganism are, however, only cultivated in about
57% o cases. Reasons or a negative culture include Suggested Readings
the initiation o antibiotic treatment prior to biopsy,
Michel SC, Pf rrmann CW, Boos N, Hodler J. CT-guided
insu f cient number o microorganisms in the specimen,
core biopsy o subchondral bone and intervertebral space
and the absence o living in ectious agents within the
in suspected spondylodiskitis. AJR Am J Roentgenol.
sampled tissue.
2006 Apr;186(4):977-980.
5. When the biopsy tract includes the subchondral bone, Phadke DM, Lucas DR, Madan S. Fine-needle aspiration
the diagnostic yield is increased substantially. This biopsy o vertebral and intervertebral disc lesions:
may be explained by the act that discitis secondary to specimen adequacy, diagnostic utility, and pit alls. Arch
hematogenous spread starts in the subchondral part o Pathol Lab Med. 2001 Nov;125(11):1463-1468.

232
Chronic backaches. HLA-B27 positive

1. What is the most likely diagnosis?

2. Which site is typically a ected?

3. Which part o the spinal column is typically


a ected?

4. Which imaging modality best images this


condition?

5. Why should these lesions not be overlooked?

233
Romanus lesion 3217
Case ranking/dif culty: Category: Vertebral body

Sagittal T2-weighted sequence of the Sagittal T1-weighted sequence of the Sagittal STIR sequence demonstrates
thoracic spine. There are multiple sharply thoracic spine shows focal areas of hyperintense vertebral corners (arrows)
marginated triangular T2 hyperintense T1 hyperintensities (arrows), which a ecting multiple thoracic vertebrae.
“corner” abnormalities (arrows). No correspond to the abnormalities seen on
associated osteophyte formation or the T2 sequence. This represents focal fatty
Schmorl nodes are seen. marrow due to chronic in ammation.

Answers 5. The sign should not be overlooked as it raises the


1. Romanus lesions are early spinal imaging f ndings in possibility o ankylosing spondylitis, which should
ankylosing spondylitis. The specif city and positive there ore be urther evaluated.
predictive value (PPV) o Romanus lesions are 96% and
92%, respectively.
2. They characteristically a ect the site o attachment Pearls
o the annulus f brosus f bers to the vertebral endplate. • Romanus lesions are early spinal imaging f ndings in
As the discovertebral junction is an enthesis, anterior ankylosing spondylitis.
and posterior spondylitis are e ectively a orm o enthesitis. • The MR corner sign re ers to a sharply marginated
3. The “corner sign” in ankylosing spondylitis is requently triangular nonerosive corner abnormality, which is
seen at the thoracolumbar junction as opposed to not associated with osteophyte ormation or Schmorl
degenerative corner lesions, which are more common in nodes.
the lumbar spine. • The most requent pattern o signal intensity in these
lesions corresponds to Modic type 2 changes.
4. MRI is more sensitive compared to radiography and can
detect changes in ankylosing spondylitis much earlier. The
MR corner sign re ers to a sharply marginated triangular
or less requently quadrant-shaped nonerosive corner Suggested Readings
abnormality, which is not associated with osteophyte Hermann KG, Altho CE, Schneider U, et al. Spinal changes
ormation or Schmorl nodes. The most requent pattern in patients with spondyloarthritis: comparison o MR
o signal intensity in these lesions corresponds to Modic imaging and radiographic appearances. Radiographics.
type 2 changes. Romanus lesions are o ten seen as ocal 2005;25(3):559-569.
T1 hyperintensities secondary to ocal atty marrow due to Kim NR, Choi JY, Hong SH, et al. “MR corner sign”: value
chronic in ammation. The “corners” appear hyperintense or predicting presence o ankylosing spondylitis. AJR Am
on both STIR and T2-weighted sequences. J Roentgenol. 2008;191(1):124-128.
234
Severe neck pain, fever, and weight loss

1. What is the most likely diagnosis?

2. Which symptoms are encountered in this


condition?

3. Which CNS mani estations are seen in this


condition?

4. Which imaging modality best images the


condition?

5. Name some risk actors or CNS involvement


in this disease.

235
Lymphoma 2998
Case ranking/dif culty: Category: Paraspinal so t tissue

Sagittal T2-weighted sequence of the Sagittal T1-weighted sequence of Axial T2 sequence con rms the large pre-
cervical and upper thoracic spine shows the cervical and upper thoracic spine vertebral soft tissue mass (arrow). Also note
a pre-vertebral soft tissue mass in the demonstrates a pre-vertebral soft tissue the conglomerate of lymph nodes along the
cervical region (arrowhead) consistent mass in the cervical region (arrowhead). right carotid sheath (arrowhead).
with a conglomerate of lymph nodes. The extent of vertebral marrow
There is obvious involvement of the involvement (arrow) is better appreciated
vertebral marrow (arrow). Note the on this sequence.
reversal of the cervical lordosis.
Answers 5. CNS involvement is estimated to happen in up to 10% o
1. Lymphoma is a relatively common heterogeneous group lymphoma patients. Immunodef ciency is the major risk
o malignancies that usually originates rom lymph nodes actor or developing CNS lymphoma.
and is subdivided into Hodgkin disease and non-Hodgkin
lymphoma (NHL) on the basis o pathology. Hodgkin
disease is primarily a nodal disease but extranodal Pearls
involvement has been described. Non-Hodgkin lymphoma • Lymphoma is a heterogeneous group o malignancies
is eight times more common than Hodgkin disease. that usually originates rom lymph nodes.
2. Lymphadenopathy is the primary presentation in • It is subdivided into Hodgkin disease and non-
lymphoma. Hodgkin lymphoma (NHL) on the basis o pathology.
Lymphoma may present with certain nonspecif c • Extranodal involvement may occur in the absence o
symptoms. These include a variety o systemic signif cant lymphadenopathy when it is regarded as
symptoms (the so-called B symptoms), which include primary extranodal lymphoma o the a ected organ.
ever, weight loss, and night sweats. • CNS involvement happens in up to 10% o lymphoma
patients.
Fatigue, dyspnea, itching, and anorexia are also
• Lymphoma is a known radiological “imitator” and
encountered.
should be included in the di erential diagnosis o
3. CNS lesions may include intracranial masses, almost any mass lesion.
leptomeningeal involvement, spinal cord compression,
and peripheral neuropathies. The latter may also
complicate chemotherapy. Suggested Readings
4. MRI is the imaging modality o choice in CNS Metser U, Lerman H, Blank A, Lievshitz G, Bokstein F, Even-
lymphoma and it is exquisitely sensitive to detect intra- Sapir E. Malignant involvement o the spine: assessment by
or extramedullary spinal involvement. Vertebral and 18F-FDG PET/CT. J Nucl Med. 2004 Feb;45(2):279284.
paravertebral involvement, invasion o the epidural space, Thomas AG, Vaidhyanath R, Kirke R, Rajesh A. Extranodal
and leptomeningeal disease may all be assessed. 18F-FDG lymphoma rom head to toe: part 1, the head and spine.
PET has been reported to be more sensitive than bone AJR Am J Roentgenol. 2011 Aug;197(2):350-356.
scintigraphy in patients with lymphoma and was shown Vanneuville B, Janssens A, Lemmerling M, de Vlam K,
to detect early bone marrow involvement be ore cortical Mielants H, Veys EM. Non-Hodgkin’s lymphoma
changes could be seen by bone scintigraphy. presenting with spinal involvement. Ann Rheum Dis. 2000
Lymphoma is a known radiological “imitator” and Jan;59(1):12-14.
should be included in the di erential diagnosis o almost
any mass lesion seen anywhere in the body.
236
Known case of thalassemia major. Incidental nding on MRI

1. What is the most likely diagnosis?

2. What should be included in the di erential


diagnosis?

3. In which conditions may EMH occur?

4. In which hematological disorders may EMH


occur?

5. In which organs does EMH occur?

237
Extramedullary hematopoiesis 2997
Case ranking/dif culty: Category: Paraspinal so t tissue

Note the di usely abnormal Coronal T1-weighted image showing two large Coronal T1-weighted image demonstrates
signal intensity of the paraspinal masses (arrows) in the lower thoracic splenomegaly (arrow) and abnormal bone
vertebral bone marrow region. Note again the abnormal vertebral bone marrow in the humeral heads (arrowheads).
(arrowheads), which marrow signal, which appears hypointense
appears hypointense and compared to the intervertebral discs.
heterogenous.

Answers
1. Extramedullary hematopoiesis (EMH) is a rare Pearls
myeloproli erative disorder o clonal origin seen in • Extramedullary hematopoiesis is a myeloproli erative
patients with prolonged anemias. It is a physiological disorder seen in prolonged anemias.
compensatory mechanism that leads to the ormation o • EMH may occur at various sites where hematopoiesis
normal blood cells outside the bone marrow when the occurred during etal li e.
latter is unable to meet the circulatory demands. • Extramedullary hematopoiesis is a rare cause o a
paraspinal mass.
2. Di erential diagnoses o a paraspinal lesion include
• The MR signal intensity o extramedullary
neurogenic tumors, lymphoma, metastasis, paravertebral
hematopoiesis varies depending on the activity o the
abscess, and lateral meningocele.
hematopoietic tissue.
3. EMH occurs with prolonged anemias including
hemolytic anemias, myeloproli erative disorders, and
neoplasia.
Suggested Readings
4. Extramedullary hematopoiesis (EMH) is seen in patients Georgiades CS, Neyman EG, Francis IR, Sneider MB,
with prolonged anemias, myeloproli erative disorders, Fishman EK. Typical and atypical presentations o
neoplasia, and ollowing marrow irradiation. extramedullary hemopoiesis. AJR Am J Roentgenol. 2002
5. EMH may occur in various sites including the spleen, Nov;179(5):1239-1243.
liver, kidneys, heart, lymph nodes, skin, thymus, breast, Ginzel AW, Kransdor MJ, Peterson JJ, Garner HW, Murphey
prostate, adrenal glands, ovaries, intestines, and the MD. Mass-like extramedullary hematopoiesis: imaging
CNS. During etal li e, hematopoiesis occurs in these eatures. Skeletal Radiol. 2012 Aug;41(8):911-916.
sites but stops just be ore birth. The extramedullary Kaleem A, Ansari S, Koirala R, Agarwal M, Chaudhary S.
hematopoietic tissues, however, retain their ability to Paraspinal and presacral extramedullary hematopoiesis:
produce red blood cells. a rare mani estation o polycythemia vera. Iran J Radiol.
2013 September;10(3):164-168.

238
Neck pain and upper extremity radicular pain

1. What is the most likely diagnosis?

2. What should be included in the di erential


diagnosis?

3. Which conditions orm part o the spectrum o


notochordal lesions?

4. Which imaging modality best images this


condition?

5. How does the condition usually present?

239
Benign notochordal cell tumor (BNCT) 2995
Case ranking/dif culty: Category: Vertebral body

Sagittal T2-weighted sequence of the cervical Sagittal T1-weighted sequence of the Contrast-enhanced sagittal T1-weighted
spine showing replacement of C5 vertebral cervical spine showing replacement of C5 sequence of the cervical spine showing
marrow with a homogenously hyperintense vertebral marrow with a homogenously replacement of C5 vertebral marrow
signal (arrow). Note that the a ected vertebra is hypointense signal (arrow). Note that the with a homogenously hypointense
not expanded. There are multilevel disc bulges in vertebral marrow should be brighter than signal (arrow). The intravertebral lesion
the mid to lower cervical spine (arrowheads) with the intervertebral disc on T1. does not enhance following contrast
cord impingement at C5-C6. The latter was likely administration.
responsible for the presenting symptoms.
include back pain, limitation o movement, and sensory
Answers (paresthesia and anesthesia) and motor disturbances.
1. Benign notochordal cell tumour (BNCT) is a benign
intravertebral lesion o notochord origin. BNCTs are
ound in the clivus or vertebral bodies in up to 20% Pearls
o autopsy series and are detected with increasing • Benign notochordal cell tumor (BNCT) is a benign
requency in vivo on cross-sectional imaging studies. intravertebral lesion o notochord origin.
2. The main di erential diagnoses include chordoma, • The spectrum o notochordal lesions includes
hemangioma, plasmacytoma, Paget disease, atypical chordoma, notochordal vestiges o the intervertebral
in ections, lymphoma, and metastatic disease. disc, and ecchordosis physaliphora.
• BNCTs are o ten ound as incidental f ndings on imaging.
3. The spectrum o notochordal lesions includes chordoma,
• MRI demonstrates a well-def ned homogenous T2
notochordal vestiges o the intervertebral disc, and
hyperintense and T1 hypointense lesion that does not
ecchordosis physaliphora.
extend beyond or destroy the a ected vertebra.
4. Small BNCTs may be radiologically occult. Larger • Di erential diagnoses include chordoma, vertebral
lesions, on the other hand, may demonstrate nonspecif c hemangioma, plasmacytoma, Paget disease, atypical
radiographic f ndings. Bone scintigraphy is typically in ections, lymphoma, and metastatic disease.
negative. BNCTs o ten appear as sclerotic lesions on CT.
MRI demonstrates a homogenous T2 hyperintense and
T1 hypointense lesion replacing the vertebral marrow. Suggested Readings
They appear as well-circumscribed lesions that do not
Amer H, Hameed M. Intraosseous benign notochordal cell
extend beyond or destroy the a ected vertebra.
tumor. Arch Pathol Lab Med. 2010 Feb;134(2):283-288.
Chordomas can o ten be distinguished radiologically Yamaguchi T, Iwata J, Sugihara S, et al. Distinguishing
by their aggressive growth as they cause extensive bone benign notochordal cell tumors rom vertebral chordoma.
destruction and so t tissue invasion. Unlike BNCTs, Skeletal Radiol. 2008 Apr;37(4):291-299.
chordomas enhance ollowing contrast administration. Yamaguchi T, Suzuki S, Ishiiwa H, Shimizu K, Ueda
5. BNCTs are o ten ound as incidental f ndings on Y. Benign notochordal cell tumors—a comparative
imaging. Less commonly larger lesions may become histological study o benign notochordal cell tumors,
symptomatic due to canal/exiting oraminal compromise. classic chordomas, and notochordal vestiges o etal
Clinical symptoms depend on the lesion location and intervertebral discs. Am J Surg Pathol. 2004;28(6):756-761.
240
Incidental nding on CT of the chest

1. What is the most likely diagnosis?

2. Which syndromes are associated with this


condition?

3. How can the condition mani est clinically?

4. Describe clinical and pathological


characteristics o this condition.

5. Which treatments may be benef cial in this


condition?

241
Hemivertebra 2981
Case ranking/dif culty: Category: Vertebral body

Coronal CT reconstructions show a wedge-shaped hemivertebra Sagittal CT reconstructions again demonstrate a wedge-shaped
in the lower thoracic spine (arrow, panel A) with a unilateral hemivertebra (arrow, panel A). A unilateral pedicle (arrow, panel B)
transverse process (arrow, panel B). The hemivertebra causes a and articulation with adjacent vertebrae is seen.
scoliotic curvature, convex to the left.
3) Nonsegmental i the hemivertebra is connected to
Answers both vertebrae above and below.
4) Incarcerated when the hemivertebra is joined to the
1. Hemivertebrae orm part o the spectrum o
adjacent levels by their pedicles.
segmentational anomalies and result rom ailure o
development o one hal o the vertebral body. Two Antenatal ultrasonography reveals an asymmetrical
lateral chondrif cation centers appear in developing vertebral body and distortion in the shape o the spine on
vertebrae during the sixth week o gestation. By the coronal and sagittal scanning.
seventh to eighth week o gestation the two centers unite 5. Treatment is conservative in asymptomatic cases. Spinal
to orm the primary ossif cation center o the vertebral cord decompression and vertebral stabilization are o ered
body. Failure o one o the chondrif cation centers to i the de ormity is progressive and or symptom relie .
develop results in a hemivertebra.
2. There are several recognized associations that include Pearls
cleidocranial dysostosis, Gorlin syndrome, VACTERL,
• Hemivertebrae orm part o the spectrum o
gastroschisis, Aicardi syndrome, and OEIS complex.
segmentational anomalies and result rom ailure o
3. A ected individuals are o ten asymptomatic. Neurologic development o one hal o the vertebral body.
problems are secondary to severe angulation o the spine, • Hemivertebrae may involve single or multiple spinal
instability o the spinal column, spinal canal stenosis, levels and are a common cause o congenital scoliosis
and subluxation or racture o the adjacent vertebrae. and kyphosis.
Symptomatic patients may present with motor (limb • The most common location o a hemivertebra is within
weakness or paralysis), sensory or autonomic dys unction the midthoracic spine.
(urinary or ecal incontinence), and back/neck pain. • A ected individuals are o ten asymptomatic.
4. Hemivertebrae may involve single or multiple spinal • Hemivertebrae are classif ed according to their
levels and are a common cause o congenital scoliosis orientation and attachment to adjacent vertebrae.
and kyphosis. The hemivertebra acts as a wedge within • Antenatal ultrasonography reveals an asymmetrical
the spinal column, resulting in a curvature away rom the vertebral body and distortion in the shape o the spine.
side where it is present. The most common location o a • Treatment is conservative in asymptomatic cases.
hemivertebra is within the midthoracic spine. • Spinal cord decompression and vertebral stabilization
Hemivertebrae are classif ed according to their are o ered i the de ormity is progressive and or
orientation (ie, dorsal, ventral, or lateral) and symptom relie .
attachment to adjacent vertebrae. The orientation o
the hemivertebra determines whether the anomalous Suggested Readings
curvature results in kyphosis, scoliosis, or lordosis.
Humbert L, Ste en JS, Vialle R, Dubousset J, Vital JM,
The attachment is described in terms o segmentation as Skalli W. 3D analysis o congenital scoliosis due to
ollows: hemivertebra using biplanar radiography. Eur Spine J.
1) Fully segmental when the hemivertebra is not 2013 Feb;22(2):379-386.
attached to either the vertebra above or below. McMaster MJ, David CV. Hemivertebra as a cause o
2) Semisegmental when the hemivertebra is used with scoliosis. A study o 104 patients. J Bone Joint Surg Br.
one o the adjacent vertebrae with no intervening disc. 1986 Aug;68(4):588-595.
242
Previousscreening
Routine resection of a posterior fossa tumor. Now presented with loss
of sensation in the lower limbs and impaired proprioception

1. What is the most likely diagnosis?

2. What is the pattern o inheritance o this


disease?

3. Which tumors are encountered in this disease?

4. Visceral cysts in this disease may a ect:

5. Which o the ollowing are diagnostic o this


condition?

243
Von Hippel-Lindau disease 2976
Case ranking/dif culty: Category: Thecal sac

Non contrast T1-weighted Contrast-enhanced T1- T2-weighted sequence of the T2-weighted sequence of the
sequence of the cervical weighted sequence of the cervical and upper thoracic lower thoracic and lumbar
and upper thoracic spine. cervical and upper thoracic spine shows evidence of spine shows a hypointense
It is di cult to identify the spine shows multiple avidly previous posterior fossa extramedullary lesion on
intradural lesions (arrows) enhancing intradural but surgery (arrowhead) for a the dorsal aspect of the cord
on this sequence, but one extramedullary lesions hemangioblastoma and a (arrow) with adjacent abnormal
can make out the abnormal (arrows) compatible with cervical cord syrinx (arrow). ow voids (arrowheads), which
adjacent ow voids. multiple hemangioblastomas. correspond to abnormal
draining/feeding vessels.

Answers
Pearls
1. von Hippel-Lindau (vHL) disease is an autosomal dominant
disorder caused by germline mutations in the vHL tumor • vHL is an autosomal dominant disorder.
suppressor gene located on chromosome 3p25-26. • CNS hemangioblastomas are a cardinal eature o vHL.
• Frequent tumors in vHL are CNS hemangioblastomas
2. vHL is an autosomal dominant disease. It demonstrates
and renal cell carcinomas.
marked phenotypic variability and age-dependent
• Less requent tumors include pheochromocytomas,
penetrance.
pancreatic islet tumors, endolymphatic sac tumors, and
3. The most requent tumors in vHL are retinal and CNS head and neck paragangliomas.
hemangioblastomas and renal cell carcinomas. • Visceral cysts are very common and may a ect
Pheochromocytomas, pancreatic islet tumors, multiple body systems.
endolymphatic sac tumors, and head and neck • vHL may be urther subclassif ed into Type 1, Type
paragangliomas are less requently encountered tumors. 2A, and Type 2B.
4. Visceral cysts in vHL are ound in bone, kidneys,
liver, pancreas, and epididymis. The spleen, omentum, Suggested Readings
mesentery, adrenal glands, and lungs may also be a ected.
Maher E, Neumann H, Richard S. von Hippel-Lindau
5. The diagnostic criteria o vHL disease are: disease: a clinical and scientif c review. Eur J Hum Genet.
a) More than one CNS hemangioblastoma June 2011;19(6):617-623.
Zhang Q, Ma L, Li W, Chen J, Ju Y, Hui X. Von Hippel-
b) One hemangioblastoma plus a visceral mani estation Lindau disease mani esting disseminated leptomeningeal
o vHL hemangioblastomatosis: surgery or medication? Acta
c) One visceral mani estation plus a known amily history Neurochirurgica. 2011 Jan;153(1):48-52.
244
Four-week history of severe back pain, now presented with acute
paraplegia

1. What is the most likely diagnosis?

2. Which eatures are typical o this condition?

3. Pulmonary involvement can be demonstrated in:

4. Which imaging modality best images this


condition?

5. Which site is pre erentially a ected?

245
Tuberculous spondylitis 2955
Case ranking/dif culty: Category: More than one category

Sagittal T2-weighted sequence of the Axial T2-weighted sequence at Coronal T1-weighted sequence of the
lower thoracic and lumbar spine showing the level of T11 vertebral body thoracic spine shows large bilateral
edematous changes in T9-T12 vertebral shows a large paraspinal abscess paraspinal collections with ring enhancement
bodies in keeping with spondylitis. Note (arrowhead), which extends through in keeping with abscess formation (arrow).
the large abscess in the anterior paraspinal the intervertebral foramina into Note the loss of disc space at T10-T11
space (arrow), and a smaller epidural the lateral recesses (arrow) causing (arrowhead) and the marrow edema in the
collection posteriorly (arrowhead). narrowing of the spinal canal. adjacent vertebral bodies.

Answers The earliest sign is involvement o the anterosuperior


1. Tuberculosis is one o the commoner in ections o the vertebral endplate or subtle irregularity o the anterior
spine and its incidence is on the increase due to the vertebral body. Focal areas o erosion and osseous
development o multiple drug-resistant strains. destruction can be seen on plain radiography at these sites.

2. The in ection usually spreads beneath the longitudinal 5. Spinal involvement results rom hematogenous seeding
ligaments to a ect multiple (o ten contiguous) vertebrae. to the vertebral bodies. There is pre erential involvement
This is re erred to as “subligamentous” spread and o the thoracolumbar region, with the cervical spine and
pre erentially occurs beneath the anterior longitudinal sacrum being rarely involved.
ligament. The posterior vertebral elements are hence
o ten spared. Slow collapse o the a ected vertebrae
results in an acute kyphotic angulation or “gibbus” Pearls
de ormity. Cord compression may result rom this • The incidence o tuberculous spondylitis is on the
angulation coupled with the ormation o large epidural increase worldwide due to the development o
collections and bone ragments. Tuberculous spondylitis multiple drug-resistant strains.
may result in large paraspinal abscesses, sometimes • Spinal involvement results rom hematogenous
being completely painless and without rank pus, when seeding to the vertebral bodies.
they are re erred to as “cold abscesses.” In ection • MRI is the imaging modality o choice as it delineates
limited to a single vertebra is less common and results in the extent o in ectious involvement, the presence and
vertebral collapse and development o a vertebra plana size o epidural collections, and canal compromise.
de ormity.
3. Pulmonary tuberculous involvement can only be
demonstrated in up to 50% o cases. Suggested Readings
4. MRI is the imaging modality o choice as it delineates Dagirmanjian A, Schils J, McHenry M, Modic MT. MR
the extent o in ectious involvement, the presence and imaging o vertebral osteomyelitis revisited. AJR Am J
size o epidural collections, and canal compromise. CT Roentgenol. 1996 Dec;167(6):1539-1543.
can demonstrate anterior vertebral body destruction or Gouliamos AD, Kehagias DT, Lahanis S, et al. MR imaging
collapse, narrowing o the intervertebral disc, and the o tuberculous vertebral osteomyelitis: pictorial review.
presence o paraspinal collections. Eur Radiol. 2001;11(4):575-579.

246
Recurrent meningitis

1. What is the most likely diagnosis?

2. Where is the anomaly requently seen?

3. Which clinical f ndings may be seen in this


condition?

4. Which vertebral anomalies are associated with


this condition?

5. Name potential complications o this condition.

247
Dorsal dermal sinus with epidermoid 2954
Case ranking/dif culty: Category: More than one category

Sagittal T2-weighted sequence shows Sagittal T1-weighted sequence again Consecutive axial T1-weighted images at the level
a linear hypointense tract extending demonstrates a linear tract (arrow) of the sacral bone demonstrates a hypointense
from the skin to the sacral canal crossing the subcutaneous tissues of tract extending from the skin surface (arrow, panel
(arrow) compatible with a dorsal the back to reach the spinal canal. The A), across the subcutaneous fat (arrow, panel B)
dermal sinus. There is an associated oval-shaped lesion in the lumbar canal to reach the sacral canal (arrowhead, panel B and
hyperintense “sausage-shaped”T2 also follows the CSF signal on T1 and arrow, panel C).
hyperintense lesion in the lower represents an epidermoid (arrowhead).
lumbar canal (arrowhead).

Answers 5. Complications include CSF leaks and recurrent


1. A dorsal dermal sinus is a rare orm o spinal in ections leading to meningitis, epidural or subdural
dysraphism in which an anomalous epithelial-lined tract abscess ormation, or less commonly a cord abscess.
extends rom the skin to the spinal cord, cauda equina, Meningitis can be either pyogenic due to bacterial ascent
or arachnoid. It results rom ailure o the cutaneous and rom the skin along the tract, or less requently chemical
neural ectoderm to separate during neurulation resulting meningitis secondary to the release o cholesterol
in a ocal segmental adhesion. crystals rom spinal inclusion tumors.
2. Dorsal dermal sinuses are commonly encountered in
the lumbosacral region. Less o ten they are seen in
the occipital region. The sinus is lined by stratif ed
Pearls
squamous epithelium and extends rom the skin sur ace
to either terminate superf cially within the subcutaneous • A dorsal dermal sinus is a rare orm o spinal
layers or else extends deeply through the ascia and dysraphism.
vertebrae to communicate directly with the thecal sac or • Up to 50% end in an associated spinal dermoid,
intradural compartment. epidermoid, or lipoma.
3. A small dimple along the back may be ound on
clinical examination and they are o ten associated with
hyperpigmentation o the overlying skin, a hairy nevus, Suggested Readings
and capillary angioma. The sinus ostium is seen in a Cox EM, Knudson KE, Manjila S, Cohen AR. Unusual
midline location or more rarely in a paramedian location. presentation o congenital dermal sinus: tethered spinal
4. Up to 50% o dorsal dermal sinuses end in an associated cord with intradural epidermoid and dual paramedian
spinal dermoid, epidermoid, or lipoma. The latter may cutaneous ostia. Neurosurg Focus. 2012 Oct;33(4):E5.
compress the adjacent neural structures resulting in Unsinn KM, Geley T, Freund MC, Gassner I. US o the
neurologic symptoms. Apart rom inclusion tumors, the spinal cord in newborns: spectrum o normal f ndings,
condition is also associated with spinal teratoma, split-cord variants, congenital anomalies, and acquired diseases.
mal ormations, and tethered spinal cord. Radiographics. 2000 Jul-Aug;20(4):923-938.

248
Poor oral intake, lethargy, and fever

1. What is the prevertebral so t tissue space?

2. What should be included in the di erential


diagnosis?

3. Describe the normal measurements o the


prevertebral space.

4. How should the radiograph be acquired?

5. Describe other so t tissue signs on the lateral


cervical radiograph.

249
Retropharyngeal abscess 2953
Case ranking/dif culty: Category: Paraspinal so t tissue

4. The patient’s neck should be in the extended position


and the radiograph should be obtained in end inspiration.
The prevertebral so t tissues may appear alsely widened
i the patient is swallowing at the time o exposure or i
the neck is exed resulting in a “pseudomass.”
5. Other so t tissue signs should be assessed when
evaluating the lateral cervical spine radiograph.
1) There is normally a “step-o ” between the posterior
wall o the trachea and the posterior pharyngeal wall
at the level o the larynx, which usually corresponds
to C4 level. This may be lost in the presence o
retropharyngeal cellulitis or abscess ormation.
2) The air–so t tissue inter ace between the posterior
Lateral radiograph of the cervical spine in an 18-month-old girl. pharynx and the prevertebral so t tissues should also
There is abnormal widening of the prevertebral soft tissue space be assessed. The inter ace is normally very sharp
(arrow) with anterior displacement of the pharynx and trachea at but may become indistinct in the presence o an
the level of C3-C4 vertebral bodies. in ammatory process in the retropharyngeal space.
3) The contour o the prevertebral so t tissue space
should ollow the contour o the anterior aspect o the
Answers cervical spine.
1. The prevertebral so t tissue space should always be
assessed care ully on a lateral radiograph o the cervical
spine, particularly in the setting o trauma where abnormal Pearls
widening o the space may point to serious spinal injury. • The prevertebral so t tissue space should always
The retropharyngeal space and the prevertebral so t be assessed care ully on a lateral radiograph o the
tissue space are two distinct spaces. The ormer extends cervical spine, particularly in the setting o trauma.
rom the skull base to the level o the carina. It is located • The patient’s neck should be in the extended position and
between the prevertebral ascia and the buccopharyngeal the radiograph should be obtained in end inspiration.
mucosa. The prevertebral so t tissue space is a potential • The retropharyngeal space and the prevertebral so t
space between the anterior aspect o the cervical spine tissue space are two distinct spaces.
and the prevertebral ascia. • The prevertebral space at C2 should measure less than
7 mm or <50% o the width o C2 body.
2. Assessment o the thickness and contours o the • At C6, the prevertebral space should not be wider than
prevertebral so t tissues is also important in the 22 mm or the width o C6 vertebral body.
evaluation o in ectious conditions o the retropharyngeal • Assessment o the thickness and contours o the
space. The prevertebral so t tissue space may also be prevertebral so t tissues is also important in the evaluation
widened in the presence o a neoplastic or in ammatory o in ectious conditions o the retropharyngeal space.
mass. • The prevertebral so t tissue space may also be widened
3. The prevertebral space is usually measured at C2 and in the presence o a neoplastic or in ammatory mass.
C6 levels. At the ormer level the space should measure • Other so t tissue signs should be assessed when
less than 7 mm or <50% o the width o C2 body. At C6, evaluating a lateral cervical spine radiograph.
the prevertebral space should not be wider than 22 mm
or the width o C6 vertebral body. Various studies have,
however, demonstrated that these measurements should Suggested Readings
only be used as a guide and so t tissue abnormalities DeBehnke DJ, Havel CJ. Utility o prevertebral so t tissue
should always be interpreted in light o other clinical measurements in identi ying patients with cervical spine
signs, osseous f ndings, and the mechanism o injury. It ractures. Ann Emerg Med.1994 Dec;24(6):1119-1124.
was also shown using these measurements in isolation Matar LD, Doyle AJ. Prevertebral so t-tissue measurements
results in a large proportion o cervical spine ractures in cervical spine injury. Australas Radiol. 1997
being missed. Aug;41(3):229-237.

250
Failure to thrive

1. What are the pertinent radiologic f ndings?

2. What conditions are included in the


di erential, and what is the most likely
diagnosis?

3. What is the pathogenesis or this entity?

4. Which orm o the disease is most severe?

5. What are some o the complications o this


entity?

251
Osteopetrosis 2947
Case ranking/dif culty: Category: Vertebral body

Another patient. Generalized increase in bone density with Another patient. Generalized increase in bone density.
Erlenmeyer ask deformity. All images courtesy of Dr. Akbar Bonakdarpour.

Answers
1. There is a generalized increase in bone density with Pearls
sclerosis in the base o skull and “sandwich” vertebrae. • Osteopetrosis is a disease resulting rom a ailure o
Although ractures are requent, none are demonstrated osteoclastic activity.
here. • There is an increase in bone density; however,
the bones are weaker and prone to ractures and
Medullary sclerosis with relative sparing o the cortices,
complications o ractures.
and Erlenmeyer ask de ormities, is typically seen in the
• Two main orms exist: the more severe in antile
extremities.
autosomal recessive orm and the milder adult
2. Lead toxicity, pyknodysostosis, Paget disease, autosomal dominant orm.
osteoblastic metastasis, and osteopetrosis are all in • Adult orms may be diagnosed incidentally, or with
the di erential or osteosclerosis. Given the age o the mild anemia.
patient, the most likely diagnosis is osteopetrosis. • The bones are dense with medullary sclerosis and
relative sparing o the cortices. Erlenmeyer ask
3. Osteopetrosis is a result o a ailure o osteoclasts to
de ormity can be seen.
resorb bone.
• Adult Type 1 orm shows uni orm increase in bone
4. The in antile autosomal recessive variant o osteopetrosis density o the long bones, spine, and skull.
has a much higher morbidity and mortality. • Adult Type 2 orm shows the classic “bone in
bone” appearance, with “sandwich vertebrae.” The
5. The bones are dense in osteopetrosis, but weak.
pelvis, spine, and skull base are o ten a ected.
There ore, ractures and racture complications such as
delayed/nonunion as well as osteomyelitis may occur.
Encroachment on the medullary space may result in
anemia. Other complications include nerve compression Suggested Readings
and abnormal dentition. Ihde LL, Forrester DM, Gottsegen CJ, et al. Sclerosing bone
dysplasias: review and di erentiation rom other causes o
osteosclerosis. Radiographics. 2011;31(7):1865-1882.
Vanhoenacker FM, De Beuckeleer LH, Van Hul W, et al.
Sclerosing bone dysplasias: genetic and radioclinical
eatures. Eur Radiol. 2000 Nov;10(9):1423-1433.

252
Quadriplegia of an acute onset

1. What is the most likely diagnosis?

2. What should be included in the di erential


diagnosis?

3. Which MR eatures are typical o this


condition?

4. Which imaging modality best images this


condition?

5. Describe clinical and pathological


characteristics o this condition.

253
Spinal cavernoma 2946
Case ranking/dif culty: Category: Spinal cord

There is an oval-shaped intramedullary lesion in the cervical Axial MRI sequences (panel A = T2; panel B = T1) of the cervical
cord (arrow, panel A), which expands the cord. The lesion is spine at the level of the odontoid peg con rms the intramedullary
surrounded by a hypointense hemosiderin rim (arrowhead, location of the lesion, which is surrounded by a hypointense
panel A). The lesion (arrow, panel B) demonstrates subtle hemosiderin rim (arrows, panels A and B). Hyperintense T1 signal
contrast enhancement (arrowhead, panel C). is noted within the lesion (arrowhead, panel B).

Answers Presentation may be acute secondary to intramedullary


1. Spinal cavernomas are well-circumscribed vascular hemorrhage or chronic when it mani ests as slowly
mal ormations composed o closely packed capillary- progressive neurological decline. This progressive
like vessels with no intervening spinal tissue. myelopathy is the result o repetitive microhemorrhages
with ormation o peri ocal hemosiderosis and reactive
2. Intramedullary lesions including primary spinal tumors gliosis. Clinical presentation also varies according to the
(eg, astrocytoma, ependymoma, and hemangioblastoma), location o the cavernoma.
metastatic lesions, and in ammatory lesions
(demyelination and transverse myelitis) should be
included in the di erential diagnosis.
Pearls
3. Cavernomas do not have prominent vascular supply or
• Cavernomas are vascular mal ormations composed o
venous drainage and are angiographically occult lesions.
closely packed capillary-like vessels.
Magnetic resonance imaging is virtually diagnostic
• Spinal cavernomas account or about 5% o all CNS
or spinal cavernomas. They are seen as having mixed
cavernomas.
signal intensity on T1-weighted sequences and are
• Presentation may be acute secondary to intramedullary
surrounded by a prominent hemosiderin ring on both T1-
hemorrhage or chronic when it mani ests as slowly
and T2-weighted images. A pathognomonic “popcorn”
progressive neurological decline.
appearance o mixed hyperintensity and hypointense
• Magnetic resonance imaging is virtually diagnostic
blood-containing locules may also be demonstrated.
or spinal cavernomas—a “popcorn” appearance is
4. Magnetic resonance imaging is virtually diagnostic o pathognomonic.
spinal cavernomas. The lesion is occult angiographically
and can be easily overlooked on CT.
5. Spinal cavernomas account or about 5% o all Suggested Readings
cavernomas and are clinically more aggressive than their Hegde A, Mohan S, Tan KK, Lim CC. Spinal cavernous
cranial counterparts. Their location is o ten precarious mal ormations: magnetic resonance imaging
and they are there ore more likely to be symptomatic and associated f ndings. Singapore Med J. 2012
compared to cranial cavernomas. Sep;53(9):582-586.
Sporadic and amilial orms have been described. The Kivelev J, Niemelä M, Hernesniemi J. Characteristics o
ormer o ten presents with a single lesion, whereas cavernomas o the brain and spine. J Clin Neurosci. 2012
multiple lesions characterize the inherited orm. May;19(5):643-648.
See-Sebastian EH, Marks ER. Spinal cord intramedullary
cavernoma: a case report. W V Med J. 2013 May-
Jun;109(3):28-30.

254
Bilateral sensorineural loss and peripheral neuropathy

1. What is the most likely diagnosis?

2. Name the diagnostic criteria o this syndrome?

3. Which conditions are considered


“neurocutaneous syndromes?”

4. Which pattern o inheritance does this


condition have?

5. Which symptoms may occur in this condition?

255
Neuro bromatosis type 2 2945
Case ranking/dif culty: Category: Nerve roots/Nerve plexus/Peripheral nerves

Contrast-enhanced scan at the level of the Post-contrast T1-weighted sequence Bilateral enhancing cerebellopontine angle
posterior fossa shows multiple enhancing demonstrates an intramedullary masses (arrowheads). The lesions extend through
extraaxial lesions including a vestibular enhancing lesion, likely an the porus acusticus into the internal auditory
schwannoma (arrowhead) and lower ependymoma (arrow) and multiple canals (arrows). Bilateral acoustic schwannomas
cranial nerve schwannomas (arrows). intradural but extramedullary lesions are diagnostic of NF-2.
(arrowheads).

Answers
1. Multiple schwannomas, meningiomas, and ependymomas Pearls
are diagnostic o neurof bromatosis type 2. The condition • NF-2 is an autosomal dominant multiple neoplasia
is sometimes re erred to as “MISME syndrome.” syndrome that results rom mutations o the NF-2
tumor suppression gene located on chromosome 22.
2. Ferner et al described three sets o diagnostic criteria or
• A ected individuals are at a propensity o developing
neurof bromatosis type 2:
multiple CNS tumors, ophthalmological lesions,
1) Bilateral vestibular schwannomas or amily history o cutaneous lesions, and peripheral neuropathy.
NF-2 plus unilateral vestibular schwannoma or any two
o meningioma, glioma, neurof broma, schwannoma,
and posterior subcapsular lenticular opacities. Suggested Readings
2) Unilateral vestibular schwannoma plus any two o
Asthagiri AR, Parry DM, Butman JA, et al. Neurof bromatosis
meningioma, glioma, neurof broma, schwannoma,
type 2. Lancet. 2009 Jun 6;373(9679):1974-1986. Review.
and posterior subcapsular lenticular opacities.
Evans DG. Neurof bromatosis type 2 (NF2): a clinical and
3) Two or more meningioma plus unilateral vestibular
molecular review. Orphanet J Rare Dis. 2009 Jun 19;4:16.
schwannoma or any two o glioma, schwannoma, and
Review.
cataract.
Ferner, Rosalie E, Susan M, Huson D. Gareth R. Evans.
3. Neurocutaneous syndromes include neurof bromatosis, Neurof bromatoses in clinical practice. Springer, 2011.
tuberous sclerosis, Sturge-Weber syndrome, von Hippel-
Lindau disease, ataxia telangiectasia, Wyburn-Mason
syndrome, incontinentia pigmenti, and nevoid basal cell
carcinoma syndrome.
4. The incidence o NF-2 is about 1 in 50,000. Hal o the
cases are secondary to de novo mutations. The a ected
gene is located on chromosome 22.
5. Vestibular schwannomas o ten present with neurosensory
hearing loss, tinnitus, and headaches. Mass e ect in the
cerebellopontine angles may also result in acial nerve
symptoms.

256
Acute paraparesis and sphincter dysfunction

1. What is the most likely diagnosis?

2. Which underlying causes have been implicated


in this condition?

3. What is a variant o this condition that a ects


three or more vertebral segments called?

4. Which imaging modality best images this


condition?

5. Name some inclusion criteria proposed by


the Transverse Myelitis Consortium Working
Group.

257
Transverse myelitis 2942
Case ranking/dif culty: Category: Spinal cord

Sagittal T2-weighted Sagittal T1-weighted Contrast-enhanced sagittal Axial T2-weighted sequence at the level of
sequence of the thoracic sequence of the thoracic T1-weighted sequence of T10 con rms the intramedullary location
spine demonstrates spine. Note that the the thoracic spine showing of the lesion (arrow). Note that the lesion
an intramedullary intramedullary lesion subtle enhancement of a ects more than 2/3 of the cross-sectional
hyperintense lesion (arrow) (arrow) at T9-T10 is barely the intramedullary lesion area of the cord.
at T9-T10, which causes seen. (arrow).
slight cord expansion.

Answers - A clearly def ned sensory level


1. Transverse myelitis is a rare in ammatory disorder o the - Exclusion o extraaxial compressive causes by
spinal cord that results in rapidly progressive weakness, neuroimaging
sensory alterations, and autonomic dys unction. - In ammation within the spinal cord as
demonstrated by CSF pleocytosis, increased IgG
2. The etiology o transverse myelitis is protean and index, or gadolinium enhancement
includes viral in ections (eg, CMV), spinal cord injury,
immune reactions, schistosomiasis, and spinal blood b) Exclusion criteria:
ow alterations. - Previous radiation to the spine in the last 10 years
- Arterial distribution clinical def cit consistent with
3. The lesions may a ect anywhere along the spinal
thrombosis o the anterior spinal artery
cord and are usually restricted to a small cord portion.
- Demonstration o a spinal AVM
A variant o the condition is termed longitudinally
extensive transverse myelitis (LETM), which extends
over three or more vertebral segments.
Pearls
4. MRI is the imaging investigation o choice and is
essential to rule out di erential diagnosis. The lesion • Transverse myelitis results in rapidly progressive
is o ten seen as an ill-def ned T2 hyperintense ocus weakness, sensory alterations, and autonomic
with variable enhancement post-contrast administration. dys unction.
The a ected cord segment is expanded and the lesion • Diagnostic criteria have been proposed by the
typically a ects more than two-thirds o the cross Transverse Myelitis Consortium Working Group and
section o the cord. include inclusion and exclusion criteria.

5. Diagnostic criteria have been proposed by the


Transverse Myelitis Consortium Working Group and Suggested Readings
include inclusion and exclusion criteria as ollows:
Desanto J, Ross JS. Spine in ection/in ammation. Radiol
a) Inclusion criteria: Clin North Am. 2011;49(1):105-27.
- Development o sensory, motor, or autonomic Transverse Myelitis Consortium Working Group. Proposed
spinal dys unction diagnostic criteria and nosology o acute transverse
- Bilateral signs and/or symptoms myelitis. Neurology. 2002;59 (4):499-505.

258
Progressive lower limb weakness

1. What is the most likely diagnosis?

2. What should be included in the di erential


diagnosis?

3. How can the condition mani est clinically?

4. Which imaging modality best images this


condition?

5. Which imaging f ndings may be seen in this


condition?

259
Neurosarcoidosis 2941
Case ranking/dif culty: Category: Meninges/Nerve sheath

Post-contrast axial T1 images of the lumbar spine. Note di use


nodular (arrows) leptomeningeal enhancement of the cauda
equina roots.

4. MRI is the imaging modality o choice. Contrast-


Sagittal T2-weighted sequence Contrast-enhanced sagittal enhanced T1-weighted sequences are essential as subtle
of the lumbar spine. The T1-weighted sequence f ndings may be otherwise overlooked. Leptomeningeal
conus terminates at L1 level. of the lumbar spine. enhancement can be ocal or di use.
Note abnormal thickening There is abnormal thick
ventral to the conus medullaris leptomeningeal enhancement 5. Spinal involvement may be urther subclassif ed into
(arrowhead) and along the at the level of the conus 1) Intradural extramedullary f ndings including
cauda equina roots (arrow). The (arrow) and along the cauda leptomeningeal enhancement, isointense-enhancing
vertebrae are normal in height equina roots (arrowhead).
root nodules, and clumping o cauda equina roots.
with preserved marrow signal.
2) Extradural f ndings include spondylodiskitis and
paraspinal masses.
Answers 3) Intramedullary sarcoidosis occurs in less than 1% o
cases and causes severe neurologic sequelae.
1. Sarcoidosis is a multisystem granulomatous disease
characterized by the ormation o noncaseating
granulomas. Neurosarcoidosis re ers to the involvement
o the central nervous system by sarcoidosis. Although Pearls
the CNS may be involved in up to 25% o patients
su ering rom systemic sarcoidosis, only 5% o those • Neurosarcoidosis re ers to the involvement o the
a ected are symptomatic. central nervous system by sarcoidosis—a multisystem
granulomatous disease characterized by the ormation
2. The diagnosis o neurosarcoidosis is challenging as o noncaseating granulomas.
both the clinical symptoms and radiological f ndings • MRI is the imaging modality o choice.
are nonspecif c and may mimic other conditions. • Early treatment with corticosteroids reduces the
Di erential diagnoses include in ectious, granulomatous, neurologic complications and disease morbidity.
demyelinating, neoplastic, and connective tissue
disorders. Def nitive diagnosis o neurosarcoidosis can
only be made by biopsy, which demonstrates epithelioid
granulomas without caseation or staining or in ectious Suggested Readings
agents. Nozaki K, Judson MA. Neurosarcoidosis. Curr Treat Options
3. Clinical mani estations o neurosarcoidosis depend on Neurol. 2013 Aug;15(4):492-504.
which site is a ected. Cranial nerve palsies, symptoms Smith JK, Matheus MG, Castillo M. Imaging mani estations
o increased intracranial pressure, pituitary hormone o neurosarcoidosis. AJR Am J Roentgenol. 2004
def ciency, seizures, motor and sensory dys unctions Feb;182(2):289-295.
may be seen. Spinal cord involvement presents with
lower extremity weakness and other signs o myelopathy.

260
Low back pain with sciatic radiation

1. What should be included in the di erential


diagnosis?

2. What is the most likely diagnosis?

3. What is the classic imaging appearance o this


condition?

4. Where do these lesions tend to occur?

5. What is the WHO classif cation o these


lesions?

261
Myxopapillary ependymoma 2940
Case ranking/dif culty: Category: Filum

Sagittal T2-weighted image shows an oval-shaped heterogeneous mass at the level of


L2 vertebral body. The mass is intradural and arises from the lum terminale. A subtle
hypointense rim along the caudal aspect of the lesion and coarse hypointense foci at the
inferior tumoral pole (arrows, Image A) represent blood products and are frequent ndings
in spinal ependymomas. The lesion enhances homogenously after contrast administration
(Image B = precontrast T1; Image C = post-contrast T1) and re ects the hypervascular nature
of the tumor. Image D clearly shows the intradural location of the tumor.

Answers
1. Schwannomas, paragangliomas, and metastases are Pearls
o ten intradural extramedullary lesions that may • Myxopapillary ependymomas are WHO grade I
be ound in the region o the f lum terminale. Disc tumors that arise rom the f lum terminale and account
prolapse is extradural, whereas cavernomas are usually or 13% o all ependymomas.
intramedullary. • They a ect a younger population with a mean
presentation age o 36 years.
2. Myxopapillary ependymoma is a slow-growing tumor
• Myxopapillary ependymomas are intradural
(WHO grade I) that arises predominantly in the region o
extramedullary masses that enhance post-contrast
the f lum terminale.
administration.
3. Myxopapillary ependymomas typically appear as a • Di erential diagnoses include other subtypes o
hyperintense sausage-shaped mass in the region o the ependymoma, astrocytoma, hemangioblastoma, and
f lum terminale. Intratumoral hemorrhage is common, nerve sheath tumors.
which may result in characteristic hemosiderin caps. • Surgical resection may be possible and is associated
Enhancement postcontrast is typical. with a more avorable outcome.
4. Myxopapillary ependymomas typically occur in the
region o the f lum terminale.
Suggested Readings
5. Myxopapillary ependymomas are slow-growing tumors Schittenhelm J, Becker R, Capper D, et al. The
and are regarded as tumors o low-grade malignancy clinico-surgico-pathological spectrum o myxopapillary
(WHO grade I). ependymomas--report o our unusal cases and
review o the literature. Clin Neuropathol. 2008
Jan-Feb;27(1):21-28.
Wippold FJ 2nd, Smirniotopoulos JG, Moran CJ, Suojanen
JN, Vollmer DG. MR imaging o myxopapillary
ependymoma: f ndings and value to determine extent o
tumor and its relation to intraspinal structures. AJR Am J
Roentgenol. 1995 Nov;165(5):1263-1267.

262
Known case of NF-1. Incidental nding on MRI

1. What is the most likely diagnosis?

2. What should be included in the di erential


diagnosis?

3. Which conditions are associated with this


f nding?

4. What is this f nding called in the absence o an


underlying condition?

5. What are the typical MR f ndings in this


condition?

263
Lateral meningocele 2939
Case ranking/dif culty: Category: Thecal sac

Axial T2-weighted sequence of the thoracic spine. Panel B Contrast-enhanced axial T1-weighted sequence of the thoracic
is at a slightly more caudal level than panel A. A right lateral spine shows a right lateral meningocele (arrow). Note the
meningocele is seen (arrow) exiting through a widened right widened intervertebral foramen.
intervertebral foramen (arrowhead).
5. Meningoceles appear as well-def ned round, smooth,
or lobulated paraspinal masses. The lesions ollow CSF
signal and appear hypointense on T1-weighted images
and hyperintense on T2-weighted images. Enlarged neural
oramina and vertebral scalloping may also be seen.

Pearls
• Lateral meningocele is a protrusion o the dura mater
and arachnoid through an enlarged intervertebral
oramen.
• Lateral meningocele may be associated with
underlying syndromes including Mar an syndrome,
Ehlers-Danlos syndrome, and neurof bromatosis
type 1.
• Occasionally multiple lateral meningoceles may occur
Sagittal T2-weighted sequence of the thoracic spine. Panel B is at
in the absence o an underlying condition when it is
a slightly more lateral plane than A. A lateral meningocele is seen
(arrow) exiting through a widened right intervertebral foramen
called Lehman syndrome.
(arrowhead). • The lesions ollow CSF signal on MRI.
• Surgical excision is only recommended in
symptomatic lesions.
Answers
1. Lateral meningocele is a rare anomaly where protrusion
o the dura mater and arachnoid extend laterally through Suggested Readings
an enlarged intervertebral oramen.
Alves D, Sampaio M, Figueiredo R, Leão M. Lateral
2. Extramedullary hematopoiesis, neurof bromatosis, meningocele syndrome: additional report and urther
and neurenteric cyst should also be included in the evidence supporting a connective tissue basis. Am J Med
di erential diagnoses. Genet A. 2013 Jul;161A(7):1768-1772.
Gripp KW, Scott CI Jr, Hughes HE, et al. Lateral
3. Lateral meningocele may be associated with underlying
meningocele syndrome: three new patients and
syndromes including Mar an syndrome, Ehlers-Danlos
review o the literature. Am J Med Genet. 1997 Jun
syndrome, and neurof bromatosis type 1.
13;70(3):229-239.
4. When multiple meningoceles are seen in the absence o Kumar BE, Hegde KV, Kumari GL, Agrawal A. Bilateral
an underlying condition, it is called Lehman syndrome or multiple level lateral meningocoele. J Clin Imaging Sci.
lateral meningocele syndrome. 2013 Jan 30;3:1.
264
Known case of a neurocutaneous syndrome.
Presented with lower back pain

1. Name some “cardinal clinical eatures” o this


condition.

2. Which conditions orm part o the


“neurocutaneous syndromes?”

3. Which conditions may cause posterior


vertebral scalloping?

4. Which pattern o inheritance does this


condition have?

5. Which CNS mani estations are seen in this


condition?

265
Neuro bromatosis type 1 2935
Case ranking/dif culty: Category: Thecal sac

Multilevel posterior vertebral There is multilevel posterior Axial image demonstrates posterior vertebral scalloping (arrow)
scalloping (arrows) is vertebral scalloping (arrow) in and widening of the left intervertebral foramen (arrowhead).
noted in the lumbar spine the lumbar spine. Also note the
secondary to dural ectasia. enlargement of the intervertebral
foramen (arrowhead). 5. CNS f ndings in NF-1 include hydrocephalus, lateral
and anterior meningoceles, cranial and spinal nerve
Answers neurof bromas, CNS hamartomas, cerebral gliomas,
1. Two out o seven “cardinal clinical eatures” need to be Moyamoya phenomenon, and spongiotic myelinopathy.
present or a diagnosis o Neurof bromatosis type 1 to be
made. These include
1) Six or more ca é-au-lait spots Pearls
2) Two or more neurof bromas or 1 plexi orm neurof broma • Neurof bromatosis type 1 is an autosomal dominant
3) Axillary reckling disorder.
• The NF-1 gene was mapped to chromosome 17q11•2.
4) Optic glioma
• The diagnosis o neurof bromatosis type 1 requires
5) Two or more Lisch nodules two o the seven “cardinal” clinical criteria to be
6) A distinctive osseous lesion present.
7) A f rst-degree relative with NF-1 • NF-1 patients have a higher incidence o tumors
including pheochromocytomas, intestinal tumors and
2. Neurocutaneous syndromes include neurof bromatosis,
malignant nerve sheath tumors.
tuberous sclerosis, Sturge-Weber syndrome, von Hippel-
• Skeletal abnormalities occur in 25%-40% o NF-1
Lindau disease, ataxia telangiectasia, Wyburn-Mason
patients.
syndrome, incontinentia pigmenti, and nevoid basal cell
• Multiple spinal mani estations (including posterior
carcinoma syndrome.
vertebral scalloping, lateral meningoceles, and
3. Posterior vertebral scalloping is seen in scoliosis) are seen in NF-1.
achondroplasia, acromegaly, neurof bromatosis type 1,
mucopolysaccharidoses, and osteogenesis imper ecta tarda.
Connective tissue disorders including Mar an disease Suggested Readings
and Ehlers-Danlos syndrome may also cause posterior
Hillier J, Moskovic E. The so t-tissue mani estations o
vertebral scalloping secondary to dural ectasia.
neurof bromatosis type 1. Clin Radiol. 2005;(9):960.
4. NF-1 has an autosomal dominant inheritance with almost Reynolds R, Browning G, Nawroz I, Campbell I. Von
a 100% penetrance. However, 50% o cases are sporadic. Recklinghausen’s neurof bromatosis: neurof bromatosis
The a ected NF-1 gene is located on chromosome 17. type 1. Lancet. 2003 May 3;361(9368):1552.
266
Low back pain

1. What eatures are demonstrated on the CT


images?

2. What is the most likely diagnosis?

3. What is the underlying abnormality in all types


o this condition?

4. Which type o the disease is usually the most


severe?

5. Histologically, this entity may resemble what


condition?

267
Osteogenesis imperfecta 2922
Case ranking/dif culty: Category: Vertebral body

Pars interarticularis fracture Numerous biconcave vertebral Marked pelvic bone and femoral deformity, osteoporosis,
(arrowhead), displaced S2 fracture bodies. and treated right femoral fracture.
(arrow), and di use osteoporosis
with biconcave vertebral bodies.

Answers
1. The bones are osteopenic with multiple ractures and Pearls
de ormity. • Osteogenesis imper ecta is a disorder o either the
quantity or quality o collagen, or both. Disorders o
2. Type 1 osteogenesis imper ecta usually has mild disease
the collagen quality lead to more severe disease.
and usually presents later in li e. Severe disease but
• The bones are osteopenic, and are prone to ractures.
with survival beyond the f rst year suggests osteogenesis
• Type 1 disease is mild, with propensity or racture
imper ecta (OI) type 3.
but no de ormity. These patients have blue sclera and
3. A de ect in the quantity and/or quality o collagen is the dea ness.
underlying problem in osteogenesis imper ecta, with a • Type 2 is most severe and is incompatible with li e.
de ect in the quality being more problematic. In the Type • Type 3 is more severe than Type 1, with multiple
2 variant both the quality and quantity o collagen are ractures and de ormity.
a ected, accounting or the severity o disease. • Type 4 is more severe than Type 1. This orm is the
type most o ten con used with nonaccidental injury.
4. Type 2 osteogenesis imper ecta is the most severe type
• Prenatal ultrasound may be diagnostic, with thin
o osteogenesis imper ecta, with death in the f rst ew
calvaria that are easily compressible or ractured.
weeks o li e. Types 5 and 6 are variants o Type 4, and
Increased acoustic through transmission occurs.
Types 7 and 8 are rare recessive orms.
5. Histologically, osteoporosis and osteogenesis imper ecta
are almost identical in appearance.
Suggested Readings
Ablin DS, Greenspan A, Reinhart M, Grix A. Di erentiation
o child abuse rom osteogenesis imper ecta. AJR Am J
Roentgenol. 1990 May;154(5):1035-1046.
Ablin DS. Osteogenesis imper ecta: a review. Can Assoc
Radiol J. 1998 Apr;49(2):110-123.

268
Neck pain and sti ness

1. What are the CT and MRI f ndings?

2. The a ected structure has three important


components. Name them.

3. Which component is most commonly a ected?

4. Which age group is most commonly a ected?

5. What is the most appropriate treatment?

269
Acute calci c tendinitis of the longus colli muscle 2918
Case ranking/dif culty: Category: Paraspinal so t tissue

Calcium hydroxyapatite deposition Calcium hydroxyapatite Prevertebral uid collection. Lentiform prevertebral uid
in the longus colli muscle (arrow). deposition in the collection.
Prevertebral uid collection longus colli muscle
(arrowhead). (arrow).

Answers
• Clinical eatures resemble retropharyngeal abscess,
1. There is calcif cation in the longus colli muscle, with a
with odynophagia, neck sti ness, and a low-grade
lenti orm uid collection in the prevertebral so t tissues.
ever.
The f ndings are consistent with acute calcif c tendinitis
• The calcif cation is amorphous within the muscle, and
o the longus colli muscle. The di erential diagnosis
best appreciated on CT.
is retropharyngeal abscess, which can be a mimic both
• The degree o calcif cation does not correlate with
clinically and radiologically.
symptoms.
2. The longus colli muscle has three components: superior • MRI (and CT) o ten shows a lenti orm prevertebral
oblique, in erior oblique, and vertical. uid collection without an enhancing wall.
• Treatment is conservative, with good response to
3. The superior oblique component is most o ten a ected
NSAIDs.
in calcif c tendinitis, o ten just in erior to the arch o C1
or at the C2-3 level.
4. Acute calcif c tendinitis o the longus colli muscle is
most common in the third to f th decades, although no Suggested Readings
age group is spared. Hall FM, Docken WP, Curtis HW. Calcif c tendinitis o the
longus coli: diagnosis by CT. AJR Am J Roentgenol.1986
5. Most patients respond very well to NSAIDs. Steroids
Oct;147(4):742-743.
are reserved or those individuals who ail to respond to
O f ah CE, Hall E. Acute calcif c tendinitis o the longus
NSAIDs.
colli muscle: spectrum o CT appearances and anatomical
correlation. Br J Radiol. 2009 Jun;82(978):e117-e1121.
Silva CF, So f a PS, Pruzzo E. Acute prevertebral calcif c
Pearls tendinitis: a source o non-surgical acute cervical pain.
Acta Radiol. 2014;55(1):91-94.
• The longus colli muscle has three important
components: superior oblique, in erior oblique, and
vertical.
• Calcium hydroxyapatite deposition typically occurs in
the superior oblique f bers, usually just in erior to the
arch o C1 or at the C2-3 levels.

270
Back pain

1. What are the major radiologic f ndings?

2. What is the di erential diagnosis?

3. What is the common age group or this entity?

4. What is the appropriate initial management or


this entity?

5. What syndromes are associated with an


increased risk o this condition?

271
Chondrosarcoma 2912
Case ranking/dif culty: Category: Vertebral body

AP lm of a contrast urogram shows a faint lucent lesion in the Lesion extends into the sacroiliac joint.
right sacrum.

Answers Pearls
1. There is a lytic expansile sacral lesion with central “ring • Chondrosarcoma should be in the di erential diagnosis
and arc” calcif cation, and extension into the SI joint. o aggressive bone lesions in middle-aged and elderly
patients. It is very rare be ore the ourth decade.
2. The di erential includes metastasis, chondrosarcoma, • The characteristic eatures o chondrosarcoma are
chordoma, plasmacytoma, and giant cell tumor. matrix mineralization. Thorough evaluation o the
The presence o central calcif cation would exclude nature o the calcif cation is essential to narrow the
plasmacytoma and giant cell tumor, as well as metastases di erential diagnosis rom other primary bone tumors,
(which is also usually multi ocal). Central calcif cation eg, osteosarcoma.
o ten occurs in chordoma, but not o this “ring and arc” • Chondrosarcoma is commonly ound in typical
conf guration, and chordomas are usually central. locations, eg, pelvis, shoulder, diametaphyseal regions
o long bones and skull base, although any bone
The diagnosis is there ore chondrosarcoma.
ossi ying rom cartilage can potentially develop a
3. Most chondrosarcomas present a ter the sixth decade o chondrosarcoma.
li e. They are very rare in adolescents, unlike osteogenic • It may occur by sarcomatous trans ormation o an
sarcomas. enchondroma or osteochondroma. Increasing pain and
an osteochondroma cartilage cap thickness greater
4. Initial management o sarcomas must be via a than 2 cm warrant re erral to a sarcoma center.
multidisciplinary sarcoma service or optimal patient • A large so t tissue mass is commonly associated with
outcome. Injudicious excisional biopsy may jeopardize chondrosarcoma, although this is also a eature o
uture curative surgery. Discussion with a specialist small round cell tumors and osteosarcoma.
surgeon is advised. • Syndromes are associated with an increased risk o
At the appropriate time, closed image-guided core chondrosarcoma (eg, Ollier—enchondromatosis,
and Ma ucci syndrome—enchondromas plus
needle biopsy is per ormed and radical surgical excision
hemangiomas).
with wide margins is the recommended treatment.
Postoperative proton beam therapy may be help ul in
some cases. Suggested Readings
5. Ma ucci and Ollier syndromes are associated with Knoeller SM, Uhl M, Gahr N, Adler CP, Herget GW.
an increased risk o chondrosarcoma. The ormer Di erential diagnosis o primary malignant bone tumors
is the association o multiple enchondromas with in the spine and sacrum.The radiological and clinical
cutaneous hemangiomas, which increases the risk o spectrum: minireview. Neoplasma. 2008 Dec;55(1):16-22.
chondrosarcoma by a actor as much as 20 rom the Nguyen BD, Da ner RH, Dash N, Roth us WE, Nathan G,
normal population. Ollier disease is also associated with Toca AR. Case report 790. Mesenchymal chondrosarcoma
an increased risk o chondrosarcoma. o the sacrum. Skeletal Radiol. 1993;22(5):362-366.
Stuckey RM, Marco RA. Chondrosarcoma o the mobile
spine and sacrum. Sarcoma. 2011 Jul;2011(2011):274281.
272
Severe neck pain after diving

1. What is the mechanism o this injury?

2. What is used to determine the stability o the


injury shown?

3. What ligament prevents posterior displacement


o the odontoid peg?

4. What is the usual treatment or a stable burst


racture o C1?

5. What is the treatment required or this injury?

273
Je erson fracture 2911
Case ranking/dif culty: Category: Vertebral body

Mild widening of Lateral slippage of the lateral Fracture of the arch of C1 both anteriorly
the predental space, masses of C1, more than 7 mm and posteriorly.
break in the arch of C1 combined, indicating transverse
(arrowhead), and mild ligament injury and instability.
prevertebral soft tissue
swelling.

Answers • Stability is determined by the integrity o the


1. Axial loading causes most burst ractures, especially transverse ligament. I the transverse ligament is
C1 Je erson ractures. The usual mechanism is sports- interrupted, stability depends on the integrity o the
related injury or diving into a shallow swimming pool. alar ligaments.
• I the lateral masses o C1 are displaced by a total o 7
2. I the combined lateral mass displacement with respect
mm relative to C2, the transverse ligament is torn; the
to the articular sur aces o the axis is greater than 7 mm,
racture is termed unstable and may require surgical
the transverse ligament is considered to be ruptured, and
f xation.
the racture is inherently unstable.
• MRI and CT are complementary investigations in
3. The transverse ligament o the atlas is the only structure cervical spine trauma to assess the cord and bony
that prevents posterior displacement o the odontoid peg. structures respectively. CTA is also indicated to
It is a very strong ligament, the integrity o which is vital exclude vertebral artery injury.
to prevent cord injury. It is at risk o rupture with Je erson • Initial assessment by plain f lms is adequate i due
racture, especially when the lateral masses are involved. care and attention is paid to the open-mouth odontoid
I it is compromised, stability depends on the integrity o peg view or displacement o the lateral masses o C1.
the alar ligaments. • Pre-vertebral so t tissue swelling o greater than a
third o the vertebral body rom C1 to C3 in trauma
4. The usual treatment or an undisplaced C1 racture is a cases should immediately trigger a CT examination to
halo collar. assess or an occult racture, although the decision or
5. A displaced Je erson racture generally requires a CT scan is o ten based on clinical parameters.
surgical stabilization o the cervical spine i the
transverse ligament is ruptured, especially i associated
with other cervical ractures. Many authorities will
Suggested Readings
attempt nonoperative stabilization with a halo initially,
depending on the severity o the injury. Korinth MC, Kapser A, Weinzierl MR. Je erson racture in
a child—illustrative case report. Pediatr Neurosurg. 2007
Mar;43(6):526-530.
Looby S, Flanders A. Spine trauma. Radiol Clin North Am.
Pearls 2011 Jan;49(1):129-163.
• An axial-loading injury results in a burst racture, Pratt H, Davies E, King L.Traumatic injuries o the c1/c2
which includes a Je erson (C1) racture. complex: computed tomographic imaging appearances.
Curr Probl Diagn Radiol. 2007 Mar;37(1):26-38.
274
Recurrent aspiration when feeding

1. What are the pertinent radiographic f ndings?

2. What is the diagnosis?

3. What are the common cardiac conditions


associated with this entity?

4. What umbilical vessel association is seen in


this entity?

5. What is the association between the renal and


limb abnormalities?

275
VACTERLsyndrome 2910
Case ranking/dif culty: Category: More than one category

Another patient with an absent radius. Another patient with a hypoplastic radius. All images courtesy of
the Children’s Hospital of Philadelphia.

Answers
• TE: Esophageal atresia with tracheoesophageal
1. Vertebral segmentation anomalies with synostosis o the
f stula.
upper right ribs are demonstrated. There is esophageal
• R: Renal anomalies including renal mal ormations,
atresia, as evidenced by ailure to advance a nasogastric
UVJ obstruction, and vesicoureteric re ux.
tube.
• L: Limb anomalies including syndactyly,
2. The clinical and radiologic eatures are consistent with polydactyly, and hypoplastic thumb. Radial ray
VACTERL syndrome. anomalies are also a eature.
• Occurrence is sporadic with no clear genetic
3. Numerous cardiac anomalies are described in
predisposition. In ants o diabetic mothers have a
VACTERL, the most common being ventricular and
somewhat higher risk.
atrial septal de ects. Transposition o the great vessels,
• Treatment in the early stages includes prompt
tetralogy o Fallot, and truncus arteriosus have also been
management o the gastrointestinal, cardiac, and
described. This patient had tetralogy o Fallot and an
genitourinary eatures.
aberrant right subclavian artery.
4. 35% o VACTERL cases have a single umbilical artery.
The normal anatomy is two umbilical arteries and one vein.
Suggested Readings
5. Babies with unilateral limb abnormalities tend to Solomon BD, Pineda-Alvarez DE, Raam MS, et al. Analysis
have renal or urologic abnormalities on the same side. o component f ndings in 79 patients diagnosed with
I the limb de ects are bilateral, the genitourinary VACTERL association. Am J Med Genet A. 2010
abnormalities are usually bilateral. Sep;152A(9):2236-2244.
Solomon BD. VACTERL/VATER Association. Orphanet J
Rare Dis. 2011 Jul;6(6):56.
Pearls
• VACTERL syndrome is a nonrandom co-occurrence
o numerous birth de ects:
• V: Vertebral de ects, including segmentation
anomalies and hypoplastic vertebrae.
• A: Anorectal de ects include anal atresia and
imper orate anus.
• C: Cardiac de ects including septal de ects and
tetralogy o Fallot.

276
Low back pain

1. What are the f ndings on the radiograph?

2. What is the di erential diagnosis?

3. What is the most likely diagnosis?

4. This entity may occur secondary to what


condition?

5. What are the usual MRI signal characteristics


o the lesion?

277
Giant cell tumor 2907
Case ranking/dif culty: Category: Vertebral body

Internal necrosis and areas of hemorrhage. The Mass arising from the sacrum. Heterogenous increased T2 signal.
uterus is displaced by the mass (arrow). Areas of low T1 and T2 signal likely Areas of low T1 and T2 signal likely
represents hemosiderin. represents hemosiderin. There are
no uid- uid levels in this case,
Answers although these are usually common.
1. There is a lytic expansile sacral lesion with coarse
trabeculae and cortical thinning. Peripheral sclerosis is
seen, which is an unusual mani estation. • Uncommon locations include the vertebrae, pelvic
2. The di erential includes sacral chordoma, bones, sacrum, and skull base. In the sacrum, lesions
chondrosarcoma, metastasis, plasmacytoma, and also are usually central but o ten extend to involve the
f brous dysplasia. Aneurysmal bone cyst is also included entire sacrum.
in the di erential. • MRI may show uid- uid levels, and hemosiderin in
over 60% o cases.
3. The age group would make metastasis or plasmacytoma • Recurrence is up to 50% i extended curettage is
unlikely. Chordomas and chondrosarcomas typically not per ormed, 10% i it is. Metastasis can occur in
calci y. The most likely diagnosis is giant cell tumor. 10%-25% o patients, usually to the lungs.
4. Giant cell tumor may occur as a rare complication o
Paget disease. These typically occur in the skull and
acial bones. Suggested Readings
5. T1 hypointensity, T2 hyperintensity, and signal Aoki J, Tanikawa H, Ishii K, et al. MR f ndings indicative o
heterogeneity. Over 60% o cases have hemosiderin hemosiderin in giant-cell tumor o bone: requency, cause,
rom prior hemorrhage, and will bloom on gradient echo and diagnostic signif cance. AJR Am J Roentgenol.1996
sequences. Fluid- uid levels are common. Jan;166(1):145-148.
Swanger R, Maldjian C, Murali R, Tenner M. Three cases
o benign giant cell tumor with unusual imaging eatures.
Clin Imaging. 2008;32(5):407-410.
Pearls
• Giant cells tumors typically occur at the end o long
bones.
• The distal phalanges are also common especially in
cases o multiple giant cell tumors.
• The lesions extend to the subarticular cortex, with
expansion o bone and cortical thinning. Lesions are
eccentric to the long axis o bone, and do not usually
have a sclerotic rim.

278
Motor vehicle hitting pedestrian

1. What are the imaging f ndings?

2. What are the major causes o this injury?

3. What is the normal lateral mass interval at the


a ected level?

4. The injury is associated with a high morbidity


and mortality. True or False?

5. Adults are at greater risk o craniocervical


dissociation, compared to children. True or
False?

279
Vertical C1-C2 dissociation 2906
Case ranking/dif culty: Category: Vertebral body

Vertical C1-C2 dissociation, predental widening. Extensive ligamentous injury and cord transection.

Answers
1. There is vertical dissociation o C1-C2, with marked Pearls
prevertebral so t tissue swelling. MRI shows extensive • Vertical C1-C2 dissociation is rare.
ligamentous injury with cord transection. • It may occur as a result o a high-velocity injury
with a distractive orce, o ten a ter a motor vehicle
2. Vertical C1-C2 dissociation occurs as a result o a high
accident. The injury may also occur, or be accentuated
velocity injury with a distractive orce, as can occur in a
by, excessive cervical traction or this or other cervical
motor vehicle accident. Injudicious cervical halo traction
spine injuries.
can also cause or accentuate the separation.
• Normal C1-C2 acet joint distance should measure
3. The normal lateral mass interval at C2-C3 is 2.6 mm or 3 mm or less, and care ul evaluation o this space is
less. An increase in this distance should raise the concern indicated when ligamentous injury is suspected.
or ligamentous injury, and should trigger an MRI to • Lesions have a high morbidity and mortality.
assess or ligamentous injury i the patient is stable. • Treatment is with cessation o traction, and cervical
Cervical traction must be avoided. usion.
4. True. Associated injuries include spinal cord injury,
cranial nerve palsies, and vertebrobasilar artery injuries.
There is a high mortality. Suggested Readings
5. False. Because o shallow atlantooccipital joints, lax Botelho RV, de Souza Palma AM, Abgussen CM, Fontoura
craniocervical ligaments and relatively large head size, EA. Traumatic vertical atlantoaxial instability: the risk
children are more prone to craniocervical dissociation. associated with skull traction. Case report and literature
review. Eur Spine J. 2000 Oct;9(5):430-433.
Gould S, Hishmeh S, McKinney B, Stephen M. Combined
traumatic occiput-C1 and C1-C2 dissociation: 2
case reports. Am J Orthop (Belle Mead NJ). 2010
Aug;39(8):392-395.

280
Mild leg weakness

1. What are the major radiologic f ndings?

2. What is the diagnosis?

3. A bony septum is always demonstrated. True


or False?

4. What are some o the associated clinical and


radiologic f ndings?

5. Patients are always symptomatic. True or


False?

281
Diastematomyelia (split cord malformation) 2903
Case ranking/dif culty: Category: Spinal cord

No bony or calci ed septum. Similar ndings.

Answers
• Each hemicord has a single set o anterior and
1. The lower thoracic and upper lumbar spinal cord is split,
posterior nerve roots.
with a single dural sac. No bony septum is demonstrated.
• More than 80% o cases have associated cord
The cord was tethered, although not demonstrated on
tethering, with atty or thickened f lum terminale.
these images.
Hydromyelia is also o ten present.
2. The diagnosis is diastematomyelia, part o the split cord • Clinically, Type 1 disease is more severe. 50% o the
mal ormation syndrome. These all within the category disease occurs at L1-3. 25% at T7-12.
o occult spinal dysraphism. • Treatment is with cord untethering and septum
resection.
3. False. A bony or f brocartilaginous septum is typically
seen in Type 1 SCM, but in Type 2 SCM, a f brous
septum may be seen or there may be no septum.
Suggested Readings
4. Associated eatures include hydromyelia, skin
pigmentation, hair patches (hypertrichosis), congenital Gan YC, Sgouros S, Walsh AR, Hockley AD.
vertebral anomalies, scoliosis, as well as hemangiomas. Diastematomyelia in children: treatment outcome and
natural history o associated syringomyelia. Childs Nerv
5. False. Patients with Type 2 SCM may have mild Syst. 2007 May;23(5):515-519.
symptoms or may be completely asymptomatic. Ru ener SL, Ibrahim M, Raybaud CA, Parmar HA.
Congenital spine and spinal cord mal ormations—
sel -assessment module. AJR Am J Roentgenol. 2010
Pearls Mar;194(3 suppl):S38-S40.
• Diastematomyelia belongs to the spectrum o split
cord mal ormations (SCM).
• Type 1 SCM has a bony or f brocartilaginous septum
dividing the two hemicords. Type 2 has a f brous
septum or no septum can be demonstrated.
• Two dural sacs are seen in Type 1 SCM, and one in
Type 2.

282
History of trauma, now with severe neck pain

1. What are the f ndings on the radiographs?

2. What is the epidemiology o the f ndings in the


spinal canal?

3. What is the natural history o the intraspinal


f nding?

4. What is the best imaging modality when


investigating this f nding?

5. What are the other associations o the


intraspinal f nding?

283
Ossi cation of the posterior longitudinal ligament (OPLL) 2901
Case ranking/dif culty: Category: Ligaments

Extensive di use The ossi cation of the posterior More lateral sagittal image better C3 body fracture
idiopathic skeletal longitudinal ligament (OPLL) is well demonstrates a C3 fracture, the (arrow), OPLL, and cord
hyperostosis (DISH) demonstrated. presenting event. compression with mild
changes with ossi cation cord edema (arrowhead)
of the posterior are well demonstrated.
longitudinal ligament
(arrowhead).

Answers
1. There is ossif cation o the posterior longitudinal • The cervical spine at C3-C5 is most commonly
ligament (OPLL) and anterior paraspinal ossif cation a ected (75%).
consistent with DISH. The acute presenting eature was • Plain f lm and CT show linear ossif cation posterior
an acute traumatic racture o the anterior in erior C3 to the vertebral bodies and possibly discs, separated
vertebral body, not demonstrated in these radiographs. by a lucent line (basivertebral venous plexus). CT will
also show the degree o spinal stenosis.
2. Ossif cation o the posterior longitudinal ligament • On MRI, OPLL shows T1 and T2 hypointensity with
(OPLL) is most common in older Japanese males. possible central T1 hyperintensity i marrow at is
3. OPLL is progressive and may result in spinal stenosis. present. MRI has the advantage o also showing cord
changes.
4. MRI is the best imaging modality, as it will indicate the • MRI will also show spinal stenosis and cord changes
extent o OPLL as well as show the degree o spinal including cord edema and myelomalacia.
stenosis. It will also show cord changes o edema and
myelomalacia.
5. 50% o patient with OPLL have DISH. There is also an Suggested Readings
association o OPLL with the spondyloarthropathies.
Munday TL, Johnson MH, Hayes CW, Thompson EO,
Smoker WR. Musculoskeletal causes o spinal axis
compromise: beyond the usual suspects. Radiographics.
Pearls 1994 Nov;14(6):1225-145.
• Ossif cation o the posterior longitudinal ligament Widder DJ. MR imaging o ossif cation o the posterior
(OPLL) occurs most commonly among Japanese. longitudinal ligament. AJR Am J Roentgenol. 1989
• It may be classif ed as continuous i the disc is Jul;153(1):194-195.
involved (either segmentally or nonsegmentally), or
noncontinuous i the disc is not involved.

284
Headache and neurological symptoms, history of a chronic illness

1. What are the relevant radiographic f ndings?

2. What is the di erence between basilar


impression and basilar invagination?

3. What are the most important radiological lines


used in evaluating this entity?

4. What are the indications or surgery?

5. What are some o the congenital causes or this


entity?

285
Rheumatoid arthritis with basilar invagination 2900
Case ranking/dif culty: Category: More than one category

Basilar invagination, with the tip of the odontoid process (blue McRae line (between blue asterisks), Chamberlain line (between
asterisk) lying above Chamberlain line. This line is drawn from the green asterisks), and McGregor line (between red and green hard
hard palate to the posterior edge of the foramen magnum and is palate asterisks).
di cult to see on these plain radiographs.

5. Congenital causes or basilar invagination include


Answers osteogenesis imper ecta, achondroplasia, Klippel-Feil
1. The tip o the odontoid process projects into the oramen syndrome, cleidocranial dysostosis, and Schwartz-
magnum and is consistent with basilar invagination, Jampel syndrome.
an acquired abnormality due to bone so tening. The
presence o carpal bone erosions conf rms rheumatoid
arthritis as the etiology. Pearls
2. Primary basilar impression is a diagnosis o exclusion, • Basilar invagination occurs when the odontoid process
and is not a result o bone so tening but is believed to be migrates superiorly into the oramen magnum.
amilial with an autosomal dominant inheritance pattern • It is a result o a bone so tening process, including
and incomplete penetrance. Basilar invagination is a rheumatoid arthritis, Paget disease, and rickets/
result o bone-so tening conditions such as rheumatoid osteomalacia.
arthritis and Paget disease. • Primary basilar impression is a diagnosis o exclusion,
where there is no evidence o bone so tening and is
3. MacGregor line is measured rom the hard palate to the
believed to be a amilial developmental abnormality.
caudal posterior occiput curve, and the dens tip should
• Platybasia is a attening o the skull base and may
be less than 4.5 mm below this line.
be a result o a bone-so tening process, or may be
Chamberlain line is measured rom the hard palate to congenital.
the oramen magnum, and the dens should be less than • Important lines to consider include Chamberlain line,
6 mm below the line. MacGregor line, and McRae line.
McRae line extends rom the posterior clivus to the • Treatment includes odontoid resection by a
oramen magnum, and the dens tip should be below the transoral or anterior retropharyngeal approach, or
line although minimal protrusion may be acceptable. occipitocervical usion.
Ranawat line is measured rom the center o the C2
pedicle to a line connecting the anterior and posterior
arches o C1, and should be 13 mm in a emale or Suggested Readings
15 mm in a male, with reduced measurements indicating
Smoker WR. Craniovertebral junction: normal anatomy,
impaction.
craniometry, and congenital anomalies. Radiographics.
4. Progressive neurologic compromise including signs o 1994 Mar;14(2):255-277.
brainstem compression, syringomyelia or hydromyelia, Smoker WR. MR imaging o the craniovertebral junction.
apnea, and cranial nerve dys unction. Magn Reson Imaging Clin N Am. 2000 Aug;8(3):635-650.
286
Chronic disease; worsening back pain

1. What are the f ndings in the spine?

2. What are included in the di erential, and what


is the most likely diagnosis?

3. What is the next most appropriate study?

4. Where are the common sites a ected in this


condition?

5. What are the f ndings to be expected on MRI?

287
Amyloid spondyloarthropathy 2899
Case ranking/dif culty: Category: More than one category

More pronounced disease in another Corresponding MRI image shows heterogenous Heterogenous signal, with foci of
patient, with marked destructive signal at the disc. Note anterior prevertebral low T2 signal in the disc and anterior
changes and kyphosis in the amyloid deposits (arrowheads). Note the prevertebral soft tissues, consistent with
midthoracic spine. similarity to infective spondylodiscitis. amyloid deposits (arrowheads). Note the
impingement on the cord (arrow).

Answers
1. There is di use osteosclerosis, with multilevel Pearls
erosions and cysts at the endplates, and mild disc space • Amyloid arthropathy results rom the extracellular
narrowing. deposition o amyloid in the joint spaces and so t
2. The major di erentials are in ective and amyloid tissues.
spondylodiscitis. The di use osteosclerosis and • In dialysis patients, it is related to the inability o the
multilevel involvement, along with a history o long-term dialysis membrane to remove amyloid protein.
dialysis, would make amyloid the most likely diagnosis. • Cervical spine and shoulders are most commonly
a ected.
The presence o increased T2 signal in the disc is, • Plain f lms show cysts, erosions, and periarticular
however, not uncommon, o ten making di erentiation osteopenia. The joint space is preserved or widened
rom in ection more di f cult. until late in the disease.
Metastases and lymphoma could demonstrate • In the spine, there are erosions at the endplates and
osteosclerosis, but would not be expected to a ect the corners o the vertebrae, with disc space narrowing
discs. and o ten so t tissue masses in the prevertebral so t
3. An MRI would demonstrate the signal characteristics tissues or in the acet/interspinous ligaments. Cord
o the discs, the presence o low T2 signal amyloid, and compression may occur.
would also show any compressive e ects on the cord. • Amyloid is low signal on all MR sequences with no
paramagnetic e ect, di erentiating it rom PVNS.
4. Amyloid spondyloarthropathy occurs predominantly,
Contrast enhancement is variable.
but not exclusively, in dialysis-related amyloid. The
shoulders, spine (particularly the lower cervical and
lower lumbar spine), wrists, hips, knees, and carpal
tunnels are most commonly a ected. Suggested Readings
5. The disc change o amyloid spondyloarthropathy is Cobby MJ, Adler RS, Swartz R, Martel W. Dialysis-related
typically iso- to hypointense on all pulse sequences; amyloid arthropathy: MR f ndings in our patients. AJR
however, the presence o increased T2 signal in the Am J Roentgenol. 1991 Nov;157(5):1023-1027.
disc is not uncommon. Hence, di erentiating rom Kiss E, Keusch G, Zanetti M, et al. Dialysis-related
in ective spondylodiscitis can be di f cult. Paravertebral amyloidosis revisited. AJR Am J Roentgenol. 2005
or ligamentous so t tissue masses would show similar Dec;185(6):1460-1467.
signal. Contrast enhancement is variable.

288
Motor vehicle accident

1. What are the imaging f ndings?

2. What is the mechanism o injury?

3. What injuries to the cervical spine are


unstable?

4. In what injury is the hamburger sign seen?

5. What percentage o patients with bilateral


injury have neurological compromise?

289
Jumped/perched facets 2882
Case ranking/dif culty: Category: Vertebral body

Anterolisthesis of C4 on Right “hamburger” sign demonstrated. Arrowhead is Jumped facet


Perched facet on
C5, with a jumped facet the inferior facet of the C4 vertebra, and arrow the with an associated
the left side.
(arrowhead) and multiple superior facet of C5. fracture (arrow).
spinous process fractures.

Answers
1. There is a hyper exion injury with a jumped acet, Pearls
anterolisthesis o C4 on C5, and multiple spinous • A jumped acet is a result o a severe injury, usually
process ractures. The “hamburger” and “naked acet” MVA or a all.
signs are the same, and are demonstrated here. • The mechanism is hyper exion rotation, and is
unstable when bilateral or associated with a acet
2. The mechanism or this injury is hyper exion. A rotary
racture.
component to the injury is contributory.
• In bilateral jumped acets, there is typically >50%
3. Flexion teardrop, Je erson and Hangman ractures, Type anterolisthesis and the subsequent spinal canal and
2 dens ractures, and bilateral locked acets are unstable neuro oraminal stenosis results in neurologic def cit in
ractures. A unilateral locked acet with a ractured acet up to 75% o cases.
is also unstable. In addition, hyperextension dislocation • Cord injury ranges rom cord edema to cord
is unstable. transection.
• Associated ractures are common, including spinous
4. A “hamburger” sign is seen in a jumped acet. On
process, acet, and a triangular corner racture o the
axial CT, the normal appearance o the acets is said to
anterosuperior margin o the in erior involved vertebra.
resemble a hamburger, with the superior articular acet o
• When the acets lie on to top o each other rather than
the vertebra below orming the top bun and the in erior
overlapping, it is called a perched acet.
acet o the vertebra above orming the bottom bun, and
• The “hamburger” or “naked acet” sign is seen on axial
the space between the “meat.” With a jumped acet, the
CT images.
bottom “bun” now lies anterior to the top “bun.” This is
• Both CT and MRI are indicated: CT to def ne the
known as the hamburger or naked acet sign.
racture anatomy, and MRI to evaluate the cord,
5. 75% o patients with bilateral jumped acets have disc, and ligamentous structures.
neurological compromise. The listhesis that occurs
results in severe canal and oraminal stenosis, with cord
compression or transection in severe cases. Suggested Reading
Kornberg M. The computed tomographic appearance
o a unilateral jumped cervical acet (the “ alse”
acet joint sign). Spine (Phila Pa 1976). 1986
Dec;11(10):1038-1040.

290
Sudden back pain with neurological compromise

1. What are the radiographic f ndings?

2. What are the MRI f ndings?

3. What is the diagnosis?

4. What are the MR eatures avoring


osteoporotic over metastatic vertebral collapse?

5. Contrast enhancement is use ul in di erentiating


acute osteoporotic cord compression rom
metastatic cord compression. True or False?

291
Osteoporotic cauda equina compression 2881
Case ranking/dif culty: Category: More than one category

Low T2 signal band (arrow) is demonstrated with osteoporotic Retropulsed fragment (arrowhead) and low-intensity band (arrow).
vertebral collapse.

• Osteoporosis is one o the major causes o


Answers
nontraumatic vertebral collapse, and it is important to
1. Vertebral compression racture. di erentiate rom metastatic collapse.
2. The MRI f ndings are a T1 and T2 low signal intensity • Plain x-ray and MRI are the main diagnostic tools.
band, retropulsion o a posterior bone ragment, and • Plain x-ray: Collapsed vertebra at one or multiple
spared marrow signal within the a ected vertebral levels. Decreased density o bones with vertical
body and other bodies. There is no posterior element striations, biconcave vertebrae, Schmorl nodes,
involvement, and no paraspinal mass. and picture raming (prominence o the cortical
outline due to disproportionate trabecula resorption
3. The f ndings are suggestive o osteoporotic vertebral and peripheral trabecula rein orcement).
collapse. • MRI: Low T1 and T2 signal intensity bands, normal
4. Low signal intensity band on T1 and T2, areas o marrow signal intensity in other vertebrae and
preserved bone marrow signal intensity in the a ected preserved marrow signal in the a ected vertebra,
and other vertebrae, multilevel collapse, no posterior posterior bone ragment retropulsion, absence o a
element involvement, and no paraspinal mass. These paraspinal or epidural mass, and the presence o other
MR eatures avor osteoporotic collapse. compression ractures avor osteoporotic collapse over
metastatic collapse.
5. True. Although it is controversial, some authors suggest
heterogenous contrast enhancement in metastatic
inf ltration may be help ul in distinguishing this entity Suggested Readings
rom cord compression secondary to osteoporotic
Cuénod CA, Laredo JD, Chevret S, et al. Acute vertebral
collapse. An enhancing so t tissue mass is also not seen
collapse due to osteoporosis or malignancy: appearance
in osteoporosis, but can be seen in metastases.
on unenhanced and gadolinium-enhanced MR images.
Radiology.1996 May;199(2):541-549.
Jung HS, Jee WH, McCauley TR, Ha KY, Choi KH.
Pearls Discrimination o metastatic rom acute osteoporotic
compression spinal ractures with MR imaging.
• Osteoporosis is a metabolic disorder a ecting the
Radiographics. 2003;23(1):179-187.
modeling o bone, resulting in bone resorption. This
Rumpel H, Chong Y, Porter DA, Chan LL. Benign versus
leads to a reduction in bone mineral density.
metastatic vertebral compression ractures: combined
• It is more common in emales, and one in three
di usion-weighted MRI and MR spectroscopy aids
women over the age o 65 are a ected.
di erentiation. Eur Radiol. 2013 Feb;23(2):541-550. doi:
10.1007/s00330-012-2620-1. Epub 2012 Aug 18.
292
Sacral and coccygeal pain

1. What are the major imaging f ndings?

2. What is the diagnosis?

3. What are the typical associations o intradural


lipomas?

4. The lesions are neoplastic. True or False?

5. What is the typical behavior o these lesions?

293
Intradural lipoma 2879
Case ranking/dif culty: Category: Spinal canal

T1 hyperintense fatty mass with the sacral canal, Complete fat suppression on fat-suppressed
displacing the nerve roots. T2-weighted images.

Answers
1. There is a well-def ned lenti orm intraspinal atty mass
Similar ndings. Note overlying
that incorporates the nerve roots anteriorly. There is subcutaneous lipoma (arrow) and
associated pelvic lipomatosis and proli eration o the pelvic lipomatosis.
subcutaneous at.
2. The presence o a atty mass occupying the entire sacral
canal is consistent with an intraspinal lipoma. There is an
• Slow growth may occur, and symptoms are related to
overlying subcutaneous lipoma and pelvic lipomatosis.
mass e ect.
3. Intradural lipomas typically have no associated f ndings, • Subcutaneous lipomas may overly the intradural
unlike lipomas associated with spinal dysraphism, which lipoma.
may be associated with renal, vertebral, and dermal • Incorporation o the nerve roots may occur, o ten
abnormalities. resulting in the lesion appearing intramedullary at
surgery.
4. False. Although the etiology o intraspinal lipomas is
• On MRI, the lesions completely suppress on at-
not clearly def ned, most authorities believe that they are
suppressed FSE T2-weighted images and there is no
nonneoplastic and are probably hamartomatous lesions.
contrast enhancement.
5. Slow growth o intradural lipomas is typical. Since the • Plain radiographs may show widening o the spinal
lesions are believed to be hamartomatous tissue, growth canal with thinning o the pedicles and a widened
o the lesions may occur along with growth in the at interpedicular distance.
pool, hence the importance o weight control especially • Treatment is with surgical decompression, although
in patients with partial surgical decompression and complete excision is usually not possible.
recurrent or residual symptoms.

Suggested Reading
Pearls Finn MA, Walker ML. Spinal lipomas: clinical spectrum,
• Intradural spinal lipomas most commonly a ect embryology, and treatment. Neurosurg Focus.
the thoracic spine in adults and the cervical spine in 2007;23(2):E10.
children.
• They are not associated with dermal, vertebral, or
renal abnormalities like lipomas associated with spinal
dysraphism.
• The lesions are not considered neoplastic, although
their etiology is poorly understood. They are probably
hamartomatous tissue.

294
Acute neurological symptoms with back pain. No history of trauma

1. What are the MRI f ndings?

2. What are the di erential diagnoses?

3. What are the specif c distinguishing eatures o


this entity on MRI?

4. T2-weighted images best to diagnose marrow


replacement. True or False?

5. What sequences are not use ul in


demonstrating this condition?

295
Metastatic cauda equina compression 2880
Case ranking/dif culty: Category: Vertebral body

Metastatic destruction of the Axial image showing no room for CSF ow, and the Contrast-enhanced image shows “sugar
vertebral body with increased epidural fat is compressed completely. coating” appearance of dural metastases
signal and di use posterior in another patient (arrowheads).
bulge, with cauda equina
compression. Note subchondral
low-intensity line (arrowhead), a Pearls
feature more commonly seen in • Skeletal metastases are the third commonest site or
osteoporotic collapse.
metastases, and in the skeletal system, the spinal
column is the most common site.
Answers • Abnormal marrow inf ltration on T1 is an important
indicator. T1 di use marrow inf ltration should also
1. T1 marrow replacement, vertebral collapse and di use be searched or, using the disc signal as a control
posterior wall bulge, low signal intensity band, cauda (marrow signal should be higher than disc on the
equina compression, and single-level involvement. T1-weighted images).
Although a low intensity band is seen in this case, it • The ollowing criteria on MRI avor metastatic
is more commonly seen in osteoporotic compression collapse over osteoporotic collapse: di use posterior
ractures. bulge, paraspinal or epidural mass, involvement o
2. The di erential diagnosis includes osteoporotic racture, the posterior elements, and multiple-level marrow
metastatic collapse, in ection, posttraumatic collapse, involvement. A band o low T1 and T2 signal intensity
and chordoma. is nonspecif c, but avors osteoporotic collapse.
• The use o contrast is o debatable use ulness in
3. MRI eatures suggesting metastatic compression include evaluating bone metastases. However, “sugar coating” in
a di use posterior vertebral bulge, involvement o the the case o dural metastases and paraspinal or epidural
posterior elements, paraspinal mass, epidural mass, other masses are well demonstrated on post-contrast images.
spinal involvement, and “sugar coating” with contrast • Urgent treatment is with cord decompression and
images ( or dural metastases). T1-weighted images are spine stabilization. In palliative cases, vertebroplasty
use ul to demonstrate multilevel marrow involvement. and kyphoplasty are o ered or pain control.
4. False. T1 is the sequence o choice or evaluation o
marrow replacement. Suggested Readings
5. STIR and T2-weighted images can show increased Berwouts D, Remery M, Van Den Berghe T. Vertebral
signal in the inf ltrated marrow, but it is nonspecif c. collapse caused by bone metastasis. J Thorac Oncol. 2011
Post-contrast images are use ul to see dural metastases Apr;6(4):823.
and paraspinal/epidural masses, but it is o uncertain Jung HS, Jee WH, McCauley TR, Ha KY, Choi KH.
use ulness in evaluating bony metastases. Gradient echo Discrimination o metastatic rom acute osteoporotic
and proton density images are o little use. compression spinal ractures with MR imaging.
Radiographics. 2003;23(1):179-187.
La orgue P, Bayle O, Massonnat J, et al. [MRI in
osteoporotic and metastatic vertebral compressions:
apropos o 60 cases]. Ann Radiol (Paris). 1991
Feb;34(3):157-166.
296
Gait disturbance

1. What is the most likely diagnosis?

2. How does the condition usually mani est?

3. Describe clinical and pathological


characteristics o this entity.

4. Which imaging modality best images the


condition?

5. What are the typical MR f ndings o this


entity?

297
Filar lipoma 747
Case ranking/dif culty: Category: Filum

Sagittal T1-weighted sequence of the lumbar spine shows a thin Axial T1-weighted sequence at the level of L2 vertebral body
cylindrical hyperintensity (arrows) within the lum terminale in con rms the presence of fat within the lum terminale (arrow) in
keeping with fat. Note that the cord terminates at L1-L2. There is keeping with a lar lipoma.
no evidence of associated spinal dysraphism.

Answers 5. Filar lipoma appears as a linear T1- and


1. Filar lipoma is a relatively common asymptomatic T2-hyperintensity within the f lum terminale. Chemical
incidental f nding and represents the most common shi t arti act may be seen on T2* or gradient echo
intraspinal lipoma. Filar lipomas are secondary to sequences. The lesional signal saturates ollowing
persistence o caudal cells that di erentiate toward at. at suppression. No enhancement o the lesion is
By def nition a atty f lum that is thicker than 2 mm in demonstrated ollowing contrast administration.
cross section is classif ed as a f lar lipoma. Intradural f lar lipomas are o ten usi orm in shape and
2. Filar lipoma is a relatively common asymptomatic taper down to the site where the f lum pierces the dura.
incidental f nding. It may, however, be associated with
tethered cord and spinal dysraphism.
Pearls
3. Filar lipomas may be associated with lipomas o the
caudal hal o the conus medullaris, as the latter also • Filar lipoma is a relatively common asymptomatic
orms by canalization and retrogressive di erentiation. incidental f nding.
Fat within the f lum terminale is seen in up to 5% o • It may be associated with tethered cord and spinal
lumbar spine MR examinations. dysraphism.
• Fat within the f lum terminale is seen in up to 5% o
The f lum terminale is thickened due to the presence o
lumbar spine MR examinations.
at and appears thicker than its neighboring nerve roots.
• MRI is the examination o choice especially when at
Filar lipomas may involve the intra- or extradural portion
suppression sequences are added.
o the f lum terminale or both.
• When associated with tethered cord syndrome, surgical
Filar lipomas may be identif ed on computed intervention is appropriate.
tomography as a cylindrical at density within the lumbar
canal. The lesional Hounsf eld units may be measured,
which conf rm at density (−90 to −30 HU). Suggested Readings
When associated with tethered cord syndrome, surgical Lowe LH, Johanek AJ, Moore CW. Sonography o the
intervention is appropriate and involves sectioning o the neonatal spine: part 2, spinal disorders. AJR Am J
f lum terminale. Roentgenol. 2007 Mar;188(3):739-744. Review.
4. MRI is the examination o choice especially when at Park HJ, Jeon YH, Rho MH, et al. Incidental f ndings o
suppression sequences are added. The lipoma o ten the lumbar spine at MRI during herniated intervertebral
extends over several vertebral segments and its signal disk disease evaluation. AJR Am J Roentgenol. 2011
ollows that o at. May;196(5):1151-1155.

298
Motor vehicle accident

1. What should be included in the di erential


diagnosis?

2. What structures are involved?

3. Why does instability arise?

4. What are the mechanisms o injury?

5. What are the treatment options?

299
Fracture C2—type 3 2196
Case ranking/dif culty: Category: Vertebral body

Pearls
• Type 3 odontoid ractures extend into the vertebral
body.
• These are inherently unstable ractures; however, they
o ten heal well with immobilization.
• These ractures have a very low rate o nonunion given
the involvement o cancellous bone.
• Traction may be required to maintain anatomic
position and some patients may require surgical
xation.

Suggested Readings
Greene KA, Dickman CA, Marciano FF, Drabier JB, Hadley
Irregular lucency through the synchondrosis, consistent with a MN, Sonntag VKH. Acute axis ractures: analysis o
type 3 odontoid fracture. management and outcome in 340 consecutive cases.
Spine. 1997;22:1843-1852.
Rao SK, Wasyliw C, Nunez DB. Spectrum o imaging
Answers
ndings in hyperextension injuries o the neck.
1. All types o C2 ractures could be considered; however, Radiographics. 2005;25:1239-1254.
type 3 ractures extend into the vertebral body.
2. Type 3 ractures extend rom the base o the dens into
the vertebral body.
3. Instability in Type 3 ractures arises rom the atlas and
occiput moving together.
4. The mechanism is thought to be primarily rom f exion
with rebound extension.
5. Most patients have good union with a combination o
traction and halo xation with surgical xation reserved
or patients with nonunion.

300
Fibrous dysplasia, enlarging painless paraspinal mass

1. What should be included in the di erential


diagnosis?

2. What are common presenting symptoms?

3. Which syndromes are associated with osseous


and so t tissue lesions?

4. What are classic imaging characteristics?

5. What are the treatment options?

301
Intramuscular myxoma, Mazabraud syndrome 2242
Case ranking/dif culty: Category: Paraspinal so t tissue

T2 image demonstrates hyperintense left paraspinal muscle T1 image demonstrates isointense left paraspinal muscle
enlargement. enlargement.

Answers
1. Paraspinal muscle enlargement and T2 hyperintensity Pearls
can be seen with intramuscular myxoma, so t tissue • Mazabraud syndrome is a rare syndrome in which
sarcoma, posttraumatic edema, myositis, and early solitary or multiple intramuscular myxomas are seen
denervation injury. in conjunction with either monostotic or polyostotic
brous dysplasia.
2. Myxomas may enlarge and cause restriction o
• The etiology and pathophysiology o Mazabraud
movement; they are generally painless. Pathologic
are unknown. There does not appear to be a genetic
racture could result rom associated brous dysplasia.
predilection.
3. Mazabraud syndrome consists o so t tissue myxomas • Treatment consists o surgical excision o
and brous dysplasia. Ma ucci syndrome consists o symptomatic masses.
enchondromas and so t tissue hemangiomas. • Asymptomatic masses may be ollowed; there is a
small risk o malignant trans ormation.
4. The classic imaging appearance o myxoma are CT
hypodensity, no activity on PET, T1 hypointensity, T2
hyperintensity, and no signi cant enhancement.
Suggested Readings
5. Symptomatic lesions may be resected; however,
asymptomatic lesions require no treatment. Some Case DB, Chapman CN, Freeman JK, Polga JP. Atypical
argue they should be ollowed as there is a low risk o presentation o polyostotic brous dysplasia with myxoma
malignant trans ormation. (Mazabraud syndrome). Radiographics. 2010;30:827-832.
Iwasko N, Steinbach LS, Disler D, et al. Imaging ndings in
Mazabraud’s syndrome: seven new cases. Skeletal Radiol.
2002;31:81-87.

302
Chiari I malformation

1. What are presenting symptoms?

2. Which ndings indicate disruption o


cerebrospinal f uid f ow?

3. What are the advantages o cine phase-contrast


MRI sequences?

4. What is the normal pattern o cerebrospinal


f uid f ow with respect to the cardiac cycle?

5. What are the treatment options?

303
Cerebrospinal uid ow study 2207
Case ranking/dif culty: Category: Thecal sac

4. During systole, there is increased intracranial pressure


secondary to inf ow o blood, leading to caudal f ow o
cerebrospinal f uid. The reverse is true during diastole.
5. Depending on the f ow studies and associated ndings,
suboccipital decompression, ventricular shunting, and
syringosubarachnoid shunt are potential treatment
options.

Pearls
• Chiari I mal ormation is o ten associated with
disruption o cerebrospinal f uid f ow at the
craniocervical junction.
• Classically, patients present with Valsalva-induced
occipital headaches, but any kind o headache can be
present.
Sagittal T2 CUBE image demonstrates hypointensity at the • Myelopathic symptoms may occur in the setting o
foramen magnum, consistent with dephasing of owing signi cant syrinx or presyrinx.
cerebrospinal uid. • Cerebrospinal f uid f ow studies help categorize and
quanti y disruption o cerebrospinal f uid f ow i
present.
Answers
• This can help identi y patients who will bene t rom
1. Classically, patients present with Valsalva-induced suboccipital craniectomy.
occipital headaches, but any kind o headache can be
present. Myelopathic symptoms may occur in the setting
o signi cant syrinx or presyrinx.
Suggested Readings
2. Spatial inhomogeneity, elevated peak velocity,
Iskandar BJ, Quigley M, Haughton VM. Foramen magnum
simultaneous bidirectional f ow, and increased cranial
cerebrospinal f uid f ow characteristics in children with
f ow can indicate disruption o normal cerebrospinal
Chiari I mal ormation be ore and a ter craniocervical
f uid f ow in the setting o Chiari I mal ormation.
decompression. J Neurosurg. 2004;101:169-178.
3. Cine phase-contrast sequences do not require contrast McGirt MJ, Nimjee SM, Floyd J, Bulsara KR, George
and can be acquired in a relatively short amount o TM. Correlation o cerebrospinal f uid f ow dynamics
time (less than 15 minutes). The spatial resolution is and headache in Chiari I mal ormation. Neurosurgery.
good, but not as high as volumetric sequences, but the 2005;56:716-721.
phase-contrast sequence gives additional directional and
velocity in ormation.

304
Sprengel deformity, club foot, arthrogryposis

1. What should be included in the di erential


diagnosis?

2. What are common presenting symptoms?

3. What is the mode o transmission?

4. What portion o the spine is most commonly


a ected?

5. What are the treatment options?

305
Multiple hereditary exostoses 2221
Case ranking/dif culty: Category: More than one category

4. The cervical spine is most commonly a ected.


5. Asymptomatic patients may be managed conservatively
with consideration or imaging ollow-up due to the
low but real risk o malignant degeneration. Surgical
resection is recommended or symptomatic lesions.

Pearls
• Multiple hereditary exostoses are characterized by
multiple osteochondromas, which commonly a ect
the long bones.
• Most patients (approximately 2/3) will have spinal
involvement, with involvement o the posterior
elements and cervical spine most common.
• The disorder is inherited in an autosomal dominant
manner.
• Usually these lesions are asymptomatic; however,
patients may develop radicular or myelopathic
Parasagittal T2 image demonstrates osseous exostosis extending symptoms depending on location.
from the anterior C7 vertebral body; incidentally noted fusion of • Continued ollow-up is recommended as there is a
the posterior elements of C1-C3. small (<2%) risk o malignant trans ormation o the
overlying cartilaginous cap into chondrosarcoma.

Answers
1. Enthesopathic changes, brous dysplasia, osteoma, and Suggested Readings
osteochondroma can all be included in the di erential
diagnosis. Ezra N, Tetteh B, Diament M, Jonas AJ, Dickson
P. Hereditary multiple exostoses with spine
2. Presenting symptoms include pain, restriction o involvement in a 4-year-old boy. Am J Med Genet A.
movement, radiculopathy, and myelopathy. 2010;152A:1264-1267.
3. Multiple hereditary exostoses are inherited in an So ka CM, Saboeiro GR, Schneider R. Multiple hereditary
autosomal dominant manner. exostoses. HSSJ. 2005;1:49-51.

306
Palpable neck mass

1. What should be included in the di erential


diagnosis?

2. What are common presenting symptoms?

3. Where do these lesions tend to occur?

4. What are the di erent WHO classi cations o


benign lipomatous tumors?

5. What are the treatment options?

307
Lipoblastoma 2220
Case ranking/dif culty: Category: Paraspinal so t tissue

Axial T1 image demonstrates lobulated hyperintense mass within Axial T1 image following gadolinium administration, using
the prevertebral soft tissues, extending into the left neck. a chemical-shift-based method to eliminate signal from fat,
demonstrates enhancement of multiple septations in the
prevertebral and left neck mass with loss of signal from the
Answers central portions, consistent with fat content.
1. Fat-containing lesions include lipoma, lipomatosis,
lipoblastoma, lipoblastomatosis, and liposarcoma, which
can be di erentiated based on patient age, encapsulation, Pearls
number, and cellular atypia. • Lipoblastomas are benign atty tumors o the
so t tissues, which have a predisposition or the
2. The most common presenting symptom is a palpable
extremities, but also can a ect the neck, mediastinum,
or enlarging mass; depending on location, there may
abdomen, and retroperitoneum.
be associated pain or limping (i the lesion is within
• They are almost exclusively seen in children—an
an extremity). Compression o the airway could cause
important act to remember, as they look similar
stridor/wheezing.
to liposarcoma on imaging; however, the latter is
3. Lipoblastomas can occur anywhere that there is atty generally a disease o adult patients.
tissue; however, the most common location are the • The classic appearance is a at intensity mass with
limbs. enhancing linear septations. There may be a prominent
myxoid matrix and the lesion is usually well
4. Recognized subtypes o benign lipomatous
circumscribed; however, the lesions may be in ltrative
tumors include lipoma/lipomatosis, angiolipoma,
in nature.
lipoblastoma/lipoblastomatosis, myolipoma,
• Key characteristics in an imaging description include
chondroid lipoma, lipomatosis o nerve, spindle cell
proximity to nerves and vasculature and compression
lipoma/pleomorphic lipoma, and hibernoma.
o adjacent structures, such as the airway in this case.
5. For symptomatic lesions, surgical resection is • Treatment is surgical excision; however, these lesions
recommended; however, up to one-quarter o patients have a propensity to recur and may necessitate urther
will recur, necessitating urther surgery. Conservative operative intervention.
management may be appropriate or asymptomatic
lesions, particularly as they have a propensity to
di erentiate into lipomas with increased patient age. Suggested Readings
Bancro t LW, Kransdor MJ, Peterson JJ, O’Connor MI.
Benign atty tumors: classi cation, clinical course,
imaging appearance and treatment. Skeletal Radiol.
2006;35:719-733.
Speer AL, Scho eld DE, Wang KS, et al. Contemporary
management o lipoblastoma. J Ped Surg. 2008;43:
1295-1300.
308
Follow up known cavernomas and developmental venous anomalies

1. What should be included in the di erential


diagnosis?

2. Where do lesions that cause this typically


occur?

3. What are common presenting symptoms?

4. What are the recognized phases?

5. What are the treatment options?

309
Hypertrophic olivary degeneration 2217
Case ranking/dif culty: Category: Spinal cord

Pearls
• Hypertrophic olivary degeneration is a orm o
transsynaptic degeneration, which can pose a
diagnostic dilemma given the associated expansion
instead o volume loss.
• The role o the radiologist is to correctly make the
diagnosis to avoid unnecessary biopsy/workup.
• It is important to have a working knowledge o the
Guillain-Mollaret triangle to assess the three patterns
that can be seen:
1. Ipsilateral hypertrophic olivary degeneration—
lesion within the central tegmental tract
2. Contralateral hypertrophic olivary degeneration—
lesion within the superior cerebellar peduncle or
dentate nucleus
3. Bilateral hypertrophic olivary degeneration—lesion
within the central tegmental tract and superior
cerebellar peduncle
Axial T2 image demonstrates hyperintensity and expansion of the
medullary olive.
Suggested Readings

Answers Kitajima M, Korogi Y, Shimomura O, et al. Hypertrophic


olivary degeneration: MR imaging and pathologic
1. Etiologies o intramedullary T2 hyperintensity and ndings. Radiology. 1994;192:539-543.
expansion include demyelinating disease, astrocytoma, Shah R, Markert J, Bag AK, Cure JK. Di usion tensor
metastasis, in arction, and lymphoma. imaging in hypertrophic olivary degeneration. AJNR.
2. Lesions within the superior cerebellar peduncle, 2010;31:1729-1731.
cerebellum, dentate nucleus, and central tegmental tract
can all produce hypertrophic olivary degeneration.
3. Palatal myoclonus is the most common presentation.
4. The phases o hypertrophic olivary degeneration are T2
hyperintensity, T2 hyperintensity, and hypertrophy and
nally T2 hyperintensity without hypertrophy.
5. Correctly recognizing this entity eliminates the need or
unnecessary workup or treatment.

310
Sudden-onset diplopia, dysarthria, and ataxia

1. What should be included in the di erential


diagnosis?

2. What other disorders are considered to be


along the same spectrum?

3. What are common presenting symptoms?

4. What are treatment options?

5. What is the prognosis and outcome or this


disorder?

311
Bickersta encephalitis 2186
Case ranking/dif culty: Category: Spinal cord

Pearls
• Bickersta encephalitis is a rare disorder characterized
by drowsiness, ophthalmoplegia, ataxia, positive
Babinski sign, and hemisensory loss.
• It is along the spectrum o Guillain-Barré syndrome.
• Approximately 2/3 o patients will demonstrate a
positive serum anti-GQ1b IgG antibody and treatment
can be geared toward immunophoresis. In the
remaining 1/3, treatment is supportive.
• The disorder generally has a monophasic course and a
good outcome.
• Only 1/3 o patients demonstrate abnormalities on
MRI, generally mani ested as T2 hyperintensity within
the brainstem.

Suggested Readings
Bickersta ER, Cloake PCP. Mesencephalitis and
Axial T2 image demonstrates increased signal within the ventral
medulla bilaterally. rhombencephalitis. Br Med J. 1951 Jul 14;77-81.
Odaka M, Yuki N, Yamada M, et al. Bickersta ’s brainstem
encephalitis: clinical eatures o 62 cases and a subgroup
Answers associated with Guillain-Barré syndrome. Brain.
1. Potential etiologies include viral encephalitis, brainstem 2003;126:2279-2290.
tumor, Bickersta encephalitis, lupus encephalitis, and
paraneoplastic syndrome.
2. Bickersta encephalitis, Miller-Fisher syndrome, and
Guillain-Barré syndrome are along a continuum and
patients o ten present with components o multiple
syndromes.
3. Bickersta encephalitis is composed o drowsiness,
ophthalmoplegia, ataxia, positive Babinski sign, and
hemisensory loss.
4. Immunophoresis directed at the anti-GQ1b IgG
antibody i present is indicated; otherwise, supportive
management is usually su cient.
5. Most patients have a complete recovery; however, there
is the potential or morbidity and mortality as well
as some patients who develop a chronic relapsing—
remitting course.

312
Neck pain

1. What should be included in the di erential


diagnosis?

2. What are common presenting symptoms?

3. What are potential causative actors?

4. What are the most common locations within


the cervical spine?

5. What are the treatment options?

313
Multiple synovial cysts 2179
Case ranking/dif culty: Category: More than one category

Sagittal T2 image demonstrates well-circumscribed uid intensity Sagittal T2 image demonstrates well-circumscribed uid intensity
mass in the dorsal spinal canal at the C7 level. mass in the ventral spinal canal at the C3-C4 level.

Answers
• Synovial cysts likely arise rom degenerative,
1. The di erential or a cystic lesion include synovial
traumatic, or inf ammatory changes o the adjacent
cyst, ganglion cyst, ligamentum f avum cyst, and
acet joints.
schwannoma.
• In the cervical spine, they are most common in the
2. Pain, radiculopathy, and myelopathy are the most atlantoaxial and cervicothoracic regions.
common presenting symptoms. Synovial cysts unto • Treatment is reserved or symptomatic lesions and
themselves do not cause instability. includes resection o the cysts. Spinal usion is
generally not indicated.
3. Rheumatoid arthritis, osteoarthritis, trauma, and
in ection can be associated with synovial cyst ormation.
4. These lesions are most common at the craniocervical and
cervicothoracic junction. Suggested Readings
5. Asymptomatic lesions may be le t alone; however, Costa F, Menghetti C, Cardia A, Formari M, Ortolina A.
symptomatic lesions are best treated with resection. Cervical synovial cyst: case report and review o the
literature. Eur Spine J. 2010;19:S100-S102.
Lyons MK, Birch BD, Krauss WE, Patel NP, Nottmeier EW,
Boucher OK. Subaxial cervical synovial cysts. Spine.
Pearls 2011;36:E1285-E1289.
• Synovial cysts are a common cause o lumbar back
pain; however, they are not commonly seen in the
cervical spine.
• There is debate over the proper terminology o
these lesions, which may be based on histologic
characteristics.

314
Left-sided numbness

1. What should be included in the di erential


diagnosis?

2. What are common presenting symptoms?

3. What are the components o this syndrome?

4. What portion o the cord is most commonly


a ected?

5. What are the treatment options?

315
Transdural cord herniation 1527
Case ranking/dif culty: Category: More than one category

4. The thoracic cord, speci cally the upper thoracic cord, is


most commonly a ected by transdural cord herniation.
5. Treatment consists o surgical reduction o the cord with
repair o the dural de ect, using gra ts as necessary.

Pearls
• Transdural cord herniation is a rare abnormality, but
an important etiology to di erentiate rom a dorsal
arachnoid cyst.
• There is controversy over the actors that lead to
transdural cord herniation; however, the dominant
theory is that a herniated disc or osteophyte tears the
dura, allowing the cord to herniate. This is supported
by the data that show most o these lesions occur at
a disc level, o ten with superimposed degenerative
changes.
• Patients o ten present with lower extremity weakness
or sensory disturbance.
• Some patients present with Brown-Sequard syndrome
rom hemicord injury.
• On imaging, there is ventral displacement o the spinal
cord, which may be complicated by superimposed
edema.
Sagittal T2 image demonstrates focal ventral displacement of the
spinal cord at the T3-T4 level. • The main di erential diagnosis is a dorsal arachnoid
cyst. This can be di erentiated using myelography—
arachnoid cysts either will not ll (i extradural) or
Answers may ll in a delayed manner (i intradural).
1. The di erential diagnosis or displacement and thinning • Treatment is surgical with reduction o the herniation
o the cord would include myelomalacia, postoperative and repair o the dural de ect.
adhesions, or mass e ect rom a cystic lesion, such as
arachnoid cyst or epidermoid cyst.
2. Lower extremity paresthesia and weakness are the Suggested Readings
most common presenting symptoms, occasionally with Batzdor U, Holly LT. Idiopathic thoracic spinal cord
associated spasticity. Pain and postural headache have herniation: report o 10 patients and descriptions
been reported, but in requently. o surgical approach. J Spinal Disord Tech.
2012;25:157-162.
3. The components o Brown-Sequard syndrome include
Brus-Ramer M, Dillon WP. Idiopathic thoracic spinal cord
ipsilateral loss o motor unction and touch and
herniation: retrospective analysis supporting a mechanism
contralateral loss o pain and temperature sensation,
o diskogenic dural injury and subsequent tamponade.
resulting rom a hemicord injury.
AJNR. 2012;33:52-56.

316
G4P2 with an abnormal screening ultrasound

1. What should be included in the di erential


diagnosis?

2. What are the components o the Altman


classi cation sequence?

3. What are potential intrauterine/obstetric


complications?

4. What are the components o the Currarino


triad?

5. What are the treatment options?

317
Sacrococcygeal teratoma 1523
Case ranking/dif culty: Category: More than one category

Sagittal T2 SSFSE image from a fetal MRI demonstrates a low Axial T2 SSFSE image from a fetal MRI demonstrates a low
presacral complex solid and cystic mass. presacral complex solid and cystic mass.

Answers
1. Neonatal lumbosacral masses include anterior sacral Pearls
meningocele, chordoma, dermoid cyst, hemangioma, and • Sacrococcygeal teratomas are the most common solid
terminal myelocystocele. tumor in neonates; two-thirds are benign.
• Classi cation is based on intrapelvic and extrapelvic
2. The Altman classi cation:
location, which is important or surgical planning and
Type 1—external component risk o malignant degeneration.
Type 2—external and small presacral components • These are o ten diagnosed antenatally on ultrasound
Type 3—internal and external components or MRI.
• Depending on lesion size, there may be implications
Type 4—internal component
or delivery and neonatal cardiorespiratory status.
The greater the proportion o internal components, the • Surgical resection is the treatment o choice with a risk
more di cult the resection and the higher the risk o or malignant degeneration in residual tissue.
malignant degeneration.
3. Potential complications, which can be seen in three-
ourths o patients carrying etuses diagnosed with Suggested Readings
sacrococcygeal teratomas, include HELLP syndrome,
Kocaoglu M, Frush DP. Pediatric presacral masses.
hyperemesis, oligohydramnios, polyhydramnios, and
Radiographics. 2006;26:833-857.
preterm labor.
Woodward PJ, Sohaey R, Kennedy A, Koeller KK. A
4. The Currarino triad includes anorectal mal ormation, comprehensive review o etal tumors with pathologic
presacral mass, and sacrococcygeal osseous de ect. One- correlation. Radiographics. 2005;25:215-242.
th o sacrococcygeal teratomas are associated with
additional abnormalities.
5. Surgical resection is the treatment o choice with
chemotherapy and/or radiation or malignant lesions. In
utero drainage o cystic components or resection may be
per ormed to acilitate delivery.

318
Persistent postural headaches

1. What should be included in the di erential


diagnosis o headache with pachymeningeal
enhancement?

2. What imaging studies can be per ormed or


urther evaluation?

3. What are potential etiologies?

4. Gadolinium myelography should be per ormed


with what parameters?

5. What are the treatment options?

319
Cerebrospinal uid leak evaluation 2188
Case ranking/dif culty: Category: Thecal sac

4. Imaging parameters should include T1 pre- and post-


intrathecal contrast. Fat suppression is imperative or
accurate evaluation o the thecal sac and nerve root
sleeves; however, given the presence o potential arti act,
at suppression should also be applied be ore contrast
administration to allow or equal comparison. Immediate
and delayed imaging should be considered, as well as
concurrent CT myelography.
5. Treatment options include epidural blood patch, brin
injection, or surgical repair.

Pearls
• Spontaneous intracranial hypotension presents a
diagnostic dilemma to the clinician.
• It can be easily con used with other causes o postural
headache, and diagnosing the exact location o
cerebrospinal f uid leak can be di cult.
• Classically, myelography was per ormed in
association with post-contrast CT and occasionally
Sagittal T1 image with fat suppression following intrathecal nuclear medicine studies.
contrast administration demonstrates multiple sacral nerve root • There is new interest in the use o gadolinium
cysts. myelography, which can be per ormed in conjunction
with traditional imaging methods, as it has increased
spatial resolution.
Answers
1. Pachymeningeal enhancement involves the dura with
sparing o the leptomeninges. Etiologies include
Suggested Readings
intracranial hypotension, dural metastasis, and dural
lymphoma. Akbar JJ, Luetmer PH, Schwartz KM, Hunt CH, Diehn FE,
Eckel LJ. The role o MR myelography with intrathecal
2. Intrathecal injection o myelographic contrast, gadolinium in localization o spinal CSF leaks in patients
gadolinium, and indium-111 can be per ormed with spontaneous intracranial hypotension. AJNR.
simultaneously with acquisition o multiple images to 2012;33:535-540.
improve diagnostic accuracy. Kraemer N, Berlis A, Schumacher M. Intrathecal
3. Potential causes o decreased cerebrospinal f uid gadolinium-enhanced MR myelography showing multiple
pressure include trauma with dural tear, nerve root sleeve dural leakages in a patient with Mar an syndrome. AJR.
cysts, dural ectasis, lumbar puncture, and spinal surgery. 2005;185:92-94.

320
History of tethered cord release

1. What should be included in the di erential


diagnosis?

2. What are common presenting symptoms?

3. What are associated symptoms?

4. What is the most common location within the


spine?

5. What are the treatment options?

321
Postoperative dermoid 1521
Case ranking/dif culty: Category: Thecal sac

Sagittal T1 image demonstrates intradural hyperintense mass at Sagittal T2 image demonstrates intradural hyperintense mass at
the level of L5. the level of L5.

Answers
• Acquired lesions are iatrogenic and o ten arise
1. Lesions that tend to occur around the conus and may
secondary to previous surgery or lumbar puncture with
have complex signal characteristics include lipoma,
introduction o dermal and epidermal elements into
lipomyelocele, dermoid cyst, and epidermoid cyst.
the thecal sac.
2. Common presenting symptoms include headache, back • Patients are o ten asymptomatic with lesions
pain, radiculopathy, myelopathy, and urinary retention; discovered incidentally at imaging ollow-up.
however, the presence o headache raises the concern or • Symptoms may include slowly progressive
rupture with associated chemical meningitis. radiculopathy or myelopathy depending on the
location o the dermoid.
3. Acute dermoid cyst rupture can lead to secondary
• Rupture o a dermoid may cause chemical meningitis.
chemical meningitis and present with headache, nausea,
• Treatment is resection to prevent complications/
vomiting, mental status changes, and coma.
progressive symptoms.
4. 80% are located within the lumbosacral region or
associated with the cauda equina.
5. Complete surgical resection is the optimal treatment Suggested Readings
to prevent progressive symptoms or rupture. Cysts
De Maio PN, Mikulis DJ, Kiehl T-R, Guha A. Spinal conus
may recur and are more likely to recur in the setting o
dermoid cyst with lipid dissemination. Radiographics.
incomplete resection.
2012;32:1215-1221.
Liu H, Zhang J-N, Zhu T. Microsurgical treatment o spinal
epidermoid and dermoid cysts in the lumbosacral region.
Pearls J Clin Neurosci. 2012;19:712-717.
• Spinal dermoid cysts can be divided into two
categories.
• The congenital lesions arise rom residual dermal rests
or rom expansion o a dermal sinus.

322
Follow up known disease

1. What should be included in the di erential


diagnosis or an osseous lesion?

2. What should be included in the di erential


diagnosis o so t tissue lesion?

3. What are common presenting symptoms?

4. What are associated ndings?

5. What are the treatment options?

323
Myo broma 1518
Case ranking/dif culty: Category: Posterior elements

5. So t tissue lesions are usually resected or pathologic


diagnosis. When there are multiple lesions and
involvement o multiple organ systems, symptomatic
lesions may be resected. Other treatment options,
including physical therapy, bracing, and surgical xation,
may be considered i complications develop.

Pearls
• Myo broma is the most common so t tissue tumor o
in ancy and the solitary orm may be resected without
recurrence.
• However, once patients have multiple lesions, the
prognosis is slightly worse.
• Multicentric myo bromatosis may have so t
tissue, osseous, and visceral involvement, with
symptomatology based on system a ected.
• The so t tissue tumors can be con used with benign
Parasagittal T2 image demonstrates expansile lesion within the and malignant so t tissue tumors.
posterior elements of T5. Partially visualized expansion of T7 • Osseous lesions are generally lytic and must be
posterior elements.
di erentiated rom metastases, in ection, and
histiocytosis.
Answers • Lesions o ten demonstrate early growth ollowed by
stabilization or regression.
1. Etiologies or lytic osseous lesions include metastasis, • Large or symptomatic lesions can be resected; there is
lymphoma, Langerhans cell histiocytosis, in ection, and no other known treatment.
giant cell tumor.
2. So t tissue masses that must be di erentiated rom
myo broma include hemangiopericytoma, brosarcoma, Suggested Readings
leiomyoma, leiomyosarcoma, and neuro broma.
Green MC, Dor man HD, Villanueva-Siles E, et al.
3. Palpable mass, pain, and pathologic racture are Aggressively recurrent in antile myo broma o the axilla
the most common presenting symptoms o osseous and shoulder girdle. Skeletal Radiol. 2011;40:357-361.
myo bromatosis. Holzer-Fruehwald L, Blaser S, Roosi A, Fruehwald-Pallamar J,
4. Potential associated ndings include lytic vertebral Thurner MM. Imaging ndings in seven cases o congenital
body lesion, epidural extension, vertebra plana, cord in antile myo bromatosis with cerebral, spinal or head and
compression, and intramedullary extension; however, neck involvement. Neuroradiology. 2012;54:1389-1398.
intramedullary extension has only been reported once in
the literature.

324
Neck pain

1. What should be included in the di erential


diagnosis?

2. What are common presenting symptoms?

3. What are the components o McCune Albright


syndrome?

4. What are the components o Mazabraud


syndrome?

5. What are the treatment options?

325
Multifocal brous dysplasia with pathologic fracture 1517
Case ranking/dif culty: Category: More than one category

5. Treatment options include conservative management,


bisphosphonate therapy, biopsy, surgical xation, and
curettage with packing; the decision is based on number
and location o lesions and presence o complications
(growth disturbance, scoliosis, pathologic racture).

Pearls
• Fibrous dysplasia can have multiple appearances,
including internal ground glass opacity and primary
lytic appearance.
• The spine is generally only involved in cases o
polyostotic brous dysplasia, which is less common
than the monostotic orm.
• Polyostotic brous dysplasia should be considered in
the di erential diagnosis o multiple osseous lesions.
• In the adult patient, the leading diagnosis or this case
should be metastatic disease, even in the absence o a
known primary.

Suggested Readings
Case DB, Chapman CN, Freeman JK, Polga JP. Atypical
Sagittal CT image demonstrates mixed attenuation in ltration of presentation o polyostotic brous dysplasia with myxoma
the dens with areas of “ground glass” attenuation. Burst fracture (Mazabraud syndrome). Radiographics. 2010;30:827-832.
of C3 with lytic region in the vertebral body. Stanton RP, Ippolito E, Spring eld D, Lindaman L,
Wientroub S, Leet A. The surgical management o
brous dysplasia o bone. Orphanet J Rare Dis.
Answers
2012;7(suppl 1):S1.
1. Multiple lytic osseous lesions can be seen in in ection,
metastasis, Paget disease, brous dysplasia, and multiple
myeloma.
2. Potential presenting symptoms include scoliosis, back
pain, and pathologic racture; endocrine dys unction
could be seen in McCune Albright syndrome and so t
tissue mass could be seen in Mazabraud syndrome.
3. McCune Albright syndrome is composed o ca e au lait
skin lesion, precocious puberty, and polyostotic brous
dysplasia.
4. Mazabraud syndrome consists o monostotic or
polyostotic brous dysplasia in conjunction with a single
or multiple so t tissue myxomas, o ten intramuscular.

326
Weakness

1. What should be included in the di erential


diagnosis?

2. What are common presenting symptoms?

3. What additional imaging ndings can be seen?

4. What additional ndings are necessary to make


the diagnosis?

5. What are the treatment options?

327
Neuromyelitis optica 1479
Case ranking/dif culty: Category: Spinal cord

Sagittal T2 image demonstrates extensive T2 hyperintensity and Sagittal T1 image following contrast administration demonstrates
expansion of the cord. vague amorphous enhancement.

Answers
lack o intracranial lesions, extensive T2 hyperintensity
1. T2 hyperintensity and expansion o the cord can be seen
within the cord, and positive NMO-IgG titers.
in acute disseminated encephalomyelitis, in ectious
• Patients o ten present with symptoms o optic neuritis,
myelitis, multiple sclerosis, primary cord tumor, and
in conjunction with myelopathy.
transverse myelitis.
• This disorder is more common in A rican American
2. Patients o ten present with symptoms o optic neuritis patients.
in conjunction with variable myelopathic symptoms, • Lumbar puncture may show oligoclonal bands, but
including bowel and bladder dys unction, sensory cellular pleocytosis is the rule.
dysesthesia, and weakness. • Treatment is aimed at decreasing inf ammation with
steroids, with addition o immunosuppressive therapy
3. Most cases (over three-quarters) will demonstrate optic
a ter the acute attack.
nerve enhancement.
• The overall prognosis o NMO tends to be worse than
4. Cerebrospinal f uid pleocytosis is a major criterion or multiple sclerosis, with multiple relapses being the
the diagnosis o NMO, as is negative brain imaging. rule.
NMO-IgG is not currently incorporated in the criteria;
however, it is being considered.
5. First-line therapy is steroids with consideration o Suggested Readings
immunosuppression in select cases. Plasmapheresis is Aboul-Enein F, Krssak M, Ho tberger R, Prayer D,
considered or recalcitrant cases. Kristo eritsch W. Di use white matter damage is absent
in neuromyelitis optica. AJNR. 2010;31:76-79.
Pearls Wingerchuk DM, Lennon VA, Pittock SJ, Lucchinetti
CF, Weinshenker BG. Revised diagnostic criteria or
• Neuromyelitis optica (NMO) is a demyelinating neuromyelitis optica. Neurology. 2006;66:1485-1489.
disease that a ects the spinal cord and optic nerves.
• It can be di cult to di erentiate rom other
demyelinating disease but the diagnosis is based on

328
Neck pain and wasting of the hand muscles

1. What is this procedure called?

2. What are the advantages o an anterior


approach?

3. What is the purpose o this procedure?

4. Describe the important surgical steps o the


procedure.

5. Name potential complications o this


procedure.

329
Anterior cervical discectomy and fusion (ACDF) 3303
Case ranking/dif culty: Category: Disc

Sagittal T2-weighted Axial T2-weighted image at the level of C5 AP uoroscopic image Lateral uoroscopic image
sequence of the cervical showing a central disc protrusion (arrow) that acquired during the acquired during surgery. A
spine showing multilevel impinges the cervical cord. procedure. A titanium titanium plate (arrow) was
disc degenerative change plate (arrow) was xed anteriorly using screws
with disc protrusions secured using screws (arrowheads).
(arrows) between C4 and (arrowheads) following
C6. The discs abut the discectomy.
cervical cord anteriorly
and the central canal is
narrowed. 5. ACDF is considered a airly sa e procedure and major
complication occur in only about 1%-2%. Most patients
Answers will experience odyno- or dysphagia in the rst ew
1. Anterior cervical discectomy and usion (ACDF) is a days postop due to retraction o the esophagus during
surgical procedure per ormed or cervical instability, the procedure. Potential complications o ACDF include
pain ul cervical disc herniations, osteophytes compressing injury to the larynx, laryngeal nerves, esophagus, carotid
the nerve roots or spinal cord, and degenerative disc artery, spinal cord, and nerve roots. In ection and implant
disease. ACDF is o ten per ormed when conservative ailure, movement, or malposition may also occur.
measures have ailed. This procedure is per ormed on the
cervical spine using an anterior approach. It involves a
discectomy ollowed by stabilization o the cervical spine. Pearls
2. The advantages o using an anterior (as opposed to a • Anterior cervical discectomy and usion (ACDF)
posterior) approach include a better access to the cervical involves a discectomy ollowed by stabilization o the
spine rom C2 down to the cervicothoracic junction and cervical spine.
a signi cant reduction in postoperative pain as the spine • The purpose o ACDF is to ree the impinged
can be accessed via anatomical planes. nerves or spinal cord by removing the causative disc
3. The purpose o ACDF is to ree the impinged nerves or herniations or osteophytes and restore the height o the
spinal cord by removing the causative disc herniations or a ected disc.
osteophytes and restore the height o the a ected disc. • ACDF is considered a airly sa e procedure and major
complication occur in only about 1%-2%.
4. A ter an anterior incision the surgeon divides the
platysma and then ollows anatomic planes right down
to the spine. The intervertebral disc and any osteophytes
Suggested Readings
are then completely removed. The intervertebral oramen
may also be widened in order to make more room or the Fountas KN, Kapsalaki EZ, Nikolakakos LG, et al. Anterior
exiting nerve root. The intervertebral space is then lled cervical discectomy and usion associated complications.
with bone gra t to increase stability. A titanium plate Spine (Phila Pa 1976). 2007 Oct 1;32(21):2310-2317.
may be screwed on the anterior aspect o the cervical Riley LH 3rd, Skolasky RL, Albert TJ, Vaccaro AR, Heller
vertebrae to ensure stability during usion, particularly JG. Dysphagia a ter anterior cervical decompression and
when there is more than one disc level involved. The usion: prevalence and risk actors rom a longitudinal
patient may need to wear a neck brace or collar or the cohort study. Spine (Phila Pa 1976). 2005 Nov
rst ew weeks to ensure proper spinal alignment. 15;30(22):2564-2569.
330
Left arm and leg numbness

1. What should be included in the di erential


diagnosis?

2. What are common presenting symptoms?

3. What are components o Cobb syndrome?

4. Which part o the spine is most commonly


involved?

5. What are the treatment options?

331
Juvenile arteriovenous malformation 1513
Case ranking/dif culty: Category: More than one category

Pearls
• Type 3 (juvenile) arteriovenous mal ormations o the
spine have both intramedullary and extramedullary
components.
• They may present with progressive weakness or
myelopathic symptoms or subarachnoid hemorrhage.
• The diagnosis is based on seeing intramedullary and
extramedullary vessels with a de nable nidus.
• There are o ten T2 hyperintense changes within the
spinal cord, which may be secondary to ischemic steal
or venous hypertension.
• There may be an extraspinal component to these
lesions.
• Treatment is generally per ormed with endovascular
embolization.
• Given the di use nature o these lesions, complete
resection is usually not possible.
• Overall prognosis is poor with progressive
neurological de cits i embolization is not success ul.

Sagittal T2 image demonstrates hyperintensity and expansion of


the cord with ow voids surrounding and within the cord. Suggested Readings
Niimi Y, Uchiyama N, Elijovich L, Berenstein A.
Answers Spinal arteriovenous metameric syndrome: clinical
mani estations and endovascular management. AJNR.
1. The di erential diagnosis or intramedullary T2 2012; epub
hyperintensity includes astrocytoma, ependymoma, Spetzler RF, Detwiler PW, Riina HA, Porter RW. Modi ed
cavernoma, hemangioblastoma, type 1 juvenile classi cation o spinal cord vascular lesions. J Neurosurg.
arteriovenous mal ormation, and type 4 arteriovenous 2002;96:145-156.
stula.
2. Common presenting symptoms include pain, weakness,
dysesthesia, bowel and bladder incontinence, and
headache; headache can result rom subarachnoid
hemorrhage.
3. Cobb syndrome (spinal arteriovenous metameric
syndrome) is characterized by osseous, cutaneous, and
spinal cord vascular mal ormations.
4. Cervical and thoracic regions are most common;
however, they can occur anywhere.
5. Embolization is the treatment o choice; partial surgical
resection may be possible. However, total resection is
o ten not technically easible secondary to the large
nidus.

332
Lower extremity weakness

1. What nerve roots orm the lumbar plexus?

2. What nerve connects the lumbar plexus and


sacral plexus to orm the lumbosacral plexus?

3. What is the di erential diagnosis or


enlargement and enhancement o the
lumbosacral plexus?

4. What are common presenting symptoms?

5. What are common malignancies that in ltrate


the lumbosacral plexus?

333
Lumbosacral plexus metastatic in ltration 1499
Case ranking/dif culty: Category: Nerve roots/Nerve plexus/Peripheral nerves

Pearls
• In ltration o the lumbosacral plexus most commonly
occurs ollowing direct invasion rom pelvic tumors.
• Hematogenous spread can occur.
• Presenting symptoms include pain and weakness with
sensory de cits. The symptoms may involve multiple
nerve distributions and can be either unilateral or
bilateral.
• Lumbosacral plexus in ltration generally presents as
enlarged, T2 hyperintense, enhancing nerve roots.
• Treatment is directed at treating the primary
malignancy.

Suggested Readings
Grisariu S, Avni B, Batchelor TT, et al. Neurolymphomatosis:
an international primary CNS lymphoma collaborative
Coronal T1 image following gadolinium administration
group report. Blood. 2010;115:5005-5011.
demonstrates enlargement and enhancement of the S1 and S2
Petchprapa CN, Rosenberg ZS, Scon enza M, et al.
nerve roots on the left.
MR imaging o entrapment neuropathies o the lower
extremity. Radiographics. 2010;30:983-1000.
Answers
1. The lumbar plexus is ormed rom the ventral rami o L1
through L4. Thus, imaging should extend to the L1 level.
2. The lumbosacral trunk, composed primarily o the L5
nerve root with a small contribution rom L4, connects
the lumbar and sacral plexi to orm the lumbosacral
plexus.
3. Enlargement and enhancement o the lumbosacral plexus
can be caused by metastatic in ltration, plexi orm
neuro broma, lymphoma, or post-radiation plexopathy.
4. Presenting symptoms can include pain and sensory
de cits, as well as proximal muscle weakness. Rarely
patients can present with incontinence.
5. Colorectal, uterine, ovarian, and cervical carcinomas, as
well as lymphoma, are common causes o lumbosacral
plexus in ltration.

334
Severe back pain with radiculopathy, patient is postpartum

1. What is the most likely diagnosis?

2. How does the condition mani est clinically?

3. Name some causes o this condition.

4. Which imaging modality best images this


condition?

5. How is this condition managed?

335
Spinal subarachnoid hemorrhage (SSH) 3298
Case ranking/dif culty: Category: Thecal sac

Sagittal T1-weighted Contrast-enhanced T1- Axial T1-weighted MRI con rms the intradural location of the
sequence of the lumbar spine weighted sequence of hematoma (arrow). Note the lum terminale passing through the
demonstrates a T1 hyperintense the lumbar spine. The central part of the hematoma. The thecal sac is seen as a linear
lesion (arrow) in the caudal lesion does not enhance hypointensity (arrowhead).
aspect of the thecal sac. The following contrast
lesion was isointense on T2 and administration (arrow). Note
is compatible with an intradural abnormal leptomeningeal
sac with no evidence o an “inverted Mercedes star”
hematoma. Note is made of an enhancement (arrowheads) sign. The latter is typical o subdural hematoma, which
enlarged uterus in keeping with likely secondary to o ten has a semicircular appearance and tends to be more
recent pregnancy and delivery. complicating arachnoiditis. crescentic on axial sequences. Di erentiation between
subdural and subarachnoid hemorrhage may, however,
be di cult, and at times, only surgical exploration will
determine the exact location o the hematoma.
Answers
1. Spinal hemorrhage is a rare entity and may be 5. Urgent decompressive surgery remains the treatment
intramedullary (hematomyelia) or within the epidural, o choice in spinal subarachnoid hemorrhage with
subdural, or subarachnoid spaces. Spinal subarachnoid rapidly progressive neurological ndings. Conservative
hemorrhage (SSH) accounts or <1% o all subarachnoid treatment is reserved or patients without signi cant
hemorrhages. It is a potentially dangerous condition that neurological impairment.
may have disastrous consequences.
2. SSH presents with severe back pain or headache, which
may be accompanied by acute sciatic pain, sensory Pearls
disturbances, paraparesis, and sphincter disturbance. • Spinal hemorrhage is a rare entity and may be
3. SSH is o ten secondary to trauma (in >50% o intramedullary (hematomyelia) or within the epidural,
cases), bleeding diatheses (including anticoagulant subdural, or subarachnoid spaces.
medication), underlying vascular mal ormations • Urgent decompressive surgery remains the treatment
(arteriovenous mal ormation, spinal angioma, spinal o choice in spinal subarachnoid hemorrhage with
artery, or intracranial aneurysm), neoplasia (intratumoral rapidly progressive neurological ndings.
hemorrhage), and lumbar puncture. Rarer causes include
systemic lupus erythematous, periarteritis nodosa,
coarctation o the aorta, hypertension, and Behcet disease. Suggested Readings
Spinal SAH may also complicate intracranial SAH.
Kim JS, Lee SH. Spontaneous spinal subarachnoid
4. MRI is essential to determine the exact location and hemorrhage with spontaneous resolution. J Korean
extent o spinal hemorrhage. It may also reveal the Neurosurg Soc. 2009 Apr;45(4):253-255.
causative lesion or a vascular mal ormation. A pure Kim YH, Cho KT, Chung CK, Kim HJ. Idiopathic
subarachnoid hematoma can be easily di erentiated rom spontaneous spinal subarachnoid hemorrhage. Spinal
a subdural hematoma. SSHs are located within the thecal Cord. 2004 Sep;42(9):545-547.
336
Dropped head syndrome

1. What are the etiologies o this abnormality?

2. What is this abnormality known as when there


is no de nable cause?

3. What should be included in the imaging


evaluation?

4. What are common presenting symptoms?

5. What are the treatment options?

337
Denervation edema; dropped head syndrome 1497
Case ranking/dif culty: Category: Paraspinal so t tissue

4. While the weakness is o ten most pro ound in the


neck extensors, other muscle groups can be involved,
including the neck f exors and shoulder abductors.
5. There is controversy over the treatment o dropped head
syndrome with little evidence or e cacy o a single
treatment plan. Treatment options include conservative
management, steroids, intravenous immunoglobulins,
plasmapheresis, and surgical xation. Surgical xation is
reserved or re ractory causes in patients who are good
surgical candidates.

Pearls
• Dropped head syndrome is a general term applied to a
variety o disorders that lead to ocal cervical kyphosis
and inability to extend one’s spine against gravity.
• It can initially be di cult to di erentiate rom changes
in alignment rom the normal aging process.
Axial T2 image demonstrates increased T2 signal within the
posterior paraspinal muscles.
• Etiologies include myasthenia gravis, chronic
inf ammatory demyelinating polyneuropathy,
hypothyroidism, mitochondrial disease, amyotrophic
Answers lateral sclerosis, polymyositis, camptocormia, and
1. There are numerous etiologies or dropped head systemic sclerosis.
syndrome, including polymyositis, myasthenia, systemic • Treatment is aimed at the underlying abnormality with
sclerosis, amyotrophic lateral sclerosis, and chronic surgical usion reserved or recalcitrant cases in which
inf ammatory demyelinating polyneuropathy. the patients are elt to be good surgical candidates.

2. Isolated neck extensor myopathy is a cause o dropped


head syndrome and important to di erentiate rom other
causes. Suggested Readings
Katz JS, Wol e GI, Burns DK, Bryan WW, Fleckenstein
3. The evaluation o dropped head syndrome may include
JL, Barohn RJ. Isolated neck extensor myopathy: a
MRI cervical spine, electromyography, nerve conduction
common cause o dropped head syndrome. Neurology.
studies, and laboratory studies, depending on the clinical
1996;46:917-921.
situation.
Rosato E, Rossi C, Salsano F. Dropped head syndrome and
systemic sclerosis. Joint Bone Spine. 2009;76:301-303.

338
Cervical pain

1. What should be included in the di erential


diagnosis?

2. What are common presenting symptoms?

3. This abnormality arises rom a developmental


insult prior to what age?

4. What organ systems can be involved?

5. What are the treatment options?

339
Lymphangiomatosis 1487
Case ranking/dif culty: Category: More than one category

4. Lymphangiomatosis can involve any organ system.


5. Treatment options include conservative management,
surgical xation, radiation, bisphosphonate therapy, and
inter eron therapy. While these are all potential treatment
options, there is no set success ul treatment algorithm.

Pearls
• Lymphangiomatosis is a rare cause o multiple
osseous lytic lesions.
• It can also present with visceral and mediastinal
involvement.
• Lymphangiomatosis is usually diagnosed in childhood.
• There is no known etiology; however, it is associated
with persistent dilation o lymphatics secondary to
Lateral radiograph demonstrates multiple lucent lesions disrupted development in utero.
throughout the osseous structures; alignment is maintained. • Patients should be monitored or the development o
pain, which may signi y pathological racture.
• I extensive lesions threaten osseous stability,
Answers prophylactic surgical xation may be per ormed.
1. The di erential or multiple lytic osseous lesions
includes metastases, in ection, Langerhans cell
histiocytosis, and lymphangiomatosis. Letterer-Siwe Suggested Readings
disease is the previous name or multi ocal multisystem
Langerhans cell histiocytosis. Kwag E, Shim SS, Kim Y, Chang JH, Kim KC. CT eatures
o generalized lymphangiomatosis in adult patients. Clin
2. In the setting o primary osseous involvement, pain and Imag. 2013;37(4):723-727.
pathologic racture are the most common presenting Wunderbaldinger P, Paya K, Partik B, et al. CT and MR
symptoms; other symptoms include palpable mass and imaging o generalized cystic lympangiomatosis in
chylous pleural e usion. pediatric patients. AJR. 2000;174:827-832.
3. The insult is elt to likely occur between 14 and 20
weeks.

340
Lower spine lump

1. What should be included in the di erential


diagnosis?

2. When per orming radiographs or evaluation


o skeletal dysplasia, what are the spinal
parameters that should be reported?

3. What are the imaging ndings?

4. What are the treatment options?

5. What should be considered in the timing o


potential surgical correction?

341
Gibbus deformity 1481
Case ranking/dif culty: Category: Spinal canal

3. Findings that can be seen in mucopolysaccharidoses


include hypoplasia o the dens, platyspondyly, dural
thickening, and oramen magnum stenosis.
4. External bracing has not been shown to change the
outcome. However, surgery is generally reserved or
high-degree kyphosis or scoliosis.
5. Generally delaying surgical usion as long as possible
is ideal to allow maximum growth and make the
surgery technically easier. Other actors include rate
o progression o kyphosis/scoliosis and myelopathic
symptoms.

Pearls
• Gibbus de ormity is an unusual radiographic nding
that should prompt an evaluation or underlying
mucopolysaccharidosis.
• The kyphosis should be measured with radiographic
ollow-up to document progression.
• Kyphosis greater than 40o is more likely to progress.
Lateral radiograph demonstrates focal thoracolumbar kyphosis • Progressive kyphosis may necessitate surgical usion.
with mild anterior beaking of the T12 vertebral body.

Answers Suggested Readings


1. Gibbus de ormity is most commonly associated Parnell SE, Phillips GS. Neonatal skeletal dysplasias. Pediatr
with mucopolysaccharidoses; other etiologies Radiol. 2012;42:S150-S157.
include Scheuermann disease, idiopathic scoliosis, White KK. Orthopaedic aspects o mucopolysaccharidoses.
achondroplasia, and spondyloepiphyseal dysplasia. Rheumatology. 2011;50:v26-v33.
Gibbus de ormity may also be acquired eg, in
tuberculosis.
2. Alignment, pedicular length, vertebral body shape, and
usion or segmentation anomalies should all be reported.

342
Belted passenger in rollover motor vehicle collision

1. Which is the correct order o the brachial


plexus components rom proximal to distal?

2. What are common presenting symptoms?

3. Where are the components o the brachial


plexus located with respect to osseous and so t
tissue landmarks?

4. What imaging studies can be used in the


evaluation o brachial plexus injuries?

5. What are the treatment options?

343
Brachial plexus avulsion with spinal cord injury 1495
and pseudomeningocele
Case ranking/dif culty: Category: Nerve roots/Nerve plexus/Peripheral nerves

4. CT myelography and MRI are equivalent or diagnosis.


While myelography is invasive, discussion with the
ordering provider regarding their pre erence is important.
Chest radiographs can be help ul; i diaphragmatic
paralysis is present, this generally implies a permanent
de cit.
5. Treatment options include physical therapy and surgical
exploration or either reanastomosis or gra ting.
Immobilization is contraindicated and may lead to a
rozen joint.

Pearls
• Brachial plexopathy symptoms tend to be vague and
nonspeci c.
• Cases o neuropraxic (stretching) injuries and
avulsions tend to present with motor symptoms.
• Brachial plexus injuries can occur during delivery,
particularly in the setting o shoulder dystocia.
AP image of the cervical spine following intrathecal contrast
• A radiologist should attempt to de ne the gap distance
administration demonstrates focal outpouching of contrast at the
between the avulsed nerve segments, which can be
level of the left T1 and T2 nerve roots.
important in ormation or a surgeon planning potential
reanastomosis.
Answers • Additionally, evaluation or associated injuries,
including to the spinal cord should be per ormed.
1. The correct order rom proximal to distal is roots, trunks,
divisions, cords, and branches.
2. The brachial plexus has both motor and sensory
Suggested Readings
components and both can be a ected by injury. Horner
syndrome can occur rom injury to the in raclavicular Castillo M. Imaging the anatomy o the brachial
plexus. Diaphragmatic paralysis can occur rom injuries plexus: review and sel -assessment module. AJR.
involving the C3, C4, and C5 nerve roots. 2005;185:S196-S204.
Sureka J, Cherian RA, Alexander M, Thomas BP. MRI o
3. Roots are located in the neural oramina; trunks are brachial plexopathies. Clin Radiol. 2009;64:208-218.
located between the scalene muscles; divisions are
located posterior to the clavicle; cords are located
in erior to the clavicle. Knowing the anatomic location is
important or identi ying abnormalities.

344
Multiple congenital abnormalities

1. What should be considered in the di erential


diagnosis or neonatal scoliosis?

2. What are the etiologies?

3. What are prenatal ultrasound ndings?

4. What are associated neuropathic


abnormalities?

5. What are the treatment options?

345
Arthrogryposis multiplex congenita 1480
Case ranking/dif culty: Category: Spinal canal

3. Prenatal ultrasound ndings include xed f exion


de ormities, intrauterine growth retardation, increased
nuchal translucency, and scoliosis.
4. The neuropathic abnormalities seen in arthrogryposis are
varied and can include cerebellar hypoplasia, anterior
horn cell loss, dorsal column degeneration, pyramidal
tract degeneration, and peripheral neuropathy. Additional
abnormalities include cortical rontal atrophy, neuronal
migration abnormalities, and olivopontocerebellar
degeneration.
5. Therapy and splinting are the mainstays o treatment
with adjunct orthopedic surgery. Spinal cord stimulation
and ventriculoperitoneal shunt placement may be
considered in select patients.

Pearls
• Arthrogryposis is a rare disorder that causes multiple
joint contractures.
• It is multi actorial and may be secondary to
neurological disorders in the neonate or causes o
AP radiograph demonstrates C-shaped scoliosis and multiple restricted intrauterine movement.
joint contractures.
• It should be considered in the evaluation o the
neonate with scoliosis.
Answers • Treatment is geared at orthopedic abnormalities and
includes xation/surgical intervention as indicated, as
1. The di erential diagnosis or neonatal scoliosis includes
well as physical and occupational therapy.
hemivertebra, vertebral bar, neuromuscular scoliosis, and
paraspinal mass.
2. The primary abnormality is elt to be impaired
Suggested Readings
intrauterine etal movement, which can arise rom
multiple causes, including neurologic abnormalities, Gordon N. Arthrogryposis multiplex congenita. Brain
intrauterine space compromise, placental insu ciency, Dev.1998;30:507-511.
teratogenic exposure, and muscle abnormalities. Jacobsen HG, Herbert EA, Poppel MH. Arthrogryposis
Additionally, connective tissue abnormalities, maternal multiplex congenita. Radiology. 1955;65:8-18.
disease (such as diabetes mellitus, myotonic dystrophy, Kalampokas E, Kalampokas T, So oudis C, Deligeoroglou
myasthenia gravis, and multiple sclerosis), and in ection E, Bostis D. Diagnosing arthrogryposis multiplex
have been implicated. congenita: a review. ISRN Obstet Gynecol. 2012; Article
ID 264198, 6 pages.

346
Fetal demise soon after birth

1. What are the major di erential diagnoses?

2. What is the likely diagnosis, and what are the


major eatures that suggest this diagnosis?

3. What is the typical li espan?

4. What are some o the nonskeletal-associated


eatures?

5. What is the presumed etiology o this entity?

347
Campomelic dysplasia 3266
Case ranking/dif culty: Category: More than one category

Absent pedicle T1-10, angular bowing of lower Absent pedicles from T1-10, Note the “ at face”
extremities. Short ribs and hypoplastic scapulae. Skull with hypoplastic vertebrae. appearance and
fractures (arrows) with cephalohematoma due to hydrocephalus.
hydrocephalus and traumatic birth. The acetabulae and
iliac bones are dysplastic.

Answers
1. The di erential includes any dwar sm, including Pearls
thanatophoric and diastrophic dwar sm, achondroplasia, • Campomelic dysplasia is a rare dwar sm with an
osteogenesis imper ecta, and campomelic dwar sm. autosomal dominant inheritance pattern.
• Characteristic radiologic eatures include:
2. The hypoplastic scapulae, vertical iliac bones, and
• 11 short ribs
hypoplastic vertebrae are characteristic o campomelic
• Absent thoracic pedicles and vertebral hypoplasia
dwar sm.
• Angular bowing o the emora and tibiae
3. The severe respiratory insu ciency that is a result o the • Hypoplastic bulae
hypoplastic ribs, laryngotracheomalacia, and abnormal • Hypoplastic scapulae
thorax typically results in death within the rst year o • Dysplastic iliac bones and acetabulae with
li e. dislocated hips
• “Flat” acies
4. Associated eatures include hearing loss, congenital
• Associated eatures include:
heart disease, hydronephrosis, hydrocephalus, and
• Congenital heart disease
laryngotracheomalacia. The latter, along with the
• Hydrocephalus
hypoplastic ribs, results in the severe respiratory
• Hydronephrosis
compromise that is typically atal.
• Death is usually due to respiratory insu ciency, o ten
5. Campomelic dysplasia is caused by an alteration in in the rst year o li e.
the SOX9 gene. This gene is responsible or both bone
ormation and testes development. This may result in
sex reversal and occurs in 66% o genetic males, with Suggested Readings
ambiguous genitalia. It is also the primary reason or the Dahdaleh NS, Albert GW, Hasan DM. Campomelic
marked skeletal abnormalities. dysplasia: a rare cause o congenital spinal de ormity.
J Clin Neurosci. 2010 May;17(5):664-666.
Gimovsky M, Rosa E, Tolbert T, Guzman G, Nazir M,
Koscica K. Campomelic dysplasia: case report and review.
J Perinatol. 2008 Jan;28(1):71-73.

348
Back pain with left L5 radiculopathy

1. What is the most likely diagnosis?

2. How do these lesions usually behave?

3. What should be included in the di erential


diagnosis?

4. Which o the ollowing modalities best image


the condition?

5. How is the diagnosis con rmed?

349
Melanocytic schwannoma 3191
Case ranking/dif culty: Category: Nerve roots/Nerve plexus/Peripheral nerves

Axial T2-weighted images at the level of L5 vertebra (panel A) Coronal (Panel A) and axial (Panel B) T1-weighted images show a
and the sacroiliac joints (panel B). A hypointense heterogeneous T1-hyperintense lesion (arrows, panels A and B) extending from
lesion with a “dumbbell” shape is seen to arise from the left the left L5 intervertebral foramen (arrowhead, panel B) along the
exiting L5 nerve root (arrow, panel A) and extends caudally left sacral ala.
along the anterior surface of the left sacral ala (arrow, panel B).
The lesion causes expansion of the left intervertebral foramen
(arrowhead, panel A).

Answers
1. Melanocytic schwannomas are very rare tumors derived Pearls
rom the neural crest that arise in the spinal nerve roots • Melanocytic schwannomas are very rare tumors
but may also originate rom the central nervous system derived rom the neural crest, which o ten arise in the
and in so t tissues. spinal nerve roots.
• The clinical outcomes are o ten disappointing, and
2. Tumor behavior is di cult to predict, but they are o ten
local recurrence and invasion tend to occur despite
locally aggressive and have the capacity to metastasize.
surgery and radiotherapy.
The clinical outcomes are o ten disappointing and local
• MRI is the imaging investigation o choice and shows
recurrence and invasion tend to occur despite surgery
characteristic intrinsic high signal intensity on T1-
and radiotherapy.
weighted images due to the presence o melanin.
It is thought that melanin is acquired by the Schwann • A percutaneous biopsy is o ten needed or de nite
cells rom nearby nonneoplastic melanocytes via diagnosis.
a process called cytocrine injection. Alternatively,
Schwann cells may phagocytize melanin.
3. Di erential diagnosis includes other melanocytic Suggested Readings
neoplasms, ganglioneuroblastoma, melanotic
Killeen RM, Davy CL, Bauserman SC. Melanocytic
medulloblastoma, pigmented neuro broma, pigmented
schwannoma. Cancer. 1988 Jul 1;62(1):174-183.
neuroblastoma, melanotic neuroendocrine carcinomas,
Liessi G, Barbazza R, Sartori F, Sabbadin P, Scapinello A.
carcinoids, and neurotropic melanoma.
CT and MR imaging o melanocytic schwannomas; report
4. Melanocytic schwannomas cannot be distinguished rom o three cases. Eur J Radiol. 1990 Sep-Oct;11(2):138-142.
other neurogenic tumors on CT as it does not show any
characteristic eatures. MRI is the imaging investigation
o choice and characteristically shows intrinsic high
signal intensity on T1-weighted images due to the
presence o melanin. Melanin shortens both T1 and T2
relaxation times.
5. A percutaneous biopsy is o ten needed or de nite
diagnosis.

350
Progressive limb weakness and sensory disturbance

1. What is the most likely diagnosis?

2. What should be included in the di erential


diagnosis?

3. Which symptoms are encountered in this


condition?

4. Which imaging modality best images this


condition?

5. Which treatments may be bene cial?

351
Chronic in ammatory demyelinating polyneuropathy (CIDP) 3100
Case ranking/dif culty: Category: Nerve roots/Nerve plexus/Peripheral nerves

Sagittal T2-weighted sequence of the Coronal T1-weighted sequence of the Axial T2-weighted image of the lumbar spine.
lumbar spine. Note that the CSF (arrow, lumbar spine demonstrates bilateral There is clumping of the hypertrophied
panel A) is completely e aced in the hypertrophied S1 exiting nerve cauda equina roots (arrow). Note the
lumbar canal due to hypertrophy of the roots (arrows). Note is also made of increased cross-sectional diameter of the
cauda equina roots. The left L5 exiting splenomegaly. exiting nerve roots (arrowhead).
nerve root is also hypertrophied (arrow,
panel B).

Answers 5. Treatment includes the administration o corticosteroids,


1. Chronic inf ammatory demyelinating polyneuropathy which may be prescribed alone or in combination with
(CIDP) is a chronic progressive or relapsing symmetric immunosuppressant drugs.
sensory-motor disorder. Physical therapy also has an important role and improves
It is considered the chronic counterpart o Guillain-Barre muscle strength, unction, and mobility.
syndrome.
2. Di erential diagnosis include acute inf ammatory
demyelinating polyradiculoneuropathy, inclusion Pearls
body myositis, cervical myelopathy, dermatomyositis,
polymyositis, HIV-1–associated progressive • Chronic inf ammatory demyelinating polyneuropathy
polyradiculopathy, Lambert-Eaton myasthenic (CIDP) is a chronic progressive or relapsing
syndrome, diabetic neuropathy, amyotrophic lateral symmetric sensory-motor disorder.
sclerosis, myasthenia gravis, syringomyelia, metabolic • MRI demonstrates abnormal hypertrophy and
myopathies, neurosarcoidosis, and nutritional clumping o the nerve roots and pathological contrast
neuropathies. enhancement.
3. A ected individuals may present with motor, sensory,
or autonomic disturbances. Symptoms include initial
proximal and distal limb weakness, paresthesia Suggested Readings
(characteristically in a stocking-glove distribution), gait
Kale HA, Sklar E. Magnetic resonance imaging ndings in
disturbances, atigue, and orthostatic dizziness.
chronic inf ammatory demyelinating polyneuropathy with
4. MRI demonstrates abnormal hypertrophy o the intracranial ndings and enhancing, thickened cranial and
nerve roots (in the cauda equina, brachial, and spinal nerves. Australas Radiol. 2007 Oct;51 Spec No.
lumbosacral plexuses) which is likely secondary to B21-4.
repeated demyelination and remyelination. Clumping Kitakule MM, McNeal A. Massive nerve root
o the nerve roots is also seen. Abnormal contrast hypertrophy in chronic inf ammatory demyelinating
enhancement o the roots may be seen particularly in polyradiculoneuropathy. J Assoc Acad Minor Phys.
active disease. 1997;8(3):55-57.
352
Sudden loss of power of neck extensor musculature

1. What is the most likely diagnosis?

2. What should be included in the di erential


diagnosis?

3. Which treatments have proved to be bene cial?

4. Describe clinical and pathological


characteristics o this condition.

5. Which investigations may aid diagnosis?

353
Dropped head syndrome due to inclusion body myositis (IBM) 3065
Case ranking/dif culty: Category: Miscellaneous

Stacked sagittal T2-weighted MR images of the cervicothoracic


spine. Exaggerated thoracic kyphosis with compensatory Postoperative frontal (image A) and lateral (image B) radiographs
cervical lordosis is seen, although the changes are less dramatic show posterior instrumented xation of the cervical spine, using
compared to plain radiography as the patient was scanned in the a double rod and screw construct.
supine position. The vertebrae have normal heights and marrow
signal. Prominent CSF ow artifacts in the upper thoracic spine wheelchair bound or bedridden. IBM can be an indirect
(arrow, image B) re ect altered CSF ow dynamics. There are no cause o death due to respiratory ailure or in ection.
signi cant intervertebral disc protrusions or spinal canal stenosis.
5. Elevated (up to 10 times the normal range) creatine
Answers kinase levels are typical o inclusion body myositis
1. Inclusion body myositis (IBM) is an inf ammatory but patients can also present with normal CK levels.
muscle disease characterized by slowly progressive Electromyographic studies o ten display abnormalities.
weakness and wasting o both distal and proximal Inclusion body myositis is a pathological challenge and
muscles, mostly apparent in the musculature o the upper even with a biopsy, the diagnosis can be ambiguous.
and lower extremities.
The condition mani ests itsel in two di erent ways: Pearls
sporadic inclusion body myositis and hereditary
inclusion body myopathy. The histopathological changes • Inclusion body myositis (IBM) is an inf ammatory
in IBM were rst described in the mid-1960s, although muscle disease characterized by slowly progressive
the disorder was not distinguished rom polymyositis weakness and wasting o both distal and proximal
and named until 1971. muscles.
• It has been described in a number o neurological
2. Dropped head syndrome results rom neck extensor disorders.
muscle weakness and has been described in a number • The condition mani ests itsel in two di erent ways:
o neurological disorders. These include amyotrophic sporadic inclusion body myositis and hereditary
lateral sclerosis, multiple sclerosis, and extensor inclusion body myopathy.
myopathy. Radiotherapy and cervical cord injury have • Age o presentation is an important prognostic predictor.
also been implicated.
• IBM is relatively resistant to standard
3. In contrast to other inf ammatory myopathies such as immunosuppressive therapy.
dermatomyositis and polymyositis, IBM is relatively • IBM can be an indirect cause o death due to
resistant to standard immunosuppressive therapy, respiratory ailure or in ection.
with muscle strength improving minimally i at all to
corticosteroids and other agents.
Suggested Readings
Some studies have reported a partial response to
corticosteroids with either mild improvement in or Peng A, Ko man BM, Malley JD, Dalakas MC. Disease
stabilization o muscle strength. Serum creatine kinase levels progression in sporadic inclusion body myositis:
o ten all and may even normalize. observations in 78 patients. Neurology. 2000;55:296.
Phillips BA, Zilko PJ, Mastaglia FL. Prevalence o sporadic
4. Patients with inclusion body myositis usually present
inclusion body myositis in Western Australia. Muscle
a ter several years o gradually worsening muscle
Nerve. 2000;23:970.
weakness. Age o presentation is an important prognostic
Wilson FC, Ytterberg SR, St Sauver JL, Reed AM.
predictor. The older the age o onset o the disease,
Epidemiology o sporadic inclusion body myositis
the more rapid is the loss o strength and unction. By
and polymyositis in Olmsted County, Minnesota. J
15 years, most patients will require assistance to cope
Rheumatol. 2008;35:445.
with basic activities o daily living and some become
354
Chronic low back pain

1. What is the most likely diagnosis?

2. What should be included in the di erential


diagnosis?

3. Where are these lesions most commonly


encountered?

4. Which vertebral anomalies are associated with


the condition?

5. Which conditions rom part o the


bronchopulmonary oregut mal ormations?

355
Neurenteric cyst 2938
Case ranking/dif culty: Category: Spinal cord

Sagittal T2- (Panel A) and T1- (Panel B) weighted MRI of the lower Axial T1-weighted sequence at the level of T12 vertebral
thoracic and lumbar spine. There is a well-de ned intramedullary body shows a well-de ned intramedullary cyst causing slight
cyst at the level of T11-T12. The cyst is thin walled and follows expansion of the spinal cord (arrow). The thecal sac is marked
uid signal on both T1 and T2 sequences. with an arrowhead.

Answers
1. Enterogenous cysts o the central nervous system Pearls
account or 0.7%-1.3% o spinal cord tumors. They are • Neurenteric cysts are cystic lesions o endodermal
cystic lesions o endodermal origin lined by a single origin lined by a single epithelial layer.
epithelial layer that resembles cells o the alimentary • They represent the rarest orm o bronchopulmonary
canal. They result rom incomplete separation o oregut mal ormations.
the oregut and notochord during the third week o • Occur primarily in children and young adults when
embryonic development resulting in the persistence o they are o ten associated with other congenital spinal
the canal o Kovalevski, which joins the yolk sac to the abnormalities.
notochord. • Most requently encountered in the spinal canal,
particularly at the cervicothoracic region.
2. The main di erential diagnoses are a posttraumatic cyst,
• Generally located in the intradural extramedullary
a demyelinating plaque, and syrinx.
compartment with occasional intramedullary
3. They are generally located in the intradural involvement.
extramedullary compartment with occasional
intramedullary involvement.
4. Neurenteric cysts occur primarily in children Suggested Readings
and young adults where they are o ten associated Rotondo M, D’Avanzo R, Natale M, et al. Intramedullary
with other congenital spinal abnormalities. Such neurenteric cysts o the spine. Report o three cases.
abnormalities include anterior and posterior spina J Neurosurg Spine. 2005 Mar;2(3):372-376.
bi da, hemivertebrae, absent vertebrae, scoliosis, used Savage JJ, Casey JN, McNeill IT, Sherman JH. Neurenteric
vertebrae, butterf y vertebrae, and diastematomyelia. cysts o the spine. J Craniovertebr Junction Spine. 2010
5. Neurenteric cysts represent the rarest orm o Jan;1(1):58-63.
bronchopulmonary oregut mal ormations, which also Singhal BS, Parekh HN, Ursekar M, Deopujari
include pulmonary sequestrations, bronchogenic cysts, CE, Manghani DK. Intramedullary neurenteric cyst in
and enteric cysts. mid thoracic spine in an adult: a case report. Neurol India.
2001 Sep;49(3):302-304.

356
Worsening cervical myelopathy and radiculopathy.

1. What is the most likely diagnosis?

2. Which conditions are also part o the


“mucopolysaccharidosis” spectrum?

3. Name some mani estations o this syndrome.

4. Describe clinical and pathological


characteristics o this condition.

5. In which conditions is in erior vertebral


beaking seen?

357
Hurler syndrome (Mucopolysaccharidosis type 1) 2936
Case ranking/dif culty: Category: Vertebral body

Lateral radiograph (A) and sagittal CT (B) show hypoplastic Sagittal T2 (A) and contrast-enhanced T1 (B) sequences show
cervical vertebrae with pathognomonic “inferior beaking”(arrows). expansion of the cervical cord, with no identi able mass lesion or
Previous posterior cervical decompression and characteristic syrinx.
hypoplasia of the odontoid process (arrowhead) is seen.

Answers
1. Hurler syndrome is a rare lysosomal storage disorder Pearls
with a prevalence o 1 in 100 000. It is caused by a • Hurler syndrome is a rare autosomal recessive,
de ective IDUA gene that codes or α -L-iduronidase lysosomal storage disorder.
and has an autosomal recessive inheritance. Enzyme • A ected individuals demonstrate typical clinical
de ciency results in accumulation o dermatan and mani estations, and the diagnosis is con rmed by
heparan sul ate in multiple tissues, which leads to demonstrating α -L-iduronidase de ciency.
progressive deterioration and eventual death. • Characteristic radiological ndings include in erior
vertebral beaking and odontoid hypoplasia.
2. Seven distinct clinical types and numerous subtypes o
mucopolysaccharidoses have been identi ed. They are
a group o metabolic disorders, which include Hunter
syndrome, Sly syndrome, San lippo syndrome, and Suggested Readings
Scheie syndrome. Belani KG, Krivit W, Carpenter BL, et al. Children
3. Clinical mani estations o Hurler syndrome include with mucopolysaccharidosis: perioperative care,
pro ound intellectual disability, cardiac disease, corneal morbidity, mortality, and new ndings. J Pediatr Surg.
clouding, coarse acial eatures, and a low nasal bridge. 1993;28:403-408.
Characteristic musculoskeletal mani estations, cervical Kachur E, Del Maestro R. Mucopolysaccharidoses and spinal
myelopathy, and excessive hair growth may also be cord compression: case report and review o the literature
encountered. with implications o bone marrow transplantation.
Neurosurgery-Baltimore. 2000;47:223-229.
4. Hurler syndrome is an autosomal recessive condition. Kirkpatrick K, Ellwood J, Walker RW.
A ected individuals o ten succumb to the condition Mucopolysaccharidosis type 1 (Hurler syndrome) and
in the rst decade, rom respiratory and cardiac anesthesia: the impact o bone marrow transplantation,
complications. enzyme replacement therapy, and beroptic intubation on
5. In erior vertebral beaks are seen in achondroplasia, airway management. Paediatr Anaesth. 2012;22:745-751.
pseudoachondroplasia, trisomy 21, and congenital
hypothyroidism.
The vertebral body beaks in Morquio syndrome arise
rom the central part (middle third) o the anterior
vertebral body.
358
Chronic back pain

1. What are the ndings on CT?

2. What is the di erential diagnosis, and what is


the most likely etiology?

3. What are common locations or this lesion?

4. What di erentiates the aggressive orm o this


lesion rom the nonaggressive orm?

5. What is the management or these lesions?

359
Aggressive osteoblastoma 2897
Case ranking/dif culty: Category: Vertebral Body

Expansile posterior element lesion Similar ndings. Coronal reformatted images


with a central mineralized matrix, show the highly mineralized
cortical expansion, and breakthrough matrix and bone expansion.
with adjacent vertebral sclerosis.

Answers
1. Lucent expansile lesion with central mineralization
Pearls
located in the posterior elements, and with cortical • Osteoblastomas are histologically similar to osteoid
breakthrough. Sclerosis is seen in the vertebral body osteoma.
with bony remodeling. • They are, however, larger, greater than 2 cm in size.
• Nocturnal pain and relie with aspirin, as seen in an
2. Di erential includes osteoblastoma, aneurysmal bone osteoid osteoma, are not typical eatures.
cyst, giant cell tumor, and a low-grade osteosarcoma. • The lesion is commonly located in the posterior
Given the age o the patient, size o lesion, and bony elements o the spine and the diaphysis o long bones.
expansion, aggressive osteoblastoma is the avored • Lesions are lucent, and may have central
diagnosis. Osteoid osteoma is not expansile. mineralization. There can be mild expansion,
3. Osteoblastomas typically occur in the posterior elements especially in the spine, with surrounding sclerosis.
o the spine or the diaphysis o long bones. The Variable central mineralization is present.
metaphysis is less common. • An aggressive osteoblastoma with cortical expansion
and breakthrough and in ltration o the so t tissues has
4. An aggressive osteoblastoma is characterized by been described. These aggressive lesions can recur, but
bony expansion, cortical breakthrough, and so t tissue have no metastatic potential.
in ltration, as shown in this case. Tumor recurrence • Aggressive osteoblastomas are usually greater than 3 cm.
is common. Metastasis is extremely uncommon and • MRI shows isointensity to hypointensity on T1-
usually occurs i there is malignant trans ormation to an weighted images, with variable T2 signal.
osteosarcoma, ollowing radiation therapy.
5. Aggressive osteoblastomas usually require surgical
excision, but un ortunately there is a high recurrence rate Suggested Readings
o up to 50%.
Abramovici L, Kenan S, Hytiroglou P, Ra i M, Steiner
In surgically unresectable tumors, radiation therapy and GC. Osteoblastoma-like osteosarcoma o the distal tibia.
chemotherapy have been tried, but there is the risk o Skeletal Radiol. 2002 Mar;31(3):179-182.
post-radiation sarcoma. Ramirez JA, Sandoz JC, Kaakaji Y, Nietzschman HR. Case 3:
Aggressive osteoblastoma. AJR Am J Roentgenol. 1998
Sep;171(3):863, 867-868.
360
Incidental ndings on a trauma CT

1. What are the major radiologic ndings?

2. What is the likely diagnosis?

3. What are the classic osseous mani estations o


this entity?

4. What are two rare osseous mani estations o


this entity?

5. What are the expected radionuclide bone scan


ndings?

361
Tuberous sclerosis 2884
Case ranking/dif culty: Category: Miscellanous

Multiple fat attenuating renal Multiple sclerotic lesions. Radionuclide bone scan shows
angiomyolipomas (arrowheads), the sclerotic lesions have normal
and a complicated cyst (arrow). uptake.

Answers
• The mani estations are protean. They include
1. There are multiple at-containing renal lesions consistent
• Cutaneous: adenoma sebaceum
with angiomyolipomas, as well as multiple sclerotic
• Neurologic: subependymal tubers and giant cell
lesions. A hyperdense le t renal lesion was a complex
astrocytomas, cortical tubers
cyst.
• Cardiac: rhabdomyomas
2. The combination o renal angiomyolipomas, renal cysts, • Ophthalmic: retinal astrocytomas
and sclerotic bone lesions is consistent with tuberous • Pulmonary: cystic pulmonary abnormalities
sclerosis. • Renal: cysts, angiomyolipomas (AMLs), renal cell
carcinomas
3. TSC bone lesions included dense sclerosis due to
• Dental: pitting o the enamel, gingival bromas
calvarial thickening and periosteal thickening in the long
• Gastrointestinal: hamartomas and polyposis
bones. Multiple bones cysts can occur, especially in
• Hepatic: cysts and hepatic AMLs
the metacarpals and phalanges, which, in combination
• Skeletal: densely sclerotic lesions with calvarial
with pulmonary disease, can lead to a misdiagnosis o
and periosteal bone thickening, occasional cysts.
neuro bromatosis or sarcoid. Erosions o the terminal tu ts
The di erential or the skeletal lesions includes
o the phalanges rom subungual angio bromas may occur.
osteopoikilosis or metastatic disease, depending on
Dysplasia o the body o the sphenoid in patients with the patient’s age.
retinal hamartomas can be seen (unlike the sphenoid • I the osseous sclerotic lesions are combined with
wing dysplasia in neuro bromatosis). bone cysts and pulmonary lesions, the di erential will
4. There are case reports o a clivus chordoma in a child include sarcoid and neuro bromatosis.
with TSC, and o occipital thinning in the bone overlying
a cortical tuber.
Suggested Readings
5. The radionuclide bone scan appearance is variable.
However, the scan is usually normal, as seen in this patient. Evans JC, Curtis J. The radiological appearances o tuberous
sclerosis. Br J Radiol. 2000 Jan;73(865):91-98.
Schroeder BA, Wells RG, Starshak RJ, Sty JR. Clivus
chordoma in a child with tuberous sclerosis: CT
Pearls and MR demonstration. J Comput Assist Tomogr.
• Tuberous sclerosis (TSC) is a multisystem disorder 1987;11(1):195-196.
that is a result o a spontaneous mutation o two genes, Terada T, Nakai E, Moriwaki H, Hayashi S, Komai N.
TSC1 and TSC2, which code or the proteins hamartin Tuberous sclerosis with an atypical radiological skull
and tuberin, respectively. These proteins act as tumor change: case report. Neurosurgery. 1985 Jun;16(6):804-807.
growth suppressors.

362
Bilateral lower back pain

1. What is the diagnosis?

2. What are the common patient demographics


or this condition?

3. De ciency o which structure is usually


responsible or this speci c entity?

4. What are the usual complications o this


condition?

5. What is the treatment o choice or progressive


de ormity/complications?

363
Dysplastic spondylolisthesis 2883
Case ranking/dif culty: Category: Posterior elements

The dysplastic (horizontal) superior articular facets of S1 are well There is a diminutive fused L5/S1 articulation and compensatory
demonstrated on the sagittal images. lumbar hyperlordosis.

Answers
1. Dysplastic spondylolisthesis is a common cause o slips as a result o pars de ects, and narrowed as a result o
identi ed in adolescents and young adults. It relates to acet joint dysplasia and subluxation.
congenital dysplasia o the posterior elements, usually • The identi cation o a dysplastic (horizontal)
superior articular acets o S1. superior horizontal articular acet o S1 is virtually
pathognomic o this condition, although other
2. Dysplastic spondylolisthesis is congenital, more dysplastic etiologies include abnormal vertically
common in women and ound more requently in oriented acet joints.
Caucasians. • Spondyloptosis (100% slip) is probably
3. Dysplastic spondylolisthesis usually results rom a only commonly seen in cases o dysplastic
congenitally mal ormed superior articular acet o S1. spondylolisthesis; hence, i this degree o slip is noted,
a congenital dysplastic etiology should be considered.
4. Severe de ormity leads to a hyperlordotic lumbar spine. • The L5/S1 articulation is o ten partly used and
There is usually L5/S1 disc degeneration. As the slip diminutive in this type o spondylolisthesis.
progresses, there may be severe central canal and lateral • Regular ollow-up is advised or consideration o
recess stenosis. surgery as high-grade dysplastic spondylolisthesis may
5. As slips progress, they may require surgical intervention. have severe central canal stenosis and de ormity.
A posterior decompression with usion in situ is the
pre erred approach, as it allows the central canal to be
decompressed, and xation prevents urther progression Suggested Readings
o the slip and de ormity. Pucher A, Jankowski R, Szulc A, Stryczyński P, Strzyzewski
W. Surgical treatment o dysplastic and isthmic
spondylolisthesis. Ortop Traumatol Rehabil. 2005
Pearls Dec;7(6):639-645.
Vialle R, Dauzac C, Khouri N, Wicart P, Glorion C, Guigui P.
• Dysplastic spondylolisthesis is a common cause o a
Sacral and lumbar-pelvic morphology in high-grade
slip in an adolescent or young adult at L5/S1.
spondylolisthesis. Orthopedics. 2007 Aug;30(8):642-649.
• The key distinction between an isthmic and a
dysplastic cause is the spinal canal, which is widened

364
Acute on chronic back pain with sudden loss of bladder and bowel
function. History of prior spine surgery

1. What are the MRI ndings?

2. What is the di erential diagnosis o intradural,


extramedullary lesions?

3. What is the diagnosis?

4. What makes the diagnosis challenging?

5. How are these lesions classi ed?

365
Intradural disc herniation 2878
Case ranking/dif culty: Category: Disc

Central canal is obscured by cystic disc material, and the normal Normal nerve roots for comparison.
nerve roots are displaced peripherally.

Answers
1. Intradural and extramedullary lesion that is T2 • They can be di cult to diagnose especially when they
hyperintense, peripherally displacing nerve roots. undergo cystic or calci ed changes.
• According to Mut et al, they can be classi ed into two
2. The di erential diagnosis includes herniated intradural types:
disc material, meningioma, neuro broma, and • Type A: Herniation o a disc material into the
metastasis. dural sac.
3. Cystic disc material is seen displacing the normal nerve • Type B: Herniation into the dural sheath in the
roots peripherally, consistent with herniated intradural pre-ganglionic region.
disc material. • MRI is the investigation o choice. Contrast-enhanced
MRI is used to di erentiate rom intradural and
4. The diagnosis can be made challenging by the extramedullary lesions such as neuro broma and
presence o cystic changes and calci cation o the disc meningioma, and also rom intradural lesions such
suggesting other etiologies. In this case, the cystic as epidermoid and dermoid. Disc material does not
disc material resembles CSF, and the nerve roots are enhance immediately a ter administration o contrast.
displaced peripherally. The postoperative status o a • Treatment usually involves urgent removal o the disc,
patient, and the presence o congenital adhesions and a especially when causing compressive symptoms.
constitutionally narrow spinal canal can also make the
diagnosis di cult.
5. Type A: Herniation o the disc into the dural sac. Suggested Readings
Type B: Herniation o the disc into the dural sheath in
Arnold PM, Wakwaya YT. Intradural disk herniation
the preganglionic region.
at L1-L2: report o two cases. J Spinal Cord Med.
2011;34(3):312-314.
Singh PK, Shrivastava S, Dulani R, Banode P, Gupta S.
Pearls Dorsal herniation o cauda equina due to sequestrated
intradural disc. Asian Spine J. 2012 Jun;6(2):145-147.
• Intradural herniated disc is a rare cause o cord or
Mut M, Berker M, Palaoğlu S. Intraradicular disc herniations
cauda equina compression.
in the lumbar spine and a new classi cation o intradural
• It accounts or 0.26% or 0.3% o all herniated disc
disc herniations. Spinal Cord. 2001 Oct;39(10):545-548.
cases, most commonly in the lumbar region, usually at
L4/5.
• Congenital adhesion between the ventral sur ace o the
thecal sac and posterior longitudinal ligament acts as a
predisposing actor.

366
Subject Index
Note: Numbers in parentheses re er to Case IDs.

Achondroplasia (2916) 128 Conjoined nerve root (2599) 158 Juvenile idiopathic arthritis (2917) 126
Acute calcif c tendinitis o the longus Cystic schwannoma (1510) 52 Juvenile pilocytic astrocytoma (1493) 230
colli muscle (2918) 270
Aggressive osteoblastoma (2897) 360 Denervation edema; dropped head Klippel-Feil syndrome (2190) 36
Amyloid spondyloarthropathy (2899) 288 syndrome (1497) 338 Kummell disease (1514) 80
Ankylosing spondylitis (2905) 142 Diastematomyelia (split cord
Annular f ssure (2951) 114 mal ormation) (2903) 282 Lateral meningocele (2939) 264
Annular rupture (3052) 92 Di use idiopathic skeletal hyperostosis with Leptomeningeal metastasis (lung
Anterior cervical discectomy and racture (1511) 50 adenocarcinoma) (2944) 116
usion (ACDF) (3303) 330 Di use marrow inf ltration (2201) 24 Ligamentous injury (1516) 200
Arachnoid cyst (1483) 222 Disc desiccation (3104) 88 Ligamentous instability (2909) 138
Arachnoiditis (1501) 38 Disc ragment migration (2193) 34 Limbus vertebra (2191) 184
Arthrogryposis multiplex congenita (1480) 346 Disc herniation (2187) 14 Lipoblastoma (2220) 308
Artif cial disc replacement (ADR) (3101) 74 Disc herniation—extrusion (3047) 100 Lipomyelocele (2194) 32
Atlantooccipital assimilation (2483) 10 Disc herniation—protrusion (3051) 70 Lumbosacral epidural lipomatosis (1500) 62
Atlantooccipital dislocation (2200) 178 Diskogenic endplate changes (2205) 22 Lumbosacral plexus metastatic
Dorsal dermal sinus with epidermoid (2954) 248 inf ltration (1499) 334
Baastrup phenomenon (2915) 130 Dropped head syndrome due to inclusion body Lumbosacral transitional vertebrae
Bacterial spinal meningitis (2185) 164 myositis (IBM) (3065) 354 (LSTV) (3048) 98
Benign notochordal cell Dural arteriovenous f stula (1478) 224 Lymphangiomatosis (1487) 340
tumor (BNCT) (2995) 240 Dural ectasia, Mar an syndrome (1515) 202 Lymphoma (2998) 236
Bertolotti syndrome (2996) 106 Dysplastic spondylolisthesis (2883) 364
Bickersta encephalitis (2186) 312 Malignant peripheral nerve sheath
Brachial plexus avulsion with Enterovirus myelitis (2183) 196 tumor (1519) 198
pseudomeningocele (1496) 228 Epidural abscess (3000) 102 Melanocytic schwannoma (3191) 350
Brachial plexus avulsion with spinal cord injury Extension teardrop racture (2919) 124 Meningioma, myelography (1359) 76
and pseudomeningocele (1495) 344 Extramedullary hematopoiesis (2997) 238 Metastatic cauda equina
Brachial plexus hematoma and retracted nerve compression (2880) 296
roots (1494) 194 Facet joint injection (2937) 132 Mixed injection on myelogram (1525) 48
Brachial plexus metastatic inf ltration (1498) 206 Filar lipoma (747) 298 Multi ocal f brous dysplasia with pathologic
Brucellosis (2181) 42 “Fishmouth” vertebra in sickle cell racture (1517) 326
Burst racture (2894) 150 disease (2216) 18 Multiple hereditary exostoses (2221) 306
Butter y vertebra (2189) 40 Fracture C2—type 1 (2195) 30 Multiple myeloma (2920) 122
Fracture C2—type 2 (2597) 2 Multiple synovial cysts (2179) 314
C1 osseous metastasis (1477) 68 Fracture C2—type 3 (2196) 300 Myelography (3216) 56
Calcif c arachnoiditis (arachnoiditis Myelomalacia (2208) 174
ossif cans) (1502) 60 Giant cell tumor (2907) 278 Myof broma (1518) 324
Calcium pyrophosphate deposition (1508) 54 Gibbus de ormity (1481) 342 Myxopapillary ependymoma (2940) 262
Campomelic dysplasia (3266) 348 Glioblastoma multi orme, cervical
Caudal regression (1491) 78 cord (2239) 162 Neurenteric cyst (2938) 356
Cerebrospinal uid ow study (2207) 304 Neuroblastoma (2215) 168
Cerebrospinal uid leak evaluation (2188) 320 Hangman racture (2895) 148 Neurof bromatosis type 1 (2935) 266
Cerebrospinal uid pulsation arti act (2198) 28 Hemangioblastoma (1506) 204 Neurof bromatosis type 2 (2945) 256
Cervical aneurysmal bone cyst (1486) 218 Hemangioma (2886) 152 Neuromyelitis optica (1479) 328
Cervical cord contusion (1505) 208 Hemivertebra (2981) 242 Neurosarcoidosis (2941) 260
Cervical ependymoma (1504) 58 Hemorrhagic cord in arction (2180) 188
Cervical myelomeningocele (1492) 210 Hurler syndrome (Mucopolysaccharidosis Ossif cation o the posterior longitudinal
Cervical rib (3050) 94 type 1) (2936) 358 ligament (OPLL) (2901) 284
Cervical spondylotic myelopathy (CSM) (3313) 82 Hypertrophic olivary degeneration (2217) 310 Osteoblastic metastases (2885) 154
Chance racture (2913) 136 Osteogenesis imper ecta (2922) 268
Chiari I mal ormation (2206) 20 Intradural disc herniation (2878) 366 Osteoid osteoma (2482) 6
Chiari I mal ormation with syrinx (1475) 72 Intradural lipoma (2879) 294 Osteopetrosis (2947) 252
Chiari II mal ormation (2600) 156 Intramedullary metastasis (1476) 226 Osteoporotic cauda equina
Chondrosarcoma (2912) 272 Intramuscular myxoma, Mazabraud compression (2881) 292
Chronic in ammatory demyelinating syndrome (2242) 302 Osteoporotic compression ractures (2914) 134
polyneuropathy (CIDP) (3100) 352 Ivory vertebra sign (3222) 86 Osteosarcoma (1488) 216
Clay shoveler racture (2908) 140
Clival chordoma (2219) 16 Je erson racture (2911) 274 Paget disease (multi ocal) (2203) 176
Complication, lumbar puncture—inadvertent Jumped/perched acets (2882) 290 Pelvic osteomyelitis with associated
epidural anesthetic injection (2241) 12 Juvenile arteriovenous mal ormation (1513) 332 sacroiliitis (1489) 212

367
368 Subject Index

Percutaneous disc aspiration and Romanus lesion (3217) 234 Subacute combined degeneration (2192) 182
biopsy (PDAB) (3231) 232 Rotatory subluxation (1522) 192 Syringomyelia (2202) 26
Percutaneous vertebroplasty (2982) 108
Peridural f brosis (2177) 190 Sacral aneurysmal bone cyst (1485) 220 Tarlov cyst (2921) 120
Platybasia (2598) 4 Sacral chordoma (2512) 8 Tethered cord (2980) 110
Posterior lumbar interbody Sacral insu f ciency racture (2218) 166 Thoracic epidural lipomatosis (2176) 46
usion (PLIF) (3312) 214 Sacrococcygeal teratoma (1523) 318 Thoracic paraspinal lines (TPL) (3246) 84
Postoperative dermoid (1521) 322 Scheuermann disease (1482) 66 Transdural cord herniation (1527) 316
Postoperative in ection lumbar spine (2240) 160 Septic acet arthritis (2184) 172 Transverse myelitis (2942) 258
Pott disease (2182) 186 Solitary plasmacytoma o Tuberculous spondylitis (2955) 246
Psoriatic spondylitis (2904) 144 bone (SPB) (2979) 112 Tuberous sclerosis (2884) 362
Spinal cavernoma (2946) 254
Radiation changes (2197) 180 Spinal dysraphism, in utero (2214) 170 VACTERL syndrome (2910) 276
Renal osteodystrophy: rugger jersey Spinal subarachnoid hemorrhage Vertebra plana (2178) 44
spine (2898) 146 (SSH) (3298) 336 Vertebral biopsy (2943) 118
Retropharyngeal abscess (2953) 250 Spondylolysis (pars de ect) (3049) 96 Vertebral body replacement (VBR) (3102) 90
Rheumatoid arthritis with basilar Spondyloptosis (1484) 64 Vertical C1-C2 dissociation (2906) 280
invagination (2900) 286 Spontaneous epidural hematoma (2999) 104 Von Hippel-Lindau disease (2976) 244
Chapter Index
Note: Numbers in parentheses re er to Case IDs.

Disc Multi ocal f brous dysplasia with pathologic Bertolotti syndrome (2996) 106
Annular f ssure (2951) 114 racture (1517) 326 Cervical aneurysmal bone cyst (1486) 218
Annular rupture (3052) 92 Multiple hereditary exostoses (2221) 306 Dysplastic spondylolisthesis (2883) 364
Anterior cervical discectomy and usion Multiple myeloma (2920) 122 Facet joint injection (2937) 132
(ACDF) (3303) 330 Multiple synovial cysts (2179) 314 Myof broma (1518) 324
Artif cial disc replacement (ADR) (3101) 74 Myelomalacia (2208) 174 Osteoid osteoma (2482) 6
Disc desiccation (3104) 88 Osteoporotic cauda equina Spondylolysis (pars de ect) (3049) 96
Disc ragment migration (2193) 34 compression (2881) 292 Spondyloptosis (1484) 64
Disc herniation—extrusion (3047) 100 Pelvic osteomyelitis with associated
Disc herniation—protrusion (3051) 70 sacroiliitis (1489) 212 Spinal canal
Intradural disc herniation (2878) 366 Peridural f brosis (2177) 190 Arachnoid cyst (1483) 222
Percutaneous disc aspiration and biopsy Pott disease (2182) 186 Arthrogryposis multiplex
(PDAB) (3231) 232 Psoriatic spondylitis (2904) 144 congenita (1480) 346
Radiation changes (2197) 180 Clival chordoma (2219) 16
Filum Rheumatoid arthritis with basilar Complication, lumbar puncture-inadvertent
Arachnoiditis (1501) 38 invagination (2900) 286 epidural anesthetic injection (2241) 12
Calcif c arachnoiditis (arachnoiditis Sacrococcygeal teratoma (1523) 318 Epidural abscess (3000) 102
ossif cans) (1502) 60 Septic acet arthritis (2184) 172 Gibbus de ormity (1481) 342
Filar lipoma (747) 298 Spinal dysraphism, in utero (2214) 170 Intradural lipoma (2879) 294
Myxopapillary ependymoma (2940) 262 Transdural cord herniation (1527) 316 Lumbosacral epidural lipomatosis (1500) 62
Tuberculous spondylitis (2955) 246 Meningioma, myelography (1359) 76
Hardware/Lines/Tubes VACTERL syndrome (2910) 276 Mixed injection on myelogram (1525) 48
Postoperative in ection lumbar Spontaneous epidural hematoma (2999) 104
spine (2240) 160 Nerve roots/Nerve plexus/Peripheral nerves Thoracic epidural lipomatosis (2176) 46
Brachial plexus avulsion with
Ligaments pseudomeningocele (1496) 228 Spinal cord
Ossif cation o the posterior longitudinal Brachial plexus avulsion with spinal cord injury Bickersta encephalitis (2186) 312
ligament (OPLL) (2901) 284 and pseudomeningocele (1495) 344 Cervical cord contusion (1505) 208
Brachial plexus hematoma and retracted Cervical ependymoma (1504) 58
Meninges/Nerve sheath nerve roots (1494) 194 Cervical spondylotic myelopathy
Cystic schwannoma (1510) 52 Brachial plexus metastatic (CSM) (3313) 82
Neurosarcoidosis (2941) 260 inf ltration (1498 ) 206 Chiari I mal ormation (2206) 20
Chronic in ammatory demyelinating Chiari I mal ormation with syrinx (1475) 72
Miscellaneous polyneuropathy (CIDP) (3100) 352 Diastematomyelia (split cord
Cervical rib (3050) 94 Conjoined nerve root (2599) 158 mal ormation) (2903) 282
Dropped head syndrome due to inclusion body Disc herniation (2187) 14 Dural arteriovenous f stula (1478) 224
myositis (IBM) (3065) 354 Leptomeningeal metastasis (lung Enterovirus myelitis (2183) 196
Tuberous sclerosis (2884) 362 adenocarcinoma) (2944) 116 Glioblastoma multi orme, cervical
Lumbosacral plexus metastatic inf ltration cord (2239) 162
More than one category (1499) 334 Hemangioblastoma (1506) 204
Amyloid spondyloarthropathy (2899) 288 Malignant peripheral nerve sheath Hemorrhagic cord in arction (2180) 188
Atlantooccipital assimilation (2483) 10 tumor (1519) 198 Hypertrophic olivary degeneration (2217) 310
Bacterial spinal meningitis (2185) 164 Melanocytic schwannoma (3191) 350 Intramedullary metastasis (1476) 226
Brucellosis (2181) 42 Neurof bromatosis type 2 (2945) 256 Juvenile pilocytic astrocytoma (1493) 230
Campomelic dysplasia (3266) 348 Neurenteric cyst (2938) 356
Caudal regression (1491) 78 Paraspinal soft tissue Neuromyelitis optica (1479) 328
Chance racture (2913) 136 Acute calcif c tendinitis o the longus Spinal cavernoma (2946) 254
Chiari II mal ormation (2600) 156 colli muscle (2918) 270 Subacute combined degeneration (2192) 182
Di use idiopathic skeletal hyperostosis Denervation edema; dropped head Syringomyelia (2202) 26
with racture (1511) 50 syndrome (1497) 338 Tethered cord (2980) 110
Dorsal dermal sinus with Extramedullary hematopoiesis (2997) 238 Transverse myelitis (2942) 258
epidermoid (2954) 248 Intramuscular myxoma, Mazabraud
Dural ectasia, Mar an syndrome (1515) 202 syndrome (2242) 302 Thecal sac
Juvenile arteriovenous Lipoblastoma (2220) 308 Cerebrospinal uid ow study (2207) 304
mal ormation (1513) 332 Lymphoma (2998) 236 Cerebrospinal uid leak evaluation (2188) 320
Juvenile idiopathic arthritis (2917) 126 Neuroblastoma (2215) 168 Cerebrospinal uid pulsation
Ligamentous injury (1516) 200 Retropharyngeal abscess (2953) 250 arti act (2198) 28
Ligamentous instability (2909) 138 Cervical myelomeningocele (1492) 210
Lipomyelocele (2194) 32 Posterior elements Lateral meningocele (2939) 264
Lymphangiomatosis (1487) 340 Baastrup phenomenon (2915) 130 Myelography (3216) 56

369
370 Chapter Index

Neurof bromatosis type 1 (2935) 266 Extension teardrop racture (2919) 124 Osteopetrosis (2947) 252
Postoperative dermoid (1521) 322 “Fishmouth” vertebra in sickle cell disease (2216) 18 Osteoporotic compression ractures (2914) 134
Spinal subarachnoid hemorrhage Fracture C2—type 1 (2195) 30 Osteosarcoma (1488) 216
(SSH) (3298) 336 Fracture C2—type 2 (2597) 2 Paget disease (multi ocal) (2203) 176
Tarlov cyst (2921) 120 Fracture C2—type 3 (2196) 300 Percutaneous vertebroplasty (2982) 108
Von Hippel-Lindau disease (2976) 244 Giant cell tumor (2907) 278 Platybasia (2598) 4
Hangman racture (2895) 148 Posterior lumbar interbody usion
Vertebral body Hemangioma (2886) 152 (PLIF) (3312) 214
Achondroplasia (2916) 128 Hemivertebra (2981) 242 Renal osteodystrophy: rugger jersey
Aggressive osteoblastoma (2897) 360 Hurler syndrome (Mucopolysaccharidosis spine (2898) 146
Ankylosing spondylitis (2905) 142 type I) (2936) 358 Romanus lesion (3217) 234
Atlantooccipital dislocation (2200) 178 Ivory vertebra sign (3222) 86 Rotatory subluxation (1522) 192
Benign notochordal cell tumor (BNCT) Je erson racture (2911) 274 Sacral aneurysmal bone cyst (1485) 220
(2995) 240 Jumped/perched acets (2882) 290 Sacral chordoma (2512) 8
Burst racture (2894) 150 Klippel-Feil syndrome (2190) 36 Sacral insu f ciency racture (2218) 166
Butter y vertebra (2189) 40 Kummell disease (1514) 80 Scheuermann disease (1482) 66
C1 osseous metastasis (1477) 68 Limbus vertebra (2191) 184 Solitary plasmacytoma o bone (SPB) (2979) 112
Calcium pyrophosphate deposition (1508) 54 Lumbosacral transitional vertebrae Thoracic paraspinal lines (TPL) (3246) 84
Chondrosarcoma (2912) 272 (LSTV) (3048) 98 Vertebra plana (2178) 44
Clay shoveler racture (2908) 140 Metastatic cauda equina compression (2880) 296 Vertebral biopsy (2943) 118
Di use marrow inf ltration (2201) 24 Osteoblastic metastases (2885) 154 Vertebral body replacement (VBR) (3102) 90
Diskogenic endplate changes (2205) 22 Osteogenesis imper ecta (2922) 268 Vertical C1-C2 dissociation (2906) 280
Subchapter Index
Note: Numbers in parentheses re er to Case IDs.

Cervical Vertebra plana (2178) 44 Posterior lumbar interbody usion


Acute calcif c tendinitis o the longus Vertebral body replacement (VBR) (3102) 90 (PLIF) (3312) 214
colli muscle (2918) 270 Vertical C1-C2 dissociation (2906) 280 Postoperative in ection lumbar spine (2240) 160
Anterior cervical discectomy and usion Postoperative dermoid (1521) 322
(ACDF) (3303) 330 Craniocervical junction Spondylolysis (pars de ect) (3049) 96
Artif cial disc replacement (ADR) (3101) 74 Atlantooccipital dislocation (2200) 178 Vertebral biopsy (2943) 118
Benign notochordal cell tumor (BNCT) (2995) 240 Bickersta encephalitis (2186) 312
Brachial plexus avulsion with C1 osseous metastasis (1477) 68 More than one subcategory
pseudomeningocele (1496) 228 Cerebrospinal uid ow study (2207) 304 Achondroplasia (2916) 128
Brachial plexus avulsion with spinal cord injury Chiari I mal ormation (2206) 20 Amyloid spondyloarthropathy (2899) 288
and pseudomeningocele (1495) 344 Chiari I mal ormation with syrinx (1475) 72 Ankylosing spondylitis (2905) 142
Brachial plexus hematoma and retracted Clival chordoma (2219) 16 Arthrogryposis multiplex congenita (1480) 346
nerve roots (1494) 194 Hypertrophic olivary degeneration (2217) 310 Atlantooccipital assimilation (2483) 10
Brachial plexus metastatic inf ltration (1498 ) 206 Platybasia (2598) 4 Burst racture (2894) 150
Calcium pyrophosphate deposition (1508) 54 Rheumatoid arthritis with basilar Campomelic dysplasia (3266) 348
Cervical aneurysmal bone cyst (1486) 218 invagination (2900) 286 Caudal regression (1491) 78
Cervical cord contusion (1505) 208 Spinal cavernoma (2946) 254 Cerebrospinal uid leak evaluation (2188) 320
Cervical ependymoma (1504) 58 Chiari II mal ormation (2600) 156
Cervical myelomeningocele (1492) 210 Lumbar Conjoined nerve root (2599) 158
Cervical rib (3050) 94 Annular f ssure (2951) 114 Diastematomyelia (split cord
Cervical spondylotic myelopathy (CSM) (3313) 82 Arachnoiditis (1501) 38 mal ormation) (2903) 282
Clay shoveler racture (2908) 140 Baastrup phenomenon (2915) 130 Di use marrow inf ltration (2201) 24
Denervation edema; dropped head Bacterial spinal meningitis (2185) 164 Dural ectasia, Mar an syndrome (1515) 202
syndrome (1497) 338 Bertolotti syndrome (2996) 106 Gibbus de ormity (1481) 342
Di use idiopathic skeletal hyperostosis Brucellosis (2181) 42 Juvenile idiopathic arthritis (2917) 126
with racture (1511) 50 Calcif c arachnoiditis (arachnoiditis Juvenile pilocytic astrocytoma (1493) 230
Dropped head syndrome due to inclusion body ossif cans) (1502) 60 Lipomyelocele (2194) 32
myositis (IBM) (3065) 354 Chronic in ammatory demyelinating Lumbosacral epidural lipomatosis (1500) 62
Enterovirus myelitis (2183) 196 polyneuropathy (CIDP) (3100) 352 Lumbosacral transitional vertebrae
Epidural abscess (3000) 102 Complication, lumbar puncture-inadvertent (LSTV) (3048) 98
Extension teardrop racture (2919) 124 epidural anesthetic injection (2241) 12 Multiple hereditary exostoses (2221) 306
Fracture C2—type 1 (2195) 30 Disc herniation—protrusion (3051) 70 Multiple myeloma (2920) 122
Fracture C2—type 2 (2597) 2 Disc desiccation (3104) 88 Neurof bromatosis type 2 (2945) 256
Fracture C2—type 3 (2196) 300 Disc ragment migration (2193) 34 Osteoblastic metastases (2885) 154
Glioblastoma multi orme, cervical cord (2239) 162 Disc herniation (2187) 14 Osteogenesis imper ecta (2922) 268
Hangman racture (2895) 148 Diskogenic endplate changes (2205) 22 Osteopetrosis (2947) 252
Hurler syndrome (Mucopolysaccharidosis Dorsal dermal sinus with epidermoid (2954) 248 Osteoporotic compression ractures (2914) 134
type 1) (2936) 358 Dysplastic spondylolisthesis (2883) 364 Paget disease (multi ocal) (2203) 176
Intramedullary metastasis (1476) 226 Facet joint injection (2937) 132 Psoriatic spondylitis (2904) 144
Je erson racture (2911) 274 Filar lipoma (747) 298 Radiation changes (2197) 180
Jumped/perched acets (2882) 290 “Fishmouth” vertebra in sickle cell Renal osteodystrophy: rugger jersey
Juvenile arteriovenous mal ormation (1513) 332 disease (2216) 18 spine (2898) 146
Klippel-Feil syndrome (2190) 36 Intradural disc herniation (2878) 366 Scheuermann disease (1482) 66
Ligamentous injury (1516) 200 Kummell disease (1514) 80 Spinal dysraphism, in utero (2214) 170
Ligamentous instability (2909) 138 Leptomeningeal metastasis (lung Spondyloptosis (1484) 64
Lipoblastoma (2220) 308 adenocarcinoma) (2944) 116 Tethered cord (2980) 110
Lymphangiomatosis (1487) 340 Limbus vertebra (2191) 184 Thoracic epidural lipomatosis (2176) 46
Lymphoma (2998) 236 Melanocytic schwannoma (3191) 350 Tuberous sclerosis (2884) 362
Malignant peripheral nerve sheath Metastatic cauda equina compression (2880) 296 VACTERL syndrome (2910) 276
tumor (1519) 198 Mixed injection on myelogram (1525) 48 Von Hippel-Lindau disease (2976) 244
Multi ocal f brous dysplasia with pathologic Myelography (3216) 56
racture (1517) 326 Neurof bromatosis type 1 (2935) 266 Miscellaneous
Multiple synovial cysts (2179) 314 Neurosarcoidosis (2941) 260 Myxopapillary ependymoma (2940) 262
Neuromyelitis optica (1479) 328 Osteoporotic cauda equina
Ossif cation o the posterior longitudinal compression (2881) 292 Sacrum/Coccyx
ligament (OPLL) (2901) 284 Osteosarcoma (1488) 216 Chondrosarcoma (2912) 272
Retropharyngeal abscess (2953) 250 Percutaneous disc aspiration and biopsy Giant cell tumor (2907) 278
Rotatory subluxation (1522) 192 (PDAB) (3231) 232 Intradural lipoma (2879) 294
Septic acet arthritis (2184) 172 Percutaneous vertebroplasty (2982) 108 Lumbosacral plexus metastatic
Subacute combined degeneration (2192) 182 Peridural f brosis (2177) 190 inf ltration (1499) 334

371
372 Subchapter Index

Pelvic osteomyelitis with associated Cerebrospinal uid pulsation arti act (2198) 28 Myelomalacia (2208) 174
sacroiliitis (1489) 212 Chance racture (2913) 136 Myof broma (1518) 324
Sacral aneurysmal bone cyst (1485) 220 Cystic schwannoma (1510) 52 Neurenteric cyst (2938) 356
Sacral chordoma (2512) 8 Disc herniation—extrusion (3047) 100 Neuroblastoma (2215) 168
Sacral insu f ciency racture (2218) 166 Dural arteriovenous f stula (1478) 224 Osteoid osteoma (2482) 6
Sacrococcygeal teratoma (1523) 318 Extramedullary hematopoiesis (2997) 238 Pott disease (2182) 186
Spinal subarachnoid hemorrhage Hemangioblastoma (1506) 204 Romanus lesion (3217) 234
(SSH) (3298) 336 Hemangioma (2886) 152 Solitary plasmacytoma o bone
Tarlov cyst (2921) 120 Hemivertebra (2981) 242 (SPB) (2979) 112
Hemorrhagic cord in arction (2180) 188 Spontaneous epidural hematoma (2999) 104
Thoracic Intramuscular myxoma, Mazabraud Syringomyelia (2202) 26
Aggressive osteoblastoma (2897) 360 syndrome (2242) 302 Thoracic paraspinal lines (TPL) (3246) 84
Annular rupture (3052) 92 Ivory vertebra sign (3222) 86 Transdural cord herniation (1527) 316
Arachnoid cyst (1483) 222 Lateral meningocele (2939) 264 Transverse myelitis (2942) 258
Butter y vertebra (2189) 40 Meningioma, myelography (1359) 76 Tuberculous spondylitis (2955) 246
Di culty Level Index
Note: Numbers in parentheses re er to Case IDs.

Easy Cases Mixed injection on myelogram (1525) 48 Intradural lipoma (2879) 294
Achondroplasia (2916) 128 Multiple myeloma (2920) 122 Intramedullary metastasis (1476) 226
Ankylosing spondylitis (2905) 142 Myelography (3216) 56 Je erson racture (2911) 274
Annular f ssure (2951) 114 Osteoblastic metastases (2885) 154 Jumped/perched acets (2882) 290
Annular rupture (3052) 92 Osteoid osteoma (2482) 6 Juvenile pilocytic astrocytoma (1493) 230
Arachnoiditis (1501) 38 Osteoporotic compression ractures (2914) 134 Lateral meningocele (2939) 264
Artif cial disc replacement (ADR) (3101) 74 Percutaneous vertebroplasty (2982) 108 Ligamentous injury (1516) 200
Atlantooccipital assimilation (2483) 10 Platybasia (2598) 4 Limbus vertebra (2191) 184
Baastrup phenomenon (2915) 130 Psoriatic spondylitis (2904) 144 Lymphoma (2998) 236
Bertolotti syndrome (2996) 106 Renal osteodystrophy: rugger jersey Malignant peripheral nerve sheath
Brucellosis (2181) 42 spine (2898) 146 tumor (1519) 198
Burst racture (2894) 150 Sacral chordoma (2512) 8 Metastatic cauda equina
Butter y vertebra (2189) 40 Scheuermann disease (1482) 66 compression (2880) 296
C1 osseous metastasis (1477) 68 Solitary plasmacytoma o bone (SPB) (2979) 112 Myelomalacia (2208) 174
Calcif c arachnoiditis (arachnoiditis Spondylolysis (pars de ect) (3049) 96 Myxopapillary ependymoma (2940) 262
ossif cans) (1502) 60 Spondyloptosis (1484) 64 Neuroblastoma (2215) 168
Calcium pyrophosphate deposition (1508) 54 Spontaneous epidural hematoma (2999) 104 Neurof bromatosis type 1 (2935) 266
Caudal regression (1491) 78 Syringomyelia (2202) 26 Neurof bromatosis type 2 (2945) 256
Cerebrospinal uid pulsation arti act (2198) 28 Tarlov cyst (2921) 120 Neurosarcoidosis (2941) 260
Cervical ependymoma (1504) 58 Tethered cord (2980) 110 Ossif cation o the posterior longitudinal
Cervical rib (3050) 94 Thoracic epidural lipomatosis (2176) 46 ligament (OPLL) (2901) 284
Cervical spondylotic myelopathy (CSM) (3313) 82 Thoracic paraspinal lines (TPL) (3246) 84 Osteogenesis imper ecta (2922) 268
Chance racture (2913) 136 Vertebra plana (2178) 44 Osteopetrosis (2947) 252
Chiari I mal ormation (2206) 20 Vertebral biopsy (2943) 118 Osteoporotic cauda equina
Chiari I mal ormation with syrinx (1475) 72 Vertebral body replacement (VBR) (3102) 90 compression (2881) 292
Clay shoveler racture (2908) 140 Osteosarcoma (1488) 216
Clival chordoma (2219) 16 Moderately Di f cult Cases Paget disease (multi ocal) (2203) 176
Complication, lumbar puncture—inadvertent Acute calcif c tendinitis o the longus Pelvic osteomyelitis with associated
epidural anesthetic injection (2241) 12 colli muscle (2918) 270 sacroiliitis (1489) 212
Cystic schwannoma (1510) 52 Amyloid spondyloarthropathy (2899) 288 Percutaneous disc aspiration and
Di use idiopathic skeletal hyperostosis with Arachnoid cyst (1483) 222 biopsy (PDAB) (3231) 232
racture (1511) 50 Atlantooccipital dislocation (2200) 178 Peridural f brosis (2177) 190
Di use marrow inf ltration (2201) 24 Bacterial spinal meningitis (2185) 164 Posterior lumbar interbody
Disc desiccation (3104) 88 Benign notochordal cell usion (PLIF) (3312) 214
Disc ragment migration (2193) 34 tumor (BNCT) (2995) 240 Postoperative in ection lumbar spine (2240) 160
Disc herniation (2187) 14 Brachial plexus avulsion with Pott disease (2182) 186
Disc herniation—extrusion (3047) 100 pseudomeningocele (1496) 228 Radiation changes (2197) 180
Disc herniation—protrusion (3051) 70 Brachial plexus hematoma and retracted nerve Retropharyngeal abscess (2953) 250
Diskogenic endplate changes (2205) 22 roots (1494) 194 Rheumatoid arthritis with basilar
Epidural abscess (3000) 102 Brachial plexus metastatic invagination (2900) 286
Extension teardrop racture (2919) 124 inf ltration (1498) 206 Romanus lesion (3217) 234
Facet joint injection (2937) 132 Cervical aneurysmal bone cyst (1486) 218 Rotatory subluxation (1522) 192
“Fishmouth” vertebra in sickle cell Cervical cord contusion (1505) 208 Sacral aneurysmal bone cyst (1485) 220
disease (2216) 18 Cervical myelomeningocele (1492) 210 Sacral insu f ciency racture (2218) 166
Fracture C2—type 1 (2195) 30 Chiari II mal ormation (2600) 156 Septic acet arthritis (2184) 172
Fracture C2—type 2 (2597) 2 Chondrosarcoma (2912) 272 Spinal cavernoma (2946) 254
Hangman racture (2895) 148 Conjoined nerve root (2599) 158 Spinal dysraphism, in utero (2214) 170
Hemangioma (2886) 152 Diastematomyelia (split cord Subacute combined degeneration (2192) 182
Ivory vertebra sign (3222) 86 mal ormation) (2903) 282 Transverse myelitis (2942) 258
Juvenile idiopathic arthritis (2917) 126 Dorsal dermal sinus with epidermoid (2954) 248 Tuberculous spondylitis (2955) 246
Klippel-Feil syndrome (2190) 36 Dural arteriovenous f stula (1478) 224 VACTERL syndrome (2910) 276
Kummell disease (1514) 80 Dural ectasia, Mar an syndrome (1515) 202 Vertical C1-C2 dissociation (2906) 280
Leptomeningeal metastasis (lung Enterovirus myelitis (2183) 196 Von Hippel-Lindau disease (2976) 244
adenocarcinoma) (2944) 116 Extramedullary hematopoiesis (2997) 238
Ligamentous instability (2909) 138 Filar lipoma (747) 298 Most Di f cult Cases
Lipomyelocele (2194) 32 Giant cell tumor (2907) 278 Aggressive osteoblastoma (2897) 360
Lumbosacral epidural lipomatosis (1500) 62 Glioblastoma multi orme, cervical cord (2239) 162 Anterior cervical discectomy and
Lumbosacral transitional vertebrae Hemangioblastoma (1506) 204 usion (ACDF) (3303) 330
(LSTV) (3048) 98 Hemivertebra (2981) 242 Arthrogryposis multiplex congenita (1480) 346
Meningioma, myelography (1359) 76 Hemorrhagic cord in arction (2180) 188 Bickersta encephalitis (2186) 312

373
374 Di culty Level Index

Brachial plexus avulsion with spinal cord injury Gibbus de ormity (1481) 342 Multi ocal f brous dysplasia with pathologic
and pseudomeningocele (1495) 344 Hurler syndrome (Mucopolysaccharidosis racture (1517) 326
Campomelic dysplasia (3266) 348 type 1) (2936) 358 Multiple hereditary exostoses (2221) 306
Cerebrospinal uid ow study (2207) 304 Hypertrophic olivary degeneration (2217) 310 Multiple synovial cysts (2179) 314
Cerebrospinal uid leak evaluation (2188) 320 Intradural disc herniation (2878) 366 Myof broma (1518) 324
Chronic in ammatory demyelinating Intramuscular myxoma, Mazabraud Neurenteric cyst (2938) 356
polyneuropathy (CIDP) (3100) 352 syndrome (2242) 302 Neuromyelitis optica (1479) 328
Denervation edema; dropped head Juvenile arteriovenous mal ormation (1513) 332 Postoperative dermoid (1521) 322
syndrome (1497) 338 Lipoblastoma (2220) 308 Sacrococcygeal teratoma (1523) 318
Dropped head syndrome due to inclusion body Lumbosacral plexus metastatic Spinal subarachnoid hemorrhage
myositis (IBM) (3065) 354 inf ltration (1499) 334 (SSH) (3298) 336
Dysplastic spondylolisthesis (2883) 364 Lymphangiomatosis (1487) 340 Transdural cord herniation (1527) 316
Fracture C2—type 3 (2196) 300 Melanocytic schwannoma (3191) 350 Tuberous sclerosis (2884) 362
Author Index
Note: Numbers in parentheses re er to Case IDs.

Allison Grayev Intramuscular myxoma, Mazabraud Burst racture (2894) 150


Arachnoid cyst (1483) 222 syndrome (2242) 302 Campomelic dysplasia (3266) 348
Arachnoiditis (1501) 38 Juvenile arteriovenous mal ormation (1513) 332 Chance racture (2913) 136
Arthrogryposis multiplex congenita (1480) 346 Juvenile pilocytic astrocytoma (1493) 230 Chondrosarcoma (2912) 272
Atlantooccipital assimilation (2483) 10 Klippel-Feil syndrome (2190) 36 Clay shoveler racture (2908) 140
Atlantooccipital dislocation (2200) 178 Kummell disease (1514) 80 Diastematomyelia (split cord
Bacterial spinal meningitis (2185) 164 Ligamentous injury (1516) 200 mal ormation) (2903) 282
Bickersta encephalitis (2186) 312 Limbus vertebra (2191) 184 Dysplastic spondylolisthesis (2883) 364
Brachial plexus avulsion with Lipoblastoma (2220) 308 Extension teardrop racture (2919) 124
pseudomeningocele (1496) 228 Lipomyelocele (2194) 32 Giant cell tumor (2907) 278
Brachial plexus avulsion with spinal cord injury Lumbosacral epidural lipomatosis (1500) 62 Hangman racture (2895) 148
and pseudomeningocele (1495) 344 Lumbosacral plexus metastatic Hemangioma (2886) 152
Brachial plexus hematoma and retracted nerve inf ltration (1499) 334 Intradural disc herniation (2878) 366
roots (1494) 194 Lymphangiomatosis (1487) 340 Intradural lipoma (2879) 294
Brachial plexus metastatic Malignant peripheral nerve sheath Je erson racture (2911) 274
inf ltration (1498) 206 tumor (1519) 198 Jumped/perched acets (2882) 290
Brucellosis (2181) 42 Meningioma, myelography (1359) 76 Juvenile idiopathic arthritis (2917) 126
Butter y vertebra (2189) 40 Mixed injection on myelogram (1525) 48 Ligamentous instability (2909) 138
C1 osseous metastasis (1477) 68 Multi ocal f brous dysplasia with pathologic Metastatic cauda equina
Calcif c arachnoiditis (arachnoiditis racture (1517) 326 compression (2880) 296
ossif cans) (1502) 60 Multiple hereditary exostoses (2221) 306 Multiple myeloma (2920) 122
Calcium pyrophosphate deposition (1508) 54 Multiple synovial cysts (2179) 314 Ossif cation o the posterior longitudinal
Caudal regression (1491) 78 Myelomalacia (2208) 174 ligament (OPLL) (2901) 284
Cerebrospinal uid ow study (2207) 304 Myof broma (1518) 324 Osteoblastic metastases (2885) 154
Cerebrospinal uid leak evaluation (2188) 320 Neuroblastoma (2215) 168 Osteogenesis imper ecta (2922) 268
Cerebrospinal uid pulsation arti act (2198) 28 Neuromyelitis optica (1479) 328 Osteopetrosis (2947) 252
Cervical aneurysmal bone cyst (1486) 218 Osteoid osteoma (2482) 6 Osteoporotic cauda equina
Cervical cord contusion (1505) 208 Osteosarcoma (1488) 216 compression (2881) 292
Cervical ependymoma (1504) 58 Paget disease (multi ocal) (2203) 176 Osteoporotic compression ractures (2914) 134
Cervical myelomeningocele (1492) 210 Pelvic osteomyelitis with associated Psoriatic spondylitis (2904) 144
Chiari I mal ormation (2206) 20 sacroiliitis (1489) 212 Renal osteodystrophy: rugger jersey
Chiari I mal ormation with syrinx (1475) 72 Peridural f brosis (2177) 190 spine (2898) 146
Chiari II mal ormation (2600) 156 Platybasia (2598) 4 Rheumatoid arthritis with basilar
Clival chordoma (2219) 16 Postoperative dermoid (1521) 322 invagination (2900) 286
Complication, lumbar puncture—inadvertent Postoperative in ection lumbar spine (2240) 160 Tarlov cyst (2921) 120
epidural anesthetic injection (2241) 12 Pott disease (2182) 186 Tuberous sclerosis (2884) 362
Conjoined nerve root (2599) 158 Radiation changes (2197) 180 VACTERL syndrome (2910) 276
Cystic schwannoma (1510) 52 Rotatory subluxation (1522) 192 Vertical C1-C2 dissociation (2906) 280
Denervation edema; dropped head Sacral aneurysmal bone cyst (1485) 220
syndrome (1497) 338 Sacral chordoma (2512) 8 Reuben Grech
Di use idiopathic skeletal hyperostosis with Sacral insu f ciency racture (2218) 166 Annular f ssure (2951) 114
racture (1511) 50 Sacrococcygeal teratoma (1523) 318 Annular rupture (3052) 92
Di use marrow inf ltration (2201) 24 Scheuermann disease (1482) 66 Anterior cervical discectomy and
Disc ragment migration (2193) 34 Septic acet arthritis (2184) 172 usion (ACDF) (3303) 330
Disc herniation (2187) 14 Spinal dysraphism, in utero (2214) 170 Artif cial disc replacement (ADR) (3101) 74
Diskogenic endplate changes (2205) 22 Spondyloptosis (1484) 64 Benign notochordal cell
Dural arteriovenous f stula (1478) 224 Subacute combined degeneration (2192) 182 tumor (BNCT) (2995) 240
Dural ectasia, Mar an syndrome (1515) 202 Syringomyelia (2202) 26 Bertolotti syndrome (2996) 106
Enterovirus myelitis (2183) 196 Thoracic epidural lipomatosis (2176) 46 Cervical rib (3050) 94
“Fishmouth” vertebra in sickle cell Transdural cord herniation (1527) 316 Cervical spondylotic myelopathy (CSM) (3313) 82
disease (2216) 18 Vertebra plana (2178) 44 Chronic in ammatory demyelinating
Fracture C2—type 1 (2195) 30 polyneuropathy (CIDP) (3100) 352
Fracture C2—type 2 (2597) 2 Sayed Ali Disc desiccation (3104) 88
Fracture C2—type 3 (2196) 300 Achondroplasia (2916) 128 Disc herniation—extrusion (3047) 100
Gibbus de ormity (1481) 342 Acute calcif c tendinitis o the longus Disc herniation—protrusion (3051) 70
Glioblastoma multi orme, cervical cord (2239) 162 colli muscle (2918) 270 Dorsal dermal sinus with epidermoid (2954) 248
Hemangioblastoma (1506) 204 Aggressive osteoblastoma (2897) 360 Dropped head syndrome due to inclusion body
Hemorrhagic cord in arction (2180) 188 Amyloid spondyloarthropathy (2899) 288 myositis (IBM) (3065) 354
Hypertrophic olivary degeneration (2217) 310 Ankylosing spondylitis (2905) 142 Epidural abscess (3000) 102
Intramedullary metastasis (1476) 226 Baastrup phenomenon (2915) 130 Extramedullary hematopoiesis (2997) 238

375
376 Author Index

Facet joint injection (2937) 132 Myelography (3216) 56 Solitary plasmacytoma o bone (SPB) (2979) 112
Filar lipoma (747) 298 Myxopapillary ependymoma (2940) 262 Spinal cavernoma (2946) 254
Hemivertebra (2981) 242 Neurenteric cyst (2938) 356 Spinal subarachnoid hemorrhage
Hurler syndrome (Mucopolysaccharidosis Neurof bromatosis type 1 (2935) 266 (SSH) (3298) 336
type 1) (2936) 358 Neurof bromatosis type 2 (2945) 256 Spondylolysis (pars de ect) (3049) 96
Ivory vertebra sign (3222) 86 Neurosarcoidosis (2941) 260 Spontaneous epidural hematoma (2999) 104
Lateral meningocele (2939) 264 Percutaneous disc aspiration and Tethered cord (2980) 110
Leptomeningeal metastasis (lung biopsy (PDAB) (3231) 232 Thoracic paraspinal lines (TPL) (3246) 84
adenocarcinoma) (2944) 116 Percutaneous vertebroplasty (2982) 108 Transverse myelitis (2942) 258
Lumbosacral transitional vertebrae Posterior lumbar interbody Tuberculous spondylitis (2955) 246
(LSTV) (3048) 98 usion (PLIF) (3312) 214 Vertebral biopsy (2943) 118
Lymphoma (2998) 236 Retropharyngeal abscess (2953) 250 Vertebral body replacement (VBR) (3102) 90
Melanocytic schwannoma (3191) 350 Romanus lesion (3217) 234 Von Hippel-Lindau disease (2976) 244
Acknowledgment Index
Note: Numbers in parentheses re er to Case IDs.

Adrian Mizzi Joseph N Borg


Chronic inf ammatory demyelinating polyneuropathy (CIDP) (3100) 352 Dropped head syndrome due to inclusion body myositis (IBM) (3065) 354
Extramedullary hematopoiesis (2997) 238 Posterior lumbar interbody usion (PLIF) (3312) 214
Hemivertebra (2981) 242
Sanjay Patel
Neuro bromatosis type 1 (2935) 266
Intradural disc herniation (2878) 366
Chetan Shah Metastatic cauda equina compression (2880) 296
Filar lipoma (747) 298 Osteoporotic cauda equina compression (2881) 292

Dhiren Shah Stephan Grech


Chondrosarcoma (2912) 272 Anterior cervical discectomy and usion (ACDF) (3303) 330
Dysplastic spondylolisthesis (2883) 364 Arti cial disc replacement (ADR) (3101) 74
Je erson racture (2911) 274 Vertebral body replacement (VBR) (3102) 90
Multiple myeloma (2920) 122
Osteoblastic metastases (2885) 154

John Thornton
Lateral meningocele (2939) 264
Myxopapillary ependymoma (2940) 262
Von Hippel-Lindau disease (2976) 244

377

You might also like